You are on page 1of 654

Krok 1 – 2013 Base(Biology)

Medical PG WhatsApp Group

Results
0 of 99 questions answered correctly

Your time: 00:03:50

You have reached 0 of 99 points, (0%)

RESTART QUIZ VIEW QUESTIONS

1. Question
A 16 year-old patient got numerous traumas in automobile accident. Now the
patient is having a shock. АP – 80/60 mm Hg. daily urine volume 60-80 ml.
What pathogenic mechanism leads to kidneys function violation?

1. Increased vasopressin blood concentration


2. Trauma of the urinary bladder
3. Increased pressure in Bowman’s capsule
4. Increased osmotic pressure in glomerular capillaries
5. Decreased hydrostatic pressure in glomerular capillaries

2. Question
LAKSHMAN

A teenager was irradiated with high radiation dose that resulted in serious
damages of lymphoid system, lysis of many lymphocytes. Restoration of
normal hemogram is possible due to the functioning of the following gland:

LAKSHMAN
1. Thyroid
2. Pancreas
3. Thymus
4. Adrenal
5. Liver

3. Question
It is known that the gene responsible for development of blood groups
according to AB0 system has three allele variants. If a man has IV blood
group, it can be explained by the following variability form:

1. Combinative
2. Phenocopy
3. Mutational
4. Phenotypic
5. Genocopy

4. Question
A patient complains of skin itch, especially between fingers, in the inguinal
creases, on the lower abdomen. Examination of these regions revealed there
some small vesicles. Laboratory diagnostics allowed to establish that this
condition had been caused by a representative of Arthropoda. Specify the
disease caused by this arthropod:

1. Demodicosis
2. Scabies
3. Dermatotropic leishmaniasis
4. Myiasis
5. Pediculosis

5. Question
A child complains of general weakness, loss of appetite, a troubled sleep,
itching in the perianal area. The provisional diagnosis is enterobiasis. In
order to specify this diagnosis it is necessary to perform:

1. Biopsy of muscle tissue


2. Duodenal contents analysis
3. Roentgenoscopy
4. Scraping from perianal folds
5. Immune diagnostics

6. Question
A woman delivered a dead child with multiple developmental defects. What
protozoan disease might have caused the intrauterine death?

1. Lambliasis
2. Leishmaniasis
3. Toxoplasmosis
LAKSHMAN

4. Malaria
5. Amebiasis

7. Question
Electrocardiogram of a 45-year-old man showed absence of P-wave in all the

LAKSHMAN
leads. What part of the conducting system is blocked?

1. Purkinje’s bres
2. Branches of the bundle of His
3. Common branch of the bundle of His
4. Sinu-atrial node
5. Atrioventricular node

8. Question
The patient with pneumonia was treated with antibiotics for a long period.
After treatment patient complains of frequent and watery stool, abdomenal
pain. What is the reason of intestine function disorder?

1. Hereditary enzyme defect


2. Antibiotics toxic in uence on the GIT
3. Intestinal disbacteriosis development
4. Autoimmune reaction development
5. Bacteria toxins in uence

9. Question
A patient complains of pain in the area of his liver. Duodenal intubation
revealed yellowish, oval, narrowed at the poles eggs with an operculum at the
end. Size of these eggs is the smallest among all helminth eggs. What is the
most probable diagnosis?
1. Beef tapeworm infection
2. Opisthorchosis
3. Diphyllobothriasis
4. Echinococcosis
5. Teniasis

10. Question
The conjugated protein necessarily contains special component as a non-
protein part. Choose the substance that can’t carry out this function:

1. HNO3
2. Thiamine pyrophosphate
3. Glucose
4. AMP
5. АТP

11. Question
While studing of the family tree with history of hypertrichosis
(hyperhirsutism of the ear) this sign was founded only in the men and it was
inherited from father to the son. Define the type of hypertrichosis
inheritance?

1. Connected with Y-chromosome


LAKSHMAN

2. Autosomal- recessive
3. Connected with Х-chromosome dominant
4. Connected with Х-chromosome recessive

LAKSHMAN
5. Autosomal-dominant

12. Question
A cell at the stage of mitosis anaphase was stimulated by colchicine that
inhibits chromosome separation to the poles. What type of mutation will be
caused?

1. Polyploidy
2. Duplication
3. Deletion
4. Inversion
5. Translocation

13. Question
After breathing with poisonous steams there is an increased quantity of slime
in respiratory passages of a chemical production worker. What of respiratory
tract epithelial cells participate in mucousa moistening?

1. Goblet cells
2. Endocrine cells
3. Fibroblasts
4. Intercalated cells
5. Langergans cells

14. Question
When a patient with traumatic impairment of the brain was examined, it was
discovered that he had stopped to distinguish displacement of an object on the
skin. What part of the brain was damaged?

1. Frontal zone
2. Frontal central gurus
3. Posterior central gurus
4. Occipital zone of the cortex
5. Parietal zone of the cortex

15. Question
The reason of occurrence of some diseases of an oral cavity is connected with
structural peculiarities of its mucous membrane. What morphological
attributes characterize these features?

1. Well developed muscularis, no submucosa


2. No muscularis mucosa, strati ed squamous epithelium
3. Simple columnar ciliated epithelium
4. Transitional epithelium, no muscularis mucosa
5. Transitional epithelium, no submucosa

16. Question
A man has worked in an African country for 3 years. A month after his return
LAKSHMAN

to Ukraine he consulted an ophthalmologist and complained about eye ache,


eyelid edema, lacrimation and temporary visual impairment. Underneath the
eye conjunctiva the doctor revealed helminths 30-50 mm long with elongated
filiform body. What diagnosis might be suspected?

LAKSHMAN
1. Enterobiasis
2. Trichocephaliasis
3. Diphyllobothriasis
4. Ascaridiasis
5. Filariasis

17. Question
Moving of the daughter chromatids to the poles of the cell is observed in the
mitotically dividing cell. On what stage of the mitotic cycle is this cell?

1. Anaphase
2. Interphase
3. Telophase
4. Metaphase
5. Prophase

18. Question
After the trauma, the patient’s right n.vagus was damaged. Which violation of
the cardiac activity is possible in this case?

1. Violation of a conductivity in the right auricle


2. Block of a conductivity in the atrio-ventricular node
3. Violation of the automatism of a Kiss-Fleck node
4. Violation of the automatism of a atrio-ventricular node
5. Arrhythmia

19. Question
Parents with ill child came to the infectionist. They worked in one of the Asian
countries for a long time. Child has earthy colored skin, loss of appetite,
laxity, enlarged liver, spleen, peripheral glands. What protozoan illness can
this child have?

1. Amebiasis
2. Lambliasis
3. Visceral leishmaniasis
4. Balantidiasis
5. Toxoplasmosis

20. Question
Life cycle of a cell includes the process of DNA autoreduplication. As a result
of it monochromatid chromosomes turn into bichromatid ones. What period
of cell cycle does this phenomenon fall into?

1. S
2. G2
3. Go
4. M
LAKSHMAN

5. G1

21. Question
Examination of cell culture got from a patient with lysosomal pathology

LAKSHMAN
revealed accumulation of great quantity of lipids in the lysosomes. What of
the following diseases is this disturbance typical for?

1. Tay-Sachs disease
2. Gout
3. Galactosemia
4. Phenylketonuria
5. Wilson disease

22. Question
The patient with diabetes mellitus has been delivered in hospital in the state
of unconsciousness. Arterial pressure is low. The patient has acidosis. Point
substances, which accumulation in the blood results in these manifestations:

1. Monosaccharides
2. Cholesterol esters
3. Amino acids
4. High fatty acids
5. Ketone bodies

23. Question
A patient has acne on his face. Microscopic examination of scrapings from the
affected areas revealed living porrect vermiform arthropoda 0.2-0.5 mm large
with four pairs of short extremities in the front part of their bodies. What is
the laboratory diagnosis?
1. Myiasis
2. Phthiriasis
3. Pediculosis
4. Demodicosis
5. Scabies

24. Question
For the purpose of myocardium infarction treatment a patient was injected
with embryonal stem cells derived from this very patient by means of
therapeutic cloning . What transplantation type is it?

1. Isotransplantation
2. Autotransplantation
3. Heterotransplantation
4. Allotransplantation
5. Xenotransplantation

25. Question
Examination of a patient revealed reduced contents of magnesium ions that
are necessary for attachment of ribosomes to the granular endoplasmatic
reticulum. It is known that it causes disturbance of protein biosynthesis.
What stage of protein biosynthesis will be disturbed?
LAKSHMAN

1. Aminoacid activation
2. Replication
3. Termination

LAKSHMAN
4. Transltion
5. Transcription

26. Question
The cell of the laboratory animal was overdosed with Roentgen rays. As a
result albuminous fragments formed in the cytoplasm. What cell organoid
will take part at their utilization?

1. Cells centre
2. Lysosomes
3. Golgi complex
4. Ribosome
5. Endoplasmic reticulum

27. Question
In course of practical training students studied a stained blood smear of a
mouse with bacteria phagocyted by leukocytes. What cell organella completes
digestion of these bacteria?

1. Granular endoplasmic reticulum


2. Lisosomes
3. Ribosomes
4. Golgi apparatus
5. Mytochondrions

28. Question
As a result of prophylactic medical examination a 7 year old boy was
diagnosed with Lesch-Nyhan syndrome (only boys fall ill with it). The boys
parents are healthy but his grandfather by his mothers side suffers from the
same disease. What type of disease inheritance is it?

1. Autosomal dominant
2. Dominant, sex-linked
3. Autosomal recessive
4. Recessive, sex-linked
5. Semidominance

29. Question
A 46 year-old patient has complained of headache, fatigue, thirst, pains in the
spine and joints for the last 2 years. Clinically observed disproportional
enlargement of hands, feet, nose, superciliary arches. He notes that he needed
to buy bigger shoes three times. What is the main reason of such
disproportional enlargement of different parts of the body?

1. Cartilaginous tissue proliferation under growth hormone in uence


2. Increased sensitivity of the tissues to growth hormone
3. Joints chronic in ammation development
4. Increased sensitivity of the tissues to insulin
5. Joints dystrophy development
LAKSHMAN

30. Question
Examination of newborns in one of the Ukrainian cities revealed a baby with
phenylketonuria. The baby parents don’t suffer from this disease and have
two other healthy children. Specify the most likely parents genotype with

LAKSHMAN
phenylketonuria gene:

1. Аа х аа
2. Аа х АА
3. Аа х Аа
4. АА х аа
5. Аа х аа

31. Question
An ovary specimen stained by hematoxylin-eosin presents a follicle, where
cells of follicular epithelium are placed in 1-2 layers and have cubic form,
there is a bright-red membrane around the ovocyte. What follicle is it?

1. Secondary
2. Primordial
3. Atretic
4. Primary
5. Mature

32. Question
Cytogenetic examination of a patient with reproductive dysfunction revealed
normal karyotype 46 ХY in some cells, but most cells have karyotype of
Klinefelters syndrome – 47 ХХY. Such cell heterogenity is called:

1. Duplication
2. Mosaicism
3. Transposition
4. Monomorphism
5. Inversion

33. Question
The CNS stimulation produced by methylxanthines, such as caffeine, is most
likely due to the antagonism of one of the following receptors:

1. Adenosine receptors
2. Glycine receptors
3. GABA receptors
4. Cholinergic muscarinic receptors
5. Glutamate receptors

34. Question
Highly injured person gradually died. Please choose the indicator of
biological death:

1. Autolysis and decay in the cells


2. Disarray of chemical processes
3. Absence of palpitation
LAKSHMAN

4. Loss of consciousness
5. Absence of movements

35. Question

LAKSHMAN
An alcoholic woman has born a girl with mental and physical developmental
lag. Doctors diagnosed the girl with fetal alcohol syndrome. What effect is the
cause of the girl’s state?

1. Mutagenic
2. Malignization
3. Carcinogenic
4. Teratogenic
5. Mechanic

36. Question
Medical examination at the military registration and enlistment office
revealed that a 15-year-old boy was high, with eunuchoid body proportions,
gynecomastia, female pattern of pubic hair distribution. The boy had also fat
deposits on the thighs, no facial hair, high voice, subnormal intelligence
quotient. Which karyotype corresponds with this disease?

1. 46, XY
2. 47, XXX
3. 46, XX
4. 47, XXY
5. 45, XO

37. Question
A patient suffering from thyrotoxicosis symptoms of vegetoasthenic
syndrome was revealed. What of the following would show the histological
appearance of a thyroid gland being stimulated by thyroid-stimulating
hormone (TSH)?

1. An abundance of colloid in the lumen of the follicle


2. Decreased numbers of parafollicular capillaries
3. Decreased numbers of follicular cells
4. Increased numbers of parafollicular cells
5. Columnar-shaped follicular cells

38. Question
Following exposure to radiation a lot of mutant cells appeared in a patient.
Some time later most of them were detected and destroyed by the following
cells of the immune system:

1. T-lymphocytes-killers
2. B-lymphocyte
3. Stem cells
4. Plasmoblasts
5. T-lymphocytes-supressors

39. Question
LAKSHMAN

A patient working at a pig farm complains about paroxysmal abdominal


pain, liquid feces with admixtures of mucus and blood, headache, weakness,
fever. Examination of large intestine revealed ulcers from 1 mm up to several
cm large, feces contained oval unicellular organisms with cilia. What disease
should be suspected?

LAKSHMAN
1. Balantidiasis
2. Amebiasis
3. Trichomoniasis
4. Toxoplasmosis
5. Lambliasis

40. Question
A boy has I (I0I0) blood group and his sister has IV (IAIB) blood group. What
blood groups do their parents have?

1. III (IВI0) and IV (IAIB)


2. II (IАIА) and III (IВI0)
3. I (I0I0) and III (IВI0)
4. I (I0I0) and IV (IAIB)
5. II (IАI0) and III (IВI0)

41. Question
During regular examination of schoolchildren it was revealed that a 10 year
old girl had asymmetric oval eggs with a larva in the scrape from her
perianal folds. What diagnosis should be made?

1. Enterobiasis
2. Ascariasis
3. Trichocephalosis
4. Amebiasis
5. Ankylostomiasis

42. Question
According to the phenotypic diagnosis a female patient has been provisionally
diagnosed with X-chromosome polysomia. This diagnosis can be confirmed by
a cytogenetic method. What karyotype will allow to confirm the diagnosis?

1. 48(XXXY)
2. 47(ХХХ)
3. 48(XXYY)
4. 46(XX)
5. 47(XXY)

43. Question
A 25-year-old woman with red and itchy eczematoid dermatitis visits your
office. She had a dental procedure one day earlier with administration of a
local anesthetic. There were no other findings, although she indicated that she
had a history of allergic reactions. Which of the following drugs is most likely
involved?

1. Procaine
2. Lidocaine
LAKSHMAN

3. Etidocaine
4. Bupivacaine
5. Cocaine

LAKSHMAN
44. Question
Larvae were detected occasionally on the microscopic examination of the
sputum of the patient with pneumonia. Eosinophiles were detected on the
blood examination. What helminthiasis can be diagnosed?

1. Enterobiosis
2. Opistorchis
3. Trichocephaliasis
4. Paragonimiasis
5. Ascariasis

45. Question
A 58-year-old female has undergone surgery for necrotic bowel. Despite
having been treated with antibiotics, on postoperative day 5, she develops
symptoms (fever, hypotension, tachycardia, declining urine output, and
confusion) consistent with septic shock. What hemodynamic support would be
helpful?

1. Atropine administration
2. Fluids and Dobutamine infusion
3. Antibiotic administration
4. Fluid administration
5. Dobutamine infusion

46. Question
Golgi complex exports substances from a cell due to the fusion of the
membrane saccule with the cell membrane. The saccule contents flows out.
What process is it?

1. Endocytosis
2. Exocytosis
3. Active transport
4. Facilitated diffusion
5. All answers are false

47. Question
A family of students who came from Africa got a child with anemia signs. The
child died soon. Examination revealed that the childs erythrocytes have
abnormal semilunar shape. Specify genotypes of the childs parents:

1. АА х АА
2. Аа х Аа
3. Аа х аа
4. Аа х АА
5. aа х аа

48. Question
There is the change of teeth at the 6-8-year-old children: deciduous are
LAKSHMAN

replaced by permanent. What embrionic tissues are the sources of formation


of permanent teeth tissues?

1. Ectodermal epithelium of a tooth plate and mesenhime

LAKSHMAN
2. Entodermal epithelium of a tooth plate and mesenhime
3. I, II brachial arches
4. Mesodermal epithelium and mesenhime
5. Entodermal epithelium and mesoderm

49. Question
On autopsy of a still-born infant it is revealed heart development
abnormalities: ventricles are not separated, originates from the right part
single arterial trunk. For what class of vertebrate is such heart construction
characteristic?

1. Fishes
2. Mammals
3. Birds
4. Amphibian
5. Reptiles

50. Question
The calcium canals of cardiomyocytes have been blocked on an isolated
rabbits heart. What changes in the hearts activity can happen as a result?

1. Decreased force of the contraction


2. Decreased rate and force of heart beat
3. Decreased heart beat rate
4. Heart stops in diastole
5. Heart stops in systole

51. Question
Normal, actively dividing cells of human red bone marrow are analyzed.
What number of cells chromosomes is typical for G1 period?

1. 47
2. 48
3. 46
4. 45
5. 23

52. Question
Autopsy of a newborn boy revealed polydactylia, microcephalia, cheiloschisis
and uranoschisis as well as hypertrophy of parenchimatous organs. These
defects correspond with the description of Pataus syndrome. What is the most
probable cause of this pathology?

1. Trisomy of the 18th chromosome


2. Partial monosomy
3. Trisomy of the 21st chromosome
4. Trisomy of the 13th chromosome
5. Nondisjunction of sex chromosomes
LAKSHMAN

53. Question
A woman with ІІІ (B), Rh- blood group born a child with ІІ (A) blood group. The
child is diagnosed with hemolytic disease of newborn as a result of rhesus

LAKSHMAN
incompatibility. What blood group is the child’s father likely to have?

1. II (A), Rh-
2. I (0), Rh+
3. I (0), Rh-
4. ІІ (A), Rh+
5. III (B), Rh+

54. Question
You are studying functioning of a bacterial operon. The operator gene has
been released from the repressor gene. Immediately after this the following
process will start in the cell:

1. Transcription
2. Repression
3. Replication
4. Translation
5. Processing

55. Question
A patient in a transplantation centre underwent heart transplantation. The
organ was taken from a donor who died in a road accident. Foreign heart can
be rejected as a result of development of transplantation immunity. It is
usually prevented by means of:

1. Chemotherapy
2. Enzymes
3. Immunosuppressors
4. X-ray therapy
5. Ultrasound

56. Question
Genetic structure of eukaryote is “exon-intron-exon”. This structure-
functional organization of gene caused transcription peculiarities. What will
be pro-i-RNA according to the schema?

1. Exon-exon-intron
2. Exon-intron
3. Intron-exon
4. Exon-intron-exon
5. Exon-exon

57. Question
A doctor revealed tissues injury on patients scalp with localized suppurations
and diagnosed his disease as myiasis. This infestation is caused by larvae of
the following insect:

1. Malarial mosquito
2. Wohlfahrt y
LAKSHMAN

3. Mosquito
4. Kissing bug
5. Stable y (Stomoxys calcitrans)

LAKSHMAN
58. Question
A couple has a son with haemophilia. The parents are healthy but the
maternal grandfather also has haemophilia. Specify the type of inheritance:

1. Dominant sex-linked
2. Recessive autosomal
3. Autosomal dominant
4. Semidominance
5. Recessive sex-linked

59. Question
Part of the DNA chain turned about 180 degree due to gamma radiation. What
type of mutation took place in the DNA chain?

1. Deletion
2. Translocation
3. Replication
4. Inversion
5. Doubling

60. Question
The guide of the scientific expedition in India was native who always was with
his dog. What invasive diseases can be transmitted by the dog if it is the
source of invasion?
1. Fascioliasis
2. Dicroceliasis
3. Teniasis
4. Echinococcosis
5. Paragonimiasis

61. Question
Two days after consumption of smoked pork a patient got face and eye-lid
edemata, gastrointestinal disturbances, abrupt temperature rise, muscle
pain. Blood analysis showed full-blown eosinophilia. What helminth could the
patient be infected with?

1. Pinworm
2. Whipworm
3. Trichina
4. Ascarid
5. Hookworm

62. Question
Labeled aminoacids alanine and tryptophane were introducted to a mouse in
order to study localization of protein biosynthesis in its cells. Around what
organellas will the accumulation of labeled aminoacids be observed?
LAKSHMAN

1. Cell centre
2. Agranular endoplasmic reticulum
3. Lysosomes

LAKSHMAN
4. Golgi apparatus
5. Ribosomes

63. Question
A 55-year-old patient with continuing ventricular arrhythmias was admitted
to the hospital. The patient is taking timolol drops for glaucoma, daily insulin
injections for diabetes mellitus, and an ACE inhibitor for hypertension. You
have decided to use phenytoin instead of procainamide. What is the reason?

1. The local anesthetic effect of procainamide would aggravate the hypertension


2. The local anesthetic effect of procainamide would potentiate diabetes
3. The hypertensive effects of procainamide would aggravate the hypertension
4. The anticholinergic effect of procainamide would aggravate glaucoma
5. The cholinergic effects of procainamide would aggravate the diabetes

64. Question
According to the model of double DNA helix that was suggested by Watson
and Creek, it was established that one of chains would not be lost during
replication and the second chain would be synthesized complementary to the
first one. What way of replication is it?

1. Conservative
2. Dispersed
3. Identical
4. Analogous
5. Semiconservative
65. Question
Patients with similar complaints applied to the doctor: weakness, pain in the
intestines, disorder of GIT. Examination of the faeces revealed that one
patient with four nucleus cysts should be hospitalized immediately. For what
protozoa are such cysts typical?

1. Balantidium
2. Intestinal amoeba
3. Lamblia
4. Dysenteric amoeba
5. Trichomonas

66. Question
White-haired, with blue eyes girl was born in healthy parents. Irritability,
anxiety, troubled sleep and feeding developed in the first months of life of the
infant. What method of genetic investigation should be used for the exact
diagnosis?

1. Twin
2. Biochemical
3. Population-statistical
4. Genealogical
LAKSHMAN

5. Cytological

67. Question
It was proved that a molecule of immature mRNA (precursor mRNA)
contained more triplets than amino acids found in the synthesized protein.

LAKSHMAN
The reason for that is that translation is normally preceded by:

1. Replication
2. Initiation
3. Processing
4. Reparation
5. Mutation

68. Question
A female patient consulted a physician about digestive disorder, extended
abdominal pain. Examination revealed drastic decrease in hemoglobin
concentration. It is known from the anamnesis that while living in the Far
East the patient used to eat freshly-salted caviar. Some relatives living with
her had the similar condition. What is the most likely diagnosis?

1. Diphyllobothriasis
2. Teniasis
3. Ascaridiasis
4. Echinococcosis
5. Trichiniasis

69. Question
A 10-year-old child complains of weakness, nausea, irritability. Helminthes of
white color and 5-10 mm long were found on the underwear. On microscopy of
the scrape from the perianal folds achromic ova of the unsymmetrical form
were revealed. Indicate what helminth is parasiting on the child?
1. Trichuris
2. Trichina
3. Enterobins vermicularis
4. Ascaris lumbricoides
5. Ancylostoma duodenalis

70. Question
Sex chromosomes of a woman didn’t separate and move to the opposite poles
of a cell during gametogenesis (meiosis). The ovum was impregnated with a
normal spermatozoon. Which chromosomal disease can be found in her
child?

1. Downs syndrome
2. Cat cry syndrome
3. Turners syndrome
4. Edwards syndrome
5. Pataus syndrome

71. Question
A patient has symptoms of inflammation of urogenital tracts. Examination of
a vaginal smear revealed big monocellular, pear-shaped organisms with the
pointed spike at the posterior end of body, big nucleus and undulating
membrane. What protozoa were found in the smear?
LAKSHMAN

1. Lamblia intestinalis
2. Trichomonas buccalis

LAKSHMAN
3. Trichomonas hominis
4. Trichomonas vaginalis
5. Trypanosoma gambiense

72. Question
If strong oxidizers get into the bloodstream, a methemoglobin is formed. It is
a compound, where iron (II) becomes iron (III). What has to be done to save
the patient?

1. He has to be given pure oxygen


2. Respiratory centers have to be stimulated
3. Interchangeable hemotransfusion has to be done
4. He has to be calmed down and put to bed
5. Patient has to be exposed to the fresh air

73. Question
A man suffering from a hereditary disease married a healthy woman. They
got 5 children, three girls and two boys. All the girls inherited their father’s
disease. What is the type of the disease inheritance?

1. Y-linked
2. Autosomal recessive
3. Dominant, X-linked
4. Autosomal dominant
5. Recessive, X-linked
74. Question
In the ovary specimen colored with hematoxylin-eosin, follicle is determined
where cubic-shaped follicle epithelium cells are placed in 1-2 layers, and
scarlet covering is seen around ovocyte. Name this follicle:

1. Secondary
2. Atretic
3. Primary
4. Primordial
5. Mature

75. Question
A gynaecologist was examining a patient and revealed symptoms of genital
tract inflammation. A smear from vagina contains pyriform protozoa with a
spine, flagella at their front; there is also an undulating membrane. What
disease can be suspected?

1. Urogenital trichomoniasis
2. Lambliasis
3. Intestinal trichomoniasis
4. Toxoplasmosis
5. Balantidiasis
LAKSHMAN

76. Question
Woman applied to the medico-genetic consulting centre for information about
the risk of haemophilia in her son. Her husband has been suffering from this
disease since birth. Woman and her parents are healthy (dont have

LAKSHMAN
haemophilia). Is the boy likely to have the disease in this family?

1. 50% of the boys will be ill


2. All boys will be ill
3. All boys will be healthy
4. 75% of the boys will be ill
5. 25% of the boys will be ill

77. Question
A patient consulted an urologist about pain during urination. Analysis of his
urine taken in the daytime revealed eggs with a characteristic sharp point. It
is known from the anamnesis that the patient has recently returned from
Australia. What is the most likely diagnosis?

1. Urogenital schistosomiasis
2. Intestinal schistosomiasis
3. Japanese schistosomiasis
4. Dicroceliasis
5. Opisthorchiasis

78. Question
A 28-year-old female patient consulted a gynecologist about sterility.
Examination revealed underdeveloped ovaries and uterus, irregular
menstrual cycle. Analysis of the sex chromatin revealed 2 Barr’s bodies in
most somatic cells. What chromosome disease is most likely?
1. Klinefelters syndrome
2. Edwards syndrome
3. Triple X syndrome
4. Turners syndrome
5. Pataus syndrome

79. Question
According to the data of WHO, for about 250 mln of Earth population fall ill
with malaria. This disease is mostly spread in tropical and subtropical
regions. Range of its spread falls into the areal of the following mosquitoes:

1. Anopheles
2. Aedes
3. Culiseta
4. Mansonia
5. Culex

80. Question
The B cells of endocrine portion of pancreas are selectively damaged by
alloxan poisoning. How will it be reflected in blood plasma?

1. The content of sugar increases


LAKSHMAN

2. The level of sugar decreases


3. The content of globulins decreases
4. The content of albumins decreases
5. The content of brinogen decrease

LAKSHMAN
81. Question
A boy found a spider with the following morphological characteristics: it is 2
cm long, has roundish black abdomen with two rows of red spots on its dorsal
side; four pairs of jointed limbs are covered with small black hairs. What
arthropod is it?

1. Tarantula
2. Karakurt spider
3. Solpuga
4. Scorpion
5. Mite

82. Question
Patient 54 year-old, 5th day after surgical operation. Blood count:
Erythrocytes 3.61012/l, Hemoglobin 95 g/l, Erythrocyte’s hemoglobin content
(color index) 0.78; Leukocytes 16*109/l, Platelets 450*109/l Blood picture:
anizocytosis, poikilocytosis, reticulocytes- 3.8%. What anemia does this
patient have?

1. Acquired hemolytic anemia


2. Acute posthemorragic anemia
3. Hypoplastic anemia
4. Anemia from iron de ciency
5. Chronic posthemorragic anemia
83. Question
A patient after pathological process has a thickened alveolar membrane. The
direct consequence of the process will be the reduction of:

1. Diffuse lung capacity


2. Reserve expiratiory capacity
3. Minute respiratory capacity
4. Alveolar lung ventilation
5. Oxygen capacity of blood

84. Question
A patient experienced a sudden temperature rise up to 390С. After 6 hours the
temperature normalized. On the 2-nd day the attack recurred: in the period of
paroxysm the temperature reached 410С, apyrexial period began after 8
hours. What type of temperature profile is it?

1. Intermitting
2. Hectic
3. Septic
4. Recurrent
5. Continued

85. Question
LAKSHMAN

Tuberculosis can be treated by means of combined chemotherapy that


includes substances with different mechanisms of action.What
antituberculous medication inhibits transcription of RNA into DNA in
mycobacteria?

LAKSHMAN
1. Para-aminosalicylic acid
2. Streptomycin
3. Isoniazid
4. Rifampicin
5. Ethionamide

86. Question
A 39-year-old woman has madescence in the region of mammilla, a small
ulcer with inflammatory hyperemia and cutaneous edema. Histologic
examination of tissue sampling from this area revealed in the malpighian
layer of thickened epidermis atypical cells with light and optically empty
cytoplasm, with no intracellular bridges. Such cells were also found in the
orifice of big mammal gland ducts. What is the most probable diagnosis?

1. Epidermoid cancer
2. Intraductal cancer
3. Basal cell carcinoma
4. Pagets disease
5. Melanocarcinoma

87. Question
A lymph node punctate of a patient with suspected protozoal disease was
examined. Examination of the stained specimen (Romanovskys stain)
revealed some crescent bodies with pointed end, blue cytoplasm and red
nucleus. What protozoans were revealed in the smears?
1. Dermotropic leishmania
2. Viscerotropic leishmania
3. Trypanosomes
4. Toxoplasms
5. Malarial plasmodiums

88. Question
It was revealed that T-lymphocytes were affected by HIV. Virus enzyme –
reverse transcriptase (RNA-dependent DNA polymerase) – catalyzes the
synthesis of:

1. Viral DNA on DNA matrix


2. MRNA on the matrix of virus protein
3. DNA on the matrix of virus mRNA
4. Virus informational RNA on the matrix of DNA
5. DNA on virus ribosomal RNA

89. Question
In some regions of South Africa there is a spread sickle-shaped cell anemia, in
which erythrocytes have shape of a sickle as a result of substitution of
glutamin by valine in the hemoglobin molecule. What is the cause of this
disease?
LAKSHMAN

1. Gene mutation
2. Crossing over
3. Genomic mutations

LAKSHMAN
4. Transduction
5. Disturbance of mechanisms of genetic information realization

90. Question
In the perianal folds of a 5-year-old girl her mother has found some white
“worms” that caused itch and anxiety in the child. The “worms” were sent to
the laboratory. During examination the physician saw white filiform
helminths 0.5-1 cm long, with pointed ends, some helminths had twisted ends.
What is the most likely diagnosis?

1. Enterobiasis
2. Diphyllobothriasis
3. Ascaridiasis
4. Opisthorchiasis
5. Teniasis

91. Question
Different functional groups can be presented in the structure of L-amino
acid’s radicals. Identify the group that is able to form ester bond:

1. -CONH2
2. -SH
3. -OH
4. -CH3
5. -NH2
92. Question
RNA that contains AIDS virus penetrated into a leukocyte and by means of
reverse transcriptase forced a cell to synthetize a viral DNA. This process is
based upon:

1. Convariant replication
2. Reverse transcription
3. Reverse translation
4. Operon depression
5. Operon repression

93. Question
A shepherd who has tended sheep together with dogs consulted a doctor about
pain in his right subcostal area, nausea, vomiting. Roentgenoscopy revealed a
tumour-like formation. What kind of helminthiasis might be suspected?

1. Enterobiasis
2. Echinococcosis
3. Taeniasis
4. Taeniarhynchosis
5. Ascaridiasis

94. Question
LAKSHMAN

While studying maximally spiralized chromosomes of human karyotype the


process of cell division was stopped in the following phase:

1. Anaphase

LAKSHMAN
2. Metaphase
3. Interphase
4. Prophase
5. Telophase

95. Question
A pregnant woman had been having toxicosis with severe repeated vomiting
for 24 hours. In the end of the day there appeared tetanic convulsions and
fluid loss. What shift of acid-base state caused these changes?

1. Metabolic acidosis
2. Gaseous alkalosis
3. Gaseous acidosis
4. Excretory alkalosis
5. Excretory acidosis

96. Question
A patient complains of skin itch, especially between fingers, in the inguinal
creases, on the lower abdomen. Examination of these regions revealed there
some small vesicles. Laboratory diagnostics allowed to establish that this
condition had been caused by a representative of Arthropoda. Specify the
disease caused by this arthropod:

1. Myiasis
2. Demodicosis
3. Dermatotropic leishmaniasis
4. Scabies
5. Pediculosis

97. Question
Examination of duodenal contents revealed some pyriform protozoa with
twin nuclei and four pairs of flagella. There were two supporting filaments
between the nuclei and a suctorial disc on the ventral side. What
representative of protozoa was revealed in this patient?

1. Toxoplasma
2. Lamblia
3. Leishmania
4. Intestinal trichomonad
5. Trypanosome

98. Question
46 chromosomes were revealed on karyotype examination of the 5-year-old
girl. One of the 15th pair of chromosomes is longer than usual due to
connected chromosome from the 21 pair. What type of mutation does this girl
have?

1. Inversion
LAKSHMAN

2. Insuf ciency
3. Duplication
4. Deletion
5. Translocation

LAKSHMAN
99. Question
Hartnup disease is caused by point mutation of only one gene which results in
disturbance of tryptophane absorption in the bowels and its resorption in the
renal tubules. It is the reason for disorder of both digestive and urination
systems. What genetic phenomenon is observed in this case?

1. Codominance
2. Complementary interaction
3. Polymery
4. Pleiotropy
5. Semidominance

Created by Eneutron Team


Kiev eneutron.info@gmail.com
LAKSHMAN

LAKSHMAN
Krok 1 – 2013 Base(Anatomy)

Medical PG WhatsApp Group

Results
1 of 151 questions answered correctly

Your time: 00:05:14

You have reached 1 of 151 points, (0.66%)

RESTART QUIZ VIEW QUESTIONS

1. Question
A patient operated on complicated appendicitis has the following changes of
blood count: erythrocytes – 4.0*1012/l, Нb – 120 g/l, color index – 0.9, leukocytes
– 18*109/l, basophils – 0, eosinophils – 0, myelocytes – 0, juvenile – 0, stab
neutrophils – 20, segmentonuclear neutrophils – 53, lymphocytes – 21,
monocytes – 5. How is such nuclear shift of leukocytic formula called?

1. Right shift
2. Hyperregenerative
3. Regeneratively-degenerative
4. Degenerative left shift
5. Regenerative left shift
LAKSHMAN

2. Question
In order to prevent massive haemorrhage in the region of oral cavity floor it
is required to ligate an artery which is located within Pirogovs triangle. What
artery is it?

LAKSHMAN
1. Ascending pharyngeal artery
2. Maxillary artery
3. Facial artery
4. Superior thyroid artery
5. Lingual artery

3. Question
A patient in three weeks after acute myocardial infarction has pain in the
heart and joints and pneumonia. What is the main mechanism of
development of post-infarction Dressler’s syndrome?

1. Autoimmune in ammation
2. Secondary infection

Dressler syndrome is a type of pericarditis, which is th


3. Resorption of enzymes from necrotized area of myocardium

e inflammation of the sac around the heart (pericardiu


4. Ischemia of myocardium
5. Vessels thrombosis m). It’s also called post-pericardiotomy syndrome, po
st-myocardial infarction syndrome, or post-cardiac inj
4. Question ury syndrome
A 5-year-old child was admitted to the otorhinolaryngological department
with diagnosis – suppurative inflammation of the middle ear. Disease started
from the inflammation of the nasopharynx. Through the what canal of the
temporal bone did the infection get into the tympanic cavity?

1. Canaliculus chordal tympani


2. Canaliculi caroticotympanici
3. Carotid canal
4. Tympanic Canaliculus tympanicus
5. Musculortubal canal

5. Question
After a trauma of soft tissues in the region of the posterior surface of medial
condyle of humerus a patient has got a skin prickle of medial forearm
surface. Which of the listed nerves is located in the affected region?

1. N.ulnaris
2. N.dorsalis scapularis
3. N.musculocutaneu
4. N.radialis
5. N.subscapularis

6. Question
Ovarian tumour was diagnozed in the woman. Surgery was indicated. What
ligament should be cut by the surgeon to disconnect the ovary and the uterus?

1. Suspensory ligament of ovary


2. Round ligament of uterus
LAKSHMAN

3. Broad ligament of uterus


4. The ovarial ligament
5. Lateral umbilical ligament

7. Question

LAKSHMAN
Examination of a patient revealed an abscess of pterygopalatine fossa. Where
can the infection spread to unless the disease is managed in time?

1. To the subgaleal temporal space


2. To the interpterygoid space
3. To the orbit
4. To the frontal sinus
5. To the tympanic cavity

8. Question
A 10-year-old child complains of weakness, nausea, irritability. Helminthes of
while color and 5-10 mm long were found on the underwear. On microscopy of
the scrape from the perianal folds achromic ovums of unsymmetrical form
were revealed. Indicate what helminth is parasiting on the child?

1. Trichuris
2. Trichina
3. Enterobins vermicularis
4. Ascaris lumbricoides
5. Ancylostoma duodenalis

9. Question
Examination of a 2-year-old child revealed physical developmental lag, the
child often has pneumonias. The child was diagnosed with nonclosure of
ductus arteriosus. Haemodynamics disorder was caused by the
intercommunication of the following vessels:

1. Superior cava and pulmonary trunk


2. Pulmonary trunk and pulmonary veins
3. Superior cava and aorta
4. Aorta and pulmonary veins
5. Aorta and pulmonary trunk

10. Question
A patient with neuritis of femoral nerve has disturbed flexion of thigh as well
as disturbed crus extension in the knee joint. What muscles function is
disturbed?

1. Quadriceps muscle of thigh


2. Biceps muscle of thigh
3. Semitendinous muscle
4. Triceps muscle of thigh
5. Semimembranous muscle

11. Question
A 45-year-old man applied to the trauma unit because of domestic shoulder
trauma. Objectively: flexibility, reduction and pronation functions of the
shoulder are absent. What muscle was injured?
LAKSHMAN

1. Infraspinous muscle
2. Supraspinous muscle

LAKSHMAN
3. Subscapular muscle
4. Teres major muscle
5. Teres minor muscle

12. Question
An 18-year-old man was delivered to the hospital after a road accident.
Examination at the traumatological department revealed multiple injuries of
soft tissues of face in the region of the medial eye angle. The injuries caused
massive haemorrhage. What arterial anastomosis might have been damaged
in this region?

1. A. carotis interna et a. ophthalmica


2. A. carotis externa et a. subclavia
3. A. subclavia et a. ophthalmica
4. A. carotis interna et a. subclavia
5. A. carotis externa et a. carotis interna

13. Question
A patient got a trauma that caused dysfunction of motor centres regulating
activity of head muscles. In what parts of cerebral cortex is the respective
centre normally localized?

1. Supramarginal gyrus
2. Superior part of precentral gyrus
3. Inferior part of precentral gyrus
4. Angular gyrus
5. Superior parietal lobule

14. Question
Vegetative abnormalities in the sleep, heat regulation, all kinds of
metabolism, diabetes insipidus are developing in the patient due to grouth of
the tumour in the III ventricle of brain. Irritation of the nucleus of what part
of the brain can cause this symptoms?

1. Medulla
2. Cerebral peduncles (cruces cerebri)
3. Pons cerebelli
4. Hypothalamus
5. Mesencephalic tegmentum

15. Question
Part of alveoles of a preterm infant didnt spread because of enhanced elastic
recoil of lungs. How can this recoil be reduced?

1. By arti cial pulmonary ventilation


2. By pure oxygene inhalation
3. By glycose introduction
4. By surfactant introduction
5. By uid suction from the respiratory tracts
LAKSHMAN

16. Question
A patients knee joint doesnt extend, there is no knee-jerk reflex, skin
sensitivity of the anterior femoral surface is disturbed. What nerve

LAKSHMAN
structures are damaged?

1. Superior gluteal nerve


2. Obturator nerve
3. Inferior gluteal nerve
4. Femoral nerve
5. Big bular nerve

17. Question
In the specimen of one of the parts of respiratory system a tubular organ was
found. It has low epithelium, well developed muscular tunic, glands and
cartilage are absent. Name this organ:

1. Minor bronchs
2. Larynx
3. Median bronchs
4. Major bronchs
5. Trachea

18. Question
A woman suffering from osteochondrosis felt acute pain in her humeral
articulation that became stronger when she abducted her shoulder. These
symptoms might be caused by damage of the following nerve:

1. Axillary nerve
2. Subscapular nerve
3. Dorsal scapular nerve
4. Throracodorsal nerve
5. Subclavicular nerve

19. Question
Brain tomography revealed a tumour in the region of red nucleus. What part
of brain is damaged?

1. Midbrain
2. Pons cerebelli
3. Interbrain
4. Medulla oblongata
5. Cerebellum

20. Question
Surgical approach to the thyroid gland from the transverse (collar) approach
involves opening of interaponeurotic suprasternal space. What anatomic
structure localized in this space is dangerous to be damaged?

1. Subclavicular vein
2. External jugular vein
3. Jugular venous arch
LAKSHMAN

4. Inferior thyroid artery


5. Superior thyroid artery

21. Question
After resection of the middle third of femoral artery obliterated by a thromb

LAKSHMAN
the lower extremity is supplied with blood due to the surgical bypass. Name
an artery that plays the main role in reestablishment of blood flow:

1. Deep femoral artery


2. Descending genicular artery
3. Super cial circum ex artery of hip bone
4. Deep external pudendal artery
5. Super cial epigastric artery

22. Question
The gluconeogenesis is activated in the liver after intensive physical trainings
.What substance is utilized in gluconeogenesis first of all in this case:

1. Lactate
2. Alanine
3. Glutamate
4. Pyruvate
5. Glucose

23. Question
A young man consulted a doctor about disturbed urination. Examination of
his external genitals revealed that urethra is split on top and urine runs out
of this opening. What anomaly of external genitals development is the case?

1. Hypospadia
2. Epispadia
3. Hermaphroditism
4. Phimosis
5. Paraphimosis

24. Question
Examination of a 6-month-old child revealed a delay in closure of the occipital
fontanelle. When should it normally close?

1. Before the child is born


2. Until the end of the second year of life
3. Until 3 months
4. Until the end of the rst year of life
5. Until 6 months

25. Question
A patient has lost skin sensitivity in the region of the medial surface of his
shoulder. This is the result of dysfunction of the following nerve:

1. Medial antebrachial cutaneous nerve


2. Ulnar nerve
3. Axillary nerve
LAKSHMAN

4. Radial nerve
5. Medial brachial cutaneous nerve

26. Question

LAKSHMAN
A patient complained about being unable to adduct and abduct fingers in the
metacarpophalangeal articulations towards and away from the 3rd finger.
Which muscles function is impaired?

1. Brevi exors of ngers


2. Lumbrical muscles
3. Interosseous muscles
4. Long exors of ngers
5. Extensors

27. Question
Usually the intravenous injection is done into median cubital vein because it
is slightly movable due to fixation by the soft tissues. What does it fix in the
cubital fossa?

1. Brachioradial muscle
2. Anconeus muscle
3. Aponeurosis of biceps muscle
4. Tendon of the triceps muscle
5. Brachial muscle

28. Question
The process of heart transplantation determined the viability of myocardial
cells. The determination of what myocardium parameter is the most
important?
1. Rest potential of cardiomyocytes
2. Concentration of oxygen in heart vessels
3. Heart temperature
4. Concentration of Ca-ions in heart vessels
5. Concentration of calcium-ions in myo brils

29. Question
A 35 year old man with a trauma of his left hand was admitted to the
traumatology department. Objectively: cut wound of palmar surface of left
hand; middle phalanxes of II–V fingers dont bend. What muscles are
damaged?

1. Palmar interosseous muscles


2. Super cial nger exor
3. Profound nger exor
4. Dorsal interosseous muscles
5. Lumbrical muscles

30. Question
A 54-year-old man was admitted to the hospital with complaints of pain in the
right subcostal region, vomiting with blood. Objectively: enlarged liver,
varicose veins in the stomach and esophagus. Disfunction of what vessel is
likely to be?
LAKSHMAN

1. Aorta abdominalis
2. Vena porta

LAKSHMAN
3. Vena cava inferior
4. Vena cava superior
5. Vena hepatica

31. Question
A 40-year-old patient complains of intensive heartbeats, sweating, nausea,
visual impairment, arm tremor, hypertension. From his anamnesis: 2 years
ago he was diagnosed with pheochromocytoma. Hyperproduction of what
hormones causes the given pathology?

1. Aldosterone
2. Glucocorticoids
3. Catecholamines
4. Thyroidal hormones
5. ACTH

32. Question
A patient has been brought to the hospital with the complaints of headache,
pain in left hypochondrium. He has been ill for 1.5 weeks. The sudden illness
began with the increase of body temperature up to 39.9°C. In 3 hours the
temperature decreased and hydropoiesis began. The attacks repeat
rhythmically in 48 hours. The patient had visited one an African country. The
doctors have suspected malaria. What method of laboratory diagnostics is
necessary to use?

1. Blood examination
2. Stool examination
3. Immunological tests
4. Examination of vaginal and urethral discharge
5. Urine examination

33. Question
In case of a penetrating wound of the anterior abdominal wall the wound
tract went above the lesser curvature of stomach. What peritoneum
formation is most likely to be injured?

1. Ligamentum hepatogastricum
2. Ligamentum gastrocolicum
3. Ligamentum hepatorenale
4. Ligamentum hepatoduoduodenale
5. Ligamentum triangulare sinistrum

34. Question
A 25 year old patient was examined by a medical board. Examination
revealed pathology of chest. Transverse dimensions were to small and the
sternum was strongly protruding. What chest type is it?

1. Cylindrical chest
2. Barrel chest
3. Funnel chest
LAKSHMAN

4. Keeled chest
5. Flat chest

35. Question

LAKSHMAN
A patient has a right-sided fracture in the region of the frontal third of
mandible accompanied by a haematoma in the region of chin. It is caused by
the injury of the following artery:

1. Palatine
2. Lingual
3. Inferior labial
4. Mental
5. Facial

36. Question
While examining the oral cavity a stomatologist revealed inflammation of
papillae on the border of the median and posterior third of the back of
tongue. What papillae are inflamed?

1. Papillae vallatae
2. Papillae conicae
3. Papillae liformes
4. Papillae foliatae
5. Papillae fungiformes

37. Question
A patient has pain, edema and reddening of his skin in the anterosuperior
area of his thigh and his foots thumb. What lymph nodes of his lower
extremity responded to the inflammatory process?
1. Internal longitudinal
2. Deep inguinal
3. Super cial longitudinal
4. Super cial inguinal
5. General longitudinal

38. Question
Where should the cathetor for evacuation of the lymph from the thoracic
lymph duct be inserted?

1. To the left venous corner


2. To the right venous corner
3. To the left inguinal vein
4. To the superior vena cava
5. To the inferior vena cava

39. Question
A man suffering from osteochondrosis got acute pain in the abdominal
muscles (lateral and anterior). During objective examination a physician
diagnosticated increased pain sensitivity of skin in the hypogastric region.
This pain might be caused by affection of the following nerve:

1. Iliohypogastric
LAKSHMAN

2. Genitofemoral
3. Femoral
4. Obturator

LAKSHMAN
5. Sciatic

40. Question
A 70 year old female patient was diagnosed with fracture of left femoral neck
accompanied by disruption of ligament of head of femur. The branch of the
following artery is damaged:

1. Internal pudendal
2. Femoral
3. Obturator
4. Inferior gluteal
5. External iliac

41. Question
A 35 year old patient applied to a doctor with complaints about having
intense rhinitis and loss of sense of smell for a week. Objectively: nasal cavity
contains a lot of mucus that covers mucous membrane and blocks olfactory
receptors. In what part of nasal cavity are these receptors situated?

1. Superior nasal turbinate


2. Inferior nasal turbinate
3. Median nasal turbinate
4. Vestibule of nose
5. Common nasal meatus

42. Question
A mother of a newborn complains of her baby’s constant belching with
undigested milk. Which developmental anomaly is it an evidence of?

1. Faux lupinum
2. Esophageal atresia
3. Esophageal stula
4. Anal atresia
5. Labium leporium

43. Question
A histological spacemen presents parenchymal organ, which has cortex and
medulla. Cortex consists of epitheliocytes bars with blood capillaries between
them; the bars form three zones. Medulla consists of chromaffinocytes and
venous sinusoids. Which organ has these morphological features?

1. Thymus
2. Lymph node
3. Kidney
4. Thyroid
5. Adrenal gland

44. Question
A 18-year-old patient came to the out-patient department with the complaints
LAKSHMAN

of bleeding trauma in the vestibule of his nose. On examination: the


mechanical injure of the mucous layer of the vestibule without continuation
into nasal cavity proper. What is the boundary between the vestibule and
nasal cavity proper?

LAKSHMAN
1. Nasal roller
2. Nostrils
3. Nasal septa
4. Nasal limen
5. Choanes

45. Question
Examination of a newborn boys genitals revealed a cleft of urethra that opens
on the inferior surface of his penis. What developmental anomaly is meant?

1. Cryptorchism
2. Epispadia
3. Monorchism
4. Hermaphroditism
5. Hypospadia

46. Question
A patient was admitted to the surgical department with suspected
inflammation of Meckels diverticulum. What part of bowels should be
examined in order to discover the diverticulum in course of an operation?
Meckel's diverticulum is an outpouching or bulge in the lower
part of the small intestine. The bulge is congenital (present at b
1. Caecum irth) and is a leftover of the umbilical cord. Meckel's diverticulu
m is the most common congenital defect of the gastrointestinal
tract. It occurs in about 2% to 3% of the general population.
2. Jejunum
3. Ileum
4. Duodenum
5. Colon ascendens

47. Question
A patient complains of dizziness and hearing loss. What nerve is damaged?

1. Sublingual
2. Vagus
3. Trochlear
4. Trigeminus
5. Vestibulocochlear

48. Question
While playing a child got a punch in the presternum region. As a result of this
trauma an organ located behind the presternum was damaged. Name this
organ:

1. Pericardium
2. Thymus
3. Thyroid gland
4. Heart
5. Larynx
LAKSHMAN

49. Question
A patient was diagnosed with bartholinitis (inflammation of greater
vulvovaginal glands). In which organ of urogenital system are these glands
localized?

LAKSHMAN
1. Uterus
2. Vagina
3. Large lips of pudendum
4. Small lips of pudendum
5. Clitoris

50. Question
While performing an operation in the area of axillary crease a surgeon has to
define an arterial vessel surrounded by fascicles of brachial plexus. What
artery is it?

1. A.vertebralis
2. A.transversa colli
3. A.axillaris
4. A.profunda brachii
5. A.subscapularis

51. Question
Ultrasonic examination of a patient revealed aneurism in the area of aortic
arch that caused alteration of vocal function of larynx. What nerve was
constricted?

1. Diaphragmatic
2. Superior laryngeal
3. Sublingual
4. Recurrent laryngeal
5. Mandibular

52. Question
After a 2 y.o. child has had flu, there appeared complaints about ear ache. A
doctor revealed hearing impairment and inflammation of the middle ear.
How did the infection penetrate into the middle ear?

1. Through the auditory tube


2. Through canalis caroticus
3. Through foramen jugularis
4. Through canalis nasolacrimalis
5. Through atrium mastoideum

53. Question
In course of an operation surgeon removed a part of a lung that was
ventilated by a tertiary bronchus accompanied by branches of pulmonary
artery and other vessels. What part of a lung was removed?

1. Superior lobe
2. Bronchopulmonary segment
3. Inferior lobe
LAKSHMAN

4. Pulmonary lobule
5. Middle lobe

54. Question

LAKSHMAN
Examination of a patient with impaired blood coagulation revealed
thrombosis of a branch of inferior mesenteric artery. What bowel segment is
damaged?

1. Colon ascendens
2. Colon sigmoideum
3. Colon transversum
4. Caecum
5. Ileum

55. Question
A patient suffering from chronic myeloleukemia has got the following
symptoms of anemia: decreased number of erythrocytes and low
haemoglobin concentration, oxyphilic and polychromatophilic normocytes,
microcytes. What is the leading pathogenetic mechanism of anemia
development?

1. Chronic haemorrhage
2. Reduced synthesis of erythropoietin
3. De ciency of vitamin B12
4. Intravascular hemolysis of erythrocytes
5. Substitution of haemoblast

56. Question
During the fetal period of the development in the vascular system of the fetus
large arterial (Botallos) duct is functioning which converts into lig.
arteriosum after birth. What anatomical formations does this duct connect
between each other?

1. Right and left auricle


2. Aorta and inferior vena cava
3. Pulmonary trunk and superior vena cava
4. Pulmonary trunk and aorta
5. Aorta and superior vena cava

57. Question
Children often have heavy nasal breathing resulting from excessive
development of lymphoid tissue of pharyngeal mucous membrane. What
tonsils growth may cause this effect?

1. All the mentioned tonsils


2. Tonsilla lingualis
3. Tonsilla tubaria
4. Tonsilla palatina
5. Tonsilla pharyngea

58. Question
LAKSHMAN

A 19-year-old female suffers from tachycardia in rest condition, weight loss,


excessive sweating, exophtalmos and irritability. What hormone would you
expect to find elevated in her serum?

LAKSHMAN
1. Thyroxine
2. Insulin
3. Cortisol
4. ACTH
5. Mineralocorticoids

59. Question
A 32-year-old patient has been diagnosed with bartholinitis (inflammation of
Bartholins glands ). In what part of the female urogenital system are the
Bartholins glands located?

1. The clitoris
2. The labia major
3. The labia minor
4. The vagina
5. The uterus

60. Question
A 38-year-old patient came to a traumatology centre and complained about
an injury of his right hand. Objectively: the patient has a cut wound in the
region of the thenar eminence on the right hand; distal phalanx of the I finger
cannot be flexed. What muscle was injured?

1. Abductor muscle of thumb


2. Opposer muscle of thumb
3. Long exor muscle of thumb
4. Short exor muscle of thumb
5. Short abductor muscle of thumb

61. Question
The patient has come to the hospital from the smelting workshop in the
condition of hyperthermia. What is the direct cause of loss of consciousness
at the heat stroke?

1. Dilatation of peripheral vessels


2. Decrease of heart output
3. Decreased brain blood supply
4. Arterial pressure drop
5. Increased water loss through sweating

62. Question
A 55-year-old patien was hospitalized in result of the trauma of the medial
group of femoral muscles. What kind of movements is the patient unable to
do?

1. Abduction of femur
2. Extension of femur
3. Adduction of femur
4. Flexion of femur
LAKSHMAN

5. Suppination of femur

63. Question
Victim has elbow joint trauma with avulsion of medial epicondyle of

LAKSHMAN
humerus. What nerve can be damaged in this trauma?

1. Radial
2. Medial cutaneous nerve of forearm
3. Musculocutaneous nerve
4. Ulnar
5. Cardiac cutaneous nerve

64. Question
A man with a stab wound in the area of quadrilateral foramen applied to a
doctor. Examination revealed that the patient was unable to draw his arm
aside from his body. What nerve is most probably damaged?

1. N.ulnaris
2. N.subclavius
3. N.medianus
4. N.radialis
5. N.axillaris

65. Question
A 45-year-old man with domestic apper arm injuiry came to the trauma unit.
The objective data are: there are no extension, adduction or pronation
functions of the arm. What muscle damage caused this condition?

1. Supraspinous
2. Teres major
3. Teres minor
4. Subscapular
5. Subspinous

66. Question
A 45-year-old man fell on the right knee and felt the acute pain in the joint. On
examination: severe edema on the anterior surface of the knee joint.
Crunching sounds are heard while moving the joint. Which bone is destroyed?

1. Left epicondyle of the thigh


2. Head of the thigh bone
3. Knee-cap
4. Right epicondyle of the thigh
5. Neck of the thigh bone

67. Question
A female patient with a tumour of pancreas has developed mechanic jaundice
resulting from compression of a bile-excreting duct. Which duct is
compressed?

1. Ductus cysticus
2. Ductus hepaticus dexter
LAKSHMAN

3. Ductus hepaticus sinister


4. Ductus hepaticus communis
5. Ductus choledochus

LAKSHMAN
68. Question
Nowadays about 50 minor bases have been found in the t-RNA structure
besides the main four nitrogenous bases. Choose the minor nitrogenous base:

1. Adenine
2. Cysteine
3. Dihydrouracil
4. Uracil
5. Cytosine

69. Question
A 60-year-old patient was diagnosed with hypothalamic lateral nuclei stroke.
What changes in patient’s behavior may be expected?

1. Unsatis ed hunger
2. The rejection of food
3. Depression
4. Thirst
5. Aggressive behaviour

70. Question
A patient had been taking glucocorticoids for a long time. When the
preparation was withdrawn he developed the symptoms of disease
aggravation, decreased blood pressure and weakness. What is the reason of
this condition?
1. Appearance of adrenal insuf ciency
2. Sensibilization
3. Habituation
4. Hyperproduction of ACTH
5. Cumulation

71. Question
A patient complains about edemata of legs, skin cyanosis, small ulcers on one
side of the lateral condyle. Examination revealed a swelling, enlarged veins,
formation of nodes. The pathological process has started in the following
vein:

1. V. iliaca externa
2. V. saphena magna
3. V. profunda femoris
4. V. saphena parva
5. V. femoralis

72. Question
A foreign body (a button) closed space of the right superior lobar bronchus.
What segments of the right lung wont be supplied with air?

1. Medial and lateral


LAKSHMAN

2. Apical and median basal


3. Apical, posterior, anterior
4. Superior and inferior lingular

LAKSHMAN
5. Apical and posterior basal

73. Question
A man with cut wound of his right foot sole was admitted to the hospital
ward. The patient has limited elevation of the lateral foot edge. In course of
wound management the injury of a muscle tendon was revealed. What muscle
is injured?

1. Triceps muscle of crus


2. Long peroneal
3. Long extensor muscle of toes
4. Short peroneal
5. Anterior tibial

74. Question
A patient was admitted to the surgical department with inguinal hernia.
During the operation the surgeon performs plastic surgery on posterior wall
of inguinal canal. What structure forms this wall?

1. Peritoneum
2. Transverse fascia
3. Inguinal ligament
4. Aponeurosis of abdominal external oblique muscle
5. Loose inferior edge of transverse abdominal muscle

75. Question
A woman underwent an operation on account of extrauterine (tubal)
pregnancy. In course of the operation the surgeon should ligate the branches
of the following arteries:

1. Superior cystic and ovarian


2. Uterine and ovarian
3. Uterine and inferior cystic
4. Inferior cystic and ovarian
5. Uterine and superior cystic

76. Question
A 50 y.o. patient was admitted to the hospital with complaints about pain
behind his breastbone, asphyxia during physical activity. Angiography
revealed pathological changes in the posterior interventricular branch of the
right coronary artery. What heart parts are affected?

1. Right atrium
2. Anterior wall of the right and left ventricles
3. Posterior wall of the right and left ventricles
4. Left atrium
5. Right atrioventricular valve

77. Question
LAKSHMAN

A patient has difficulties with hand movement. Examination revealed


inflammation of common synovial sheath of flexor muscles. It is known from
the patients anamnesis that he got a stab wound of finger a week ago. Which
finger was most probably damaged?

LAKSHMAN
1. Digitus minimus
2. Pollex
3. Index
4. Digitus anularis
5. Digitus medius

78. Question
A patient has tissue ischemia below the knee joint accompanied with
intermittent claudication. What artery occlusion should be suspected?

1. Peroneal artery
2. Anterior tibial artery
3. Proximal part of femoral artery
4. Posterior tibial artery
5. Popliteal artery

79. Question
Child inspired button. Where is it likely to be?

1. In the larynx
2. In the gullet
3. In the trachea
4. In the left main bronchus
5. In the right main bronchus
80. Question
During the operation on the hip joint of a 5-year-old child her ligament was
damaged which caused bleeding.What ligament was damaged?

1. Perpendicular of the acetabule


2. The head of the thigh
3. Iliofemoral
4. Ischiofemoral
5. Pubofemoral

81. Question
Preventive examination of a patient revealed an enlarged lymph node of
metastatic origin on the medial wall of the left axillary crease. Specify the
most likely localization of the primary tumour:

1. Mammary gland
2. Submandibular salivary gland
3. Stomach
4. Lung
5. Thyroid gland

82. Question
LAKSHMAN

A 6-year-old child fell on the cutting object and traumatized soft tissues
between tibia and fibula . What kind of bone connection was injured?

1. Membrane

LAKSHMAN
2. Suture
3. Gomphosis
4. Fontanel
5. Ligament

83. Question
After a trauma a patient lost ability of elbow extension. This might have been
caused by dysfunction of the following main muscle:

1. M. subscapularis
2. M. levator scapulae
3. M. infraspinatus
4. M. teres major
5. M. triceps brachii

84. Question
A patient got an injury of spinal marrow in a road accident that caused loss
of tactile sensation, posture sense, vibration sense. What conduction tracts
are damaged?

1. Fascicle of Goll and cuneate fascicle


2. Anterior spinocerebellar tract
3. Rubrospinal tract
4. Tectospinal tract
5. Reticulospinal tract
85. Question
A man with an injury of the dorsal area of his neck was admitted to the
resuscitation department. What muscle occupies this area?

1. M.latissimus dorsi
2. M.scalenus anterior
3. M.rhomboideus minor
4. M.sternocleidomastoideus
5. M.trapezius

86. Question
During complicated labour the symphysis pubis ruptured. What organ can be
damaged mostly?

1. Uterine tubes
2. Ovaria
3. Uterus
4. Urinary blader
5. Rectum

87. Question
During cytoscopy mucous membrane of urinary bladder normally makes
folds except for a single triangular area with smooth mucosa. This triangle is
LAKSHMAN

located in the following part of urinary bladder:

1. Bladder body

LAKSHMAN
2. Bladder cervix
3. Bladder apex
4. Bladder oor
5. Bladder isthmus

88. Question
The cerebrospinal fluid is being examined for the purpose of diffrential
meningitis diagnostics. At what site is the lumbal puncture safe?

1. Th XII-L I
2. L V-S I
3. L II-L III
4. L I-L II
5. L III-L IV

89. Question
Succinate dehydrogenase catalyses the dehydrogenation of succinate.
Malonic acid HOOC-CH2-COOH is used to interrupt the action of this enzyme.
Choose the inhibition type:

1. Allosteric
2. Dephosphorylation
3. Non-competitive
4. Competitive
5. Limited proteolysis
90. Question
A 53-year-old female patient was diagnosed with liver rupture resulting from
a blunt abdominal injury. The escaped blood will be assembled in the
following anatomic formation:

1. Right mesenteric sinus


2. Rectouterine pouch
3. Left mesenteric sinus
4. Omental bursa
5. Vesicouterine pouch

91. Question
Cerebral trauma caused increase of ammonia formation. What aminoacid
takes part in removal of ammonia from cerebral tissue?

1. Lisine
2. Tyrosine
3. Tryptophan
4. Valine
5. Glutamic

92. Question
Microspecimen of spinal cord contains a nucleus that should be analyzed. Its
LAKSHMAN

neurons form motor endings in the skeletal muscles. What nucleus of spinal
cord is meant?

1. Proper nucleus of the anterior horn

LAKSHMAN
2. Thoracic nucleus
3. Intermediate lateral nucleus
4. Proper nucleus of gray substance
5. Proper nucleus of the posterior horn

93. Question
During investigation of patient, it was found formation in the white substance
of cerebral hemispheres with location in the knee and frontal part of
posterior crus of internal capsule. Fibres of what conductive tract of the
brain will be disrupted?

1. Tr. frontopontinus
2. Tr. parietooccipitopontinus
3. Tr. frontothalamicus
4. Тr. thalamocorticalis
5. Tr. pyramidalis

94. Question
A surgeon has to find the common hepatic duct during the operative
intervention on account of concrements in the gall ducts. The common hepatic
duct is located between the leaves of:

1. Hepatorenal ligament
2. Hepatogastric ligament
3. Venous ligament
4. Round ligament of liver
5. Hepatoduodenal ligament

95. Question
As a result of a cold a patient has the abnormal pain and temperature
sensitivity of the frontal 2/3 of his tongue. Which nerve must have been
damaged?

1. Sublingual
2. Trigeminus
3. Accessory
4. Vagus
5. Glossopharyngeal

96. Question
While performing an inguinal canal operation on account of hernia a surgeon
damaged the canals contents. What exactly was damaged?

1. Lig. inguinalе
2. Lig. teres uteri
3. Urarchus
4. Funiculus spermaticus
LAKSHMAN

97. Question
The alveolar ventilation of the patient is 5 L/min, the breath frequency is 10
per/min, and the tidal volume is 700 ml. What is the patients dead space
ventilation?

LAKSHMAN
1. 2.0 L/min
2. 1.0 L/min
3. 0.7 L/min
4. 4.3 L/min

98. Question
A 25-year-old patient complained of the decreased vision. Accommodation
disorders, dilated pupil, not reacting on the light were revealed on
examination. Function of what muscles is disturbed?

1. Pupil narrowing muscle, ciliary


2. Lateral rectus muscle, pupil narrowing
3. Inferior oblique muscle, ciliary
4. Pupil dilating muscle, ciliary
5. Pupil narrowing and dilating muscle

99. Question
The electronic microphoto of kidney fragment has exposed afferent
glomerular arteriole, which has giant cells under its endothelium, containing
secretory granules. Name the type of these cells:

1. Juxtaglomerular
2. Smooth muscular
3. Interstitial
4. Juxtavascular
5. Mesangial

100. Question
A man after 1.5 litre blood loss has suddenly reduced diuresis. The increased
secretion of what hormone caused such diuresis alteration?

1. Cortisol
2. Parathormone
3. Vasopressin
4. Natriuretic
5. Corticotropin

101. Question
Patient complains of frequent and difficult urination. Imperfection of what
formation can cause it?

1. Prostate
2. Bulb-uretic glands
3. Testicle adnexa
4. Sperm bubbles
5. Testicles
LAKSHMAN

102. Question
A patient with cholelithiasis fell ill with mechanic jaundice. Examination
revealed that the stone was in the common bile duct. What bile-excreting
ducts make up the obturated duct?

LAKSHMAN
1. Ductus hepaticus dexter et ductus cysticus
2. Ductus hepaticus dexter et sinister
3. Ductus hepaticus sinister et ductus cysticus
4. Ductus hepaticus communis et ductus choledochus
5. Ductus hepaticus communis et ductus cysticus

103. Question
A patient who suffers from cancer of back of tongue has an intense bleeding
as a result of affection of dorsal lingual artery by the tumour. What vessel
should be ligated to stop bleeding?

1. Ascending pharyngeal artery


2. Lingual artery
3. Deep lingual artery
4. Facial artery
5. Dorsal lingual artery

104. Question
Three separate bones connected with cartilage in the area of pelvis cavity are
noticed on the X-ray of the pelvis. What are these bones?

1. Pubic, sciatic, femoral


2. Iliac, sacral, coccyx
3. Sacral, pubic, coccyx
4. Iliac, pubic, sciatic
5. Sciatic, femoral, sacral

105. Question
A 28 year old woman has been diagnosed with extrauterine pregnancy
complicated by tha fallopian tube rupture. The blood is most likely to
penetrate the following peritoneal space:

1. Vesicouterine
2. Right mesenteric sinus
3. Left mesenteric sinus
4. Rectouterine
5. Intersigmoid sinus

106. Question
It is necessary to take the cerebrospinal fluid from a patient with suspected
inflammation of brain tunics. Diagnostic puncture was performed between
the arches of the lumbar vertebras. During the puncture the needle went
through the following ligament:

1. Intertransverse
2. Yellow ( aval)
3. Posterior longitudinal
LAKSHMAN

4. Iliolumbar
5. Anterior longitudinal

107. Question

LAKSHMAN
Young man felt sharp pain in the back during active tightening on the
horizontal bar. Objectively: pain while moving of upper extremity, limited
pronation and adduction functions. Sprain of what muscle is presented?

1. М. trapezius
2. М. latissimus dorsi
3. М. romboideus major
4. М. levator scapulae
5. М. subscapularis

108. Question
Examination of a patient revealed hypertrophy and inflammation of
lymphoid tissue, edema of mucous membrane between palatine arches (acute
tonsillitis). What tonsil is normally situated in this area?

1. Tonsilla lingualis
2. Tonsilla palatina
3. Tonsilla pharyngealis
4. Tonsilla tubaria

109. Question
While preparing a patient to the operation the heart chambers pressure was
measured. In one of them the pressure changed during one heart cycle from 0
to 120 mm Hg. What chamber of heart was it?

1. Right atrium
2. Right ventricle
3. Left ventricle
4. Left atrium

110. Question
Pyeloureterography X-ray photo showed a renal pelvis with minor calyces
only (major calyces were absent). What form of urinary tracts of a kidney
was revealed?

1. Mature
2. Fetal
3. Embryonal
4. Ampullar

111. Question
An old woman was hospitalized with acute pain, edema in the right hip joint;
the movements in the joint are limited. Which bone or part of it was broken?

1. Condyle of the thigh


2. Ischial bone
3. The body of the thigh bone
4. The neck of the thigh
LAKSHMAN

5. Pubic bone

112. Question
A patient with a stab wound of the anterior stomach wall is in surgical care.
What formation of abdominal cavity did the stomach contents get into?

LAKSHMAN
1. Right mesenteric sinus
2. Left mesenteric sinus
3. Antegastrial bursa
4. Omental bursa
5. Hepatic

113. Question
A 30-year-old patient was hospitalized due to bleeding of the facial artery .
What place on the face has to be pressed to stop bleeding?

1. The molar bone


2. The mandible’s branch
3. The mandible’s edge
4. The nose’s back
5. The mental process

114. Question
An injured person was delivered to the hospital with a penetrating wound in
the left lateral region of abdomen. What part of the large intestine is most
likely damaged?

1. Colon transverses
2. Colon descendens
3. Rectum
4. Caecum
5. Colon ascendens

115. Question
A 19 year-old patient was diagnosed with appendicitis and was hospitalized.
The surgical operation on ablating appendix vermiformis is to be performed.
What artery must be fixed to stop bleeding during the surgical operation?

1. The colica sinistra


2. The colica media
3. The ileocolic artery
4. The colica dextra
5. The iliac

116. Question
Obturative jaundice developed in a 60-year-old patient because of malignant
tumour of the big papillary of the duodenal. Lumen of what anatomical
structure is squeezed with tumour?

1. Left hepatic duct


2. Hepatopancreatic ampulla
3. Common hepatic duct
4. Cystic duct
LAKSHMAN

5. Right hepatic duct

117. Question
A patient has a deep cut wound on the posterior surface of his shoulder in its

LAKSHMAN
middle third. What muscle might be injured?

1. Biceps muscle of arm


2. Brachial muscle
3. Triceps muscle of arm
4. Coracobrachial muscle
5. Anconeus muscle

118. Question
During the endoscopy the inflammation of a major papilla of the duodenum
and the disturbances of bile secretion were found. In which part of duodenum
were the problems found?

1. Upper horizontal part


2. Lower horizontal part
3. Ascendant part
4. Descendent part
5. Bulb

119. Question
A patient with infectious mononucleosis had been taking glucocorticoids for
two weeks. He was brought into remission, but he fell ill with acute attack of
chronic tonsillitis. What action of glucocorticoids caused this complication?

1. Antiallergic
2. Anti-in ammatory
3. Antitoxic
4. Immunosuppressive
5. Antishock

120. Question
Inflammation of the tympanic cavity (purulent otitis media) was complicated
by inflammation of mammillary process sockets. What wall of tympanic
cavity did the pus penetrate into the sockets through?

1. Anterior
2. Posterior
3. Lateral
4. Superior
5. Medial

121. Question
A 7-year-old child cant abduct the shoulder, raise it to the horizontal level. He
can raise the hand to the face only with dorsal side with abduction of the
shoulder (with help of supraspinous muscle ) – “bugler” arm. Active function
of what muscle is absent?

1. Infraspinous
LAKSHMAN

2. Teres minor
3. Teres major
4. Deltoid

LAKSHMAN
5. Pectoral major

122. Question
The patient with thymoma (thymus gland tumour) has cyanosis, extention of
subcutaneous venous net and edema of the soft tissues of face, neck, upper
part of the trunk and upper extremities. What venous trunk is pressed with
tumour?

1. Frontal jugular vein


2. Internal jugular vein
3. Clavicular vein
4. External jugular vein
5. Superior vena cava

123. Question
A 70 y.o. man has cut an abscess off in the area of mammiform process during
shaving. Two days later he was admitted to the hospital with inflammation of
arachnoid membranes. How did the infection penetrate into the cavity of
skull?

1. V.facialis
2. V.v.tympanicae
3. V.v.labyrinthi
4. V.v.auriculares
5. V.emissariaе mastoideaе

124. Question
A 7-year-old girl has signs of anemia. Laboratory examination revealed
pyruvate kinase deficiency in erythrocytes. What process disturbance plays
the main role in anemia development?

1. Oxidative phosphorylation
2. Peroxide decomposition
3. Tissue respiration
4. Aminoacids desamination
5. Anaerobic glycolysis

125. Question
An injured man has bleeding from branches of carotid artery. For a
temporary arrest of bleeding it is necessary to press the carotid artery to the
tubercle of a cervical vertebra. Which vertebra is it?

1. III
2. IV
3. II
4. VI
5. V

126. Question
A patient has been diagnosed with a compression fracture of a lumbar
LAKSHMAN

vertebra. As a result he has a considerable increase in curvature of the


lumbar lordosis. Which ligament damage can induce such changes in the
spine curvature?

LAKSHMAN
1. Interspinous ligament
2. Yellow ligament
3. Iliolumbar ligament
4. Anterior longitudinal ligament
5. Posterior longitudinal ligament

127. Question
Inflammatory process of modified subserous layer around cervix of the
uterus caused an intensive pain syndrome. In what region of genitals does the
pathological process take place?

1. Perimetrium
2. Endometrium
3. Parametrium
4. Mesometrium
5. Myometrium

128. Question
Slime, blood and protozoa 30-200 microns of length have been revealed in a
mans feces. The body is covered with cilias and has correct oval form with a
little bit narrowed forward and wide round shaped back end. On the forward
end a mouth is visible. In cytoplasm there are two nucleuses and two short
vacuoles. For whom are the described attributes typical?

1. Intestinal amoeba
2. Lamblia
3. Balantidium
4. Trichomonas
5. Dysenteric amoeba

129. Question
A patient complains about impaired evacuatory function of stomach (long-
term retention of food in stomach). Examination revealed a tumour of initial
part of duodenum. Specify localization of the tumour:

1. Pars inferior
2. Pars ascendens
3. Flexura duodeni inferior
4. Pars descendens
5. Pars superior

130. Question
A patient, who suffers from congenital erythropoietic porphyria, has skin
photosensitivity. The accumulation of what compound in the skin can cause
it?

1. Uroporphyrinogen 1
2. Coproporphyrinogen 3
3. Heme
LAKSHMAN

4. Protoporphyrin
5. Uroporphyrinogen 2

131. Question

LAKSHMAN
A 50 year-old patient had hemorrhage of the brain and was taken to the
hospital. The place of hemorrhage was revealed on the lateral hemispheres
surfaces during the medical examination. What artery was injured ?

1. The middle cerebral artery


2. The anterior cerebral artery
3. The posterior communicating artery
4. The anterior communicating artery
5. The posterior cerebral artery

132. Question
A 58-year-old patient with acute cardiac insufficiency has decreased volume
of daily urine – oliguria. What is the mechanism of this phenomenon?

1. Drop of oncotic blood pressure


2. Reduced permeability of renal lter
3. Rise of hydrostatic blood pressure in capillars
4. Decreased glomerular ltration
5. Decreased number of functioning glomerules

133. Question
After a craniocerebral trauma a patient lost the ability to execute learned
purposeful movements (apraxia). The injury is most likely localized in the
following region of the cerebral cortex:
1. Gyrus parahippocampalis
2. Gyrus paracentralis
3. Gyrus angularis
4. Gyrus supramarginalis
5. Gyrus lingualis

134. Question
In course of laparotomy a surgeon revealed gangrenous lesion of descending
colon. It was caused by thrombosis of the following artery:

1. Ileocolic
2. Superior mesenteric artery
3. Median colic
4. Dexter colic
5. Sinister colic

135. Question
After trauma a 44-year-old patient had a rupture of left palm muscle tendons
and of the surface of blood vessels. The bloodstream was normalized after
operation and removal of the most part of the necrotically changed muscle
tissue. What vessels have helped with restoration of bloodstream?

1. Arcus palmaris profundus


LAKSHMAN

2. Aa. perforantes
3. Aa. digitales palmares communes
4. Aa. metacarpeae palmares

LAKSHMAN
5. Arcus palmaris super cialis

136. Question
After a road accident a driver was delivered to the hospital with an injury of
the medial epicondyle of humerus. What nerve might be damaged in this case?

1. N. axillaris
2. N. ulnaris
3. N. muscolocutaneus
4. N. medianus
5. N. radialis

137. Question
A man with internal abdominal right side injury and suspicion of liver
rupture was admitted to the traumatological department. In what peritonial
structure will blood accumulate?

1. Bursa omentalis
2. Excavatio rectovesicalis
3. Fossa ischio-analis
4. Recessus duodenalis inferior
5. Recessus intersigmoideus

138. Question
Only one factor can influence the charge of amino acid radicals in the active
centre of enzyme. Name this factor:
1. The surplus of a product
2. PH medium
3. Temperature
4. Pressure
5. The presence of a competitive inhibitor

139. Question
While palpating mammary gland of a patient a doctor revealed an induration
in form of a node in the inferior medial quadrant. Metastases may extend to
the following lymph nodes:

1. Superior diaphragmal
2. Parasternal
3. Bronchopulmonary
4. Profound lateral cervical
5. Posterior mediastinal

140. Question
A 40-year-old woman was admitted to the infectious diseases department with
high body temperature. Objectively: marked meningeal symptoms. A spinal
cord punction was made. What anatomic formation was puncturated?

1. Cavum trigeminale
LAKSHMAN

2. Cisterna cerebellomedullaris posterior


3. Spatium subdurale
4. Spatium epidurale

LAKSHMAN
5. Spatium subarachnoideum

141. Question
A 60-year-old patient has reduced perception of high-frequency sounds. What
structures disorder of auditory analizer caused these changes?

1. Muscles of middle ear


2. Main membrane of cochlea near helicotrema
3. Eustachian tube
4. Tympanic membrane
5. Main membrane of cochlea near the oval window

142. Question
A patient has a malignisation of thoracic part of esophagus. What lymphatic
nodes are regional for this organ?

1. Nodi lymphatici mediastinales posteriores


2. Anulus lymphaticus cardiae
3. Nodi lymphatici laterales
4. Nodi lymphatici prevertebralis
5. Nodi lymphatici paratrachealis

143. Question
A 6 month old baby ill with bronchitis was taken for an X-ray of chest. Apart
of changes associated with bronchi the X-ray film showed a shadow of thymus
gland. What might have caused such changes?
1. It is caused by neoplastic process
2. It is caused by abnormal position
3. It is caused by thymus in ammation
4. The above-mentioned condition is a normal variant for this age
5. Its the effect of bronchitis

144. Question
A 45-year-old patient was admitted to the surgical department with
complaints of abrupt sharp pain in the epigastric region. After examination it
was diagnosed: perforated ulcer of the posterior wall of the stomach. Where
did content of the stomach flow out while perforation?

1. To the liver bursa


2. To the left mesenteric sinus
3. To the right mesenteric sinus
4. To the omental bursa
5. To the proventriculus sack

145. Question
Neurological examination of a 65 y.o. patient revealed a haemorrhage within
the superior temporal gyrus. In the blood supply area of which artery is it?

1. Basilar artery
LAKSHMAN

2. Posterior cerebral artery


3. Anterior cerebral artery
4. Anterior communicating artery

LAKSHMAN
5. Middle cerebral artery

146. Question
A patient was diagnosed with paralysis of facial and masticatory muscles.
The haematoma is inside the genu of internal capsule. What conduction tract
is damaged?

1. Tr. cortico-spinalis
2. Tr. cortico-fronto-pontinus
3. Tr. cortico-temporo-parieto-occipito-pontinus
4. Tr. cortico-nuclearis
5. Tr. cortico-thalamicus

147. Question
As a result of an accident a patient has intense painfullness and edema of the
anterior crus surface; dorsal flexion of foot is hindered. Function of which
crus muscle is most likely to be disturbed?

1. M.peroneus brevis
2. M.peroneus longus
3. M. exor digitorum longus
4. M. exor hallucis longus
5. M.tibialis anterior

148. Question
A patient with a knife wound in the left lumbal part was delivered to the
emergency hospital. In course of operation a surgeon found that internal
organs were not damaged but the knife injured one of muscles of renal pelvis.
What muscle is it?

1. Abdominal internal oblique muscle


2. Erector muscle of spine
3. Iliac muscle
4. Greater psoas muscle
5. Abdominal external oblique muscle

149. Question
A patient got a craniocerebral trauma that resulted in right-side convergent
strabismus. Damage of which craniocerebral nerve caused such
consequences?

1. N.trigeminus
2. N.abducens
3. N.facialis
4. N.trochlearis
5. N.aculomotorius

150. Question
LAKSHMAN

In course of a small pelvis operation it became necessary to ligate an ovarian


artery. What formation may be accidentally ligated together with it?

1. Internal iliac vein

LAKSHMAN
2. Ureter
3. Urethra
4. Uterine tube
5. Round ligament of uterus

151. Question
A patients blood was analyzed and the decreased erythrocyte’s sedimentation
rate (ESR) was discovered. What disease from the listed below is accompanied
with decreased ESR?

1. Vitamin B de ciency
2. Polycytemia
3. Myocardial infarction
4. Hepatitis
5. Splenomegaly

Created by Eneutron Team

Kiev eneutron.info@gmail.com
Krok 1 – 2013 Base(Biochemistry)

Medical PG WhatsApp Group

Results
0 of 181 questions answered correctly

Your time: 00:07:26

You have reached 0 of 181 points, (0%)

RESTART QUIZ VIEW QUESTIONS

1. Question
A 63-year-old woman developed signs of rheumatoid arthritis. Increase of
which indicated blood values level could be helpful in proving diagnosis?

1. R-glycosidase
2. General cholesterol
3. Acid phosphatase
4. Lipoproteids
5. Additive glycosaminoglycans

2. Question
In course of histidine catabolism a biogenic amin is formed that has powerful
LAKSHMAN

vasodilatating effect. Name it:

1. Dopamine
2. Noradrenalin

LAKSHMAN
3. Histamine
4. Serotonin
5. Dioxyphenylalanine

3. Question
A 46 year old woman suffering from chololithiasis developed jaundice. Her
urine became dark-yellow and feces became colourless. Blood serum will have
the highest concentration of the following substance:

1. Biliverdin
2. Unconjugated bilirubin
3. Mesobilirubin
4. Conjugated bilirubin
5. Urobilinogen

4. Question
Autopsy of a 12-year-old girl revealed: multiple cutaneous hemmorhages
(mostly into the skin of buttocks, lower extremities), serous and mucous
memrane hemmorhages, cerebral hemmorhages. Adrenal glands show focal
necrosis and massive hemmorhages; kidneys show necrotic nephrosis,
suppurative arthritis, iridocyclitis, vasculitis. What is the most probable
diagnosis?

1. Systemic lupus erythematosus


2. Epidemic typhus
3. Radiation sickness
4. Periarteritis nodosa
5. Meningococcemia

5. Question
Researchers isolated 5 isoenzymic forms of lactate dehydrogenase from the
human blood serum and studied their properties. What property indicates
that the isoenzymic forms were isolated from the same enzyme?

1. Catalyzation of the same reaction


2. The same physicochemical properties
3. Tissue localization
4. The same electrophoretic mobility
5. The same molecular weight

6. Question
A patient with high rate of obesity was advised to use carnitine as a food
additive in order to enhance “fat burning”. What is the role of carnitine in the
process of fat oxidation?

1. FFA activation
2. Transport of FFA from fat depots to the tissues
3. Activation of intracellular lipolysis
4. Transport of FFA (free fatty acids) from cytosol to the mitochondria
LAKSHMAN

5. It takes part in one of reactions of FFA beta-oxidation

7. Question
A businessman came to India from South America. On examination the

LAKSHMAN
physician found that the patient was suffering from sleeping-sickness. What
was the way of invasion?

1. With contaminated fruits and vegetables


2. After contact with a sick dogs
3. As a result of mosquitos bites
4. As a result of bugs bites
5. Through dirty hands

8. Question
Methotrexate (structural analogue of the folic acid which is competitive
inhibitor of the dihydrofolatreductase) is prescribed for treatment of the
malignant tumour. On which level does methotrexate inhibit synthesis of the
nucleic acids?

1. Mononucleotide synthesis
2. Replication
3. Processing
4. Transcription
5. Reparation

9. Question
A patient presents high activity of LDH1,2, aspartate aminotransferase,
creatine phosphokinase. In what organ (organs) is the development of a
pathological process the most probable?
1. In skeletal muscles (dystrophy, atrophy)
2. In liver and kidneys
3. In the heart muscle (initial stage of myocardium infarction)
4. In kidneys and adrenals
5. In connective tissue

10. Question
A 42-year man suffering from gout has increased level of urinary acid in the
blood. Allopurinol was prescribed to decrease the level of urinary acid.
Competitive inhibitor of what enzyme is allopurinol?

1. Hypoxantinphosphoribosiltransferase
2. Guaninedeaminase
3. Adenosinedeaminase
4. Adeninephosphoribosiltransferase
5. Xanthinoxidase

11. Question
A 13-year-old boy complains of general weakness, dizziness, tiredness. He is
mentally retarded. Increased level of valine, isoleucine, leucine is in the blood
and urine. Urine has specific smell. What is the diagnosis?

1. Histidinemia
LAKSHMAN

2. Addisons disease
3. Tyrosinosis
4. Maple syrup urine disease

LAKSHMAN
5. Graves disease

12. Question
Examination of a 27-year-old patient revealed pathological changes in liver
and brain. Blood plasma analysis revealed an abrupt decrease in the copper
concentration, urine analysis revealed an increased copper concentration.
The patient was diagnosed with Wilson’s degeneration. To confirm the
diagnosis it is necessary to study the activity of the following enzyme in blood
serum:

1. Carbonic anhydrase
2. Ceruloplasmin
3. Xanthine oxidase
4. Leucine aminopeptidase
5. Alcohol dehydrogenase

13. Question
A patient has pellagra. Interrogation revealed that he had lived mostly on
maize for a long time and eaten little meat. This disease had been caused by
the deficit of the following substance in the maize:

1. Proline
2. Tyrosine
3. Histidine
4. Alanine
5. Tryptophan
14. Question
An infant has apparent diarrhea resulting from improper feeding. One of the
main diarrhea effects is plentiful excretion of sodium bicarbonate. What
form of acid-base balance disorder is the case?

1. Metabolic alkalosis
2. Metabolic acidosis
3. Respiratory acidosis
4. No disorders of acid-base balance will be observed
5. Respiratory alkalosis

15. Question
Removal of gall bladder of a patient has disturbed processes of Ca absorption
through the intestinal wall. What vitamin will stimulate this process?

1. D3
2. PP
3. B12
4. K
5. C

16. Question
LAKSHMAN

Patient with encephalopathy was admitted to the neurological in-patient


department. Correlation of increasing of encephalopathy and substances
absorbed by the bloodstream from the intestines was revealed.
Whatsubstances that are created in the intestines can cause endotoxemia?

LAKSHMAN
1. Butyrate
2. Biotin
3. Acetacetate
4. Ornithine
5. Indole

17. Question
A newborn child has convulsions that have been observed after prescription
of vitamin B6. This most probable cause of this effect is that vitamin B6 is a
componet of the following enzyme:

1. Glutamate decarboxylase
2. Netoglubarate dehydromine
3. Aminolevulinate synthase
4. Glycogen phosphorylase
5. Pyruvate dehydrostase

18. Question
Marked increase of activity of MB-forms of CPK (creatinephosphokinase) and
LDH-1 were revealed on the examination of the patients blood. What is the
most likely pathology?

1. Cholecystitis
2. Hepatitis
3. Miocardial infarction
4. Pancreatitis
5. Rheumatism

19. Question
An 18-year-old patient has enlarged inguinal lymphnodes, they are painless,
thickened on palpation. In the area of genital mucous membrane there is a
small-sized ulcer with thickened edges and “laquer” bottom of greyish colour.
What is the most probable diagnosis?

1. Gonorrhea
2. Trophic ulcer
3. Syphilis
4. Tuberculosis
5. Lepra

20. Question
A 48 year old patient complained about intense pain, slight swelling and
reddening of skin over the joints, temperature rise up to 38oC. Blood analysis
revealed high concentration of urates. This condition might be caused by
disturbed metabolism of:

1. Collagen
2. Carbohydrates
LAKSHMAN

3. Pyrimidines
4. Cholesterol
5. Purines

LAKSHMAN
21. Question
Profuse foam appeared when dentist put hydrogen peroxide on the mucous of
the oral cavity. What enzyme caused such activity?

1. Acetyltransferase
2. Methemoglobinreductase
3. Glucose-6-phosphatdehydrogenase
4. Catalase
5. Cholinesterase

22. Question
Buffer capacity of blood was decreased in the worker due to exhausting
muscular work. Entry of what acid substance to the blood can this state be
explained?

1. Lactate
2. 1,3-bisphosphoglycerate
3. Pyruvate
4. a-ketoglutarate
5. 3-phosphoglycerate

23. Question
Untrained people often have muscle pain after sprints as a result of lactate
accumulation. This might be caused by intensification of the following
biochemical process:
1. Pentose phosphate pathway
2. Gluconeogenesis
3. Glycolysis
4. Lipogenesis
5. Glycogenesis

24. Question
A 62 y.o. woman complains of frequent pains in the area of her chest and
backbone, rib fractures. A doctor assumed myelomatosis (plasmocytoma).
What of the following laboratory characteristics will be of the greatest
diagnostical importance?

1. Hyperalbuminemia
2. Proteinuria
3. Hypoproteinemia
4. Hypoglobulinemia
5. Paraproteinemia

25. Question
Characteristic sign of glycogenosis is muscle pain during physical work. Blood
examination reveals usually hypoglycemia. This pathology is caused by
congenital deficiency of the following enzyme:
LAKSHMAN

1. Lysosomal glycosidase
2. Gamma amylase
3. Alpha amylase

LAKSHMAN
4. Glycogen phosphorylase
5. Glucose 6-phosphate dehydrogenase

26. Question
Tere is observed inhibited fibrillation in the patients with bile ducts
obstruction, bleeding due to low level of absorbtion of some vitamin. What
vitamin is in deficit?

1. A
2. Carotene
3. D
4. K
5. E

27. Question
A doctor examined a child and revealed symptoms of rachitis. Development of
this desease was caused by deficiency of the following compound:

1. Naphtaquinone
2. Retinol
3. Tocopherol
4. Biotin
5. 1,25 [OH]-dichydroxycholecalciferol

28. Question
A sportsman was recommended to take a medication that contains carnitine
in order to improve his results. What process is activated by carnitine the
most?

1. Synthesis of ketone bodies


2. Tissue respiration
3. Synthesis of lipids
4. Synthesis of steroid hormones
5. Fatty acids transport to mitochondrions

29. Question
Fatty of phospholipids is disordered due to fat infiltration of the liver.
Indicate which of the presented substances can enhance the process of
methylation during phospholipids synthesis?

1. Ascorbic acid
2. Citrate
3. Glucose
4. Glycerin
5. Methionine

30. Question
A 30 y.o. woman had been ill for a year when she felt pain in the area of joints
LAKSHMAN

for the first time, they got swollen and skin above them became reddened.
Provisional diagnosis is rheumatoid arthritis. One of the most probable
causes of this disease is a structure alteration of a connective tissue protein:

LAKSHMAN
1. Collagen
2. Myosin
3. Ovoalbumin
4. Mucin
5. Troponin

31. Question
Nappies of a new born have dark spots that witness of formation of
homogentisic acid. Metabolic imbalance of which substance is it connected
with?

1. Thyrosine
2. Tryptophane
3. Methionine
4. Galactose
5. Cholesterine

32. Question
A 44-year-old woman complains of common weakness, heart pain,
considerable increase of body weight. Objectively: moon-like face, hirsutism,
AP- 165/100 mm Hg, height – 164 cm, weight – 103 kg; fat is mostly accumulated
in the region of neck, upper shoulder girdle, stomach. What is the main
pathogenetic mechanism of obesity?

1. Increased production of insulin


2. Increased production of glucocorticoids
3. Decreased production of thyroidal hormones
4. Increased production of mineralocorticoids
5. Decreased production of glucagon

33. Question
Galactosemia is revealed in the child. Concentration of glucose in the blood is
not considerably changed. Deficiency of what enzyme caused this illness?

1. Galactokinase
2. Hexokinase
3. Phosphoglucomutase
4. Amylo-1,6-glucosidase
5. Galactose-1-phosphate uridyltransferase

34. Question
Pain along large nervous stems and increased amount of pyruvate in the
blood were revealed in the patient. Insufficiency of what vitamin can cause
such change?

1. B1
2. B2
3. Biotin
LAKSHMAN

4. Pantothenic acid
5. PP

35. Question
A 2-year-old child with mental and physical retardation has been delivered to

LAKSHMAN
a hospital. He presents with frequent vomiting after having meals. There is
phenylpyruvic acid in urine. Which metabolism abnormality is the reason for
this pathology?

1. Amino-acid metabolism
2. Phosphoric calcium metabolism
3. Lipidic metabolism
4. Carbohydrate metabolism
5. Water-salt metabolism

36. Question
The study of the genealogy of a family with hypertrichosis (helix excessive
pilosis) has demonstrated that this symptom is manifested in all generations
only in men and is inherited by son from his father. What is the type of
hypertrichosis inheritance?

1. X-linked dominant chromosome


2. X-linked recessive chromosome
3. Autosome-dominant
4. Y-linked chromosome
5. Autosome-recessive

37. Question
The greater amount of nitrogen is excreted from the organism in form of
urea. Inhibition of urea synthesis and accumulation of ammonia in blood and
tissues are induced by the decreased activity of the following liver enzyme:
1. Pepsin
2. Amylase
3. Carbamoyl phosphate synthetase
4. Aspartate aminotransferase
5. Urease

38. Question
A 35-year-old man under the treatment for pulmonary tuberculosis has acute-
onset of right big toe pain, swelling, and low-grade fever. The gouty arthritis
was diagnosed and high serum uric acid level was found. Which of the
following antituberculosis drugs are known for causing high uric acid levels?

1. Cycloserine
2. Pyrazinamide
3. Thiacetazone
4. Rifampicin
5. Aminosalicylic acid

39. Question
Examination of a patient with frequent hemorrhages from internals and
mucous membranes revealed proline and lysine being a part of collagene
fibers. What vitamin absence caused disturbance of their hydroxylation?
LAKSHMAN

1. Vitamin A
2. Vitamin C
3. Vitamin E

LAKSHMAN
4. Thiamine
5. Vitamin K

40. Question
A patient has been diagnosed with alkaptonuria. Choose an enzyme whose
deficiency can be the reason for this pathology:

1. Pyruvate dehydrogenase
2. Homogentisic acid oxidase
3. Phenylalanine hydroxylase
4. Glutamate dehydrogenase
5. Dioxyphenylalanine decarboxylase

41. Question
An experimental animal has been given excessive amount of carbon-labeled
glucose for a week. What compound can the label be found in?

1. Methionine
2. Vitamin A
3. Arachidonic acid
4. Choline
5. Palmitic acid

42. Question
After severe viral hepatitis a 4 year old boy presents with vommiting,
occasional loss of consciousness, convulsions. Blood test revealed
hyperammoniemia. Such condition is caused by a disorder of the following
biochemical hepatic process:

1. Activation of amino acid decarboxylation


2. Disorder of ammonia neutralization
3. Inhibition of transamination enzymes
4. Disorder of biogenic amines neutralization
5. Protein synthesis inhibition

43. Question
The penetration of the irritable cell membrane for potassium ions has been
increased during an experiment. What changes of membrane electric status
can occur?

1. No changes
2. Hyperpolarization
3. Local response
4. Action potential
5. Depolarization

44. Question
LAKSHMAN

A child is languid, apathetic. Liver is enlarged and liver biopsy revealed a


significant excess of glycogene. Glucose concentration in the blood stream is
below normal. What is the cause of low glucose concentration?

LAKSHMAN
1. Low (absent) activity of hexokinase
2. High activity of glycogen synthetase
3. Low (absent) activity of glucose 6-phosphatase
4. Low (absent) activity of glycogene phosphorylase in liver
5. De cit of a gene that is responsible for synthesis of glucose 1-phosphaturidine
transferase

45. Question
According to clinical indications a patient was administered pyridoxal
phosphate. What processes is this medication intended to correct?

1. Protein synthesis
2. Desamination of purine nucleotide
3. Transamination and decarboxylation of aminoacids
4. Oxidative decarboxylation of ketonic acids
5. Synthesis of purine and pyrimidine bases

46. Question
Examination of a patient suffering from cancer of urinary bladder revealed
high rate of serotonin and hydroxyanthranilic acid. It is caused by excess of
the following amino acid in the organism:

1. Tyrosine
2. Tryptophan
3. Histidine
4. Alanine
5. Methionine

47. Question
In patients with the biliary tract obstruction the blood coagulation is
inhibited; the patients have frequent haemorrhages caused by the subnormal
assimilation of the following vitamin:

1. D
2. A
3. E
4. C
5. K

48. Question
A woman who has been keeping to a clean-rice diet for a long time was
diagnosed with polyneuritis (beriberi). What vitamin deficit results in
development of this disease?

1. Pyridoxine
2. Thiamine
3. Ribo avin
4. Ascorbic acid
LAKSHMAN

5. Folic acid

49. Question
A patient with suspicion on epidemic typhus was admitted to the hospital.

LAKSHMAN
Some arachnids and insects have been found in his flat. Which of them may be
a carrier of the pathogen of epidemic typhus?

1. Spiders
2. Cockroaches
3. Lice
4. Bed-bugs
5. House ies

50. Question
To prevent postoperative bleeding a 6 y.o. child was administered vicasol that
is a synthetic analogue of vitamin K. Name post-translational changes of
blood coagulation factors that will be activated by vicasol:

1. Partial proteolysis
2. Phosphorylation of serine radicals
3. Polymerization
4. Glycosylation
5. Carboxylation of glutamin acid

51. Question
Donor skin transplantation was performed to a patient with extensive burns.
On the 8th day the graft became swollen and changed colour; on the 11-th day
graft rejection started. What cells take part in this process?

1. Eosinophils
2. Erythrocytes
3. Basophils
4. T-lymphocytes
5. B-lymphocytes

52. Question
Myocyte cytoplasm contains a big number of dissolved metabolites of glucose
oxidation. Name one of them that turns directly into a lactate:

1. Glycerophosphate
2. Oxaloacetate
3. Glucose 6-phosphate
4. Fructose 6-phosphate
5. Pyruvate

53. Question
The energy inputs of a healthy man have been measured. In what position was
the patient if his energy inputs were less than the main exchange?

1. Nervous exertion
2. Easy work
3. Calmness
LAKSHMAN

4. Rest
5. Sleep

54. Question

LAKSHMAN
Analysis of amniotic fluid that was obtained as a result of amniocentesis
(puncture of amniotic sac) revealed cells the nuclei of which contain sex
chromatin (Barrs body). What can it be evidence of?

1. Genetic disorders of fetus development


2. Development of female fetus
3. Polyploidy
4. Trisomy
5. Development of male fetus

55. Question
A patient suffers from hepatic cirrhosis. Examination of which of the
following substances excreted by urine can characterize the state of antitoxic
function of liver?

1. Aminoacids
2. Hippuric acid
3. Kreatinine
4. Ammonium salts
5. Uric acid

56. Question
Albinos cant be under the sunlight – they dont aquire sun-tan but get
sunburns. Disturbed metabolism of what aminoacid underlies this
phenomenon?
1. Methionine
2. Tryptophan
3. Glutamic acid
4. Phenilalanine
5. Histidine

57. Question
While examining the child the doctor revealed symmetric cheeks roughness,
diarrhea, disfunction of the nervous system. Lack of what food components
caused it?

1. Phenylalanine, pangamic acid


2. Nicotinic acid, tryptophane
3. Methionine, lipoic acid
4. Threonine, pantothenic acid
5. Lysine, ascorbic acid

58. Question
A 62-year-old female patient has developed a cataract (lenticular opacity)
secondary to the diabetes mellitus. What type of protein modification is
observed in case of diabetic cataract?

1. Methylation
LAKSHMAN

2. Glycosylation
3. Phosphorylation
4. Limited proteolysis

LAKSHMAN
5. ADP-ribosylation

59. Question
A 46-year-old female patient has a continuous history of progressive
muscular (Duchennes) dystrophy. Which blood enzyme changes will be of
diagnostic value in this case?

1. Creatine phosphokinase
2. Pyruvate dehydrogenase
3. Lactate dehydrogenase
4. Adenylate cyclase
5. Glutamate dehydrogenase

60. Question
After implantation of a cardiac valve a young man constantly takes indirect
anticoagulants. His state was complicated by hemorrhage. What substance
content has decreased in blood?

1. Ceruloplasmin
2. Prothrombin
3. Creatin
4. Haptoglobin
5. Heparin

61. Question
Vitamin B1 deficiency results in disturbance of oxidative decarboxylation of
alpha-ketoglutaric acid. This will disturb synthesis of the following coenzyme:

1. Lipoic acid
2. Coenzyme A
3. Thiamine pyrophosphate
4. Flavine adenine dinucleotide (FAD)
5. Nicotinamide adenine dinucleotide (NAD)

62. Question
A 4 y.o. child with signs of durative proteinic starvation was admitted to the
hospital. The signs were as follows: growth inhibition, anemia, edemata,
mental deficiency. Choose a cause of edemata development:

1. Reduced synthesis of albumins


2. Reduced synthesis of hemoglobin
3. Reduced synthesis of lipoproteins
4. Reduced synthesis of glycoproteins
5. Reduced synthesis of globulins

63. Question
A child’s blood presents high content of galactose, glucose concentration is
low. There are such presentations as cataract, mental deficiency, adipose
LAKSHMAN

degeneration of liver. What disease is it?

1. Fructosemia
2. Steroid diabetes

LAKSHMAN
3. Galactosemia
4. Diabetes mellitus
5. Lactosemia

64. Question
Increased amount of free fat acids is observed in the blood of the patients
with diabetes mellitus. It can be caused by:

1. Increased activity of triglyceridelipase adipocytes


2. Decreased activity of phosphatidylcholine-cholesterol-acyltransferase blood plasma
3. Activation of the ketone bodies utilization
4. Activation of the synthesis of the apolipoproteins
5. Storage of palmitatoil-CoA

65. Question
A 3 year old child with fever was given aspirin. It resulted in intensified
erythrocyte haemolysis. Hemolytic anemia might have been caused by
congenital insufficiency of the following enzyme:

1. Glycerol phosphate dehydrogenase


2. Glucose 6-phosphate dehydrogenase
3. B-glutamiltransferase
4. Glucose 6-phosphatase
5. Glycogen phosphorylase
66. Question
A child has an acute renal failure. What biochemical factor found in saliva
can confirm this diagnosis?

1. Decrease in glucose concentration


2. Decrease in nucleic acid concentration
3. Increase in concentration of higher fatty acids
4. Increase in glucose concentration
5. Increase in urea concentration

67. Question
ATP synthesis is totaly blocked in a cell. How will the value of membrane rest
potential change?

1. It will be slightly increased


2. It will disappear
3. First it will decrease, then increase
4. It will be considerably increased
5. First it will increase, then decrease

68. Question
A 1.5-year-old child presents with both mental and physical lag, decolorizing
of skin and hair, decrease in catecholamine concentration in blood. When a
LAKSHMAN

few drops of 5% solution of trichloroacetic iron had been added to the child’s
urine it turned olive green. Such alteration are typical for the following
pathology of the amino acid metabolism:

P——T——M

LAKSHMAN
1. Alkaptonuria
2. Albinism
3. Phenylketonuria
4. Xanthinuria
5. Tyrosinosis

69. Question
A 50-year-old patient complains about general weakness, appetite loss and
cardiac arrhythmia. The patient presents with muscle hypotonia, flaccid
paralyses, weakened peristaltic activity of the bowels. Such condition might
be caused by:

1. Hypoproteinemia
2. Hypophosphatemia
3. Hyperkaliemia
4. Hyponatremia
5. Hypokaliemia

70. Question
A 65 year old man suffering from gout complains of kidney pain. Ultrasound
examination revealed renal calculi. The most probable cause of calculi
formation is the strengthened concentration of the following substance:

1. Cystine
2. Uric acid
3. Bilirubin
4. Cholesterol
5. Urea

71. Question
Ammonia is a very toxic substance, especially for nervous system. What
substance takes the most active part in ammonia detoxication in brain
tissues?

1. Lysine
2. Glutamic acid
3. Proline
4. Alanine
5. Histidine

72. Question
After a serious viral infection a 3-year-old child has repeated vomiting, loss of
consciousness, convulsions. Examination revealed hyperammoniemia. What
may have caused changes of biochemical blood indices of this child?

1. Inhibited activity of transamination enzymes


2. Increased purtefaction of proteins in intestines
3. Disorder of biogenic amines neutralization
LAKSHMAN

4. Activated processes of aminoacids decarboxylation


5. Disorder of ammonia neutralization in ornithinic cycle

73. Question

LAKSHMAN
Increased breaking of vessels, enamel and dentine destruction in scurvy
patients are caused by disorder of collagen maturing. What stage of
modification of procollagen is disordered in this avitaminosis?

1. Hydroxylation of proline
2. Glycosylation of hydroxylysine residues
3. Formation of polypeptide chains
4. Detaching of N-ended peptide
5. Removal of C-ended peptide from procollagen

74. Question
Examination of a patient suffering from chronic hepatitis revealed a
significant decrease in the synthesis and secretion of bile acids. What process
will be mainly disturbed in the patient’s bowels?

1. Glycerin absorption
2. Amino acid absorption
3. Protein digestion
4. Fat emulsi cation
5. Carbohydrate digestion

75. Question
As a result of exhausting muscular work a worker has largely reduced buffer
capacity of blood. What acidic substance that came to blood caused this
phenomenon?
1. Lactate
2. 3-phosphoglycerate
3. 1,3-bisphosphoglycerate
4. Pyruvate

76. Question
A patient consulted a doctor about symmetric dermatitis of open skin areas.
It was found out that the patient lived mostly on cereals and ate too little
meat, milk and eggs. What vitamin deficiency is the most evident?

1. Biotin
2. Calciferol
3. Tocopherol
4. Folic acid
5. Nicotinamide

77. Question
A 5-year-old child who often fells ill with respiratory diseases has eczematous
appearances after consumption of some food products, tendency to prolonged
course of inflammatory processes. What kind of diathesis can be suspected in
this case?

1. Lymphohypoplastic
LAKSHMAN

2. Hemmorhagic
3. Arthritism
4. Exudative-catharral

LAKSHMAN
5. Asthenic

78. Question
Examination of a patient suffering from frequent haemorrhages in the inner
organs and mucous membranes revealed proline and lysine being included in
collagen fibers. Impairment of their hydroxylation is caused by lack of the
following vitamin:

1. D
2. C
3. E
4. K
5. A

79. Question
A 4 y.o. boy has had recently serious viral hepatitis. Now there are such
clinical presentations as vomiting, loss of consciousness, convulsions. Blood
analysis revealed hyperammoniemia. Disturbunce of which biochemical
process caused such pathological condition of the patient?

1. Inhibition of transamination enzymes


2. Disturbed neutralization of ammonia in liver
3. Activation of aminoacid decarboxylation
4. Increased putrefaction of proteins in bowels
5. Disturbed neutralization of biogenic amines

80. Question
A patient complains about dyspnea provoked by the physical activity. Clinical
examination revealed anaemia and presence of the paraprotein in the zone of
gamma-globulins. To confirm the myeloma diagnosis it is necessary to
determine the following index in the patient’s urine:

1. Antitrypsin
2. Bilirubin
3. Bence Jones protein
4. Ceruloplasmin
5. Haemoglobin

81. Question
A patient with suspected diphtheria went through bacterioscopic
examination. Examination of throat swab revealed rod-shaped bacteria with
volutin granules. What etiotropic preparation should be chosen in this case?

1. Interferon
2. Bacteriophage
3. Antidiphtheric antitoxic serum
4. Diphtheria antitoxin
5. Eubiotic

82. Question
LAKSHMAN

A nurse accidentally injected a nearly double dose of insulin to a patient with


diabetes mellitus. The patient lapsed into a hypoglycemic coma. What drug
should be injected in order to help him out of coma?

LAKSHMAN
1. Insulin
2. Glucose
3. Noradrenaline
4. Somatotropin
5. Lidase

83. Question
After a sprint an untrained person develops muscle hypoxia. This leads to the
accumulation of the following metabolite in muscles:

1. Acetyl CoA
2. Lactate
3. Oxaloacetate
4. Glucose 6-phosphate
5. Ketone bodies

84. Question
A newborn child suffers from milk curdling in stomach, this means that
soluble milk proteins (caseins) transform to insoluble proteins (paracaseins)
by means of calcium ions and a certain enzyme. What enzyme takes part in
this process?

1. Secretin
2. Lipase
3. Renin
4. Gastrin
5. Pepsin

85. Question
A patient had been ill with bronchial asthma for many years and died from
asthmatic fit. Histologic lung examination revealed: lumen of bronchioles and
small bronches contain a lot of mucus with some eosinophils, there is
sclerosis of alveolar septums, dilatation of alveole lumen. What mechanism
of development of hypersensibility reaction took place?

1. Immunocomplex reaction
2. Reagin reaction
3. Cytolysis determined by lymphocytes
4. Granulomatosis
5. Cytotoxic reaction

86. Question
Emotional stress causes activation of hormone-sensitive triglyceride lipase in
the adipocytes. What secondary mediator takes part in this process?

1. Cyclic adenosine monophosphate


2. Ions of Ca2+
3. Diacylglycerol
LAKSHMAN

4. Adenosine monophosphate
5. Cyclic guanosine monophosphate

87. Question

LAKSHMAN
Autopsy of a 46-year-old man revealed multiple brown-and-green layers and
hemmorhages on the mucous membrane of rectum and sigmoid colon; slime
and some blood in colon lumen; histologically – fibrinous colitis. In course of
bacteriological analysis of colon contents S.Sonne were found. What is the
most probable diagnosis?

1. Yersiniosis
2. Crohns disease
3. Dysentery
4. Cholera
5. Salmonellosis

88. Question
A 65-year-old suffering from the gout man complains of the pain in the
kidneys region. On ultrasonic examination the renal calculi were revealed. As
a result of what process were they formed?

1. Ornithine cycle
2. Protein catabolism
3. Restoration of cysteine
4. Heme decay
5. Decay of purine nucleotides

89. Question
Carnitine including drug was recomended to the sportsman for improving
results. What process is activated most of all with help of carnitine?
1. Synthesis of steroid hormones
2. Synthesis of lipids
3. Transport of fatty acids to the mitochondria
4. Tissue respiration
5. Synthesis of ketone bodies

90. Question
Products of some proteins hydrolysis and modification are the biologically
active substances called hormones. Lipotropin, corticotropin, melanotropin
and endorphins are synthesized in the hypophysis of the following protein:

1. Proopiomelanocortin (POMC)
2. Neuroglobulin
3. Thyreoglobulin
4. Neuroalbumin
5. Neurostromin

91. Question
A patient is ill with diabetes mellitus that is accompanied by hyperglycemia of
over 7.2 millimole/l on an empty stomach. The level of what blood plasma
protein allows to estimate the glycemia rate retrospectively (4-8 weeks before
examination)?
LAKSHMAN

1. Fibrinogen
2. Albumin
3. Glycated hemoglobin

LAKSHMAN
4. C-reactive protein
5. Ceruloplasmin

92. Question
A 36-year-old female patient has a history of collagen disease. Urine analysis
is likely to reveal an increased concentration of the following metabolite:

1. Urobilinogen
2. Creatinine
3. Urea
4. Oxyproline
5. Indican

93. Question
Vitamin A together with specific cytoreceptors penetrates through the nuclear
membranes, induces transcription processes that stimulate growth and
differentiation of cells. This biological function is realized by the following
form of vitamin A:

1. Carotin
2. Trans-retinoic acid
3. Cis-retinal
4. Trans-retinal
5. Retinol

94. Question
Concentration of pyruvate is increased in the patients blood, the most of
which is excreted with urine. What avitaminosis is observed in the patient?

1. Avitaminosis E
2. Avitaminosis B6
3. Avitaminosis B2
4. Avitaminosis B3
5. Avitaminosis B1

95. Question
A 35 y.o. patient who often consumes alcohol was treated with diuretics. There
appeared serious muscle and heart weakness, vomiting, diarrhea, AP- 100/60
mm Hg, depression. This condition is caused by intensified excretion with
urine of:

1. Chlorine
2. Phosphates
3. Potassium
4. Calcium
5. Sodium

96. Question
A 16-year-old boy was performed an appendectomy. He has been hospitalized
LAKSHMAN

for right lower quadrant abdominal pain within 18 hours. The surgical
specimen is edematous and erythematous. Infiltration by what of the
following cells is the most typical for the process occuring here?

LAKSHMAN
1. Monocytes
2. Eosinophils
3. Limphocytes
4. Neutrophils
5. Basophils

97. Question
After consumption of rich food a patient has nausea and heartburn,
steatorrhea. This condition might be caused by:

1. Amylase de ciency
2. Bile acid de ciency
3. Disturbed tripsin synthesis
4. Disturbed phospholipase synthesis
5. Increased lipase secretion

98. Question
An experimantal animal that was kept on protein-free diet developed fatty
liver infiltration, in particular as a result of deficiency of methylating agents.
This is caused by disturbed generation of the following metabolite:

1. Choline
2. Acetoacetate
3. DOPA
4. Linoleic acid
5. Cholesterol

99. Question
A 52 year-old patient with bronchial asthma was treated with glucocorticoids.
Fever reaction appeared as a result of postinjective abscess. The patient had
subfebrile temperature, which didn’t correspond to latitude and severity of
inflammatory process. Why did patient have low fever reaction?

1. Inhibited endogen pyrogens production


2. Thermoregulation center inhibition
3. Violation of heat-producing mechanisms
4. In ammatory barrier formation in injection place
5. Violation of heat loss through lungs

100. Question
Glutamate decarboxylation results in formation of inhibitory transmitter in
CNS. Name it:

1. Asparagine
2. Serotonin
3. GABA
4. Glutathione
5. Histamine
LAKSHMAN

101. Question
A 10-year-old girl often experiences acute respiratory infections with multiple
spotty haemorrages in the places of clothes friction. Hypovitaminosis of what

LAKSHMAN
vitamin is present at the girl?

1. A
2. B2
3. B6
4. C
5. B1

102. Question
During examination of an 11-month-old infant a pediatrician revealed
osteoectasia of the lower extremities and delayed mineralization of cranial
bones. Such pathology is usually provoked by the deficit of the following
vitamin:

1. Thiamin
2. Cholecalciferol
3. Pantothenic acid
4. Bio avonoids
5. Ribo avin

103. Question
A genetics specialist analyzed the genealogy of a family and found that both
males and females may have the illness, not across all the generations, and
that healthy parents may have ill children. What is the type of illness
inheritance?
1. Y-linked
2. Autosomal dominant
3. X-linked recessive
4. Autosomal recessive
5. X-linked dominant

104. Question
A patient diagnosed with carcinoid of bowels was admitted to the hospital.
Analysis revealed high production of serotonin. It is known that this
substance is formed of tryptophane aminooacid. What biochemical
mechanism underlies this process?

1. Microsomal oxydation
2. Formation of paired compounds
3. Desamination
4. Decarboxylation
5. Transamination

105. Question
Desulfiram is widely used in medical practice to prevent alcocholism. It
inhibits aldehyde dehydrogenase. Increased level of what metabolite causes
aversion to alcochol?
LAKSHMAN

1. Propionic aldehyde
2. Acetaldehyde
3. Methanol

LAKSHMAN
4. Malonyl aldehyde
5. Ethanol

106. Question
12 hours after an accute attack of retrosternal pain a patient presented a
jump of aspartate aminotransferase activity in blood serum. What pathology
is this deviation typical for?

1. Myocardium infarction
2. Diabetes mellitus
3. Viral hepatitis
4. Diabetes insipidus
5. Collagenosis

107. Question
Examination of a man who hadn’t been consuming fats but had been getting
enough carbohydrates and proteins for a long time revealed dermatitis, poor
wound healing, vision impairment. What is the probable cause of metabolic
disorder?

1. Lack of linoleic acid, vitamins A, D, E, K


2. Lack of palmitic acid
3. Low caloric value of diet
4. Lack of vitamins PP, H
5. Lack of oleic acid
108. Question
After intake of rich food a patient feels nausea and sluggishness; with time
there appeared signs of steatorrhea. Blood cholesterine concentration is 9.2
micromole/l. This condition was caused by lack of:

1. Triglycerides
2. Phospholipids
3. Bile acids
4. Fatty acids
5. Chylomicrons

109. Question
Utilization of arachidonic acid via cyclooxigenase pathway results in
formation of some bioactive substances. Name them:

1. Somatomedins
2. Thyroxine
3. Biogenic amins
4. Prostaglandins
5. Insulin-like growth factors

110. Question
A 1-year-old child with symptoms of muscle involvement was admitted to the
LAKSHMAN

hospital. Examination revealed carnitine deficiency in his muscles. What


process disturbance is the biochemical basis of this pathology?

1. Substrate phosphorylation

LAKSHMAN
2. Transporting of fatty acids to mitochodrions
3. Actin and myosin synthesis
4. Lactic acid utilization
5. Regulation of Ca2+ level in mitochondrions

111. Question
A patient with continious bronchopneumonia was admitted to the therapeutic
department. Antibiotic therapy didnt give much effect. What medication for
improvement of immune state should be added to the complex treatment of
this patient?

1. Sulfocamphocaine
2. Analgin
3. Paracetamol
4. Timaline
5. Benadryl

112. Question
A 45 y.o. woman suffers from Cushings syndrome – steroid diabetes.
Biochemical examination revealed: hyperglycemia, hypochloremia. Which of
the under-mentioned processes is the first to be activated?

1. Glucose transport to the cell


2. Glycolysis
3. Glycogenolysis
4. Glucose reabsorption
5. Gluconeogenesis

113. Question
Study of conversion of a food colouring agent revealed that neutralization of
this xenobiotic takes place only in one phase – microsomal oxydation. Name a
component of this phase:

1. Cytochrome oxidase
2. Cytochrome A
3. Cytochrome C
4. Cytochrome p-450
5. Cytochrome B

114. Question
A patient with suspected diagnosis “progressing muscular dystrophy” got his
urine tested. What compound will confirm this diagnosis if found in urine?

1. Myoglobin
2. Collagen
3. Kreatine
4. Porphyrin
LAKSHMAN

5. Calmodulin

115. Question
A 5-month-old boy was hospitalized for tonic convulsions. He has a life-time
history of this disease. Examination revealed coarse hair, thinned and fragile

LAKSHMAN
nails, pale and dry skin. In blood: calcium – 1.5 millimole/l, phosphor – 1.9
millimole/l. These changes are associated with:

1. Hypoparathyroidism
2. Hyperaldosteronism
3. Hypothyroidism
4. Hyperparathyroidism
5. Hypoaldosteronism

116. Question
Pharmacological effects of antidepressants are connected with inhibition of
an enzyme catalyzing biogenic amines noradrenaline and serotonine in the
mitochondrions of cerebral neurons. What enzyme participates in this
process?

1. Decarboxylase
2. Transaminase
3. Monoamine oxidase
4. Peptidase
5. Lyase

117. Question
Patient with diabetes mellitus experienced loss of consciousness and
convulsions after injection of insulin. What is the result of biochemical blood
analysis for concentration of the sugar?
1. 8.0 mmol/L
2. 3.3 mmol/L
3. 5.5 mmol/L
4. 1.5 mmol/L
5. 10.0 mmol/L

118. Question
Patient experienced increased susceptibility of the skin to the sunlight. His
urine after some time became dark-red. What is the most likely cause of this?

1. Alkaptonuria
2. Porphyria
3. Pellagra
4. Hemolytic jaundice
5. Albinism

119. Question
A 49-year-old driver complains about unbearable constricting pain behind the
breastbone irradiating to the neck. The pain arose 2 hours ago. Objectively:
the patient’s condition is grave, he is pale, heart tones are decreased.
Laboratory studies revealed high activity of creatine kinase and LDH1. What
disease are these symptoms typical for?
LAKSHMAN

1. Acute myocardial infarction


2. Acute pancreatitis
3. Diabetes mellitus

LAKSHMAN
4. Cholelithiasis
5. Stenocardia

120. Question
The patient with complaints of permanent thirst applied to the doctor.
Hyperglycemia, polyuria and increased concentration of 17-ketosteroids in
the urine were revealed. What disease is the most likely?

1. Type I glycogenosis
2. Steroid diabetes
3. Myxoedema
4. Addisons disease
5. Insulin-dependent diabetes mellitus

121. Question
As a result of posttranslative modifications some proteins taking part in
blood coagulation, particularly prothrombin, become capable of calcium
binding. The following vitamin takes part in this process:

1. B1
2. C
3. B2
4. K
5. A

122. Question
Toxic affection of liver results in dysfunction of protein synthesis. It is usually
accompanied by the following kind of dysproteinemia:

1. Relative hypoproteinemia
2. Absolute hypoproteinemia
3. Relative hyperproteinemia
4. Absolute hyperproteinemia
5. Paraproteinemia

123. Question
A patient has yellow skin colour, dark urine, dark-yellow feces. What
substance will have strengthened concentration in the blood serum?

1. Unconjugated bilirubin
2. Conjugated bilirubin
3. Biliverdin
4. Verdoglobin
5. Mesobilirubin

124. Question
Aspirin has antiinflammatory effect due to inhibition of the cyclooxygenase
activity. Level of what biological active acids will decrease?
LAKSHMAN

1. Leucotriens
2. Iodinethyronyns
3. Catecholamines

LAKSHMAN
4. Biogenic amines
5. Prostaglandins

125. Question
A patient was delivered to the hospital by an emergency team. Objectively:
grave condition, unconscious, adynamy. Cutaneous surfaces are dry, eyes are
sunken, face is cyanotic. There is tachycardia and smell of acetone from the
mouth. Analysis results: blood glucose – 20.1 micromole/l (standard is 3.3-5.5
micromole/l), urine glucose – 3.5% (standard is – 0). What is the most probable
diagnosis?

1. Acute alcoholic intoxication


2. Anaphylactic shock
3. Acute heart failure
4. Hyperglycemic coma
5. Hypoglycemic coma

126. Question
A mother consulted a doctor about her 5-year-old child who develops
erythemas, vesicular rash and skin itch under the influence of sun.
Laboratory studies revealed decreased iron concentration in the blood
serum, increased uroporphyrinogen I excretion with the urine. What is the
most likely inherited pathology in this child?

1. Coproporphyria
2. Erythropoietic porphyria
3. Intermittent porphyria
4. Methemoglobinemia
5. Hepatic porphyria

127. Question
Examination of a child who hasn’t got fresh fruit and vegetables during
winter revealed numerous subcutaneous hemorrhages, gingivitis, carious
cavities in teeth. What vitamin combination should be prescribed in this case?

1. Folic acid and cobalamin


2. Ribo avin and nicotinamide
3. Thiamine and pyridoxine
4. Calciferol and ascorbic acid
5. Ascorbic acid and rutin

128. Question
A 20 year old patient complains of general weakness, dizziness, quick
fatigability. Blood analysis results: Hb- 80 g/l. Microscopical examination
results: erythrocytes are of modified form. This condition might be caused by:

1. Addisons disease
2. Obturative jaundice
3. Acute intermittent porphyria
4. Sickle-cell anemia
LAKSHMAN

5. Hepatocellular jaundice

129. Question
In clinical practice tuberculosis is treated with izoniazid preparation – that is

LAKSHMAN
an antivitamin able to penetrate into the tuberculosis bacillus.
Tuberculostatic effect is induced by the interference with replication
processes and oxidation-reduction reactions due to the buildup of pseudo-
coenzyme:

1. NAD
2. FAD
3. FMN
4. CoQ
5. TDP

130. Question
A patient complains of frequent diarrheas, especially after consumption of
fattening food, and of body weight loss. Laboratory examination revealed
steatorrhea; hypocholic feces. What can be the cause of this condition?

1. Mucous membrane in ammation of small intestine


2. Lack of pancreatic phospholipase
3. Lack of pancreatic lipase
4. Unbalanced diet
5. Obturation of biliary tracts

131. Question
On the empty stomach in the patients blood glucose level was 5.65 mmol/L, in
an hour after usage of sugar it was 8.55 mmol/L, in a 2 hours – 4.95 mmol/L.
Such indicators are typical for:
1. Patient with insulin-dependent diabetes mellitus
2. Patient with hidden diabetes mellitus
3. Patient with non-insulin dependent diabetes mellitus
4. Healthy person
5. Patient with tireotoxicosis

132. Question
A 38 year old patient suffers from rheumatism in its active phase. What
laboratory characteristic of blood serum is of diagnostic importance in case
of this pathology?

1. C-reactive protein
2. Transferrin
3. Uric acid
4. Urea
5. Creatinine

133. Question
A woman with 0 (I) blood group has born a child with AB blood group. This
woman’s husband has A blood group. What genetic interaction explains this
phenomenon?

1. Incomplete dominance
LAKSHMAN

2. Polymery
3. Complementation
4. Codominance

LAKSHMAN
5. Recessive epistasis

134. Question
Cytogenetic examination of a patient with dysfunction of the reproductive
system revealed normal karyotype 46, XY in some cells, but most cells have
Klinefelters syndrome karyotype – 47,XXY. Such phenomenon of cell
inhomogeneity is called:

1. Heterogeneity
2. Inversion
3. Duplication
4. Mosaicism
5. Transposition

135. Question
Patients who suffer from severe diabetes and don’t receive insulin have
metabolic acidosis. This is caused by increased concentration of the following
metabolites:

1. Cholesterol
2. Ketone bodies
3. Fatty acids
4. Triacylglycerols
5. Unsaturated fatty acids

136. Question
In case of enterobiasis acrihine – the structural analogue of vitamin B2 – is
administered. The synthesis disorder of which enzymes does this medicine
cause in microorganisms?

1. FAD-dependent dehydrogenases
2. Peptidases
3. Cytochromeoxidases
4. Aminotransferases
5. NAD-dependet dehydrogenases

137. Question
Osteolaterism is characterized by a decrease in collagen strength caused by
much less intensive formation of cross-links in collagen fibrils. This
phenomenon is caused by the low activity of the following enzyme:

1. Lysyl oxidase
2. Prolyl hydroxylase
3. Monoamino-oxidase
4. Collagenase
5. Lysyl hydroxylase

138. Question
Pathological changes of the liver and brain were revealed in a 27-year-old
LAKSHMAN

patient.The copper concentration is abruptly decreased in blood plasma and


increased in the urine. Wilsons disease was diagnosed. Activity of what
enzyme in the blood serum should be examined to prove diagnisis?

LAKSHMAN
1. Alcoholdehydrogenaze
2. Carboanhydraze
3. Xanthioxidase
4. Leucinamineopeptidaze
5. Ceruloplasmin

139. Question
Laboratory examination of a child revealed increased concentration of
leucine, valine, isoleucine and their ketoderivatives in blood and urine. Urine
smelt of maple syrup. This disease is characterized by the deficit of the
following enzyme:

1. Glucose-6-phosphatase
2. Aminotransferase
3. Dehydrogenase of branched amino acids
4. Phosphofructokinase
5. Phosphofructomutase

140. Question
Examination of a patient revealed typical presentations of collagenosis. This
pathology is characterized by increase of the following urine index:

1. Glucose
2. Arginine
3. Hydroxyproline
4. Ammonium salts
5. Mineral salts

141. Question
A man got poisoned with mushrooms. They contain muscarine that stimulates
muscarinic cholinoreceptors. What symptom is typical for poisoning with
inedible mushrooms?

1. Mydriasis
2. Miosis
3. Bronchi dilation
4. Arterial pressure rise
5. Heart rate rise

142. Question
A 46 year old patient applied to a doctor complaining about joint pain that
becomes stronger the day before weather changes. Blood examination
revealed strengthened concentration of uric acid. The most probable cause of
the disease is the intensified disintegration of the following substance:

1. Uridine monophosphate
2. Thymidine monophosphate
3. Cytidine monophosphate
LAKSHMAN

4. Adenosine monophosphate
5. Uridine triphosphate

143. Question

LAKSHMAN
Dietary intake of a 30 year old nursing woman contains 1000 mg of calcium,
1300 mg of phosphorus and 20 mg of iron per day. It is necessary to change
content of these mineral substances in the following way:

1. To reduce iron content


2. To increase phosphorus content
3. To reduce uorine content
4. To increase calcium content
5. To increase iron content

144. Question
It was found out that some compounds, for instance fungi toxins and some
antibiotics can inhibit activity of RNA-polymerase. What process will be
disturbed in a cell in case of inhibition of this enzyme?

1. Reparation
2. Translation
3. Replication
4. Processing
5. Transcription

145. Question
During hypersensitivity test a patient got subcutaneous injection of an
antigen which caused reddening of skin, edema, pain as a result of histamine
action. This biogenic amine is generated as a result of transformation of the
following histidine amino acid:
1. Decarboxylation
2. Methylation
3. Isomerization
4. Phosphorylation
5. Deaminization

146. Question
A newborn develops dyspepsia after the milk feeding. When the milk is
substituted by the glucose solution the dyspepsia symptoms disappear. The
newborn has the subnormal activity of the following enzyme:

1. Maltase
2. Invertase
3. Isomaltase
4. Lactase
5. Amylase

147. Question
The concentration of albumins in human blood sample is lower than normal.
This leads to edema of tissues. What blood function is damaged?

1. Maintaining the Ph level


LAKSHMAN

2. Maintaining the blood sedimentation system


3. Maintaining the body temperature
4. Maintaining the oncotic blood pressure
5. All answers are correct

LAKSHMAN
148. Question
A patient with serious damage of muscular tissue was admitted to the
traumatological department. What biochemical urine index will be increased
in this case?

1. Mineral salts
2. Creatinine
3. Common lipids
4. Uric acid
5. Glucose

149. Question
A patient has an increased pyruvate concentration in blood. A large amount
of it is excreted with the urine. What vitamin is lacking in this patient?

1. B6
2. B2
3. E
4. B3
5. B1

150. Question
A 38-year-old patient died during intractable attack of bronchial asthma.
Histologic examination revealed mucus accumulation in bronchial lumen, a
lot of fat cells (labrocytes) in the wall of bronches, many of them are in the
state of degranulation, there are also a lot of eosinophils. What pathogenesis
of bronchial changes is it?

1. Immunocomplex mechanism
2. Granulomatosis
3. Cellular cytolysis
4. Atopy
5. Cytotoxic, cytolytic of antibodies

151. Question
When blood circulation in the damaged tissue is restored, then lactate
accumulation comes to a stop and glucose consumption decelerates. These
metabolic changes are caused by activation of the following process:

1. Aerobic glycolysis
2. Anaerobic glycolysis
3. Lipolysis
4. Gluconeogenesis
5. Glycogen biosynthesis

152. Question
Surgical removal of a part of stomach resulted in disturbed absorption of
vitamin B12, it is excreted with feces. The patient was diagnosed with anemia.
LAKSHMAN

What factor is necessary for absorption of this vitamin?

1. Gastromucoprotein
2. Hydrochloric acid

LAKSHMAN
3. Folic acid
4. Gastrin
5. Pepsin

153. Question
Index of pH of the blood changed and became 7.3 in the patient with diabetus
mellitus. Detecting of the components of what buffer system is used while
diagnosing disorder of the acid-base equilibrium?

1. Oxyhemoglobin
2. Protein
3. Phosphate
4. Bicarbonate
5. Hemoglobin

154. Question
RNA-polymeraseB(II) is blocked due to amanitine poisoning (poison of death-
cup). It disturbes:

1. Maturation of m-RNA
2. Synthesis of m-RNA
3. Reverse transcription
4. Synthesis of t-RNA
5. Primers synthesis
155. Question
Increased production of thyroidal hormones T3 and T4, weight loss,
tachycardia, psychic excitement and so on present on thyrotoxicosis. How do
thyroidal hormones effect energy metabolism in the mitochondrion of cells?

1. Activates phosphorylation of substance


2. Activates oxidated phosphorylation
3. Stops respiratory chain
4. Disconnect oxidation and oxidated phosphorylation
5. Stops phosphorylation of substance

156. Question
During starvation muscle proteins break up into free amino acids. These
compounds will be the most probably involved into the following process:

1. Gluconeogenesis in liver
2. Gluconeogenesis in muscles
3. Decarboxylation
4. Synthesis of higher fatty acids
5. Glycogenolysis

157. Question
Examination of a patient revealed II grade obesity. It is known that he
LAKSHMAN

consumes a lot of sweets and rich food, has sedentary way of life. Thats why
anabolic metabolism has the priority in his organism. Which of the following
pathways is amphibolic?

LAKSHMAN
1. Glycolysis
2. Cycle of tricarboxylic acids
3. Lipolysis
4. Glyconeogenesis
5. Fatty acids oxidation

158. Question
Hydroxylation of endogenous substrates and xenobiotics requires a donor of
protons. Which of the following vitamins can play this role?

1. Vitamin A
2. Vitamin P
3. Vitamin E
4. Vitamin B6
5. Vitamin C

159. Question
An oncological patient was prescribed methotrexate. With the lapse of time
target cells of the tumour lost susceptibility to this drug. There is change of
gene expression of the folowing enzyme:

1. Folate decarboxylase
2. Folate oxidase
3. Deaminase
4. Thiaminase
5. Dehydrofolate reductase

160. Question
A 3 year old child with symptoms of stomatitis, gingivitis and dermatitis of
open skin areas was delivered to a hospital. Examination revealed inherited
disturbance of neutral amino acid transporting in the bowels. These
symptoms were caused by the deficiency of the following vitamin:

1. Niacin
2. Vitamin A
3. Cobalamin
4. Pantothenic acid
5. Biotin

161. Question
A male patient has been diagnosed with acute radiation disease. Laboratory
examination revealed a considerable reduction of platelet serotonin level. The
likely cause of platelet serotonin reduction is the disturbed metabolism of the
following substance:

1. Tyrosine
2. Histidine
3. Serine
LAKSHMAN

4. Phenylalanine
5. 5-oxytryptofane

162. Question

LAKSHMAN
A newborn child was found to have reduced intensity of sucking, frequent
vomiting, hypotonia. Urine and blood exhibit increased concentration of
citrulline. What metabolic process is disturbed?

1. Ornithinic cycle
2. Glycolysis
3. Tricarboxylic acid cycle
4. Cori cycle
5. Glyconeogenesis

163. Question
A patient presents with dysfunction of cerebral cortex accompanied by
epileptic seizures. He has been administered a biogenic amine synthetized
from glutamate and responsible for central inhibition. What substance is it?

1. Serotonin
2. Acetylcholine
3. Dopamine
4. Gamma-amino butyric acid
5. Histamine

164. Question
Depressions and emotional insanities result from the deficit of noradrenalin,
serotonin and other biogenic amines in the brain. Their concentration in the
synapses can be increased by means of the antidepressants that inhibit the
following enzyme:
1. Phenylalanine-4-monooxygenase
2. Monoamine oxidase
3. L-amino-acid oxidase
4. Diamine oxidase
5. D-amino-acid oxidase

165. Question
Parodontitis is treated with calcium preparations and a hormone that
stimulates tooth mineralization and inhibits tissue resorption. What
hormone is it?

1. Calcitonin
2. Parathormone
3. Adrenalin
4. Aldosterone
5. Thyroxine

166. Question
A 9-month-old infant is fed with artificial formulas with unbalanced vitamin
B6 concentration. The infant presents with pellagral dermatitis, convulsions,
anaemia. Convulsion development might be caused by the disturbed
formation of:
LAKSHMAN

1. DOPA
2. Histamine
3. Serotonin

LAKSHMAN
4. GABA
5. Dopamine

167. Question
During metabolic process active forms of the oxygen including superoxide
anion radical are formed in the human body. With help of what enzyme is this
anion activated?

1. Catalase
2. Glutathioneperoxidase
3. Peroxidase
4. Superoxide dismutase
5. Glutathionereductase

168. Question
At the laboratory experiment the leukocyte culture was mixed with
staphylococci. Neutrophile leukocytes engulfed and digested bacterial cells.
This processes are termed:

1. Diffusion
2. Facilitated diffusion
3. Pinocytosis
4. Osmosis
5. Phagocytosis

169. Question
An oncological patient had been administered methotrexate. With time target
cells of the tumour lost sensitivity to this drug. At the same time the change in
gene expression of the following enzyme is observed:

1. Thiaminase
2. Deaminase
3. Pholate oxidase
4. Dehydropholate reductase
5. Pholate decarboxylase

170. Question
A patient had hemorrhagic stroke. Blood examination revealed strengthened
kinin concentration.The patient was prescribed contrical. It was
administered in order to inhibit the following proteinase:

1. Pepsin
2. Collagenase
3. Kallikrein
4. Trypsin
5. Chemotrypsin

171. Question
A 4 year old child with hereditary renal lesion has signs of rickets, vitamin D
LAKSHMAN

concentration in blood is normal. What is the most probable cause of rickets


development?

1. Hypofunction of parathyroid glands

LAKSHMAN
2. Impaired synthesis of calcitriol
3. Lack of calcium in food
4. Increased excretion of calcium
5. Hyperfunction of parathyroid glands

172. Question
Blood of a 12 year old boy presents low concentration of uric acid and
accumulation of xanthine and hypoxanthine. This child has genetic defect of
the following enzyme:

1. Arginase
2. Xanthine oxidase
3. Glycerylkinase
4. Urease
5. Ornithine carbamoyltransferase

173. Question
A patient complained about dizziness, memory impairment, periodical
convulsions. It was revealed that these changes were caused by a product of
decarboxylation of glutamic acid. Name this product:

1. ATP
2. THFA
3. TDP
4. GABA
5. Pyridoxal phosphate

174. Question
A 6 year old child was delivered to a hospital. Examination revealed that the
child couldn’t fix his eyes, didn’t keep his eyes on toys, eye ground had the
cherry-red spot sign. Laboratory analyses showed that brain, liver and spleen
had high rate of ganglioside glycometide. What congenital disease is the child
ill with?

1. Tay-Sachs disease
2. Turners syndrome
3. Niemann-Pick disease
4. Wilsons syndrome
5. MacArdle disease

175. Question
Diabetes mellitus causes ketosis as a result of activated oxidation of fatty
acids. What disorders of acid-base equilibrium may be caused by excessive
accumulation of ketone bodies in blood?

1. Any changes wount happen


2. Metabolic alcalosis
3. Respiratory alcalosis
LAKSHMAN

4. Metabolic acidosis
5. Respiratory acidosis

176. Question

LAKSHMAN
An experiment proved that UV-radiated cells of patients with xeroderma
pigmentosum restore the native DNA structure slower than cells of healthy
individuals as a result of reparation enzyme defection. What enzyme helps
this process?

1. RNA ligase
2. DNA polymerase III
3. Primase
4. DNA gyirase
5. Endonuclease

177. Question
Objective examination of a patient revealed: slender figure, big skull, highly
developed frontal region of face, short extremities. What constitutional type
is it characteristic for?

1. Respiratory
2. Digestive
3. Cerebral
4. Muscular
5. Mixed

178. Question
The formation of a secondary mediator is obligatory in membrane-
intracellular mechanism of hormone action. Point out the substance that is
unable to be a secondary mediator:
1. Inositol-3,4,5-triphosphate
2. Glycerol
3. Ca2+
4. Diacylglycerol
5. CAMP

179. Question
On some diseases it is observed aldosteronism with hypertension and edema
due to sodium retention in the organism. What organ of the internal secretion
is affected on aldosteronism?

1. Testicle
2. Ovaries
3. Pancreas
4. Hypophysis
5. Adrenal glands

180. Question
Cardinal symptoms of primary hyperparathyroidism are osteoporosis and
renal lesion along with development of urolithiasis. What substance makes up
the basis of these calculi in this disease?

1. Cystine
LAKSHMAN

2. Uric acid
3. Cholesterol
4. Bilirubin

LAKSHMAN
5. Calcium phosphate

181. Question
A sportsman needs to improve his sporting results. He was recommended to
take a preparation that contains carnitine. What process is activated the
most by this compound?

1. Fatty acids transporting


2. Calcium ions transporting
3. Amino acids transporting
4. Vitamin K transporting
5. Glucose transporting
Created by Eneutron Team

Kiev eneutron.info@gmail.com
LAKSHMAN

LAKSHMAN
Krok 1 – 2013 Base(Histology)

Medical PG WhatsApp Group

Results
0 of 65 questions answered correctly

Your time: 00:02:21

You have reached 0 of 65 points, (0%)

RESTART QUIZ VIEW QUESTIONS

1. Question
A 50-year-old male farm worker has been brought to the emergency room. He
was found confused in the orchard and since then has remained unconscious.
His heart rate is 45 and his blood pressure is 80/40 mm Hg. He is sweating and
salivating profusely. Which of the following should be prescribed?

1. Proserine
2. Atropine
3. Pentamine
4. Norepinephrine
5. Physostigmine

2. Question
LAKSHMAN

A patient visited a dentist with complaints of redness and edema of his mouth
mucous membrane in a month after dental prosthesis. The patient was
diagnosed with allergic stomatitis. What type of allergic reaction by Gell and
Cumbs underlies this disease?

LAKSHMAN
1. Anaphylactic
2. Immunocomplex
3. Stimulating
4. Cytotoxic
5. Delayed type hypersensitivity

3. Question
A patient has undergone an amputation of lower extremity. Some time later
painful nodules appeared in a stump. Amputatious neuromas were found out
at the microscopic examination. To what pathological processes do those
formations relate?

1. Dystrophy
2. Regeneration
3. Metaplasia
4. In ammation
5. Hyperemia

4. Question
A histological specimen presents an artery. One of the membranes of its wall
has flat cells lying on the basal membrane. What type of cells is it?

1. Endothelium
2. Mesothelium
3. Fibroblasts
4. Macrophages
5. Smooth myocytes

5. Question
A patient died 3 days after the operation because of perforated colon with
manifestations of diffuse purulent peritonitis. The autopsy revealed: colon
mucose membrane was thickened and covered with a fibrin film, isolated
ulcers penetrated at different depth. The histology result: mucous membrane
necrosis, leukocytes infiltration with hemorrhages focuses. What disease
complication caused the patient’s death?

1. Dysentery
2. Amebiasis
3. Crohns disease
4. Typhoid
5. Nonspeci c ulcerative colitis

6. Question
Most participants of Magellan expedition to America died from avitominosis.
This disease declared itself by general weakness, subcutaneous
hemmorhages, falling of teeth, gingival hemmorhages. What is the name of
this avitiminosis?
LAKSHMAN

1. Polyneuritis (beriberi)
2. Scurvy
3. Rachitis

LAKSHMAN
4. Biermers anemia
5. Pellagra

7. Question
Kidneys of a man under examination show increased resorbtion of calcium
ions and decreased resorbtion of phosphate ions. What hormone causes this
phenomenon?

1. Vasopressin
2. Parathormone
3. Hormonal form D3
4. Thyrocalcitonin
5. Aldosterone

8. Question
A patient was admitted to the hospital with an asphyxia attack provoked by a
spasm of smooth muscles of the respiratory tracts. This attack was mainly
caused by alterations in the following parts of the airways:

1. Small bronchi
2. Large bronchi
3. Median bronchi
4. Respiratory part
5. Terminal bronchioles

9. Question
Blood sampling for bulk analysis is recommended to be performed on an
empty stomack and in the morning. What changes in blood composition can
occur if to perform blood sampling after food intake?

1. Increased contents of leukocytes


2. Increased plasma proteins
3. Increased contents of erythrocytes
4. Reduced contents of erythrocytes
5. Reduced contents of thrombocytes

10. Question
Hypertrychosis of auricles is caused by a gene that is localized in Y-
chromosome. Father has this feature. What is the probability to give birth to a
boy with such anomaly?

1. 35%
2. 100%
3. 75%
4. 0%
5. 25%

11. Question
The increased intraocular tension is observed in the patient with
LAKSHMAN

glaucoma.Secretion of aqueous humor by the ciliar body is normal. Injury of


what structure of the eyeball wall caused the disorder of flow-out from the
anterior chamber?

LAKSHMAN
1. Back epithelium of cornea
2. Choroid
3. Ciliar body
4. Venous sinus
5. Ciliary muscle

12. Question
Electronic microphotography of pulmonary alveoles wall presents a big cell.
Its cytoplasm has a lot of mitochondria, developed Golgi apparatus,
osmiophil lamellated corpuscles. What is the main function of this cell?
Type I alveolar cells are squamous extremely thin ce
It absorbs microorganisms lls involved in the process of gas exchange between t
he alveoli and blood. Type II alveolar cells are involv
1.
2. It puri es the air
ed in the secretion of surfactant proteins.
3. It produces surfactant
4. It is a component of blood-air barrier
5. It warms the air

13. Question
When the pH level of the stomach lumen decreases to less than 3, the antrum
of the stomach releases peptide that acts in paracrine fashion to inhibit
gastrin release. This peptide is:

1. Vasoactive intestinal peptide (VIP)


2. Somatostatin
3. Acetylcholine
4. GIF
5. Gastrin-releasing peptide (GRP)

14. Question
A histological specimen of a kidney shows a part of the distal tubule going
between the afferent and efferent arteriole. The cells building the tubule wall
have dense nuclei; basal membrane is absent. Such structural formation is
called:

1. Juxtavascular cells
2. Juxtaglomerular cells
3. Mesangial cells
4. Macula densa

15. Question
An electronic microphotograph shows a macrophagic cell with erythrocytes
at different stages of differentiation located along its processes. This is the
cell of the following organ:

1. Red bone marrow


2. Lymph node
3. Tonsil
4. Spleen
LAKSHMAN

5. Thymus

16. Question
A patient has been given high doses of hydrocortisone for a long time. This

LAKSHMAN
caused atrophy of one of the adrenal cortex zones. Which zone is it?

1. Reticular
2. Fascial
3. Glomerular and reticular
4. Glomerular

17. Question
Low level of albumins and fibrinogen was detected in the patients blood.
Decreased activity of what organelle of the liver hepatocytes can cause it?

1. Golgi complex
2. Lysosomes
3. Mitochondrions
4. Agranular endoplasmatic net
5. Granular endoplasmatic net

18. Question
A patient with thrombophlebitis of lower extremities had got chest pains,
blood spitting, growing respiratory failure that caused his death. Autopsy
revealed multiple pulmonary infarctions. What is the most probable reason
of their development?

1. Bronchial artery embolism


2. Pulmonary venous thrombosis
3. Pulmonary artery thrombosis
4. Bronchial artery thrombosis
5. Pulmonary artery embolism

19. Question
As a result of a trauma a patient has damaged anterior roots of spinal cord.
What structures have been affected?

1. Axons of motoneurons and axons of neurons of lateral horns


2. Dendrites of neurons of spinal ganglions
3. Axons of neurons of lateral horns
4. Peripheral processes of sensitive spinal ganglions
5. Central processes of sensitive neurons of spinal ganglions

20. Question
A viral infection has damaged cells that form walls of bile capillaries. This
stimulated conditions for inflow of bile into the blood of sinusoidal
capillaries. What cells are damaged?

1. Hepatocytes
2. Ito cells
3. Kupffers cells
4. Endotheliocytes
LAKSHMAN

5. Pit-cells

21. Question
Ultramicroscopical examination of “dark” hepatocyte population in the cell
cytoplasm detected a developed granular endoplasmic reticulum. What

LAKSHMAN
function has this organella in these cells?

1. Deintoxicative function
2. Carbohydrate synthesis
3. Synthesis of blood plasma proteins
4. Calcium ion depositing
5. Bile production

22. Question
During postembryonal haemopoiesis in the red bone marrow the cells of one
of the cellular differons demonstrate a gradual decrease in cytoplasmic
basophilia as well as an increase in oxyphilia, the nucleus is being forced out.
Such morphological changes are typical for the following haemopoiesis type:

1. Erythropoiesis
2. Neutrophil cytopoiesis
3. Basophil cytopoiesis
4. Lymphopoiesis
5. Eosinophil cytopoiesis

23. Question
Autopsy of a man who died from chronic cardiacvascular collapse revealed
“tiger heart”. Sidewards of endocardium a yellowish-white banding can be
seen; myocardium is dull, dark-yellow. What process caused this pathology?

1. Fatty vascular-stromal degeneration


2. Fatty parenchymatous degeneration
3. Amyloidosis
4. Carbohydrate degeneration
5. Hyaline degeneration

24. Question
A sensitive neural ganglion consists of roundish neurocytes with one
extension that divides into axon and dendrite at some distance from the
perikaryon. What are these cells called?

1. Bipolar
2. Multipolar
3. Apolar
4. Unipolar
5. Pseudounipolar

25. Question
The pulmonalis embolism has suddenly developed in a 40 year-old patient
with opened fracture of the hip. Choose the possible kind of embolism

1. Air
2. Thrombus-embolus
LAKSHMAN

3. Fat
4. Foreign body
5. Tissue

26. Question

LAKSHMAN
A 13-year-old girl with history of asthma complained of cough, dyspnea and
wheezing. Her symptoms became so severe that her parents brought her to the
emergency room. Physical examination revealed diaphoresis, dyspnea,
tachycardia and tachypnea. Her respiratory rate was 42/min, pulse rate was
110 beats per minute, and blood pressure was 130/70 mm Hg. Choose from the
following list the most appropriate drug to reverse the bronchoconstriction
rapidly:

1. Salbutamol
2. Cromolyn
3. Beclomethasone
4. Methylprednidsolone
5. Ipratropium

27. Question
Study of fingerprints (dactylography) is used by criminalists for personal
identification as well as for diagnostics of genetic abnormalities, particularly
Dawns disease. What layer of skin determines individuality of fingerprints?

1. Basal
2. Dermopapillary
3. Horny
4. Clear (stratum lucidum epidermidis)
5. Reticular

28. Question
A histological specimen of spleen shows a vessel with a wall consisting of
endothelium and subendothelial layer, median membrane is absent, exterior
membrane inosculates with the layers of spleen connective tissue. What vessel
is it?

1. Vein of muscular type


2. Vein of non-muscular type
3. Arteriole
4. Artery of muscular type
5. Capillary

29. Question
A doctor administered Allopurinol to a 26-year-old young man with the
symptoms of gout. What pharmacological action of Allopurinol ensures

xanthine oxidase inhibitors


therapeutical effect?

1. By inhibiting leucocyte migration into the joint


2. By increasing uric acid excretion
3. By general anti-in ammatory effect
4. By general analgetic effect
5. By inhibiting uric acid synthesis

30. Question
LAKSHMAN

The low specific gravity of the secondary urine (1002) was found out in the
sick person. What is the most distant part of nephron where concentration of
secondary urine takes place?

LAKSHMAN
1. In the collecting duck
2. In proximal tubule of nephron
3. In the nephron’s glomerulus
4. In distal tubule of nephron
5. In ascending part of loop of Henle

31. Question
Examination of a 43 y.o. patient revealed that his stomach has difficulties with
digestion of protein food. Gastric juice analysis revealed low acidity. Function
of which gastric cells is disturbed in this case?

1. Endocrinous cells
2. Parietal exocrinocytes
3. Mucous cells (mucocytes)
4. Main exocrinocytes
5. Cervical mucocytes

32. Question
A histological specimen of kidney shows a structure consisting of a
glomerulus of fenestrated capillaries and a bilayer epithelial capsule. Specify
this structure:

1. Renal corpuscle
2. Proximal tubule
3. Henles loop
4. Distal tubule
5. Receiving tube

33. Question
A scheme presents an exocrinous gland that has unbranched excretory duct
with a terminal part in form of a saccule openining into the duct. How is this
gland called according to the morphological classification of exocrinous
glands?

1. Compound unbranched alveolar tubular


2. Compound unbranched alveolar
3. Simple unbranched alveolar
4. Compound branched alveolar
5. Simple branched tubular

34. Question
An electron microphotography of a fragment of proper gastric gland shows a
big irregular round-shaped cell. There are a lot of intracellular tubules and
mitochondria in the cytoplasm. Specify this cell:

1. Mucous cell
2. Endocrine cell
3. Principal cell
LAKSHMAN

4. Parietal cell
5. Undifferentiated cell

35. Question

LAKSHMAN
A histological specimen presents a receptor zone of a sensoepithelial sense
organ. Cells of this zone are placed upon the basal membrane and include the
following types: external and internal receptor cells, external and internal
phalangeal cell, stem cells, external limiting cells and external supporting
cell. The described receptor zone belongs to the following sense organ:

1. Equilibrium organ
2. Gustatory organ
3. Olfactory organ
4. Visual organ
5. Acoustic organ

36. Question
A histological specimen shows a blood vessel. Its inner coat is composed by
endothelium, subendothelium and internal elastic membrane. The middle
coat is enriched with smooth myocytes. Such morphological characteristics
are typical for the following vessel:
Muscular veins
These blood vessels are much more numerous than in
arteries of a similar size. Unlike muscular arteries, th
1. Elastic-type artery
2. Muscular-type vein ere is no internal or external elastic layer surrounding
the muscle layer (why do you think this is?). Compare
the structures of the vein and muscular vein.
3. Capillary
4. Non-muscular vein
5. Muscular-type artery

37. Question
A 22-year-old patient was admitted to the hospital with complaints of heavy
nasal breathing. During the examination of her nasal cavity the doctors
found thickened mucous membrane, a lot of mucus and nodular infiltrates
without erosions in the nose.The nasal rhinoscleroma was diagnosed. The
biopsy was taken. What typical morphological changes may be found?

1. Granulomas with Mikulicz’s cells


2. Interstitial in ammation
3. Granulomas with Langhan’s cells
4. Granulomas with foreign body cells
5. Granulomas with Virchow’s cells

38. Question
In the blood of a 26-year-old man it was revealed 18% of erythrocytes of the
spherical, ball-shaped, flat and thorn-like shape. Other eritrocytes were in the
form of the concavo-concave disks. How is such phenomenon called?

1. Erytrocytosis
2. Physiological poikilocytosis
Physiological anisocytosisterm anisopoikilocytosis is actually made up of two diff
erent terms: anisocytosis and poikilocytosis. Anisocyto
3.
4. Pathological poikilocytosissis means that there are red blood cells of varying sizes
5. Pathological anisocytosis on your blood smear. Poikilocytosis means that there a
re red blood cells of varying shapes on your blood sme
ar.
LAKSHMAN

39. Question
A patient died from acute cardiac insufficiency. The histological examination
of his heart revealed the necrotized section in myocardium of the left
ventricle, which was separated from undamaged tissue by the zone of
hyperimic vessels, small hemorrhages and leukocytic infiltration. What is the

LAKSHMAN
most likely diagnosis?

1. Myocardial infarction
2. Productive myocarditis
3. Focal exudate myocarditis
4. Diffuse exudate myocarditis
5. Myocardial ischemic dystrophy

40. Question
Live vaccine is injected into the human body. Increasing activity of what cells
of connective tissue can be expected?

1. Fibroblasts and labrocytes


2. Plasmocytes and lymphocytes
3. Pigmentocytes and pericytes
4. Macrophages and broblasts
5. Adipocytes and adventitious cells

41. Question
Decreased blood supply to the organs causes hypoxia that activates
fibroblasts function. Volume of what elements is increased in this case?
Intracellular calcium concentration increases shortly after the on
set of hypoxia. Voltage-gated calcium channels open in response
1. Nerve elements to the falling transmembrane potential, and the increasing intrac
ellular sodium concentration causes the membrane-bound Na/Ca
exchanger to reverse its activity.
2. Vessels of microcircular stream
3. Intercellular substance
4. Parenchymatous elements of the organ
5. Lymphatic vessels

42. Question
Lung of premature infant is presented on electronic photomicrography of
biopsy material. Collapse of the alveolar wall caused by the deficiency of
surfactant was revealed. Disfunction of what cells of the alveolar wall caused
it?

1. Secretory cells
2. Fibroblasts
3. Alveocytes type I
4. Alveolar macrophages
5. Alveocytes type II

43. Question
M-r S presents all signs of the hepatic coma: loss of consciousness, absence of
reflexes, cramps, convulsion, disorder of heart activity, recurrent (periodical)
respiration. What are cerebrotoxical substances which accumulate in blood
under hepar insufficiency?

1. Ammonia
2. Autoantibody
LAKSHMAN

3. Ketonic body
4. Necrosogenic substances
5. IL-1

LAKSHMAN
44. Question
In the microspecimen of red bone marrow there were revealed multiple
capillares through the walls of which mature blood cells penetrated. What
type of capillares is it?

1. Visceral
2. Sinusoidal
3. Lymphatic
4. Fenestrational
5. Somatical

45. Question
On autopsy a 35-year-old man the focus of carnification 5 cm in diametre
enclosed in a thin capsule was revealed in the second segment of the right
lung . The focus consists of a tough dry friable tissue with a dim surface. For
what disease are these morphological changes typical?

1. Chondroma
2. Tumorous form of silicosis
3. Lung cancer
4. Tuberculoma
5. Postin ammatory pneumosclerosis

46. Question
In a histological specimen parenchyma of an organ is represented by
lymphoid tissue that forms lymph nodes; the latter are arranged in a diffuse
manner and enclose a central artery. What anatomic formation has such
morphological structure?

1. Lymph node
2. Tonsil
3. Red bone marrow
4. Spleen
5. Thymus

47. Question
A microspecimen of the submandibular salivary gland shows some basket-
shaped cells concentrated around the acines and excretory ducts. These cells
surround bases of the serous cells and are called myoepitheliocytes. These
cells relate to the following tissue:

1. Special connective tissue


2. Muscular tissue
3. Epithelial tissue
4. Loose brous connective tissue
5. Neural tissue
LAKSHMAN

48. Question
The specimens present sections of haemopoetic and immunogenetic organs.
Organ has lymph tissue forming different structures (lymph nodes,lobules,
bars). In what organ does antigen-independent proliferation and
differantiation take place?

LAKSHMAN
1. Hemolymph nodes
2. Thymus
3. Spleen
4. Tonsil
5. Lymphatic nodes

49. Question
Histological examination of a 40 y.o. mans thymus revealed decreased share
of parenchymatous gland elements, increased share of adipose and loose
connective tissue, its enrichment with thymus bodies. The organs mass was
unchanged. What phenomenon is it?

1. Age involution
2. Dystrophy
3. Atrophy
4. Hypotrophy
5. Accidental involution

50. Question
An embryo displays disturbed process of dorsal mesoderm segmentation and
somite formation. What part of skin will have developmental abnormalities?

1. Sudoriferous glands
2. Hair
3. Sebaceous glands
4. Derma
5. Epidermis

51. Question
Patient with injured muscles of the lower extremities was admitted to the
traumatological department. Due to what cells is reparative regeneration of
the muscle fibers and restoration of the muscle function possible?

1. Myoepithelial cells
2. Satellite-cells
3. Myo broblasts
4. Myoblasts
5. Fibroblasts

52. Question
A patient ill with chronic gastritis went for endogastric pH-metry that
allowed to reveal decreased acidity of gastric juice. It is indicative of
diminished function of the following cells:

1. Endocrinocytes
2. Accessory cells
3. Cervical cells
LAKSHMAN

4. Chief exocrinocytes
5. Parietal exocrinocytes

53. Question

LAKSHMAN
An endocrinal gland with parenchyma consisting of epithelium and neural
tissue is under morphological examination. Epithelial trabecules have two
types of cells: chromophilic and chromophobic. Identify this organ:

1. Adrenal glands
2. Thyroid gland
3. Parathyroid gland
4. Hypothalamus
5. Hypophysis

54. Question
In course of a conditional experiment the development of mesenchyma cells
was completely inhibited. Development of the following muscular tissue will
be disturbed:

1. Cardiac muscular tissue


2. Skeletal muscular tissue
3. Smooth muscular tissue
4. Epidermal muscular tissue
5. Neural muscular tissue

55. Question
A patient complains of dryness of head skin, itching, fragility and loss of hair.
After examination he was diagnosed with seborrhea. Disturbed activity of
which cells caused this condition?
sebaceous gland is an organ located in the dermis. Its role is t
Melanocytes o synthesise and secrete sebum which is a component of the
hydrolipidic film. The purpose of sebum is to protect the skin
1.
2. Adipocytes from damages caused by external factors and from dehydrati
on. Sebum also maintains the suppleness of skin and hair.
3. Epithelial cells
4. Cells of sudoriferous glands
5. Cells of sebaceous glands

56. Question
A pathological process in bronchi resulted in epithelium desquamation. What
cells will regenerate bronchial epithelium?

1. Goblet
2. Basal

Sweat glands, also known as sudoriferous or sudoriparous glands, fro


3. Ciliate
4. Intercalary m Latin sudor 'sweat', are small tubular structures of the skin that pro
duce sweat. ... They are not significant for cooling in humans, but are
5. Endocrinal the sole effective sweat glands in hoofed animals, such as the camels,
donkeys, horses, and cattle
57. Question
An infectious disease caused contractive activity of muscles that contract and
dilate eye pupil (paralytic state). What functional eye system was damaged?

1. Lacrimal apparatus
2. Dioptric
LAKSHMAN

3. Accomodative
4. Photosensory
5. Ancillary

LAKSHMAN
58. Question
In course of an experiment a big number of stem cells of red bone marrow
was in some way destructed. Regeneration of which cell populations in the
loose connective tissue will be inhibited?

1. Of macrophages
2. Of pericytes
3. Of pigment cells
4. Of lipocytes
5. Of broblasts

59. Question
Vitamin A deficit results in the impairment of twilight vision. Name the cells
that have the above-mentioned photoreceptor function:

1. Cone receptor cells


2. Horizontal neurocytes
3. Ganglion neurocytes
4. Rod receptor cell
5. Bipolar neurons

60. Question
During pubescence the cells of male sexual glands begin to produce male sex
hormon testosterone that calls forth secondary sexual characters. What cells
of male sexual glands produce this hormone?
1. Supporting cells
2. Sustentocytes
3. Spermatozoa
4. Sertolis cells
5. Leidig cells

61. Question
Histological specimen presents a vessel the wall of which consists of
endothelium, basal membrane and loose connective tissue. What type of
vessel is it?

1. Hemocapillary
2. Vein of non-muscular type
3. Artery
4. Vein of muscular type
5. Lymphocapillary

62. Question
A 2-year-old child has got intestinal dysbacteriosis, which results in
hemorrhagic syndrome. What is the most likely cause of hemorrhage of the
child?

1. Activation of tissue thromboplastin


LAKSHMAN

2. Fibrinogen de ciency
3. Hypocalcemia
4. Vitamin K insuf ciency

LAKSHMAN
5. PP hypovitaminosis

63. Question
One of sections of central nervous system has layerwise arrangement of
neurocytes. Among them there are cells of the following forms: stellate,
fusiform, horizontal, pyramidal. What section of central nervous system is
this structure typical for?

1. Hypothalamus
2. Medulla oblongata
3. Cortex of cerebrum
4. Spinal cord
5. Cerebellum

64. Question
During histological examination of the stomach it was found out that glands
contain very small amount of pariental cells or they are totally absent.
Mucose membrane of what part of the stomach was studied?

1. Body of stomach
2. Pyloric part
3. Cardiak part
4. Fundus of stomach

65. Question
Roentgenological examination of skull base bones revealed enlargement of
sellar cavity, thinning of anterior clinoid processes, destruction of different
parts, destruction of different parts of sella turcica. Such bone destruction
might be caused by a tumour of the following wndocrinous gland:

1. Thymus gland
2. Epiphysis
3. Thyroid gland
4. Adrenal glands
5. Hypophysis

Created by Eneutron Team

Kiev eneutron.info@gmail.com
LAKSHMAN

LAKSHMAN
Krok 1 – 2013 Base(Physiology)

Medical PG WhatsApp Group

Results
0 of 161 questions answered correctly

Your time: 00:05:46

You have reached 0 of 161 points, (0%)

RESTART QUIZ VIEW QUESTIONS

1. Question
A patient has a transverse disruption of spinal cord below the IV thoracic
segment. What changes of respiration will it cause?

1. Respiration will stop


2. Respiration will become less frequent
3. Respiration will stay unchanged
4. Respiration will become more frequent
5. Respiration will become deeper

2. Question
An individual is characterized by rounded face, broad forehead, a mongolian
LAKSHMAN

type of eyelid fold, flattened nasal bridge, permanently open mouth,


projecting lower lip, protruding tongue, short neck, flat hands, and stubby
fingers. What diagnosis can be put to the patient?

LAKSHMAN
1. Downs syndrome
2. Turners syndrome
3. Supermales
4. Alkaptonuria
5. Klinefelters syndrome

3. Question
A man is being measured power inputs on an empty stomach, in the lying
position, under conditions of physical and psychic rest at a comfortable
temperature. Power inputs will reach the maximum at:

1. 2-3 p.m
2. 3-4 a.m.
3. 5-6 p.m
4. 10-12 a.m
5. 7-8 a.m

4. Question
A sportsman was examined after an intensive physical activity. The
examination revealed disorder of movement coordination but the force of
muscle contractions remained the same. It can be explained by retarded speed
of excitement conduction through:

1. Conduction tracts
2. Central synapses
3. Afferent nerves
4. Efferent nerves
5. Neuromuscular synapses

5. Question
ECG study showed that the T-waves were positive in the standard extremity
leads, their amplitude and duration were normal. The right conclusion would
be that the following process runs normally in the heart ventricles:

1. Repolarization
2. Excitement
3. Depolarization
4. Relaxation
5. Contraction

6. Question
While emotional excitement the heart rate in a 30-year-old person run up to
112 Bpm. What part of the conducting system of the heart caused it?

1. Intraventricular node
2. His bundle branches
3. Synoatrial node
4. Purkinjes bers
LAKSHMAN

5. His bundle

7. Question
Power inputs of a boy increased from 500 to 2000 kJ pro hour. What can be the
cause of it?

LAKSHMAN
1. Food intake
2. Physical exercise
3. Mental activity
4. Transition from sleep to wakefulness
5. Raise of outer temperatute

8. Question
Punctata hemorrhage was found out in the patient after application of a
tourniquet. With disfunction of what blood cells is it connected?

1. Lymphocytes
2. Neutrophiles
3. Eosinophiles
4. Monocytes
5. Platelets

9. Question
Person felt thirsty after staying in heat for a long time. Signals of what
receptors caused it first of all?

1. Glucoreceptors of hypothalamus
2. Osmoreceptorsof the liver
3. Baroreceptors of aortic arch
4. Sodium receptors of hypothalamus
5. Osmoreceptors of hypothalamus

10. Question
The person has decreased diuresis, hypernatremia, hypokalemia.
Hypersecretion of what hormon can cause such changes?

1. Auricular sodiumuretic factor


2. Aldosterone
3. Parathormone
4. Adrenalin
5. Vasopressin

11. Question
A 60-year-old man after cerebral hemorrhage felt asleep for a long time.
Damage of what structure caused this state?

1. Black substances
2. Reticular formation
3. Nuclears of the cerebral nerves
4. Hippocampus
5. Cortex of the large hemispheres

12. Question
LAKSHMAN

Inhibition of alpha-motoneuron of the extensor muscles was noticed after


stimulation of alpha-motoneuron of the flexor muscles during the experiment
on the spinal column. What type of inhibition can this process cause?

LAKSHMAN
1. Recurrent
2. Lateral
3. Presynaptic
4. Reciprocal
5. Depolarizational

13. Question
While shifting the gaze to the closely situated object the refracting power of
eyes optical mediums will increase by 10 diopters. It results from changing of
such eye structure:

1. Lens
2. Vitreous body
3. Cornea
4. Muscle that dilatates pupil
5. Liquid of the anterior chamber of eye

14. Question
Blood analysis of a patient showed signs of HIV infection (human
immunodeficiency virus). Which cells does HIV-virus primarily affect?

1. Cells that contain receptor IgM (B-lymphocytes)


2. Specialized nervous cells (neurons)
3. Cells that contain receptor T4 (T-helpers)
4. Proliferating cells (stem hematoplastic cells)
5. Mast cells

15. Question
A 2-year-old child experienced convulsions because of lowering calcium ions
concentration in the blood plasma. Function of what structure is decreased?

1. Thymus
2. Parathyroid glands
3. Pineal gland
4. Adrenal cortex
5. Hypophysis

16. Question
Intrapleural pressure is being measured in a person. In what phase has a
person hold his breath if the pressure is – 25 cm H2O?

1. Calm expiration
2. Calm inspiration
3. Speed up expiration
4. Speed up inspiration

17. Question
Purulent endometritis developed in a woman after delivery. Treating with
LAKSHMAN

antibiotics inhibitors of murein synthesis was ineffective. Wide spectrum


bactericidal antibiotic was administered to her. In 6 hours temperature
rapidly increased up to 400C with shiver. Muscle pains have appeared. BP
dropped down to 70/40 mmHg. Oligura has developed. What is the main
reason for the development of this condition?

LAKSHMAN
1. Internal bleeding
2. Toxic effect of preparation
3. Anaphylactic shock
4. Bacteremia
5. Endotoxic shock

18. Question
Blood minute volume of a 30 year old woman at rest is 5 l/m. What blood
volume is pumped through the pulmonary vessels per minute?

1. 2.5 l
2. 3.75 l
3. 5l
4. 1.5 l
5. 2.0 l

19. Question
A patient has a trauma of sternocleidomastoid muscle. This caused a
decrease in value of the following indicator of external respiration:

1. Functional residual lung capacity


2. Expiratory reserve volume
3. Residual volume
4. Respiratory capacity
5. Inspiratory reserve volume

20. Question
A 2 y.o. child has convulsions as a result of lowered concentration of calcium
ions in blood plasma. It is caused by reduced function of:

1. Hypophysis
2. Pineal gland
3. Thymus
4. Parathyroid glands
5. Adrenal cortex

21. Question
After a long training session a sportsman has developed fatigue accompanied
by abrupt performance decrement. What link of the reflex arch was the
fatigue initiated in?

1. Muscles
2. Receptors
3. Efferent conductor
4. Afferent conductor
LAKSHMAN

5. Nerve centres

22. Question
As a result of long-term starvation the glomerular filtration of a man was
accelerated by 20%. The most probable cause of filtration changes under such

LAKSHMAN
conditions is:

1. Rise of systemic arterial pressure


2. Increase of renal plasma ow
3. Growth of ltration coef cient
4. Fall of oncotic pressure of blood plasma
5. Increased permeability of renal lter

23. Question
Middle part of cochlear of internal ear was destroyed in animal while
experiment. It will cause abnormalities of the sound perception of the
following frequencies:

1. Middle
2. Low
3. High
4. No abnormalities
5. High and low

24. Question
X-ray examination discovered lungs emphysema in the patient. What is the
reason of short breath development in this case?

1. Decreasing of alveoli receptors sensitivity


2. Decreased lungs elasticity
3. Excitation of respiratory center
4. Inhibition of respiratory center
5. Increased lungs elasticity

25. Question
As a result of spinal-cord trauma a 33 y.o. man has a disturbed pain and
temperature sensitivity that is caused by damage of the following tract:

1. Anterior spinocerebellar
2. Spinothalamic
3. Posterior spinocerebellar
4. Medial spinocortical
5. Lateral spinocortical

26. Question
In course of an experiment a skeletal muscle is being stimulated by a series of
electric impulses. What type of muscle contraction will arise, if every
subsequent impulse comes in the period of shortening of the previous single
muscle contraction?

1. Holotetanus
2. Muscle contracture
3. Partial tetanus
LAKSHMAN

4. Asynchronous tetanus
5. A series of single contractions

27. Question

LAKSHMAN
Examination of a patient revealed overgrowth of facial bones and soft tissues,
tongue enlargement, wide interdental spaces in the enlarged dental arch.
What changes of the hormonal secretion are the most likely?

1. Hypersecretion of insulin
2. Hyposecretion of the somatotropic hormone
3. Hypersecretion of the somatotropic hormone
4. Hyposecretion of insulin
5. Hyposecretion of thyroxin

28. Question
Spasm of smooth muscle of bronchi developed in the patient. Usage of
activators of what membrane cytoreceptors will physiologically lead to
decrease attack?

1. M-cholinoreceptors
2. alpha-adrenoreceptors
3. H-cholinoreceptors
4. alpha- and beta-adrenoreceptors
5. beta-adrenoreceptors

29. Question
A child has abnormal formation of tooth enamel and dentin as a result of low
concentration of calcium ions in blood. Such abnormalities might be caused
by deficiency of the following hormone:
1. Parathormone
2. Triiodothyronine
3. Thyrocalcitonin
4. Thyroxin
5. Somatotropic hormone

30. Question
A concentrated solution of sodium chloride was intravenously injected to an
animal. This caused decreased reabsorption of sodium ions in the renal
tubules. It is the result of the following changes of hormonal secretion:

1. Aldosterone increase
2. Vasopressin reduction
3. Aldosterone reduction
4. Vasopressin increase
5. Reduction of atrial natriuretic factor

31. Question
Examination of a 43 y.o. anephric patient revealed anemia symptoms. What is
the cause of these symptoms?

1. Reduced synthesis of erythropoietins


LAKSHMAN

2. Iron de cit
3. Vitamin B12 de cit
4. Enhanced destruction of erythrocytes
5. Folic acid de cit

LAKSHMAN
32. Question
The temperature of the ambient environment is 38oC and relative air humidity
is 50%. What ways of heat emission provide maintaining a constant
temperature of the human body?

1. Convection
2. Convection and conduction
3. Radiation
4. Evaporation
5. Heat conduction

33. Question
A peripheral segment of vagus nerve on a dogs neck was being stimulated in
course of an experiment. The following changes of cardiac activity could be
meanwhile observed:

1. Enhancement of atrioventricular conduction


2. Increased excitability of myocardium
3. Heart rate and heart force ampli cation
4. Heart rate fall
5. Heart hurry

34. Question
A lightly dressed man is standing in a room, air temperature is +140C,
windows and doors are closed. In what way does he emit heat the most
actively?

1. Heat conduction
2. Convection
3. Heat radiation
4. Perspiration
5. Evaporation

35. Question
During the breakout of acute respiratory infection in order to diagnose
influenza the express-diagnosis, based on revealing of specific viral antigen in
the examined material (nasopharyngial lavage), is carried out. Which
reaction is used for this?

1. Precipitation
2. Opsonization
3. Agglutination
4. Immuno uorescence
5. Complement binding

36. Question
LAKSHMAN

During an experiment the myotatic reflex has been studied in frogs. After
extension in a skeletal muscle its reflectory contraction was absent. The
reason for it might be a dysfunction of the following receptors:

LAKSHMAN
1. Golgi tendon organs
2. Tactile
3. Nonriceptors
4. Articular
5. Muscle spindles

37. Question
To prevent long-term effects of 4-day malaria a 42-year-old patient was
prescribed primaquine. On the 3-rd day from the begin of treatment there
appeared stomach and heart pains, dyspepsia, general cyanosis,
hemoglobinuria. What caused side effects of the preparation?

1. Drug potentiation by other preparations


2. Delayed urinary excretion of the preparation
3. Genetic insuf ciency of glucose 6-phosphate dehydrogenase
4. Cumulation of the preparation
5. Decreased activity of microsomal liver enzymes

38. Question
While determining power inputs of a patient’s organism it was established
that the respiratory coefficient equaled 1.0. This means that in the cells of the
patient the following substances are mainly oxidized:

1. Fats
2. Proteins
3. Carbohydrates
4. Proteins and carbohydrates
5. Carbohydrates and fats

39. Question
During an experiment the dorsal roots of the spinal cord of an animal have
been cut. What changes will be observed in the innervation zone?

1. Sensitivity loss and loss of motor functions


2. Loss of motor functions
3. Increase in muscle tone
4. Sensitivity loss
5. Decrease in muscle tone

40. Question
As a result of damage to certain structures of brainstem an animal lost
orientation reflexes. What structures were damaged?

1. Vestibular nuclei
2. Black substance
3. Red nuclei
4. Medial nuclei of reticular formation
5. Quadritubercular bodies
LAKSHMAN

41. Question
A 60 year old patient was found to have a dysfunction of main digestive
enzyme of saliva. This causes the disturbance of primary hydrolysis of:

LAKSHMAN
1. Carbohydrates
2. Lactose
3. Fats
4. Cellulose
5. Proteins

42. Question
In a healthy adult speed of the excitement conduction through the
atrioventricular node is 0.02-0.05 m/sec. Atrioventricular delay enables:

1. Sequence of atrial and ventricular contractions


2. Suf cient force of ventricular contractions
3. Suf cient force of atrial contractions
4. Simultaneity of both atria contractions
5. Simultaneity of both ventricles contractions

43. Question
Microelectrode technique allowed to register a potential following “all-or-
none” law and being able of undecremental spreading. Specify this potential:

1. Inhibitory postsynaptic potential


2. Receptor potential
3. Rest potential
4. Action potential
5. Excitatory postsynaptic potential

44. Question
Glomerular filtration rate (GFR) increased for 20% due to prolonged
starvation of the person. The most likely cause of filtration changes under this
conditions is:

1. Increase of systemic blood pressure


2. Decrease of oncotic pressure of blood plasma
3. Increase of penetration of the renal lter
4. Increase of renal plasma stream
5. Increase of ltration coef cient

45. Question
In course of an experiment thalamocortical tracts of an animal were cut.
What type of sensory perception remained intact?

1. Exteroreceptive
2. Olfactory
3. Auditory
4. Visual
5. Nociceptive
LAKSHMAN

46. Question
A man has normal sensitivity of his finger skin, however he doesn’t sense his
wedding ring around the finger. What process induced by wearing of the ring
has caused this phenomenon?

LAKSHMAN
1. Development of the brous tissue
2. Impaired circulation
3. Abnormality of the receptor structure
4. Abnormality of the epidermis structure
5. Receptor adaptation

47. Question
A patient with disturbed cerebral circulation has problems with deglutition.
What part of brain was damaged?

1. Midbrain
2. Interbrain
3. Cervical part of spinal cord
4. Forebrain
5. Brainstem

48. Question
Lung ventilation in a person is increased as a result of physical activity.
Which of the following indices of the external respiration is much higher than
in a state of rest?

1. Total lung capacity


2. Inspiratory reserve volume
3. Respiratory volume
4. Vital capacity of lungs
5. Expiratory reserve volume

49. Question
In course of an experiment a toads right labyrinth was destroyed. It will
cause amyotonia of the following muscles:

1. Right and left extensors


2. Left exors
3. Left extensors
4. Right extensors
5. Right exors

50. Question
A cardiac electric stimulator was implanted to a 75 year old man with heart
rate of 40 bpm. Thereafter the heart rate rose up to 70 bpm. The electric
stimulator has undertaken the function of the following heart part:

1. Atrioventricular node
2. Sinoatrial node
3. His bundle branches
4. His bundle bers
LAKSHMAN

5. Purkinjes bers

51. Question
When measuring power inputs of a man by the method of indirect calorimetry
the following results were obtained: 1000 ml oxygen consumption and 800 ml

LAKSHMAN
carbon dioxide liberation per minute. The man under examination has the
following respiratory coefficient:

1. 0.84
2. 1.25
3. 0.9
4. 0.8
5. 1.0

52. Question
According to audiometry data a patient has a disturbed perception of
medium-frequency sounds. It might have been caused by a damage of:

1. Middle part of helix


2. Quadritubercular structure
3. Spiral ganglion
4. Lateral geniculate bodies
5. Cochlear nuclei

53. Question
Lungs of a preterm infant have areas of atelectasis (pulmonary collapse). The
main cause is:

1. Increased viscous resistance


2. Surfactant excess
3. Underdeveloped inspiration muscles
4. Surfactant de ciency
5. Diminished force of surface tension of lungs

54. Question
A 32-year-old patient consulted a doctor about the absence of lactation after
parturition. Such disorder might be explained by the deficit of the following
hormone:

1. Somatotropin
2. Thyrocalcitonin
3. Prolactin
4. Vasopressin
5. Glucagon

55. Question
A patient after hypertension stroke does not have voluntary movements in his
right arm and leg with the increased muscle tone in these extremites. What
type of disfunction of nervous system is it?

1. Peripheral paresis
2. Peripheral paralysis
3. Re ex paresis
LAKSHMAN

4. Central paralysis
5. Central paresis

56. Question

LAKSHMAN
Stimulation of an excitable cell by the electric current has led to the
depolarization of its membrane. The depolarization has been caused mainly
by the following ions penetrating into the cell through its membrane:

1. Na+
2. Ca2+
3. K+
4. Cl-
5. HCO3-

57. Question
Examination of a patient revealed hyperkaliemia and hyponatremia. Low
secretion of which hormone may cause such changes?

1. Parathormone
2. Natriuretic
3. Vasopressin
4. Aldosterone
5. Cortisol

58. Question
A man has a considerable decrease in diuresis as a result of 1.5 l blood loss.
The primary cause of such diuresis disorder is the hypersecretion of the
following hormone:
1. Parathormone
2. Natriuretic
3. Vasopressin
4. Corticotropin
5. Cortisol

59. Question
During preparation of a patient to a heart surgery it was necessary to
measure pressure in heart chambers. In one of them pressure varied from 0
mm Hg up to 120 mm Hg within one cardiac cycle. What heart chamber is it?

1. Left ventricle
2. Right ventricle
3. Left atrium
4. Right atrium

60. Question
When water affects mucous membrane of the inferior nasal meatuses, this
causes “diver reflex” that provokes:

1. Cough
2. Re ectrory dyspnea
LAKSHMAN

3. Re ectory hyperpnea
4. Re ectory apnea
5. Bronchospasm

LAKSHMAN
61. Question
People adapted to high external temperatures have such peculiarity: profuse
sweating is not accompanied by loss of large volumes of sodium chloride. It is
caused by the effect of the following hormone upon the perspiratory glands:

1. Cortisol
2. Aldosterone
3. Natriuretic
4. Thyroxin
5. Vasopressin

62. Question
After destruction of CNS structures an animal lost orientative reflexes. What
structure was destroyed?

1. Lateral vestibular nuclei


2. Quadrigeminal plate
3. Black substance
4. Red nucleus
5. Medial reticular nuclei

63. Question
A patient has a haemorrhage into the posterior central gyrus. What type of
sensitivity on the opposite side will be disturbed?

1. Visual
2. Olfactory
3. Auditory and visual
4. Skin and proprioceptive
5. Auditory

64. Question
Atria of an experimental animal were superdistended by blood that resulted
in decreased reabsorption of Na+ and water in renal tubules. This can be
explained by the influence of the following factor upon kidneys:

1. Angiotensin
2. Renin
3. Vasopressin
4. Aldosterone
5. Natriuretic hormone

65. Question
Examination of a patient revealed a strong, balanced, inert type of higher
nervous activity according to Pavlov. What temperament type does the
patient have (according to Hippocrates classification)?

1. Melancholic
2. Sanguine
LAKSHMAN

3. Choleric
4. Phlegmatic

66. Question

LAKSHMAN
Examination of an isolated cardiomyocyte revealed that it did not generate
excitation impulses automatically. This cardiomyocyte was obtained from:

1. His bundle
2. Purkinjes bers
3. Atrioventricular node
4. Sinoatrial node
5. Ventricles

67. Question
Rest potential of a cell equals -80 mV. At what stage of action potential did the
membrane potential equal +30 mV?

1. After hyperpolarization
2. Depolarization
3. After depolarization
4. Reverse polarization

68. Question
A patient has a decreased vasopressin synthesis that causes polyuria and as
a result of it evident organism dehydratation. What is the mechanism of
polyuria development?

1. Reduced tubular reabsorption of protein


2. Reduced tubular reabsorption of Na ions
3. Acceleration of glomerular ltration
4. Reduced tubular reabsorption of water
5. Reduced glucose reabsorption

69. Question
Systemic arterial pressure of an adult dropped from 120/70 to 90/50 mm Hg
that led to reflectory vasoconstriction. The vasoconstriction will be maximal
in the following organ:

1. Bowels
2. Brain
3. Adrenals
4. Kidneys
5. Heart

70. Question
An aged man had raise of arterial pressure under a stress. It was caused by
activation of:

1. Parasympathetic nucleus of vagus


2. Functions of thyroid gland
3. Sympathoadrenal system
LAKSHMAN

4. Hypophysis function
5. Functions of adrenal cortex

71. Question
A 35 year old man consulted a dentist about reduced density of dental tissue,

LAKSHMAN
high fragility of teeth during eating solid food. This patient suffers the most
probably from the deficiency of the following mineral element:

1. Magnesium
2. Calcium
3. Sodium
4. Potassium
5. Iron

72. Question
The ventral roots of 5 frontal segment of spinal cord were cut during
experiment in the animal. What changes will take place in the innervation
region?

1. Loss of proprioceptive sensitivity


2. Hypersensitivity
3. Loss of touch sensitivity
4. Loss of movements
5. Loss of temperature sensitivity

73. Question
A patient came to the hospital complaining about quick fatigability and
apparent muscle weakness. Examination revealed an autoimmune disease
that causes disorder of functional receptor condition in neuromuscular
synapses. What transmitter will be blocked?
1. Glycine
2. Serotonin
3. Acetylcholine
4. Noradrenalin
5. Dopamine

74. Question
The minute blood volume in a patient with transplanted heart has increased
as a result of physical activity. What regulative mechanism is responsible for
these changes?

1. Parasympathetic conditioned re exes


2. Sympathetic unconditioned re exes
3. Catecholamines
4. Sympathetic conditioned re exes
5. Parasympathetic unconditioned re exes

75. Question
A big dose of histamine introduction to an experimental animal caused
abrupt drop of arterial pressure as a result of:

1. Increase of heart rate


LAKSHMAN

2. Decrease of heart rate and force


3. Constriction of resistance vessels
4. Decrease of heart rate
5. Dilatation of resistance vessels

LAKSHMAN
76. Question
During influenza epidemic 40% of pupils who didn’t go in for sports were
affected by the disease, and among the pupils who regularly did physical
exercises this index was only 20%. What adaptative mechanisms determined
such a low sickness rate of pupils participating in the sports?

1. Cross adaptation
2. Speci c adaptation
3. Physiological adaptation
4. Genetic adaptation
5. Biochemical adaptation

77. Question
A 30 year old woman has subnormal concentration of enzymes in the
pancreatic juice. This might be caused by the hyposecretion of the following
gastrointestinal hormone:

1. Vaso-intestinal peptide
2. Gastro-inhibiting peptide
3. Secretin
4. Somatostatin
5. Cholecystokinin-pancreozymin

78. Question
A student is thoroughly summarising a lecture. When his groupmates begin
talking the quality of the summarising worsens greatly. What type of
inhibition in the cerebral cortex is the cause of it?

1. Dying
2. External
3. Delayed
4. Differential
5. Protective

79. Question
A 49 year old woman spent a lot of time standing. As a result of it she got leg
edema. What is the most likely cause of the edema?

1. Increase in oncotic pressure of blood plasma


2. Increase in systemic arterial pressure
3. Decrease in hydrostatic pressure of blood in arteries
4. Decrease in hydrostatic pressure of blood in veins
5. Increase in hydrostatic pressure of blood in veins

80. Question
As a result of continuous starvation the glomerular filtration rate has
increased by 20%. The most probable cause of the glomerular filtration
LAKSHMAN

alteration under the mentioned conditions is:

1. Increase of the ltartion quotient


2. Decrease in the oncotic pressure of blood plasma

LAKSHMAN
3. Increase in the systemic arterial pressure
4. Increase of the renal blood ow
5. Increase in the permeability of the renal lter

81. Question
A 35 year old man got an injury that caused complete disruption of spinal
cord at the level of the first cervical segment. What respiration changes will
be observed?

1. Thoracic respiration will be maintained, diaphragmal respiration will disappear


2. It will become infrequent and deep
3. No changes will be observed
4. It will come to a standstill
5. Diaphragmal respiration will be maintained, thoracic respiration will disappear

82. Question
A 16 year old boy after an illness has diminished function of protein synthesis
in liver as a result of vitamin K deficiency. It will cause disturbance of:

1. Erythropoietin secretion
2. Blood coagulation
3. Erythrocyte sedimentation rate
4. Anticoagulant generation
5. Osmotic blood pressure
83. Question
Osmotic pressure of a mans blood plasma is 350 mosmole/l (standard
pressure is 300 mosmole/l). First of all it will result in high secretion of the
following hormone:

1. Natriuretic
2. Cortisol
3. Adrenocorticotropin
4. Aldosteron
5. Vasopressin

84. Question
A hypertensive glucose solution was introduced to a patient. It will intensify
water movement:

1. From the capillaries to the intercellular liquid


2. From the cells to the intercellular liquid
3. From the intercellular liquid to the capillaries
4. There will be no changes of water movement
5. From the intercellular liquid to the cells

85. Question
Isolated muscle of a frog is rhythmically irritated with electric impulses.
LAKSHMAN

Every next impulse is in a period of relaxation from the previus contraction.


What contraction of the muscle appears?

1. Single

LAKSHMAN
2. Continuous(smooth) tetanus
3. Tonic
4. Asynchronous
5. Waved tetanus

86. Question
A patient presented to a hospital with complaints about quick fatigability and
significant muscle weakness. Examination revealed an autoimmune disease
that causes functional disorder of receptors in the neuromuscular synapses.
This will result in the disturbed activity of the following mediator:

1. Serotonin
2. Noradrenaline
3. Dopamine
4. Glycine
5. Acetylcholine

87. Question
Secretion of which gastrointestinal hormones is primerily decreased in
patient with removed duodenum?

1. Gastrin and histamine


2. Gastrin
3. Neurotensin
4. Histamine
5. Cholecystokinin and secretin

88. Question
A clinic observes a 49 year old patient with significant prolongation of
coagulation time, gastrointestinal haemorrhages, subcutaneous hematomas.
These symptoms might be explained by the deficiency of the following vitamin:

1. B1
2. B6
3. K
4. E
5. H

89. Question
Students who are taking examinations often have dry mouth. The mechanism
that causes this state is the realization of the following reflexes:

1. Unconditioned parasympathetic
2. Unconditioned peripheral
3. Unconditioned sympathetic
4. Conditioned parasympathetic
5. Conditioned sympathetic
LAKSHMAN

90. Question
A patient has a disturbed absorbtion of fat hydrolysates. It might have been
caused by a deficit in the small intestine cavity:

LAKSHMAN
1. Of lipolytic enzymes
2. Of sodium ions
3. Of bile acids
4. Of bile pigments
5. Of liposoluble vitamins

91. Question
In response to a change in body position from horizontal to vertical blood
circulation system develops reflectory pressor reaction. Which of the
following is its compulsory component?

1. Systemic constriction of the venous vessels


2. Decrease in the circulating blood volume
3. Systemic dilatation of the arterial resistive vessels
4. Weakening of the pumbing ability of heart
5. Increase in the heart rate

92. Question
A middle-aged man went to a foreign country because he had been offered a
job there. However he had been unemployed for quite a long time. What
endocrine glands were exhausted most of all in this man?

1. Seminal glands
2. Parathyroid glands
3. Thyroid gland
4. Substernal gland
5. Adrenal glands

93. Question
In the experiment on the animal the part of the cerebral cortex hemispheres
was removed. It caused elimination of previously formed conditioned reflex
to the light irritation. What part of the cortex was removed?

1. Limbic cortex
2. Postcentral convolution
3. Precentral convolution
4. Occipital cortex
5. Temporal lobe

94. Question
The high level of Lactate Dehydrogenase (LDH) isozymes concentration
showed the increase of LDH-1 and LDH-2 in a patient’s blood plasma. Point out
the most probable diagnosis:

1. Viral hepatitis
2. Myocardial infarction
3. Acute pancreatitis
4. Skeletal muscle dystrophy
LAKSHMAN

5. Diabetes mellitus

95. Question
A 17-year-old boy fell seriously ill, body temperature rose up to 38.50C?, there

LAKSHMAN
is cough, rhinitis, lacrimation, nasal discharges. What kind of inflammation
is it?

1. Hemorrhagic in ammation
2. Fibrinous in ammation
3. Catarrhal in ammation
4. Suppurative in ammation
5. Serous in ammation

96. Question
A patient is 44 years old. Laboratory examination of his blood revealed that
content of proteins in plasma was 40 g/l. What influence will be exerted on the
transcapillary water exchange?

1. Both ltration and reabsorption will be increased


2. Filtration will be increased, reabsorption - decreased
3. Filtration will be decreased, reabsorption - increased
4. Exchange will stay unchanged
5. Both ltration and reabsorption will be decreased

97. Question
In course of an experiment a skeletal muscle is being stimulated by a series of
electric impulses. What type of muscle contraction will arise, if every
subsequent impulse comes in the period of relaxation of single muscle
contraction?
1. Muscle contracture
2. Asynchronous tetanus
3. Partial tetanus
4. Holotetanus
5. A series of single contractions

98. Question
A 30-year-old woman was diagnosed with insufficiency of exocrinous function
of pancreas. Hydrolisis of what nutrients will be disturbed?

1. Proteins
2. Proteins, fats, carbohydrates
3. Proteins, fats
4. Fats, carbohydrates
5. Proteins, carbohydrates

99. Question
An isolated muscle fiber is under examination. It was established that the
threshold of stimulation force became significantly lower. What is the cause
of this phenomenon?

1. Activation of sodium channels of membrane


LAKSHMAN

2. Activation of potassium channels of membrane


3. Block of energy production in the cell
4. Inactivation of sodium channels of membrane
5. Inactivation of potassium channels of membrane

LAKSHMAN
100. Question
To anaesthetize the manipulation related to burn surface treatment, a patient
was intravenously injected a medication for short-acting narcosis. 1 minute
later the patient being under anaesthesia had increased blood pressure,
tachycardia, increased tone of sceletal muscles; reflexes remained. After
awakening the patient had desorientation and visual hallucinations. What
medication was the patient injected?

1. Ketamine
2. Thiopental sodium
3. Nitrous oxide
4. Sombrevin
5. Diethyl ether

101. Question
Patient with hypersecretion of the gastric juices was recomended to exclude
from the diet concentrated bouillons and vegetable decoctions because of
their stimulation of gastric secretion. What is dominating mechanism of
stimulation of secretion in this case?

1. Irritation of mechanoreceptors of the oral cavity


2. Irritation of taste receptors
3. Stimulation of excretion of secretin in the duodenum
4. Irritation of mechanoreceptors of the stomach
5. Stimulation of gastrin production by G-cells
102. Question
As a result of destruction of certain brainstem structures an animal has lost
its orientative reflexes in response to strong photic stimuli. What structures
were destroyed?

1. Posterior tubercles of quadrigeminal plate


2. Vestibular nuclei
3. Black substance
4. Red nuclei
5. Anterior tubercles of quadrigeminal plate

103. Question
Examination of a pregnant woman revealed twice as much concentration of
fibrinogen in blood plasm. What ESR can this woman have?

1. 5-10 mm/h
2. 0-5 mm/h
3. 2-12 mm/h
4. 40-50 mm/h
5. 10-15 mm/h

104. Question
LAKSHMAN

Inhabitants of territories with cold climate have high content of an adaptive


thermoregulatory hormone. What hormone is meant?

1. Insulin

LAKSHMAN
2. Glucagon
3. Somatotropin
4. Cortisol
5. Thyroxin

105. Question
A man’s intrapleural pressure is being measured. In what phase did the man
hold his breath, if his pressure is 7.5 cm Hg?

1. Forced inspiration
2. Quiet inspiration
3. Forced expiration
4. Quiet expiration

106. Question
An isolated cell of human heart automatically generates excitation impulses
with frequency 60 times pro minute. What structure does this cell belong to?

1. Atrioventricular node
2. His bundle
3. Atrium
4. Ventricle
5. Sinoatrial node

107. Question
Long-term starvation cure of a patient resulted in diminished ratio of
albumines and globulines in plasma. What of the following will be result of
these changes?

1. Decrease of hematocrit
2. Hypercoagulation
3. Decrease of ESR
4. Increase of hematocrit
5. Increase of ESR

108. Question
A month after surgical constriction of rabbits renal artery the considerable
increase of systematic arterial pressure was observed. What of the following
regulation mechanisms caused the animals pressure change?

1. Vasopressin
2. Serotonin
3. Adrenaline
4. Noradrenaline
5. Angiotensin-II

109. Question
A 38-year-old woman was admitted to the admission-diagnostic department
LAKSHMAN

with uterine bleeding. What are the most likely changes of blood?

1. Increase of haematocrite rate


2. Leucocytosis

LAKSHMAN
3. Polycythemia
4. Leukopenia
5. Reduction of haematocrite rate

110. Question
A 42 year old patient complains of pain in the epigastral area, vomiting; vomit
masses have the colour of “coffee-grounds”, the patient has also melena.
Anamnesis records gastric ulcer. Blood formula: erythrocytes – 2.81012l,
leukocytes – 8*109l, Hb- 90 g/l. What complication is it?

1. Canceration
2. Pyloric stenosis
3. Haemorrhage
4. Penetration
5. Perforation

111. Question
Due to action of electric current on the exitable cell there appeared
depolarization of its membrane. Movement of what ions through the
membrane caused depolarisation?

1. Ca2+
2. Na+
3. K+
4. Cl-
5. HCO3-

112. Question
A patient who has been treated with diazepam on account of neurosis
complains of toothache. Doctor administered him an analgetic, but its dose
was lower than average therapeutic dose. What phenomenon did the doctor
take into account while prescribing the patient an underdose?

1. Drug dependence
2. Potentiation
3. Tolerance
4. Cumulation
5. Summation

113. Question
Packed cell volume of a man was 40% before the trauma. What packed cell
volume will be observed 24 hours after blood loss of 750 ml?

1. 45%
2. 40%
3. 55%
4. 50%
5. 30%
LAKSHMAN

114. Question
Accelerated frequency of the heart rate and increased blood pressure were
marked in the sportsman on the start before the competitions. Influence of

LAKSHMAN
what parts of the CNS can explain these changes?

1. Cortex of the large hemispheres*=


2. Hypothalamus
3. Medulla
4. Diencephalon
5. Mesencephalon

115. Question
On blood grouping on the system ABO, standart serum of the I and II groups
caused erythrocytes agglutination of the examined blood and serum group of
the III did not. What agglutinogens are in this erythrocytes?

1. C
2. B
3. A
4. D and C
5. A and B

116. Question
Vagi of an experimental animal were cut on both sides. Which respiration
changes will be observed?

1. It will become deep and infrequent


2. It will become deep and frequent
3. It will become shallow and frequent
4. It will become shallow and infrequent
5. No changes will be observed

117. Question
Vagus branches which innervate the heart are being stimulated during an
experiment. It caused reduction of heart rate due to the intensification of the
following process (through the cell membrane of cardiac pacemaker):

1. Calcium and potassium ion yield


2. Potassium ion entry
3. Calcium ion yield
4. Calcium ion entry
5. Potassium ion yield

118. Question
Child asked you to puff up the balloon as much as possible for a one
exhalation. What air volume will you use?

1. Inspiration volume
2. Backup volume of the inspiration
3. Functional residual volume
LAKSHMAN

4. Vital volume of the lungs


5. Total volume of the lungs

119. Question
Vagus branches that innervate heart are being stimulated in course of an

LAKSHMAN
experiment. As a result of it the excitement conduction from atria to the
ventricles was brought to a stop. It is caused by electrophysical changes in the
following structures:

1. Ventricles
2. His bundle
3. Atria
4. Atrioventricular node
5. Sinoatrial node

120. Question
A patient staggers and walks astraddle. He has hypomyotonia of arm and leg
muscles, staccato speech. In what brain section is this affection localized?

1. Putamen
2. Motor cortex
3. Red nucleus
4. Cerebellum
5. Caudate nucleus

121. Question
On experiment on the dog the peripheral part of nervus vagus of the neck was
irritated. What changes of the heart function would be observed?

1. Increased myocardial excitability


2. Decreased contraction rate
3. Increased atrioventricular conduction
4. Increased contraction force and rate
5. Increased contraction force

122. Question
ECG of a patient with hyperfunction of thyroid gland showed heart hurry. It is
indicated by depression of the following ECG element:

1. QRS complex
2. P-Q segment
3. R-R interval
4. P-T interval
5. P-Q interval

123. Question
What heat transfer mechanism is the most effective while the man being at
80% of air moisture and the temperature +350C ?

1. Heat conduction
2. Radiation
3. Evaporation
LAKSHMAN

4. Convection

124. Question
A man was intoxicated with mushrooms. They contain muscarine that
stimulates muscarinic cholinoreceptors. What symptoms signalize

LAKSHMAN
intoxication with inedible mushrooms?

1. Increased heart rate


2. Mydriatic pupils
3. Rise of arterial pressure
4. Bronchi dilatation
5. Myotic pupils

125. Question
A pregnant woman had her blood group identified. Reaction of erythrocyte
agglutination with standard serums of 0a/b (I), Ba(III) groups didn’t proceed
with standard serum of Ab (II) group. The blood group under examination is:

1. Ba (III)
2. 0a/b (I)
3. Ab (II)
4. AB (IV)

126. Question
There is only one hormone among the neurohormones which refers to the
derivatives of amino acids according to classification. Point it out:

1. Vasopressin
2. Thyroliberin
3. Melatonin
4. Oxytocin
5. Somatotropin

127. Question
Blood group of a 30 year old man was specified before an operation. His blood
is Rh-positive. Reaction of erythrocyte agglutination was absent with
standard sera of 0(I), A(II), B(III) groups. The analysed blood is of the
following group:

1. B (III)
2. 0 (I)
3. AB (IV)
4. A (II)

128. Question
A man presents with increased heart rate, mydriatic pupils, dry mouth. This
condition results from the activation of the following system of function
regulation:

1. Metasympathetic
2. Parasympathetic
3. Sympathetic
4. Vago-insular
LAKSHMAN

5. Hypothalamo-pituitary-adrenal

129. Question
Respiratory coefficient was studied in the patient who strictly kept certain

LAKSHMAN
diet for 10 days. It was determined that it is 1. What diet does the patient
follow?

1. With domination of proteins and carbohydrates


2. With domination of carbohydrates
3. Mixed
4. With domination of proteins and fat
5. With domination of fat and carbohydrates

130. Question
A doctor asked a patient to breath out fully after taking a normal breath.
What muscles contract during such exhalation?

1. Trapezius muscles
2. Abdominal muscles
3. External intercostal muscles
4. Pectoral muscles
5. Diaphragm

131. Question
In course of an experiment a peripheral section of vagus of an expiremental
animal is being stimulated. What changes will be observed?

1. Pupil dilation
2. Heart rate fall
3. Heart hurry
4. Increase of respiration rate
5. Bronchi dilation

132. Question
A young woman who entered a production department where it strongly
smelt of paints and varnishes had a bronchospasm. This reflex was caused by
irritation of the following receptors:

1. Pleura receptors
2. Central chemoreceptors
3. Peripheral chemoreceptors
4. Irritant
5. Juxtaglomerular

133. Question
An animal has an increased tonus of extensor muscles. This the result of
intensified information transmission to the motoneurons of the spinal cord
through the following descending pathways:

1. Medial corticospinal
2. Lateral corticospinal
3. Rubrospinal
LAKSHMAN

4. Vestibulospinal
5. Reticulospinal

134. Question

LAKSHMAN
Which muscle contraction will be observed in the upper extremity during
holding (not moving) a load in a certain position?

1. Auxotonic
2. Isometric
3. Isotonic
4. Excentric
5. Concentric

135. Question
Usage of oral contraceptives with sex hormones inhibits secretion of the
hypophysiae hormones. Secretion of which of the indicated hormones is
inhibited while using oral contraceptives with sex hormones?

1. Somatotropic
2. Follicle-stimulating
3. Vasopressin
4. Oxytocin
5. Thyrotropic

136. Question
The preventive radioprotector was given to a worker of a nuclear power
station. What mechanism from the below mentioned is considered to be the
main mechanism of radioprotection?
1. Inhibition of free radicals formation
2. Activation of oxidation reactions
3. Increasing of tissue blood supply
4. Increasing of respiration
5. Prevention of tissue’s hypoxia

137. Question
ECG of a patient shows prolongation of T-wave. This is caused by deceleration
in ventricles of:

1. Repolarization
2. Depolarization
3. Depolarization and repolarization
4. Relaxation
5. Contraction

138. Question
The sterile Petri dishes and pipettes are necessary to prepare for
microbiological tests in bacteriological laboratory. What way of sterilization
should be applied in this case?

1. Boiling
LAKSHMAN

2. Pasteurization
3. Tyndallization
4. Steam sterilization in autoclave
5. Dry-heat sterilization

LAKSHMAN
139. Question
A patient consumed a lot of reach in proteins food that caused increase of
rate of proteolytic enzymes of pancreatic juice. It is also accompanied by
increase of rate of the following enzyme:

1. Pepsin
2. Enterokinase
3. Gastricsin
4. Renin
5. Tripsin

140. Question
Parents of a 10 y.o. boy consulted a doctor about extension of hair-covering,
growth of beard and moustache, low voice. Intensified secretion of which
hormone must be assumed?

1. Of progesterone
2. Of oestrogen
3. Of somatotropin
4. Of testosterone
5. Of cortisol

141. Question
A human body cools in water much faster that in the air. What way of heat
emission in water is much more efficient?
1. Sweat evaporation
2. Convection
3. Heat radiation
4. Heat conduction

142. Question
A patient under test was subjected to a moderate physical stress. His minute
blood volume amounted 10 l/min. What blood volume was pumped through his
lung vessels every minute?

1. 4 l/min
2. 6 l/min
3. 10 l/min
4. 5 l/min
5. 7 l/min

143. Question
Voluntary breath-holding caused increase of respiration depth and frequency.
The main factor stimulating these changes of external respiration is:

1. Decreased tension of CO2 in blood


2. Increased tension of CO2 in blood
LAKSHMAN

3. Increased tension of O2 in blood


4. Decreased tension of O2 in blood
5. Decreased concentration of H+ in blood

LAKSHMAN
144. Question
If a man has an attack of bronchiospasm it is necessary to reduce the effect of
vagus on smooth muscles of bronchi. What membrane cytoreceptors should
be blocked for this purpose?

1. beta-adrenoreceptors
2. alpha- and beta-adrenoreceptors
3. N-cholinoreceptors
4. alpha-adrenoreceptors
5. M-cholinoreceptors

145. Question
A 60-year-old patient presents with weakened peristaltic activity of the
bowels. Which of the following foodstuffs would stimulate peristalsis most of
all?

1. Lard
2. Tea
3. White bread
4. Meat
5. Brown bread

146. Question
A patient has delayed conduction of excitement through the atrioventricular
node. What changes of ECG will be observed?
1. Prolongation of Q-S interval
2. Prolongation of Q-T interval
3. S-T-segment displacement
4. Prolongation of P-Q interval
5. Negative T wave

147. Question
On examination of the person it was revealed that minute volume of heart is
3500 mL, systolic volume is 50 mL. What is the frequency of cardiac
contraction?

1. 60 bpm
2. 50 bpm
3. 90 bpm
4. 70 bpm
5. 80 bpm

148. Question
A man who went for a ride on a roundabout had amplification of heart rate,
sweating and nausea. What receptors stimulation is it primarily connected
with?

1. Proprioceptors
LAKSHMAN

2. Auditory
3. Visual
4. Tactors

LAKSHMAN
5. Vestibular

149. Question
It was established that agglutination of the recipient’s blood erythrocytes had
been caused by the standard sera from the I and II groups. Serum from the III
group as well as anti-Rh serum hadn‘t provoke any agglutination. Which
blood group and rhesus is allowed to be transfused this recipient?

1. 0 ab (I) Rh+
2. Ab (II) Rh-
3. AB (IV), Rh+
4. AB (IV), Rh-
5. aB (III) Rh-

150. Question
A patient presents with the following motor activity disturbances: tremor,
ataxia and asynergia movements, dysarthria. The disturbances are most
likely to be localized in:

1. Medulla oblongata
2. Cerebellum
3. Limbic system
4. Brainstem
5. Basal ganglions

151. Question
Heart rate of a man permanently equals to 40 beats per minute. What is the
pacemaker?

1. Atriventricular node
2. His bundle
3. Purkinjes bers
4. His bundle branches
5. Sinoatrial node

152. Question
Due to activation of ion channels of external membrane of excitable cell its
rest potential has significantly increased. What channels were activated?

1. Natrium channels
2. Fast calcium channels
3. Potassium channels
4. Slow calcium channels
5. Natrium and calcium channels

153. Question
A man took a quiet expiration. Name an air volume that is meanwhile
contained in his lungs:
LAKSHMAN

1. Vital lung capacity


2. Residual volume
3. Expiratory reserve volume

LAKSHMAN
4. Functional residual capacity
5. Respiratory volume

154. Question
A man weighs 80 kg, after long physical activity his circulating blood volume
is reduced down to 5.4 l, hematocrit makes up 50%, whole blood protein is 80
g/l. These blood characteristics are determined first of all by:

1. Increased circulating blood volume


2. Increased diuresis
3. Water loss with sweat
4. Increased number of erythrocytes
5. Increased protein concentration in plasm

155. Question
Due to cranial trauma the patient developed the symptoms: intention tremor,
dysmetry, adiadochokinesis, dysarthria. What structure of the brain is
injured?

1. Striatum
2. Black substance
3. Cerebellum
4. Pale sphere
5. Motor cortex

156. Question
Examination of a man established that cardiac output equaled 3500 ml,
systolic output – 50 ml. What is the man’s heart rate pro minute?

1. 90
2. 60
3. 50
4. 70
5. 80

157. Question
After a surgery a 36-year-old woman was given an intravenous injection of
concentrated albumin solution. This has induced intensified water movement
in the following direction:

1. From the capillaries to the intercellular uid


2. No changes of water movement will be observed
3. From the intercellular uid to the cells
4. From the intercellular uid to the capillaries
5. From the cells to the intercellular uid

158. Question
Surface with an intact toad on it was inclined to the right. Tone of extensor
muscles became reflectory higher due to the activation of the following
LAKSHMAN

receptors:

1. Proprioreceptors
2. Vestibuloreceptors of semicircular ducts

LAKSHMAN
3. Vestibuloreceptors of utricle and saccule
4. Photoreceptors of retina
5. Mechanoreceptors of foot skin

159. Question
Workers of a hothouse farm work under conditions of unfavourable
microclimate: air temperature is +37oC, relative humidity is 90%, air speed is
0.2 m/s. The way of heat emission under these conditions will be:

1. Evaporation
2. All the ways
3. Radiation
4. Heat conduction
5. Convection

160. Question
Short-term physical activity resulted in reflex amplification of heart rate and
raise of systemic arterial pressure. What receptors activation was the main
cause of pressor reflex realization?

1. Vascular chemoreceptors
2. Vascular baroceptors
3. Vascular volume receptors
4. Hypothalamus thermoreceptors
5. Proprioreceptors of active muscles
161. Question
Examination of a 35 year old patient revealed high acidity of gastric juice.
What receptors should be blocked in order to reduce it?

1. b1-adrenoreceptors
2. a2-adrenoreceptors
3. Histamine
4. b2-adrenoreceptors
5. a1-adrenoreceptors

Created by Eneutron Team

Kiev eneutron.info@gmail.com
LAKSHMAN

LAKSHMAN
Krok 1 – 2013 Base(Microbiology)

Medical PG WhatsApp Group

Results
0 of 85 questions answered correctly

Your time: 00:02:54

You have reached 0 of 85 points, (0%)

RESTART QUIZ VIEW QUESTIONS

1. Question
A patient with clinical presentations of immunodeficiency went through
immunological examinations. They revealed significant loss of cells that form
rosettes with erythrocytes of a ram. What conclusion can be made according
to the analysis data?

1. Decrease of B-lymphocytes rate


2. Decrease of complement system rate
3. Insuf ciency of effector cells of humoral immunity
4. Decrease of natural killer cell rate
5. Decrease of T-lymphocytes rate

2. Question
LAKSHMAN

A patient recovered from Sonne dysentery and was once more infected with
the same causative agent. What is such infection form called?

LAKSHMAN
1. Superinfection
2. Recidivation
3. Chronic infection
4. Reinfection
5. Persisting infection

3. Question
There was a record of some anthrax cases among animals in a countryside.
The spread of disease can be prevented by means of immunization. What kind
of vaccine should be used?

1. Salk vaccine
2. Sabins vaccine
3. BCG vaccine
4. Diphteria and tetanus toxoids and pertussis vaccine
5. STI live vaccine

4. Question
Examination of a patient with pustular skin lesions allowed to isolate a
causative agent that forms in the blood agar roundish yellow middle-sized
colonies surrounded by haemolysis zone. Smears from the colonies contain
irregular-shaped clusters of gram-positive cocci. The culture is oxidase- and
catalase-positive, ferments mannitol and synthesizes plasmocoagulase. What
causative agent was isolated?

1. Streptococcus pyogenes
2. Staphylococcus epidermidis
3. Streptococcus agalactiae
4. Staphylococcus saprophyticus
5. Staphylococcus aureus

5. Question
A virological laboratory obtained pathological material (mucous discharges
from nasal meatuses) taken from a patient with provisional diagnosis
“influenza”. What quick test will allow to reveal specific viral antigen in the
material under examination?

1. Direct and indirect uorescence immunoassay


2. Radioimmunoassay
3. Direct and indirect immuno uorescence test
4. Hemagglutination inhibition assay

6. Question
Examination of a child revealed some whitish spots looking like coagulated
milk on the mucous membrane of his cheeks and tongue. Analysis of smears
revealed gram-positive oval yeast-like cells. What causative agents are they?

1. Actinomycetes
2. Diphtheria bacillus
LAKSHMAN

3. Fusobacteria
4. Staphylococci
5. Candida

LAKSHMAN
7. Question
Bacteriological laboratory examines canned meat whether it contains
botulinum toxin. For this purpose an extract of test specimen and antitoxic
antibotulinic serum of A, B, E types were introducted to a group of mice under
examination; a control group of mice got the extract without antibotulinic
serum. What serological reaction was applied?

1. Neutralization
2. Complement binding
3. Precipitation
4. Double immune diffusion
5. Opsono-phagocytic

8. Question
During the regular sanitary-epidemiological inspection of a pharmacy, the
bacteriological analysis of air was performed. The air was found to have
bacilli, yeast fungi, hemolytic streptococci, micrococci. Which of the detected
microorganisms indicate the direct epidemic danger?

1. Yeast fungi
2. Micrococci
3. Bacilli
4. Haemolytic streptococci

9. Question
A bacteriological laboratory received sputum sample of a patient suffering
from tuberculosis. Bacterioscopic examination of smears and detection of
tuberculosis bacillus can be realized by one of enrichment methods that
involves processing of sputum only with solution of caustic soda. What is this
method called?

1. Neutralization
2. Flotation
3. Inactivation
4. Filtration
5. Homogenization

10. Question
A man who was bitten by the unknown dog applied to the surgeon. Wide
ragged woundes were localised on the face. What curative-prophylactic aid
should be given to prevent rabies?

1. Hospitalize the patient and keep under the doctors supervision


2. Start immunisation with rabies vaccine
3. Prescribe combine antibiotic therapy
4. Immediate injection of DPT(Diphtheria, Pertusis, Tetanus) vaccine
5. Immediately inject normal gamma globulin
LAKSHMAN

11. Question
A duodenal content smear of a patient with indigestion contains protosoa 10-
18 mm large. They have piriform bodies, 4 pairs of filaments, two
symmetrically located nuclei in the broadened part of body. What kind of the
lowest organisms is it?

LAKSHMAN
1. Trichomonas
2. Balantidium
3. Dysentery ameba
4. Intestinal ameba
5. Lamblia

12. Question
Inoculum from pharynx of a patient ill with angina was inoculated into
blood-tellurite agar. It resulted in growth of grey, radially striated (in form of
rosettes) colonies 4-5 mm in diameter. Gram-positive bacilli with clublike
thickenings on their ends placed in form of spread wide apart fingers are
visible by microscope. What microorganisms are there?

1. Streptococci
2. Botulism clostridia
3. Diphtheroids
4. Diphtheria corynebacteria
5. Streptobacilli

13. Question
Clinical diagnosis of a female patient was gonorrhoea. What examination
method can be applied for confirmation of this diagnosis?

1. Hemagglutination reaction
2. Immobilization reaction
3. Infection of laboratory animals
4. Test with bacteriophage
5. Microscopy of pathological material

14. Question
Bacteriological examination of a patient with food poisoning required
inoculation of a pure culture of bacteria with the following properties: gram-
negative movable bacillus that grows in the Endos medium in form of
colourless colonies. A representative of which species caused this disease?

1. Iersinia
2. Shigella
3. Salmonella
4. Esherichia
5. Citrobacter

15. Question
A patient has been suffering from elevated temperature and attacks of typical
cough for 10 days. Doctor administered inoculation of mucus from the
patients nasopharynx on the agar. What microorganism is presumed?

1. Listeria
LAKSHMAN

2. Pfeiffers bacillus
3. Staphylococcus
4. Klebsiella

LAKSHMAN
5. Pertussis bacillus

16. Question
Microscopical examination of a microbal culture revealed fusiform spore-
forming microorganisms that get violet-blue Grams stain. What
microorganisms were revealed?

1. Diplococci
2. Actinomycete
3. Streptococci
4. Clostridia
5. Spirochaete

17. Question
While studying a microslide obtained from the punctuate of a regional lymph
node and stained by Romanovsky-Giemsa method a physician revealed some
light-pink thin microorganisms with 12-14 regular spiral coils and pointed
ends, up to 10-13 micrometer long. This might be the causative agent of the
following disease:

1. Relapsing fever
2. Leishmaniasis
3. Trypanosomiasis
4. Leptospirosis
5. Syphilis

18. Question
A patient suffering from periodical attacks caused by inhalation of different
flavoring substances was diagnosed with atopic bronchial asthma. IgE level
was increased. This is typical for the following type of reactions:

1. Delayed-type hypersensitivity
2. Autoimmune reactions
3. Cytotoxic reactions
4. Immunocomplex reactions
5. Anaphylactic reactions

19. Question
A patient with clinical signs of encephalitis was delivered to the infectious
diseases hospital. Anamnesis registers a tick bite. Hemagglutination-
inhibition reaction helped to reveal antibodies to the causative agent of tick-
borne encephalitis in the dilution 1:20 which is not diagnostic. What actions
should the doctor take after he had got such result?

1. To apply more sensitive reaction


2. To examine the same serum
3. To repeat examination with another diagnosticum
4. To deny diagnosis of tick-borne encephalitis
5. To repeat the examination with serum taken 10 days later
LAKSHMAN

20. Question
HIV has gp41 and gp120 on its surface interacts with target cells of an
organism. Which of the following human lymphocyte antigens is gp120
complementary bound with?

LAKSHMAN
1. CD 28
2. CD 4
3. CD 8
4. CD 19
5. CD 3

21. Question
A 65-year-old man has purulent abscess on his neck. Analyses revealed a
culture of gram-positive cocci with plasmocoagulase activity. This culture
relates most likely to:

1. Staphylococcus aureus
2. Staphylococcus saprophyticus
3. Staphylococcus epidermidis
4. Streptococcus pyogenes

22. Question
A 4-year-old child presents with general weakness, sore throat and deglutitive
problem. After his examination a doctor suspected diphtheria and sent the
material to the bacteriological laboratory. In order to determine the
diphtheria causative agent the material should be inoculated into the
following differential diagnostic medium:

1. Levenshtein-Yessen agar
2. Ploskyrevs agar
3. Endos agar
4. Blood tellurite agar
5. Sabourauds agar

23. Question
In order to determine toxigenicity of diphtheria bacilli a strip of filter paper
impregnated with antitoxic diphtherial serum was put on the dense nutrient
medium. There were also inoculated a studied microbal culture and a strain
that is known to be toxigenic. If the microbal culture under examination
produces exotoxin, it will result in formation of:

1. Precipitin ring
2. Zones of diffuse opaci cation
3. Precipitin lines
4. Zones of lecithovitellinous activity
5. Haemolysis zones

24. Question
A 16 y.o. boy from a countryside entered an educational establishment.
Scheduled Manteux test revealed that the boy had negative reaction. What are
the most reasonable actions in this case?
T.B
1. To repeat the reaction in a month
LAKSHMAN

2. To perform rapid Price diagnostics


3. To isolate the boy temporarily from his mates
4. To perform BCG vaccination
5. To perform serodiagnostics of tuberculosis

LAKSHMAN
25. Question
During the repeated Widals agglutination test it was noticed that the ratio of
antibody titers and O-antigens S.typhi in the patients serum had increased
from 1:100 to 1:400. How would you interpret these results?

1. The patient was previously vaccinated against typhoid fever


2. The patient has typhoid fever
3. The patient is a chronic carrier of typhoid microbes
4. The patient is an acute carrier of typhoid microbes
5. The patient previously had typhoid fever

26. Question
During examination of a patient a dentist revealed a lot of “white spots” –
zones of enamel demineralization. What microorganisms take part in the
development of this process?

1. Veilonella parvula
2. Staphylococcus epidermidis
3. Streptococcus pyogenes
4. Streptococcus mutans
5. Streptococcus salivarius

27. Question
From pharynx of a child with suspected diphtheria a pure culture of
microorganisms was isolated. Their morphological, tinctorial, cultural and
biochemical properties appeared to be typical for diphtheria causative
agents. What study should be conducted in order to draw a conclusion that
this is a pathogenic diphtheria bacillus?

1. Estimation of urease activity


2. Estimation of proteolytic properties
3. Estimation of ability to decompose starch
4. Estimation of toxigenic properties
5. Estimation of cystinous activity

28. Question
A consumptive patient has an open pulmonary form of disease. Choose what
sputum staining should be selected for finding out the tubercle (Kochs)
bacillus?

1. Method of Neisser
2. Method of Romanowsky-Giemsa
3. Method of Gram
4. Method of Ziel-Neelsen
5. Method of Burry-Gins
LAKSHMAN

29. Question
For the purpose of retrtospective diagnostics of recent bacterial dysentery it
was decided to perform serological examination of blood serum in order to
determine antibody titer towards Shiga bacilli. What of the following
reactions should be applied?

LAKSHMAN
1. Precipitation
2. Bacteriolysis
3. Hemolysis
4. Passive hemagglutination
5. Bordet-Gengou test

30. Question
Planned mass vaccination of all newborn 5-7 day old children against
tuberulosis plays an important role in tuberculosis prevention. In this case
the following vaccine is applied:

1. Adsorbed diphtheria vaccine


2. Diphteria and tetanus toxoids and pertussis vaccine
3. Diphtheria and tetanus anatoxin vaccine
4. BCG

31. Question
A patient with complaints of 3-day-long fever, general weakness, loss of
appetite came to visit the infectionist. The doctor suspected enteric fever.
Which method of laboratory diagnosis is the best to confirm the diagnosis?

1. Detachment of blood culture


2. Detachment of feces culture
3. Detachment of myeloculture
4. Detachment of urine culture
5. Detachment of pure culture

32. Question
Material taken from a patient with provisional diagnosis “influenza” was
referred to a laboratory. For virological examination the hemadsorption
reaction was applied. This reaction can be applied for detection of the
following viruses:

1. All the complex viruses


2. Viruses containing hemagglutinins
3. DNA-genomic viruses
4. Any viruses
5. All the simple viruses

33. Question
In a patient with clinical signs of immunodeficiency the number and
functional activity of T and B lymphocytes are not changed. Defect with
dysfunction of antigen-presentation to the immunocompetent cells was found
during investigation on the molecule level. Defect of what cells is the most
probable?

1. Macrophages, monocytes
2. B-lymphocytes
LAKSHMAN

3. Fibroblasts, Т-lymphocytes, В-lymphocytes


4. Т-lymphocytes, В-lymphocytes
5. NK-cells

LAKSHMAN
34. Question
Urine examination of a patient with acute cystitis revealed leukocytes and a
lot of gram-negative bacilli. Inoculation resulted in growth of colonies of
mucous nature that formed green soluble pigment. What microorganism is
the most probable cause of the disease?

1. Pseudomonas aeruginosa
2. Escherihia coli
3. Salmonella enteritidis
4. Proteus mirabilis
5. Klebsiella pneumoniae

35. Question
Bacterioscopy of nasopharyngeal mucus taken from a 2.5 year old child with
nasopharyngitis revealed gram-positive diplococci looking like coffee grains.
What organs of the child are most likely to be affected if these
microorganisms penetrate the blood?

1. Lymph nodes
2. Urogenital tracts
3. Renal glomeruli
4. Cardiac valves
5. Brain tunics

36. Question
A female patient underwent liver transplantation. 1.5 month after it her
condition became worse because of reaction of transplant rejection. What
factor of immune system plays the leading part in this reaction?

1. T-helpers
2. T-killers
3. Natural killers
4. Interleukin-1
5. B-lymphocytes

37. Question
A pregnant woman was registered in an antenatal clinic and underwent
complex examination for a number of infections. Blood serum contained IgM
to the rubella virus. What is this result indicative of?

1. Of a chronic process
2. Of recurring infection with rubella virus
3. Of exacerbation of a chronic disease
4. The woman is healthy
5. Of primary infection

38. Question
Bacteriological examination of purulent discharges from the urethra
LAKSHMAN

revealed gram-negative bacteria looking like coffee beans. They were


localized in the leukocytes and could decompose glucose and maltose to acid.
These are the causative agents of the following disease:

LAKSHMAN
1. Veneral lymphogranulomatosis
2. Syphilis
3. Gonorrhoea
4. Soft chancre
5. Melioidosis

39. Question
From the defecation of a 6-year-old ill child, who has artificial feeding, the
intestinal bacillus with antigen structure 0-111 is excreted. What is the
diagnosis?

1. Food poisoning
2. Gastroenteritis
3. Cholera-like diseasis
4. Coli-enteritis
5. Disentery-like diseasis

40. Question
Study of bacteriological sputum specimens stained by the Ziel-Neelsen method
revealed some bright-red acid-resistant bacilli that were found in groups or
singularly. When they were inoculated onto the nutrient media, the signs of
their growth show up on the 10-15 day. These bacteria relate to the following
family:

1. Coxiella burnettii
2. Yersinia pseudotuberculosis
3. Histoplasma dubrosii
4. Micobacterium tuberculosis
5. Klebsiella rhinoscleromatis

41. Question
Analysis of sputum taken from a patient with suspected pneumonia revealed
rather elongated gram-positive diplococci with somewhat pointed opposite
ends. What microorganisms were revealed in the sputum?

1. Neisseria meningitidis
2. Neisseria gonorrhoeae
3. Streptococcus pneumoniae
4. Staphylococcus aureus
5. Klebsiella pneumoniae

42. Question
A specimen stained by Ozheshko method contains rod-like microorganisms
stained blue with round terminal components stained red. What are these
components called?

1. Mesosomas
2. Flagella
3. Capsules
LAKSHMAN

4. Cilia
5. Spores

43. Question

LAKSHMAN
Vomiting material of a patient suspected of having cholera was delivered to
the bacteriological laboratory. The material was used for preparing a
“hanging drop” specimen. What type of microscopy will be applied for
identification of the causative agent by its mobility?

Living bacteria=phase...
1. Immersion microscopy
2. Fluorescence microscopy
3. Immune and electron microscopy
4. Phase-contrast microscopy
5. Electron microscopy

44. Question
The donor who didn’t donate the blood for a long time was investigated with
IFA method. Anti-HBs antibodies were revealed. What does positive result of
IFA in this case mean?

1. Acute hepatitis C
2. Acute hepatitis B
3. Previous hepatitis B
4. Chronic hepatitis В
5. Chronic hepatitis С

45. Question
In a 2-year-old child with catarrhal presentations and skin rash a
pediatrician suspected scarlet fever. The child was given intracutaneously a
small dose of serum antibody to the streptococcal erythrogenic toxin; on the
site of injection the rash disappeared. What do the reaction results mean?

1. The disease wasnt caused by haemolytic streptococcus


2. The child has very weak immune system
3. The child has hypersensitivity to the erythrogenic toxin
4. The clinical diagnosis was con rmed
5. The whole serum dose may be injected intravenously

46. Question
Gramnegative bin-shaped diplococcus inside and outside of leucocytes were
detected on bacteriological examination of the purulent exudates from the
cervix of the uterus. Name the causative agent of purulent inflammation of
the cervix of the uterus

1. Neisseria gonorroeae
2. Chlamidia trachomatis
3. Calymmatobacterium granulomatis
4. Trichomonas vaginalis
5. Haemophilus vaginalis

47. Question
Serological diagnostics of infectious diseases is based upon specific
interaction with antigenes. Specify the serological reaction that underlies
LAKSHMAN

adhesion of microorganisms when they are affected by specific antibodies in


presence of an electrolyte:

1. Complement-binding reaction

LAKSHMAN
2. Neutralization reaction
3. Agglutination reaction
4. Precipitation reaction
5. Hemadsorption reaction

48. Question
A patient of surgical department complains about pain in the small of her
back and in the lower part of her belly; painful and frequent urination.
Bacteriological examination of urine revealed gram-negative oxidase-positive
rod-like bacteria forming greenish mucoid colonies with specific smell. What
causative agent can it be?

1. Proteus mirabilis
2. Pseudomonas aeruginosa
3. Mycoplasma pneumonie
4. Str.pyogenes
5. E.coli

49. Question
A 50-year-old patient with typhoid fever was treated with Levomycetin, the
next day his condition became worse, temperature rised to 39.6°С. What
caused worthening?

1. Reinfection
2. Secondary infection addition
3. The effect of endotoxin agent
4. Irresponsiveness of an agent to the levomycetin
5. Allergic reaction

50. Question
A 7 year old child often suffers from streprococcic angina. Doctor suspected
development of rheumatism and administered serological examination. The
provisional diagnosis will be most probably confirmed by presence of
antibodies to the following streptococcic antigen:

1. C-carbohydrate
2. Erythrogenic toxin
3. M-protein
4. O-streptolysin
5. Capsular polysaccharide

51. Question
A man died from an acute infectious disease accompanied by fever, jaundice,
haemorrhagic rash on the skin and mucous membranes as well as by acute
renal insufficiency. Histological examination of renal tissue (stained by
Romanovsky-Giemsa method) revealed some convoluted bacteria looking like
C und S letters. What bacteria were revealed?

1. Spirilla
2. Treponema
LAKSHMAN

3. Leptospira
4. Borrelia
5. Campilobacteria

LAKSHMAN
52. Question
On bacteriological study of rinsing water of the patient with food poisoning,
the pure bacterial culture was inoculated with the following properties:
gram-negative motile bacillus in the Endo environment grows like achromic
colony. Representative of what genus has caused the illness?

1. Escherichia
2. Citrobacter
3. Salmonella
4. Yersinia
5. Shigella

53. Question
Microscopic examination of a Gram-stained scrape from patients tongue
revealed oval, round, elongated chains of dark-violet gemmating cells. What
disease can be caused by this causative agent?

1. Actinomycosis
2. Staphylococcic infection
3. Diphtheria
4. Streptococcic infection
5. Candidosis

54. Question
A 42-year-old man who has been injured in a car accident is brought into the
emergency room. His blood alcohol level on admission is 250 mg/dL. Hospital
records show a prior hospitalization for alcohol related seizures. His wife
confirms that he has been drinking heavily for 3 weeks. What treatment
should be provided to the patient if he goes into withdrawal?

Diazepam belongs to a group of medicines called benzodi


1. Phenytoin azepines. It's used to treat anxiety, muscle spasms and fit
Pentobarbital s (seizures). It's also used in hospital to reduce alcohol wit
hdrawal symptoms, such as sweating or difficulty sleepin
2.
3. Phenobarbital g.
4. None
5. Diazepam

55. Question
A 10-year-old child had the mantoux tuberculin test administered. 48 hours
later a papule up to 8 mm in diameter appeared on the site of the injection.
What type of hypersensitivity reaction developed after the tuberculin
injection?

1. Type II hypersensitivity reaction


2. Type IV hypersensitivity reaction
3. Seroreaction
4. Arthus phenomenon
5. Atopic reaction
LAKSHMAN

56. Question
From the nasopharynx of a 5-year-old child it was excreted amicroorganism
which is identical to Corynebacterium diphtheriae dose according to
morphological and biochemical signs.Microorganism does not produce

LAKSHMAN
exotoxin. As a result of what process can this microorganism become
toxigenic?

1. Growing with antitoxic serum


2. Phage conversion
3. Cultivation in the telluric environment
4. Passing through the organism of the sensative animals
5. Chromosome mutation

57. Question
The immunoblot detected gp120 protein in the blood serum. This protein is
typical for the following disease:

1. HIV-infection

Gp41 & receptor=CD4 co rec-CCR


2. Tuberculosis
3. Poliomyelitis 5
4. Syphilis
5. Virus B hepatitis

58. Question
While registering the child to the school Mantus test was made to define
whether revaccination was needed test result is negative. What does this
result of the test mean?

1. Absence of cell immunity to the tuberculosis


Mantoux test is a widely used test for latent TB.
2. Presence of antibodies for tubercle bacillus It involves injecting a small amount of a subs
ance called PPD tuberculin into the skin of you
r forearm. It's also called the tuberculin skin te
3. Absence of antibodies for tubercle bacillus
4. Presence of cell immunity to the tuberculosis st (TST)
5. Absence of antitoxic immunity to the tuberculosis

59. Question
A laboratory received a material from a patients wound. Ppreliminary
diagnosis is gaseous gangrene. What microbiological method should be
applied to determine species of causative agent?

1. Serological
2. Bacteriological
3. RIA
4. Allergic
5. Bacterioscopic

60. Question
Patient with diarrhoea was admitted to the infection unit. Gramnegative
curved rod-like bacteria were founded on bacterioscopic examination of
faecal masses. What is the most likely disease in this patient?

1. Intestinal form of plague


2. Cholera
LAKSHMAN

3. Salmonellosis gastroenteritis
4. Typhoid fever
5. Diphtheria

LAKSHMAN
61. Question
The person was selling “homemade pork” sausages on the market. State
sanitary inspector suspected falcification of the sausages.With help of what
serological immune reaction can food substance be identified?

1. Precipitation test
Precipitation tests measure an antigen or antibody i
n body fluids by the degree of visible precipitation o
2. Agglutination test
3. Indirect hemagglutination test f antigen-antibody complexes within a gel (agarose)
4. Complement- xation test or in solution
5. Immuno uorescence test

62. Question
In order to estimate toxigenity of diphtheria agents obtained from patients
the cultures were inoculated on Petri dish with nutrient agar on either side of
a filter paper strip that was put into the centre and moistened with
antidiphtheric antitoxic serum. After incubation of inoculations in agar the
strip-like areas of medium turbidity were found between separate cultures
and the strip of filter paper. What immunological reaction was conducted?

Precipitation reactions are based on the interaction of antibodi


1. Opsonization reaction es and antigens. They are based on two soluble reactants that c
ome together to make one insoluble product, the precipitate. T
hese reactions depend on the formation of lattices (cross-links
2. Coombs test
3. Precipitation gel reaction ) when antigen and antibody exist in optimal proportions
4. Agglutination reaction
5. Rings precipitation reaction

63. Question
Sanitary bacteriological research on water by the membrane filter method
revealed two red colonies on a membrane filter (Endo agar) through which
500 ml of analyzed water were passed. Calculate the coli index and coli titer
of the analyzed water:

1. 4 and 250
2. 500 and 2
3. 250 and 2
4. 2 and 500
5. 250 and 4

64. Question
Among junior children of an orphanage an outbreak of intestinal infection
with signs of colienteritis was registered. In order to identify isolated
causative agent it is necessary to:

1. To study biochemical properties of the causative agent


2. To study sensitivity to bacteriophages
3. To determine sensitivity to antibiotics
4. To study virulence of the causative agent
5. Study antigenic properties of the causative agent

65. Question
LAKSHMAN

As a result of durative antibiotic therapy a 37-year old patient developed


intestinal dysbacteriosis. What type of drugs should be used in order to
normalize intestinal microflora?

LAKSHMAN
1. Autovaccines
2. Bacteriophages
3. Eubiotics
4. Vitamins
5. Sulfanilamides

66. Question
Scraps of the mycelium of a fungus, spores, air bubbles and fat drops were
discovered on microscopy of the patients hair excluded from the infected
areas. For what fungus disease is this microscopic picture characteristic?

Favus is a chronic infection caused most commonly by T schoenlei


1. Trichophytosis nii and, occasionally, by T violaceum or Microsporum gypsum. Scal
p lesions are characterized by the presence of yellow cup-shaped cr
usts termed scutula, which surround the infected hair follicles
2. Microspory
3. Sporotrichosis
4. Favus
5. Epidermophytosis

67. Question
During surgical operation a blood transfusion was made. The blood must be
checked to find antigens of some disease. What disease is expected to be
found?

1. Adenovirus
2. Virus of hepatitis B
3. Virus of hepatitis E
4. Enterovirus
5. Virus of hepatitis A

68. Question
Patient with vomiting, dizziness, sensation of dubble vision, difficult
swallowing was admitted to the hospital. Doctor suspects botulism. What
diagnostic methods should be used for diagnosis approving?

1. Allergic test, serological


2. Bacteriological, mycological
3. Biological test, bacteriological
4. Protozoological, microscopical

69. Question
Examination of a young man in the AIDS centre produced a positive result of
immune-enzyme assay with HIV antigens. Patient didn’t complain on state of
his health. What can the positive result of immune-enzyme assay be evidence
of?

1. Being infected with HBV


2. HIV infection
3. HBV persistence
4. Having had AIDS recently
LAKSHMAN

5. Being ill with AIDS

70. Question
In the surgical department of a hospital there was an outbreak of hospital

LAKSHMAN
infection that showed itself in often postoperative wound abscesses.
Bacteriological examination of pus revealed aurococcus. What examination
shall be conducted to find out the source of this causative agent among the
department personnel?
A subdivision of a species distinguished from other
strains therein by sensitivity to a certain bacteriop
hage or set of bacteriophages.
1. Estimation of antibiotic susceptibility
2. Serological identi cation
3. Phagotyping
4. Biochemical identi cation
5. Microscopical examination

71. Question
The contents of vesicles that appeared on the mucous membrane of a patient
with variola was sent to a virological laboratory. Which of the listed changes
were revealed during the smear microscopy?
particles that occur in the cells of skin rashes in pa
1. Babes-Negri bodies tients with cowpox or smallpox; they are thought t
o be the virus particles
2. Syncytium
3. Guarnieri bodies
4. Paschen bodies
5. Babes-Ernst bodies

72. Question
For serological diagnostics of the whooping cough it was made large-scale
reaction with parapertussis and pertussis diagnosticums. At the bottom of
the test-tubes with diagnosticum of Bordetella parapertussis grain-like
sediment formed. What antibodies have this reaction revealed?
1. Opsonins
2. Agglutinins
3. Antitoxins
4. Bacteriolysins
5. Precipitins

73. Question
While examining a patient an otolaryngologist noticed hyperaemia and
significantly edematous tonsils with a grayish film upon them. Microscopical
examination of this film revealed some gram-positive bacilli placed at an
angle with each other. What disease might be suspected?

1. Scarlet fever
2. Meningococcal nasopharyngitis
3. Diphtheria
4. Angina
5. Epidemic parotitis

74. Question
A patient who came to the doctor because of his infertility was administered
to make tests for toxoplasmosis and chronic gonorrhoea. Which reaction
should be performed to reveal latent toxoplasmosis and chronic gonorrhoea
of the patient? complement fixation test is an immunological medical t
est that can be used to detect the presence of either spe
LAKSHMAN

cific antibody or specific antigen in a patient's serum, b


ased on whether complement fixation occurs
1. Immunoblot analysis
2. IFA - Immuno uorescence assay

LAKSHMAN
3. RDHA - Reverse direct hemagglutination assay
4. (R)CFT- Reiters complement xation test
5. RIHA - Reverse indirect hemagglutination assay

75. Question
Reaction of passive hemagglutination conducted with erythrocytic typhoid Vi-
diagnosticum helped to reveal some antibodies in the dilution of the patients
serum at a ratio of 1:80 that exceeds the diagnostic titer. Such result
witnesses of:

1. Incubation period of typhoid fever


2. Being ill with acute typhoid fever
3. Reconvalescence of a patient ill with typhoid fever
4. Typhoid fever recurrence
5. Being a potential carrier of typhoid bacilli

76. Question
On bacteriological examination of the defecation of a 4-months-old baby with
the symptoms of acute bowel infection there were revealed red colonies
spread in the large quantity in the Endo environment. What microorganism
can it be?

1. Streptococcus
2. Staphylococcus
3. Salmonella
4. Shigell
5. Escherichia

77. Question
Quite often the cause of secondary immunodeficiency is an infection
involvement, when the causative agents propagate directly in the cells of
immune system and destroy it. The following diseases are characterized by:

1. Q-febris, epidemic typhus


2. Dysentery, cholera
3. Infectious mononucleosis, AIDS
4. Tuberculosis, mycobacteriosis
5. Poliomyelitis, type A hepatitis

78. Question
It is planned to use the territory of an old cattle burial ground (which is not
used for more than 50 years) for building houses. But ground analysis
revealed presence of the pathogen of the very dangerous illness. Which of the
indicated microorgonisms is likely to remain in the ground for such a long
time?

1. Bacillus anthracis
Bacillus anthracis is a Gram-positive and rod-shaped bacte
rium that causes anthrax, a deadly disease to livestock and
2. Francisella tularensis
3. Brucella abortus , occasionally, to humans. It is the only permanent pathog
Mycobacterium bovisen within the genus Bacillus. Its infection is a type of zoon
LAKSHMAN

osis, as it is transmitted from animals to humans


4.
5. Yersinia pestis

79. Question

LAKSHMAN
A man was admitted to the hospital on the 5th day of disease that manifested
itself by jaundice, muscle aching, chill, nose bleedings. In course of laboratory
diagnostics a bacteriologist performed dark-field microscopy of the patients
blood drop. Name a causative agent of this disease:

1. Rickettsia mooseri
2. Leptospira interrogans
3. Bartonella bacilloformis Leptospira interrogans is a Gram negative, obligate
aerobe spirochete, with periplasmic flagella. When
viewed through a dark-field microscope, it often res
4. Calymmatobacterium granulomatis
5. Borrelia dutlonii embles a question mark, and this gives the species i
ts name
80. Question
Blood of a patient with presumable sepsis was inoculated into sugar broth.
There appeared bottom sediment. Repeated inoculation into blood agar
caused growth of small transparent round colonies surrounded by hemolysis
zone. Examination of a smear from the sediment revealed gram-positive cocci
in form of long chains. What microorganisms are present in blood of this
patient?

1. Tetracocci
2. Streptococci
3. Micrococci
4. Staphylococci
5. Sarcina

81. Question
The first grade pupils were examined in order to sort out children for
tuberculosis revaccination. What test was applied for this purpose?

1. Anthraxine test
2. Supracutaneous tularin test
3. Schick test
4. Burnet test
5. Mantoux test

82. Question
Microscopy of stained (Ziehl-Neelsen staining) smears taken from the sputum
of a patient with chronic pulmonary disease revealed red bacilli. What
property of tuberculous bacillus was shown up?

1. Alcohol resistance
2. Acid resistance
3. Spori cation
4. Capsule formation
5. Alkali resistance

83. Question
In order to speed up healing of a wound of oral mucosa a patient was
prescribed a drug that is a thermostable protein occuring in tears, saliva,
LAKSHMAN

mothers milk as well as in a new-laid hens egg. It is known that this protein is
a factor of natural resistance of an organism. What is it called?

1. Lysozyme

LAKSHMAN
2. Interleukin
3. Complement
4. Imanine
5. Interferon

84. Question
A culture of monkey cells (Vero) and a group of mouse sucklings were infected
with an inoculum taken from a child with provisional diagnosis “enterovirus
infection”. There was no cytopathic effect on the cell culture but mouse
sucklings died. What enteric viruses might have caused disease of this child?
Symptoms of fever, poor appetite, runny nose and sore thr
oat can appear three to five days after exposure. A blister-l
ike rash on the hands, feet and in the mouth usually devel
1. ECHO virus
2. Coxsackie B ops one to two days after the initial symptoms
3. Unclassi ed enteric viruses 68-71
4. Coxsackie A
5. Polioviruses

85. Question
A patient underwent esophagogastroduodenoscopy. Analysis of the biopsy
material enabled doctors to diagnose him with helicobacteriosis. What
property of the bacteria found in this patient had to be obligatory taken into
account during their cultivation?

1. Colonisation of gastral cells


2. Absence of spores and capsules
3. Microaerophilic ability
4. Presence of urease
5. Presence of six polar agella

Created by Eneutron Team

Kiev eneutron.info@gmail.com
LAKSHMAN

LAKSHMAN
Krok 1 – 2013 Base(Path-Anatomy)

Medical PG WhatsApp Group

Results
0 of 122 questions answered correctly

Your time: 00:04:50

You have reached 0 of 122 points, (0%)

RESTART QUIZ VIEW QUESTIONS

1. Question
Autopsy of a man, who had been suffering from the multiple bronchiectasis
for 5 years and died from chronic renal insufficiency, revealed that kidneys
were dense and enlarged, with thickened cortical layer of white colour with
greasy lustre. What renal disease might be suspected?

1. Glomerulonephritis
2. Secondary amyloidosis
3. Chronic pyelonephritis
4. Necrotic nephrosis

2. Question
A 40 year old man noticed a reddening and an edema of skin in the area of his
LAKSHMAN

neck that later developed into a small abscess. The incised focus is dense,
yellowish-green. The pus contains white granules. Histological examination
revealed drusen of a fungus, plasmatic and xanthome cells, macrophages.
What type of mycosis is the most probable?

LAKSHMAN
1. Actinomycosis
2. Aspergillosis
3. Coccidioidomycosis
4. Sporotrichosis
5. Candidosis

3. Question
A 22 y.o. woman has enlarged lymph nodes. Histologically: a lymph node
contains lymphocytes, histiocytes, reticular cells, small and big Hodgkins
cells, multinucleated Sternberg cells, isolated foci of caseous necrosis. What
disease are these changes typical for?

1. Lymphogranulomatosis
2. Lymphosarcoma
3. Lung cancer metastasis
4. Chronic leukosis
5. Acute leukosis

4. Question
A 30 year old man had been suffering from acute respiratory disease and died
from cardiopulmonary decompensation. Autopsy revealed fibrinous-
haemorrhagic inflammation in the mucous membrane of larynx and trachea,
destructive panbronchitis, enlarged lungs that look black due to the multiple
abcesses, haemorrhages, necrosis. What is the most probable postmortem
diagnosis?
1. In uenza
2. Measles
3. Parain uenza
4. Respiratory syncytial infection
5. Adenoviral infection

5. Question
During surgery in a 17-year-old patient it was revealed the tumour of
4.5*5.0*3.5 cm in size on the lower surface of the liver with subserose
localization, of dark-red color. On the section tumour has cavities with
marked amount of blood. What is preliminary diagnosis?

1. Hemangiopericytoma
2. Lymphangioma
3. Cavernous hemangioma
4. Hemangioendothelioma
5. Capillar hemangioma

6. Question
A 4 year old child complained of pain during deglutition, indisposition.
Objectively: palatine arches and tonsils are moderately edematic and
hyperemic, there are greyish-white films up to 1 mm thick closely adhering to
the subjacent tissues. What pathological process are these changes typical
LAKSHMAN

for?

1. In ammation
2. Metaplasia

LAKSHMAN
3. Dystrophy
4. Necrosis
5. Organization

7. Question
Autopsy of a 17 year old girl who died from pulmonary failure revealed a
small area of caseous necrosis in the inferior lobe of the right lung, and
occurences of caseous necrosis in the bronchopulmonary, bronchial and
bifurcational lymph nodes. What is the most probable postmortem diagnosis?

1. Primary tuberculosis
2. Hematogenous progression of primary tuberculosis
3. Tuberculoma
4. Hematogenous tuberculosis with predominant lung affection
5. Caseous pneumonia under secondary tuberculosis

8. Question
Examination of a 66 year old patient revealed a lytic tumour in the locus of
pathological rib fracture. Histologically this tumour consists of atypical
plasmoblasts. Further examination revealed osteoporosis in the bones of
vertebral column and pelvis. These changes are typical for:

1. Metastatic lung cancer


2. Neuroblastoma
3. Myelomatosis
4. Tuberculous osteomyelitis
5. Ewings osteosarcoma

9. Question
Autopsy of a 73-year-old man who had been suffering from the coronary heart
disease along with cardiac insufficiency for a long time revealed: nutmeg
liver, brown induration of lungs, cyanotic induration of kidneys and spleen.
What kind of circulation disorder was the cause of such effects?

1. Acute anaemia
2. General chronic venous congestion
3. Arterial hyperaemia
4. General acute venous congestion
5. Chronic anaemia

10. Question
Autopsy of a 49-year-old woman who died from chronic renal insufficiency,
revealed: kidneys were dense, reduced, multicoloured, with haemorrhagic
areas. Microscopic examination revealed some hematoxylin bodies in the
nuclei of the renal tubule epithelium; “wire-loop” thickening of the glomerular
capillary basement membrane; here and there in the capillaries some hyaline
thrombi and foci of fibrinoid necrosis were present. What is the most likely
diagnosis?
LAKSHMAN

1. Rheumatism
2. Systemic lupus erythematosus
3. Amyloidosis

LAKSHMAN
4. Atherosclerotic nephrosclerosis
5. Arteriosclerotic pneumosclerosis

11. Question
Autopsy of a 50-year-old man revealed the following changes: his right lung
was moderately compact in all parts, the dissected tissue was found to be
airless, fine-grained, dryish. Visceral pleura had greyish-brown layers of
fibrin. What is the most likely diagnosis?

1. Bronchopneumonia
2. Interstitial pneumonia
3. Pneumo brosis
4. Croupous pneumonia
5. Tuberculosis

12. Question
For a long time a 49-year-old woman was suffering from glomerulonephritis
which caused death.On autopsy it was revealed that kidneys size was 7-3*2.5
cm, weight is 65.0 g, they are dence and small-grained. Microscopically:
fibrinogenous inflammation of serous and mucous capsules, dystrophic
changes of parenchymatous organs, brain edema. What complication can
cause such changes of serous capsules and inner organs?

1. Uraemia
2. Sepsis
3. Anemia
4. Thrombopenia
5. DIC-syndrome

13. Question
Autopsy of a 1.5-year-old child revealed haemorrhagic skin rash, moderate
hyperaemia and edema of nasopharyngeal mucous membrane, small
haemorrhages in the mucous membranes and internal organs; dramatic
dystrophic alterations in liver and myocardium; acute necrotic nephrosis;
massive haemorrhages in the adrenal glands. What disease are these
alterations the most typical for?

1. Epidemic typhus
2. Scarlet fever
3. Measles
4. Meningococcal infection
5. Diphtheria

14. Question
Autopsy of a 56 y.o. man revealed in the right temporal part of brain a big
focus of softened grey matter that was semi-liquid and light grey. Arteries of
cerebral tela contain multiple whitish-yellow thickenings of intima that
abruptly narrow the lumen. What is your diagnosis?

1. Ischemic stroke
2. Brain abscess
LAKSHMAN

3. Hemorrhagic infarction
4. Brain edema
5. Hemorrhage

LAKSHMAN
15. Question
2 hours after a skeletal extension was performed to a 27 year old patient with
multiple traumas (closed injury of chest, closed fracture of right thigh) his
condition abruptly became worse and the patient died from acute
cardiopulmonary decompensation. Histological examination of pulmonary
and cerebral vessels stained with Sudan III revealed orange drops occluding
the vessel lumen. What complication of polytrauma was developed?

1. Microbal embolism
2. Gaseous embolism
3. Thromboembolism
4. Air embolism
5. Fat embolism

16. Question
6 months after delivery a woman had uterine bleeding. Gynecological
examination revealed in the uterine cavity a dark-red tissue with multiple
cavities that resembled of “sponge”. Microscopic examination of the tumour
revealed some atypic light epithelial Langhans cells and giant cells of
cyncytiotrophoblast in blood lacunas. What tumour is it?

1. Squamous cell nonkeratinous carcinoma


2. Adenocarcinoma
3. Fibromyoma
4. Chorioepithelioma
5. Vesicular mole
17. Question
A 33 year old man died from uraemia. Autopsy revealed enlarged kidneys
weighing 500g   each and consisting of multiple cavities 0.5-2 cm in diameter.
The cavities were full of light-yellow transparent liquid. Renal pelvis and
ureters had no pecularities. What renal disease caused uraemia?

1. Bilateral polycystic renal disease


2. Renal tumour
3. Rapidly progressing glomerulonephritis
4. Renal tuberculosis
5. Chronic pyelonephritis

18. Question
A stillborn child was found to have thickened skin resembling of the tortoise
shell, underdeveloped auricles. Histological examination of skin revealed
hyperkeratosis, atrophy of the granular epidermis layer; inflammatory
changes were not present. What is the most likely diagnosis?

1. Xerodermia
2. Dermatomyositis
3. Ichthyosis
4. Erythroplakia
LAKSHMAN

5. Leukoplakia

19. Question
On autopsy of the 40-year-old woman suffering from rheumatic arthritis, the
enlarged solid spleen was revealed. On section its tissue is of the mahogany

LAKSHMAN
color with enlarged follicles, which look like semi-transparent grayish-whitish
grains. What pathological process is the most likely?

1. Glaze spleen
2. Sago spleen
3. Hyaline spleen
4. Waxy spleen
5. Porphyric spleen

20. Question
Autopsy of a man who had been working as a miner for many years and died
from cardiopulmonary decompensation revealed that his lungs were airless,
sclerosed, their apexex had emphysematous changes, the lung surface was
greyish-black, the incised lung tissue was coal-black. What disease caused
death?

1. Talcosis
2. Silicosis
3. Anthracosis
4. Aluminosis
5. Asbestosis

21. Question
The upper lobe of the right lung is enlarged, grey and airless, the inscision
surface is dripping with turbid liquid, the pleura has many fibrinogenous
films; microscopical examination of alveoles revealed exudate containing
neutrophils, desquamated alveolocytes and fibrin fibers. The bronchus wall is
intact. What is the most probable diagnosis?

1. Croupous pneumonia
2. Pulmonary abscess
3. In uenzal pneumonia
4. Interstitial pneumonia
5. Focal pneumonia

22. Question
Microscopical renal examination of a 36 y.o. woman who died from renal
insufficiency revealed in the glomerules proliferation of capsule
nephrothelium as well as of podocytes and phagocytes accompanied by
formation of “crescents”, capillary loop necrosis, fibrinous thrombs in their
lumens; sclerosis and hyalinosis of glomerules, atrophy of tubules and
fibrosis of renal stroma. What is the most probable diagnosis?

1. Chronic glomerulonephritis
2. Subacute glomerulonephritis
3. Membranous nephropathy
4. Focal segmentary sclerosis
5. Acute glomerulonephritis
LAKSHMAN

23. Question
Autopsy of a woman with cerebral atherosclerosis revealed in the left
cerebral hemisphere a certain focus that is presented by flabby, anhistic,
greyish and yellowish tissue with indistinct edges. What pathological process
is the case?

LAKSHMAN
1. Senile encephalopathy
2. Focal encephalitis
3. Multifocal tumor growth with cystic degeneration
4. Ischemic stroke
5. Multiple foci of fresh and old cerebral hemorrhage

24. Question
48 hours after performing tuberculin test (Mantoux test) to a child a 10 mm
papule appeared on the spot of tuberculin introduction. What
hypersensitivity mechanism underlies these changes?

1. Anaphylaxis
2. Cellular cytotoxicity
3. Immune complex cytotoxicity
4. Granulomatosis
5. Antibody-dependent cytotoxicity

25. Question
A female patient suffering from bronchial asthma had got a viral infection
that provoked status asthmaticus with fatal outcome. Histological
examination of lungs revealed spasm and edema of bronchioles, apparent
infiltration of their walls with lymphocytes, eosinophils and other leukocytes;
labrocyte degranulation. What mechanism of hypersensitivity underlies the
described alterations?
1. Immune complex
2. In ammatory
3. Autoimmune
4. Reagin reaction
5. Immune cytolysis

26. Question
On autopsy it is revealed that kidneys are enlarged, surface is large-granular
because of multiple cavities with smooth wall, which are filled with clear
fluid. What kidney disease did the patient have?

1. Infarction
2. Polycystic kidney
3. Pyelonephritis
4. Necrotic nephrosis
5. Glomerulonephritis

27. Question
Diagnostic scraping was performed to the woman with dysfunctional uterine
bleeding. Multiple convoluted glands, ganglially dilated cavities of some
glands were revealed histologically in the scrape. Name the type of general
pathological process
LAKSHMAN

1. Displasia
2. Hypertrophic excrescence
3. Atrophy

LAKSHMAN
4. Glandulo-gangliac hyperplasia
5. Metaplasia

28. Question
On autopsy of the man with alcohol abuse for a long time it was revealed:
dense, small-knobby, small size liver. Microscopically: small pseudo-lobules,
divided with thin layers of connective tissue with lymphomacrophagial
infiltrates; hepatocytes in the state of globular fatty dystrophy. What is the
most likely diagnosis?

1. Chronic persistent alcohol hepatitis


2. Chronic active alcohol hepatitis
3. Fatty hepatosis
4. Toxic liver dystrophy
5. Alcohol cirrhosis

29. Question
Chronic inflammation and transformation of the one-layer ciliated
epithelium into multiple-layers flat epithelium was revealed in the thickened
mucous membrane of the bronchus bioptate of the patient with smoke abuse.
Which of the processes is the most likely?

1. Hyperplasia of the epithelium


2. Leucoplacia
3. Epithelium hypertrophy
4. Metaplasia
5. Squamous cancer

30. Question
A 9 m.o. child has delayed dentition, it is also out of order. Upper jaw
configuration is horizontal (“high” palate); microscopically – irregular
mineralization of tooth enamel, wrinkled enamel prisms, some of them are
vacuolized. Predentin zone is extended; there are solitary denticles. What
disease is it?

1. Hypervitaminosis D
2. Early rickets
3. Osteomalacia
4. Late rickets
5. Gout

31. Question
Local lymphonodules enlarged near the infected wound. Increased amount of
macrophages, lymphocytes, lymphatic follicles in the cortical layer and large
amount of plasma cells were revealed on histological examination. What
process in the lymphatic nodules represent these histological changes?

1. Antigen stimulation
2. Acquired insuf ciency of the lymphoid tissue
3. Tumour transformation
LAKSHMAN

4. Hypersensibility reaction
5. Innate insuf ciency of the lymphoid tissue

32. Question

LAKSHMAN
Extensive thromboembolic infarction of the left cerebral hemispheres, large
septic spleen, immunocomplex glomerulonephritis, ulcers on the edges of the
aortic valves, covered with polypous thrombus with colonies of
staphylococcus were revealed on autopsy of the young man who died in coma.
What disease caused cerebral thromboemboly?

1. Septic bacterial endocarditis


2. Acute rheumatic valvulitis
3. Rheumatic thromboendocarditis
4. Septicopyemia
5. Septicemia

33. Question
Examination of the anterior abdominal wall of a pregnant woman revealed a
tumour-like formation that arose on the spot of a tumour that was removed
two years ago. The neoplasm was well-defined, dense, 2*1 cm large.
Histological examination revealed that the tumour was composed of
differentiated connective tissue with prevailing collagen fibres. What tumour
might be suspected?

1. Hibernoma
2. Lipoma
3. Leiomyoma
4. Desmoid
5. Fibrosarcoma
34. Question
Gynecological examination of the uterine cervix in a 30-year-old woman
revealed some bright-red lustrous spots that easily bleed when touched.
Biopsy showed that a part of the uterine cervix was covered with cylindrical
epithelium with papillary outgrowths; in the depth of tissue the growth of
glands was present. What pathology of the uterine cervix was revealed?

1. Glandular hyperplasia
2. Leukoplakia
3. True erosion
4. Endocervicitis
5. Pseudoerosion

35. Question
An experimental animal was first sensibilized where upon an antigen dose
was introduced subcutaneously. This injection resulted in the development of
a fibrinous inflammation with alteration of vessel walls, basal substance and
fibrous structures of connective tissue in form of mucoid and fibrinoid
swelling and necrosis. What immunological reaction took place?

1. Normergic reaction
2. Immediate hypersensitivity
3. Delayed-type hypersensitivity
LAKSHMAN

4. Granulomatosis
5. Reaction of transplantation immunity

36. Question

LAKSHMAN
Tuberculine was injected intracutaneously to the child for tuberculin test.
Marked hyperemia, tissue infiltration developed on the place of injection in 24
hours. What mechanism caused these modifications?

1. Granuloma formation
2. Immunocomplex cytotoxity
3. Antibody cytotoxity
4. Reagin type cytotoxity
5. Cells cytotoxity

37. Question
A 20 year old woman died in 8 days of intoxication after artificial illegal
abortion performed in her 14-15th week of pregnancy. Autopsy of the corpse
revealed yellowish colour of eye sclera and of skin, necrotic suppurative
endometritis, multiple pulmonary abscesses, spleen hyperplasia with a big
number of neutrophils in its sinuses. What complication after abortion was
developed?

1. Chroniosepsis
2. Viral hepatitis type A
3. Hemorrhagic shock
4. Septicemia
5. Septicopyemia

38. Question
2 days after labour a woman developed shock along with DIC syndrome that
caused her death. Autopsy revealed purulent endomyometritis, regional
purulent lymphangitis, lymphadenitis and purulent thrombophlebitis. There
were also dystrophic alterations and interstitial inflammation of
parenchymal organs. What is the most likely diagnosis?

1. Chorioadenoma destruens
2. Septicemia
3. Tuberculosis of genital organs
4. Hydatid mole
5. Syphilis

39. Question
A man had worked in a coal mine for over 20 years. After his death autopsy
revealed that his lungs were dense, grayish-black and had large areas of
neogenic connective tissue containing a lot of microphages with black pigment
in the cytoplasm. What is the most likely diagnosis?

1. Anthracosilicosis
2. Talcosis
3. Siderosis
4. Silicoanthracosis
5. Anthracosis
LAKSHMAN

40. Question
Autopsy of a 48 y.o. man revealed a round formation 5 cm in diameter with
clear-cut outlines in the region of the 1st segment of his right lung. This
formation was encircled with a thin layer of connective tissue full of white

LAKSHMAN
brittle masses. Make a diagnosis of the secondary tuberculosis form:

1. Fibrous cavernous tuberculosis


2. Acute focal tuberculosis
3. Tuberculoma
4. Caseous pneumonia
5. Acute cavernous tuberculosis

41. Question
The intraoperational biopsy of mammal gland has revealed the signs of
atypical tissue with disorder of parenchyma stroma proportion with
domination of the last, gland structures of the different size and shape, lined
with single-layer proliferative epithelium. What is the most appropriate
diagnosis?

1. Nonin ltrative cancer


2. Fibroadenoma
3. Mastitis
4. In ltrative cancer
5. Papilloma

42. Question
A 45 year old male died from disseminated tuberculosis. On autopsy the
symptoms of tuberculosis were confirmed by both microscopical and
histological analyses. All the affected organs had epithelioid cell granulomas
with caseous necrosis in the centre. What kind of hypersensitivity reaction
underlies the process of granuloma development?

1. Complement-dependent cytotoxicity
2. Immune complex
3. Delayed
4. Antibody-dependent cytotoxicity
5. Anaphylactic

43. Question
In course of severe respiratory viral infection there appeared clinical signs of
progressing cardiac insufficiency that caused death of a patient in the 2nd
week of disease. Autopsy revealed that the heart was sluggish, with
significant cavity dilatation. Histological examination of myocardium
revealed plethora of microvessels and diffuse infiltration of stroma by
lymphocytes and histiocytes. What disease corresponds with the described
picture?

1. Myocardium infarction
2. Stenocardia
3. Myocarditis
4. Cardiomyopathy
5. Acute coronary insuf ciency
LAKSHMAN

44. Question
An 8-year-old child was admitted to the infectious department with fever (up
to 38°C) and punctuate bright-red skin rash. The child was diagnosed as
having scarlet fever. Objectively: mucous membrane of pharynx is apparently

LAKSHMAN
hyperaemic and edematic, the tonsils are enlarged and have dull yellowish-
grey foci with some black areas. What inflammation is the reason for the
pharynx alterations?

1. Purulent necrotic
2. Haemorrhagic
3. Fibrinous
4. Catarrhal
5. Serous

45. Question
A sick man with high temperature and a lot of tiny wounds on the body has
been admitted to the hospital. Lice have been found in the folds of his clothing.
What disease can be suspected in the patient?

1. Tularemia
2. Epidemic typhus
3. Plague
4. Scabies
5. Malaria

46. Question
Histological examination of a skin tissue sampling revealed granulomas
consisting of macrophagal nodules with lymphocytes and plasmatic cells.
There are also some big macrophages with fatty vacuoles containing
causative agents of a disease packed up in form of spheres (Virchows cells).
Granulation tissue is well vascularized. What disease is this granuloma
typical for?

1. Rhinoscleroma
2. Glanders
3. Tuberculosis
4. Lepra
5. Syphilis

47. Question
A section of the left lung was shown an area of dense red tissue. The area was
cone-shaped, stood out distinctly from the healthy tissue, with its base
directed to the pleura. The dissected tissue was granular, dark-red. What is
the most likely diagnosis?

1. Lung abscess
2. Primary tuberculous affection
3. Lung gangrene
4. Haemorrhagic infarction
5. Croupous pneumonia

48. Question
LAKSHMAN

A forensic medical expert examines the body of a 58 y.o. man who had been
consuming large amounts of alcochol for a long time and died at home.
Microscopicaly: the right lung is dense and enlarged, its incision revealed
that the tissue is greyish and homogenous, pleura is covered with greyish
layers. Microscopically – alveolar cavities contain fibrin, hemolyzed

LAKSHMAN
erythrocytes. Make a diagnosis:

1. Primary pulmonary tuberculosis


2. Focal pneumonia
3. Interstitial pneumonia
4. Caseous pneumonia
5. Croupous pneumonia

49. Question
Autopsy of a man with a malignant stomach tumour who had died from
cancer intoxication revealed in the posteroinferior lung fields some dense,
grayish-red irregular foci protruding above the section surface. Microscopic
examination revealed exudate containing a large amount of neutrophils in
the lumen and walls of small bronchi and alveoles. Such pulmonary
alterations indicate the following disease:

1. Croupous pneumonia
2. Acute serous bronchopneumonia
3. Acute bronchitis
4. Acute purulent bronchopneumonia
5. Intermittent pneumonia

50. Question
Examination of a 55 year old woman revealed under the skin of
submandibular area a movable slowly growing pasty formation with distinct
borders 1.0×0.7 cm large. Histological examination revealed lipocytes that
form segments of diffrent forms and sizes separated from each other by thin
layers of connective tissue with vessels. What is the most probable diagnosis?

1. Lipoma
2. Fibroma
3. Angioma
4. Liposarcoma
5. Fibrosarcoma

51. Question
A 39 year old man who had been operated for the stomach ulcer died 7 days
after the surgery. Autopsy revealed that peritoneal leaves were dull,
plethoric, covered with massive yellow-greenish films, the peritoneal cavity
contained for about 300 ml of thick yellow-greenish liquid. What pathologic
process was revealed in the peritoneal cavity?

1. Fibrinous serous peritonitis


2. Peritoneal commissures
3. Fibrinous suppurative peritonitis
4. Serous peritonitis
5. Fibrinous haemorrhagic peritonitis

52. Question
LAKSHMAN

A patient died from acute cardiac insufficiency, among clinical presentations


there was gastrointestinal haemorrhage. Examination of mucous membrane
of stomach revealed some defects reaching myenteron; their edges and
bottom were mostly even and loose, some of them contained dark-red blood.
What pathological process was revealed?

LAKSHMAN
1. Acute ulcers
2. Thrombosis
3. Erosions
4. In ammation
5. Chronic ulcers

53. Question
A 71 year old man had been presenting with diarrhea for 10 days. The feces
had admixtures of blood and mucus. He was delivered to a hospital in grave
condition and died 2 days later. Autopsy of the body revealed the following:
diphtheritic colitis with multiple irregularly-shaped ulcers of different depth
in both sigmoid colon and rectus. Bacteriological analysis revealed Shigella.
What was the main disease?

1. Typhoid fever
2. Salmonellosis
3. Nonspeci c ulcerous colitis
4. Dysentery
5. Yersiniosis

54. Question
Microscopical examination of an enlarged cervical lymph node revealed
blurring of its structure, absence of lymphoid follicles; all the microscopic
fields showed cells with roundish nuclei and thin limbus of basophil
cytoplasm. It is known from the clinical data that other groups of lymph
nodes are also enlarged as well as spleen and liver. What disease might be
suspected?

1. Lymphoid leukosis
2. Multiple myeloma
3. Lymphosarcoma
4. Lymphogranulomatosis
5. Myeloid leukosis

55. Question
A 2 year old child had acute respiratory viral infection and died from
cardiopulmonary decompensation. Autopsy revealed that his right lung was
hyperemic; in the 2nd, 6th and 10th segments and on the incision there were
airless yellowish foci of irregular form, from several mm up to 1 cm large.
Microscopical examination revealed exudate consisting mainly of neutrophils
in the given areas of pulmonary tissue in the alveoles, bronchioles and
bronchial tubes. What is the most probable diagnosis?

1. Pulmonary abscess
2. Interstitial pneumonia
3. Acute bronchitis
4. Focal pneumonia
LAKSHMAN

5. Croupous pneumonia

56. Question
Examination of a bronchial tissue sample revealed atrophy of mucous
membrane, cystic degeneration of glands, focal metaplastic changes of lining

LAKSHMAN
prismatic epithelial cells into multilayer squamous cells; increase in goblet
cell number; in some parts of bronchial wall and especially in the mucous
membrane there was marked cellular inflammatory infiltration and growth
of granulation tissue bulging into the bronchial lumen in form of a polyp.
What is the most likely diagnosis?

1. Chronic bronchitis
2. Acute bronchitis
3. Bronchopneumonia
4. Lobar pneumonia
5. Interstitial pneumonia

57. Question
A patient died under conditions of cardiovascular insufficiency. Autopsy
results: postinfarction cardiosclerosis, myocardium hypertrophy and
dilatation of its cavities, especially of its right ventricle. Liver is enlarged, its
surface is smooth, incision revealed that it was plethoric, with dark-red
specks against the background of brownish tissue. Histologically: plethora of
central parts of lobules; peritheral parts around portal tracts contain
hepatocytes in a state of adipose degeneration. How are these liver changes
called?

1. Liver steatosis
2. Nutmeg liver
3. Amyloidosis
4. Pseudonutmeg liver
5. Liver cirrhosis

58. Question
A 38 year old patient with full-blown jaundice, small cutaneous hemorrhages,
general weakness and loss of appetite underwent puncture biopsy of liver.
Histological examination revealed disseminated dystrophy, hepatocyte
necrosis, Councilmans bodies. Lobule periphery has signs of significant
infiltration by lymphocytes, there are also individual multinuclear
hepatocytes. What is the most probable diagnosis?

1. Acute alcoholic hepatitis


2. Miliary hepatic cirrhosis
3. Acute viral hepatitis
4. Toxic degeneration of liver
5. Chronic hepatitis

59. Question
Arterial hypertension, hyperglycemia, glucosuria were observed clinically for
a long time in the patient with upper type of obesity. Death was due to the
cerebral haemorrhage. Basophilic hypophysis adenoma, hyperplasia of
adrenal gland cortex were revealed on pathomorphological examination.
What is the likely diagnosis?

1. Acromegaly
LAKSHMAN

2. Diabetes mellitus
3. Adiposogenitalis dystrophy
4. Cushing disease

LAKSHMAN
5. Hypophysis nanism

60. Question
Unpainfull formation without marked borders appeared in the soft tissues of
the thigh in the young man. On the tissue bioptate the formation lookes like a
meat of a fish, consisting of the immature fibroblast-like cells with multiple
mitosis, which grow through the muscles. What is the most likely diagnosis?

1. Myoma
2. Fibrosarcoma
3. Cancer
4. Myosarcoma
5. Fibroma

61. Question
A 30 year old woman has applied a lipstick with a fluorescent substance for a
long time. Then she got a limited erythema and slight peeling on her lip
border, later there appeared transversal striae and cracks. Special methods
of microscopic examination of the affected area helped to reveal sensibilized
lymphocytes and macrophages in the connective tissue; cytolysis. What type
of immunological hypersensitivity was developed?

1. III type (immune complex cytotoxicity)


2. I type (reaginic)
3. Granulomatosis
4. II type (antibody cytotoxicity)
5. IV type (cellular cytotoxicity)
62. Question
A boy is 7 y.o. Objectively: against the background of hyperemic skin there is
knobby bright-pink rash on his forehead, neck, at the bottom of abdomen, in
the popliteal spaces; nasolabial triangle is pale. Examination of
oropharyngeal surface revealed localized bright-red hyperemia; tonsils are
swollen, soft, lacunas contain pus, tongue is crimson. Cervical lymph nodes
are enlarged, dense and painful. What is the most probable diagnosis?

1. Rubella
2. Whooping cough
3. Scarlet fever
4. Infectious mononucleosis
5. Diphtheria

63. Question
Acute renal impairment caused death of a bleeding patient. Autopsy revealed
enlarged kidneys with a broad pale pink cortical layer expressively
demarcated from dark red renal pyramids. Macroscopic examination
revealed lack of epithelial nuclei of convoluted tubules, tubulorrhexis,
phlebostasis. The cell nuclei of choroid glomus and straight tubules were
present. What pathology is it?

1. Pyelonephritis
LAKSHMAN

2. Infarction
3. Glomerulonephritis
4. Necronephrosis
5. Nephrosis

LAKSHMAN
64. Question
Examination of coronary arteries revealed atherosclerotic calcific plaques
that close vessel lumen by 1/3. The muscle has multiple whitish layers of
connective tissue. What process was revealed in myocardium?

1. Postinfarction cardiosclerosis
2. Tiger heart
3. Diffuse cardiosclerosis
4. Myocarditis
5. Myocardium infarction

65. Question
A 30-year-old patient with bacteriologically proved dysentery developed the
signs of paraproctitis. What is the stage of local changes in this patient?
Paraproctitis is an infection inflammation of the
1. Catarrhal colitis tissues around the rectum.
2. Healing of the ulcers stage
3. Fibrinous colitis
4. Follicular colitis
5. Ulceration stage

66. Question
A 46 year-old man complains of difficult nose breathing. Mikulich cells,
storage of epithelioid cells, plasmocytes, lymphocytes, hyaline balls are
discovered in the biopsy material of the nose thickening. What is the most
likely diagnosis?

1. Scleroma
2. Virus rhinitis
3. Meningococcal nasopharyngitis
4. Allergic rhinitis
5. Rhinovirus infection

67. Question
A 63 y.o. man fell ill with acute tracheitis and bronchitis accompanied by
bronchial pneumonia. On the 10th day the patient died from
cardiopulmonary insufficiency. Autopsy revealed fibrinous hemorrhagic
laryngotracheobronchitis; lungs were enlarged, their incision revealed the
“coal-miners” effect caused by interlacing of sections of bronchial pneumonia,
hemorrhages into the pulmonary parenchyma, acute abscesses and
atelectases. Internal organs have discirculatory and dystrophic changes.
What is the most probable diagnosis?

1. Adenoviral infection
2. In uenza, severe form
3. Parain uenza
4. Respiratory syncytial infection
5. Moderately severe in uenza
LAKSHMAN

68. Question
A 59-year-old man has signs of the parenchymatous jaundice and portal
hypertension. On histological examination of the puncture of the liver
bioptate, it was revealed: beam-lobule structure is affected, part of

LAKSHMAN
hepatocytes has signs of fat dystrophy, port-portal connective tissue septa
with formation of pseudo-lobules,with periportal lympho-macrophage
infiltrations. What is the most probable diagnosis?

1. Chronic hepatosis
2. Alcohol hepatitis
3. Liver cirrhosis
4. Viral hepatitis
5. Toxic dystrophy

69. Question
A 46 year old patient who had been suffering from tuberculosis for 6 years
died from massive pulmonary haemorrhage. Autopsy revealed different-sixed
foci of sclerosis and caseous necrosis in lungs, in the upper part of the right
lung there was a cavity 5 cm in diameter with dense grey walls, the cavity
contained liquid blood and blood clots. What type of tuberculosis is it?

1. Fibrous focal
2. Acute focal
3. Acute cavernous
4. In ltrative
5. Fibrocavernous

70. Question
A man with a wound of his limb that had been suppurating for a long time
died from intoxication. Autopsy revealed extreme emaciation, dehydration,
brown atrophy of liver, myocardium, spleen and cross-striated muscles as
well as renal amyloidosis. What diagnosis corresponds with the described
picture?

1. Chernogubovs syndrome
2. Chroniosepsis
3. Brucellosis
4. Septicopyemia
5. Septicemia

71. Question
A patient who has been abusing tobacco smoking for a long time has got
cough accompanied by excretion of viscous mucus; weakness after minor
physical stress, pale skin. The patient has also lost 12.0 kg of body weight.
Endoscopic examination of biosy material his illness was diagnosed as
squamous cell carcinoma. Name a pathological process that preceded
formation of the tumour:

1. Metaplasia
2. Hyperplasia
3. Hypoplasia
4. Sclerosis
LAKSHMAN

5. Necrosis

72. Question
Histologic analysis of uterus mucous membrane revealed twisting glands,

LAKSHMAN
serrated and spinned, they were extended by stroma growth with
proliferation of its cells. Formulate a diagnosis:

1. Leiomyoma
2. Acute endometritis
3. Cystic mole
4. Glandular hyperplasia of endometrium
5. Placental polyp

73. Question
Autopsy of a man who died from influenza revealed that his heart was
slightly enlarged, pastous, myocardium was dull and had specks.
Microscopical examination of myocardium revealed signs of parenchymatous
adipose and hydropic dystrophy; stroma was edematic with poor
macrophagal and lymphocytic infiltration, vessels were plethoric;
perivascular analysis revealed petechial hemorrhages. What type of
myocarditis was developed in this case?

1. Serous diffuse
2. Serous focal
3. Interstitial proliferative
4. Granulomatous
5. Purulent

74. Question
A 28 year old patient had high arterial pressure, hematuria and facial
edemata. In spite of treatment renal insufficiency was progressing. 6 months
later the patient died from uremia. Microscopic examination of his kidneys
and their glomerules revealed proliferation of capsule nephrothelium and of
podocytes with “semilune” formation, sclerosis and hyalinosis of glomerules.
What disease corresponds with the described picture?

1. Chronic glomerulonephritis
2. Acute glomerulonephritis
3. Subacute glomerulonephritis
4. Nephrotic syndrome
5. Acute pyelonephritis

75. Question
A patient who abuses smoking has chronic bronchitis. Biopsy of his primary
bronchus revealed multilayer pavement epithelium. What pathological
process was revealed in the bronchus?

1. Dysplasia
2. Hyperplasia
3. Reparative regeneration
4. Physiological regeneration
5. Metaplasia
LAKSHMAN

76. Question
Autopsy of a man who died from burn disease revealed brain edema, liver
enlargement as well as enlargement of kidneys with wide light-grey cortical
layer and plethoric medullary area. Microscopic examination revealed

LAKSHMAN
necrosis of tubules of main segments along with destruction of basal
membranes, intersticium edema with leukocytic infiltration and
haemorrhages. What is the most probable postmortem diagnosis?

1. Tubulointerstitial nephritis
2. Myeloma kidney
3. Gouty kidney
4. Necrotic nephrosis
5. Pyelonephritis

77. Question
On autopsy it was revealed: large (1-2 cm) brownish-red, easy crumbling
formations covering ulcerative defects on the external surface of the aortic
valve. What is the most likely diagnosis?

1. Fibroplastic endocarditis
2. Recurrent warty endocarditis
3. Acute warty endocarditis
4. Polypus-ulcerative endocarditis
5. Diffusive endocarditis

78. Question
A woman suffering from dysfunctional metrorrhagia was made a diagnostic
abortion. Histologically in the scrape there were a lot of small stamped
glandulars covered with multirowed epithelium. The lumens of some
glandulars were cystically extended. Choose the variant of general pathologic
process in the endometrium

1. Atrophy of endometrium
2. Metaplasia of endometrium
3. Glandular-cystic hyperplasia of endometrium
4. Hypertrophic growth
5. Neoplasm of endometrium

79. Question
Skin of a man who died from cardiac insufficiency has an eruption in form of
spots and specks. There are also bedsores in the area of sacrum and spinous
vertebral processes. Microscopical examination of CNS, skin, adrenal glands
revealed in the vessels of microcirculatory bed and in small arteries
destructive-proliferative endothrombovasculitis with Popovs granulomas;
interstitial myocarditis. What diagnosis corresponds with the described
picture?

1. HIV
2. Q fever
3. Nodular periarteritis
4. Spotted fever
5. Enteric fever
LAKSHMAN

80. Question
Microscopical examination of a removed appendix revealed an edema, diffuse
neutrophilic infiltration of appendix wall along with necrosis and defect of
mucous membrane with affection of its muscle plate. What form of

LAKSHMAN
appendicitis was developed?

1. Phlegmonous
2. Super cial
3. Gangrenous
4. Ulcerophlegmonous
5. Apostematous

81. Question
A 50 year old patient underwent resection of tumour of large intestine wall.
Microscopically it presents itself as fascicles of divergent collagen fibers of
different thickness and form and some monomorphous fusiform cells that are
irregularly distributed among the fibers. Cellular atypia is not evident. What
tumour is it?

1. Desmoma
2. Fibrosarcoma
3. Soft broma
4. Hard broma
5. Fibromyoma

82. Question
A patient died from cardiopulmonary decompensation. Histological
examination revealed diffused pulmonary lesion together with interstitial
edema, infiltration of tissue by limphocytes, macrophages, plasmocytes;
pulmonary fibrosis, panacinar emphysema. What disease corresponds with
the described picture?

1. Fibrosing alveolitis
2. Bronchial asthma
3. Bronchopneumonia
4. Pulmonary atelectasis
5. Chronic bronchitis

83. Question
On autopsy it is revealed enlarged dense right lung, fibrin layers on the
pleura. Lung tissue is light green color on incision with muddy liqued
exudates. What lung disease are these symptoms typical for?

1. Pulmonary gangrene
2. Bronchopneumonia
3. Interstitial pneumonia
4. Fibrosing alveolitis
5. Lung-fever

84. Question
A patient with high-titer antinuclear antibodies died from progressing renal
impairment. Autopsy revealed mesangioproliferative glomerulonephritis and
LAKSHMAN

abacterial polypous endocarditis. There was periarterial bulbar sclerosis in


spleen and productive proliferative vasculitis in skin. What is the most likely
diagnosis?

LAKSHMAN
1. Periarteritis nodosa
2. Dermatomyositis
3. Rheumatism
4. Nephrotic syndrome
5. Systemic lupus erythematosus

85. Question
Autopsy of a 75 year old patient who had been suffering from disseminated
atherosclerosis and died under chronic cardiac failure revealed constriction
and deformation of coronary arteries, tuberous intima whose section
appeared to be white and petrosal. Specify the stage of atherosclerosis
morphogenesis:

1. Bilipid
2. Atheromatosis
3. Lipoidosis
4. Liposclerosis
5. Atherocalcinosis

86. Question
A worker of a cattle farm fell acutely ill and then died from the progressing
intoxication. Autopsy revealed enlarged, hyposthenic spleen of dark-cherry
colour when dissected; excessive pulp scraping. At the base and fornix of
brain pia maters are edematous, soaked with blood, dark-red (“scarlet hat”).
Microscopic examination revealed serous haemorrhagic inflammation of
brain tissues and tunics along with destruction of small vessel walls. What is
the most likely diagnosis?
1. Cholera
2. Anthrax
3. Tularemia
4. Plaque
5. Brucellosis

87. Question
On microscopic examination of the enlarged neck gland of a 14-year-old girl it
was revealed destruction of the tissue structure of the node, absence of the
lymph follicles, sclerotic and necrosis parts, cell constitution of the node is
polymorphous, lymphocites, eosinophiles, atypical cells of the large size with
multiple-lobule nuclei (Beresovsky-Shternberg cells) and onenucleus cells of
the large size are present. What is the most likely diagnosis?

1. Lymphogranulomatous
2. Berkitts lymphoma
3. Acute lympholeucosis
4. Chronic lympholeucosis
5. Fungous mycosis

88. Question
Purulent endometritis with fatal outcome was progressing in the woman
after abortion performed not at the hospital. On autopsy multiple lung
LAKSHMAN

abscesses, subcapsule ulcers in the kidneys, spleen hyperplasia were revealed.


What form of sepsis developed in the patient?

1. Septicopyemia

LAKSHMAN
2. Chroniosepsis
3. Urosepsis
4. Lung sepsis
5. Septicemia

89. Question
A physician examined a patient and found inguinal hernia. Through what
anatomic formation does it penetrate into the skin?

1. Canalis adductorius
2. Hiatus saphenus
3. Anulus inguinalis super cialis
4. Lacuna musculorum
5. Anulus femoralis

90. Question
A patient has been syffering from diarrhea for 5 day. On the fith day
colonoscopy revealed that membrane of rectum was inflamed, there were
greyish-green films closely adhering to the subjacent tissue. What is the most
probable diagnosis?

1. Crohns disease
2. Dysentery
3. Typhoid fever
4. Salmonellosis
5. Nonspeci c ulcerous colitis

91. Question
Autopsy of a 5 year old child revealed in the area of vermis of cerebellum a
soft greyish-pink node 2 cm in diameter with areas of haemorrhage.
Histologically this tumour consisted of atypical monomorphous small
roundish cells with big polymorphous nuclei. What tumour is it?

1. Oligodendroglioma
2. Medulloblastoma
3. Glioblastoma
4. Meningioma
5. Astrocytoma

92. Question
A 63 year old male patient who had been suffering from chronic diffuse
obstructive disease, pulmonary emphysema, for 15 years died from cardiac
insufficiency. Autopsy revealed nutmeg liver cirrhosis, cyanotic induration of
kidneys and spleen, ascites, edemata of lower limbs. These changes of internal
organs are typical for the following disease:

1. Acute right-ventricular insuf ciency


2. Acute left-ventricular insuf ciency
3. General cardiac insuf ciency
LAKSHMAN

4. Chronic right-ventricular insuf ciency


5. Chronic left-ventricular insuf ciency

93. Question

LAKSHMAN
Autopsy of a man who died from ethylene glycol poisoning revealed that his
kidneys are a little bit enlarged, edematic; their capsule can be easily
removed. Cortical substance is broad and light-grey. Medullary substance is
dark-red. What pathology had this man?

1. Acute tubular-interstitial nephritis


2. Acute pyelonephritis
3. Acute glomerulonephritis
4. Lipoid nephrosis
5. Necrotic nephrosis

94. Question
A patient with android-type obesity had been suffering from arterial
hypertension, hyperglycemia, glycosuria for a long time and died from the
cerebral haemorrhage. Pathologic examination revealed pituitary basophil
adenoma, adrenal cortex hyperplasia. What is the most likely diagnosis?

1. Pituitary nanism
2. Acromegalia
3. Itsenko-Cushings syndrome
4. Adiposogenital dystrophy
5. Diabetes mellitus

95. Question
A patient ill with tuberculosis died from progressing cardiopulmonary
decompensation. Autopsy in the area of the right lung apex revealed a cavity 5
cm in diameter communicating with lumen of a segmental bronchus. On the
inside cavity walls are covered with caseous masses with epithelioid and
Langhans cells between them. What morphological form of tuberculosis is it?

1. Acute focal tuberculosis


2. Caseous pneumonia
3. In ltrative tuberculosis
4. Tuberculoma
5. Acute cavernous tuberculosis

96. Question
A 50 year old man who was referred to the hospital for treatment of cervical
lymphadenitis underwent test for induvidual sensitivity to penicillin. 30
seconds after he went hot all over, AP dropped down to 0 mm Hg that led to
cardiac arrest. Resuscitation was unsuccessful. Autopsy results: acute venous
plethora of internal organs; histological examination of skin (from the site of
injection) revealed degranulation of mast cells (tissue basophils).
Degranulation was also revealed in myocardium and lungs. What type of
hypersensitivity reaction is it?

1. Immunocomplex-mediated
2. Complement-mediated cytotoxic
LAKSHMAN

3. Anaphylactic
4. Delayed-type hypersensitivity

97. Question

LAKSHMAN
Autopsy of a man ill with severe hypothyroidism revealed that connective
tissue, organ stroma, adipose and cartilaginous tissues were swollen,
semitransparent, mucus-like. Microscopic examination of tissues revealed
stellate cells having processes with mucus between them. What type of
dystrophy is it?

1. Stromal-vascular carbohydrate
2. Parenchymatous proteinaceous
3. Stromal-vascular proteinaceous
4. Parenchymatous adipose
5. Stromal-vascular adipose

98. Question
Autopsy of a man who died from the sepsis in his femoral bone revealed
phlegmonous inflammation that affected the marrow, haversian canals and
periosteum. Under the periosteum there are multiple abscesses, adjoining soft
tissues of thigh also have signs of phlegmonous inflammation. What
pathological process was described?

1. Osteopetrosis
2. Acute hematogenous osteomyelitis
3. Chronic hematogenous osteomielitis
4. Osteoporosis

99. Question
A man died 8 days after the beginning of the disease. He was diagnosed with
dysentery. At the autopsy it was found out a thickened wall of the sigma and
rectum, fibrinous membrane on the surface of mucous membrane.
Histologically: there is a deep necrosis of mucous membrane with infiltration
of necrotic masses with fibrin. What kind of colitis does correspond to the
changes?

1. Gangrenous
2. Diphtheritic
3. Ulcerative
4. Chronic
5. Catarrhal

100. Question
A 22 year old patient from the West Ukraine complains of laboured nasal
breathing. Morphological examination of biopsy material of nasal mucous
membrane revealed lymphoid, epithelioid, plasma cells as well as Mikuliczs
cells. What is the most probable diagnosis?

1. Glanders
2. Tuberculosis
3. Rhinoscleroma
4. Leprosy
LAKSHMAN

5. Syphilis

101. Question
Colonoscopy of a patient ill with dysentery revealed that mucous membrane
of his large intestine is hyperemic, edematic, its surface was covered with

LAKSHMAN
grey-and-green coats. Name the morphological form of dysenteric collitis:

1. Purulent
2. Fibrinous
3. Catarrhal
4. Ulcerous
5. Necrotic

102. Question
A 45 y.o. patient consulted a doctor about plaque-shaped formation on his
neck. Histological examination of biopsy skin material revealed tumourous
cells of round and oval form with thin ring of basophilic cytoplasma that
resemble of cells of basal epidermal layer. What tumour is it?

1. Basalioma
2. Syringoadenoma
3. Trichoepithelioma
4. Hydradenoma
5. Epidermal cancer

103. Question
A patient had been suffering from profuse diarrhea and vomiting for 2 days.
He died from acute dehydration. Autopsy revealed that the intestinal wall
was edematic and hyperemic, with multiple haemorrhages in the mucous
membrane. Intestine lumen contains whitish fluid resembling of rice water.
What disease caused death?
1. Enterocolitis
2. Salmonellosis
3. Dysentery
4. Typhoid fever
5. Cholera

104. Question
Analysis of a punction biopsy material of liver revealed hepatocyte dystrophy
with necroses as well as sclerosis with disorder of beam and lobulous
structure, with formation of pseudolobules and regenerative nodes. What is
the most probable diagnosis:

1. Acute hepatitis
2. Progressive massive liver necrosis
3. Liver cirrhosis
4. Chronic hepatosis
5. Chronic hepatitis

105. Question
A patient has a cluster of matted together dense lymph nodes on his neck.
Histological examination of a removed lymph node revealed proliferation of
reticular cells, presense of Reed-Sternberg cells. What disease is meant?
LAKSHMAN

1. Myelocytic leukosis
2. Lymphoblastic leukosis
3. Myeloblastic leukosis

LAKSHMAN
4. Lymphogranulomatosis
5. Lymphocytic leukosis

106. Question
A patient ill with diabetes mellitus felt acute pain in his right foot. Objectively:
foot thumb is black, foot tissues are edematous, there are foci of epidermis
desquamation, stinking discharges. What clinicopathological form of
necrosis is it?

1. Sequestrum
2. Bedsore
3. Moist gangrene
4. Dry gangrene
5. Infarction

107. Question
A 23 year old man has perforation of hard palate. In the area of this
perforation there was a compact well-defined formation. Microscopic
examination of the resected formation revealed a large focus of caseous
necrosis surrounded by granulation tissue with endovasculitis, cellular
infiltration composed of lymphocytes, epithelioid cells (mainly plasmocytes).
What is the most probable diagnosis?

1. Sarcoma
2. Syphilis
3. Tuberculosis
4. Scleroma
5. Leprosy

108. Question
Mucous membrane of the right palatine tonsil has a painless ulcer with
smooth lacquer fundus and regular cartilagenous edges. Microscopically:
inflammatory infiltration that consists of lymphocytes, plasmocytes, a small
number of neutrophils and epithelioid cells; endovasculitis and perivasculitis.
What disease is it?

1. Ulcerous necrotic Vincents angina


2. Pharyngeal diphtheria
3. Actinomycosis
4. Syphilis
5. Tuberculosis

109. Question
A 50 year old patient has been taking treatment thrice for the last 6 months
because of fractures caused by domestic accidents. Microscopical
examination of bony tissue revealed foci of lacunar resolution, giant-cell
granulomas in the tumour-like formations, cysts. Bony tissue was substituted
by fibrous connective tissue. Examination revealed also adenoma of
parathyroid gland and hypercalcemia. What is the most probable diagnosis?
LAKSHMAN

1. Parathyroid osteodystrophy
2. Osteomyelitis
3. Pagets disease

LAKSHMAN
4. Osteopetrosis
5. Myelomatosis

110. Question
A pathology-histology laboratory received a vermiform appendix up to 2.0 cm
thick. Its serous membrane was pale, thick and covered with yellowish-green
films. The wall was flaccid, of grayish-red colour. The appendix lumen was
dilated and filled with yellowish-green substance. Histological examination
revealed that the appendix wall was infiltrated with neutrophils. Specify the
appendix disease:

1. Acute super cial appendicitis


2. Acute simple appendicitis
3. Chronic appendicitis
4. Acute phlegmonous appendicitis
5. Acute gangrenous appendicitis

111. Question
A 39 y.o. woman went through an operation in course of which surgeons
removed her uterine tube that was enlarged and a part of an ovary with a big
cyst. Histological examination of a tube wall revealed decidual cells, chorion
villi. What was the most probable diagnosis made after examination of the
uterine tube?

1. Choriocarcinoma
2. Papyraceous fetus
3. Placental polyp
4. Lithopedion
5. Tubal pregnancy

112. Question
Examination of a young woman revealed a node-like, soft and elastic
homogenous tumour of pinkish-white colour along the acoustic nerve. The
tumour contains cell bundles with oval nuclei. Cellular fibrous bundles form
rhythmic structures made up by parallel rows of regularly oriented cells
arranged in form of a palisade with cell-free homogenous zone (Verocay
bodies) between them. What tumour is it?

1. Neurinoma
2. Ganglioneuroblastoma
3. Neuroblastoma
4. Malignant neurinoma
5. Ganglioneurinoma

113. Question
A 40-year-old woman has had a feeling of abdominal discomfort for the past 8
months. On pelvic examination, there is the right adnexal mass. Abdominal
CT scan demonstrates a 7 cm cystic mass involving the right ovary with small
areas of calcification. The uterus is normal in size. The right fallopian tube
and ovary have been removed surgically. Grossly, the mass on sectioning is
filled with abundant hair and sebum. Microscopically, the mass has
glandular spaces lined by columnar epithelium, squamous epithelium with
LAKSHMAN

hair follicles, cartilage, and dense connective tissue. What type of tumour is
it?

1. Metastase of cervical carcinoma

LAKSHMAN
2. Sarcoma of ovary
3. Melanoma
4. Teratoma
5. Squamous cell carcinoma of ovary

114. Question
On autopsy of the 58-year-old man it is revealed: mitral valve is deformed,
thickened, not totally closed. Microscopically: centers of collagen fibers are
eosinophilic, have positive fibrin reaction. The most likely it is:

1. Hyalinosis
2. Amyloidosis
3. Mucoid swelling
4. Fibrinoid in ammation
5. Fibrinoid swelling

115. Question
A man with a long-term history of bronchial asthma died from asphyxia.
Histological examination of his lungs revealed that the lumens of bronchioles
and minor bronchi contained a lot of mucus with some eosinophils. There was
also sclerosis of interalveolar septa, dilatation of alveole lumens. What
mechanism accounts for the development of hypersensitivity reaction?

1. Granulomatosis
2. Lymphocyte-mediated cytolysis
3. Reagine reaction
4. Immune complex reaction
5. Cytotoxic reaction

116. Question
Autopsy of a man who had tuberculosis revealed a 3×2 cm large cavity in the
superior lobe of the right lung. The cavity was interconnected with a
bronchus, its wall was dense and consisted of three layers: the internal layer
was pyogenic, the middle layer was made by tuberculous granulation tissue
and the external one was made by connective tissue. What is the most likely
diagnosis?

1. Acute focal tuberculosis


2. Acute cavernous tuberculosis
3. Fibrous focal tuberculosis
4. Fibrous cavernous tuberculosis
5. Tuberculoma

117. Question
A 50-year-old man has felt vague abdominal discomfort within past 4 months.
Physical examination revealed no lymphadenopathy, and no abdominal
masses or organomegaly at palpation. Bowel sounds are heard. An
abdominal CT scan shows a 20 cm retroperitoneal soft tissue mass obscuring
the left psoas muscle. A stool specimen tested for occult blood is negative.
LAKSHMAN

Which of the following neoplasms is this man most likely to have?

1. Lymphoma
2. Adenocarcinoma

LAKSHMAN
3. Lipoma
4. Melanoma
5. Hamartoma

118. Question
Examination of a patient revealed a dense, movable skin tumour that is
standing out distinctly from the surrounding tissues. Its section is found to be
white and composed of fibrous tissue. Microscopic examination revealed
interlacing collagen fibers and few cells. What tumour is it?

1. Fibroma
2. Dermato broma
3. Desmoid
4. Histiocytoma
5. Myoma

119. Question
On autopsy it is revealed: soft arachnoid membrane of the upper parts of
cerebral hemisphere is plethoric, it is of yellowish-green color, soaked with
purulent and fibrose exudate, it lookes like cap. For what disease is it
characteristical picture

1. Meningitis at anthrax
2. Meningitis at typhus
3. In uenza meningitis
4. Meningococcal meningitis
5. Tuberculous meningitis

120. Question
A denaturation of proteins can be found in some substances. Specify the
substance that is used for the incomplete denaturation of hemoglobin:

1. Sodium hydroxide
2. Toluene
3. Urea
4. Nitric acid
5. Sulfuric acid

121. Question
Patient suffering from trombophlebitis of the deep veins suddenly died.
Autopsy has shown freely lying red friable masses with dim crimped surface
in the trunk and bifurcation of the pulmonary artery. What pathologic
process was revealed by morbid anatomist?

1. Tromboembolism
2. Tissue embolism
3. Thrombosis
4. Fat embolism
LAKSHMAN

5. Embolism with foreign body

122. Question
Multiple oval ulcers along the intestine were revealed on autopsy of the

LAKSHMAN
person, who died from diffuse of peritonitis in the distal part of the small
intestine. Bottom parts of the ulcers are clear, smooth, formed with muscular
or serous covering, edges of ulcers are flat, rounded. There are perforations
up to 0.5 cm in diameter in two ulcers. What diseasis can be diagnosed?

1. Tuberculosis
2. Typhus
3. Dysentery
4. Cholera
5. Typhoid fever

Created by Eneutron Team

Kiev eneutron.info@gmail.com
Krok 1 – 2013 Base(Path-Physiology)

Medical PG WhatsApp Group

Results
0 of 157 questions answered correctly

Your time: 00:05:55

You have reached 0 of 157 points, (0%)

RESTART QUIZ VIEW QUESTIONS

1. Question
After transfusion of 200 ml of blood a patient presented with body
temperature rise up to 37,9 oC Which of the following substances is the most
likely cause of temperature rise?

1. Interleukin-2
2. Tumour necrosis factor
3. Interleukin-1
4. Interleukin-4
5. Interleukin-3

2. Question
LAKSHMAN

ECG of a patient shows such alterations: P-wave is normal, P-Q-interval is


short, ventricular QRST complex is wide, R-wave is double-peak or two-phase.
What form of arrhythmia is it?

LAKSHMAN
1. Ventricular brillation
2. Atrioventricular block
3. WPW syndrome (Wolff-Parkinson-White)
4. Fredericks syndrome (atrial utter)
5. Ciliary arrhythmia

3. Question
A 56 y.o. patient has been suffering from thyreotoxicosis for a long time. What
type of hypoxia can be developed?

1. Respiratory
2. Tissue
3. Circulatory
4. Hemic
5. Mixed

4. Question
A patient who suffers from heart failure has enlarged liver, edemata of lower
extremities, ascites. What is the leading mechanism in the development of this
edema?

1. Membranogenic
2. Hydrodynamic
3. Lymphogenous
4. Colloid osmotic
5. Question
Blood analysis of a patient with jaundice reveals conjugated bilirubinemia,
increased concentration of bile acids. There is no stercobilinogen in urine.
What type of jaundice is it?

1. Cythemolytic jaundice
2. Hemolytic jaundice
3. Obstructive jaundice
4. Parenchymatous jaundice
5. Hepatocellular jaundice

6. Question
A patient with obliterating endarteritis underwent ganglionic
sympathectomy. What type of arterial hyperaemia should have developed as a
result of the surgery?

1. Reactive
2. Functional
3. Neurotonic
4. Neuroparalytic
5. Metabolic

7. Question
LAKSHMAN

Patient with diabetes didn’t get insulin injection in time that caused
hyperglycemic coma (glucose in the blood 50 mmol/L). What mechanism is
prevalent in the development of the coma?

LAKSHMAN
1. Hypokaliemia
2. Hyperosmia
3. Acidosis
4. Hyponatremia
5. Hypoxia

8. Question
A girl is diagnosed with adrenogenital syndrome (pseudohermaphroditism).
This pathology was caused by hypersecretion of the following adrenal
hormone:

1. Estrogen
2. Cortisol
3. Androgen
4. Aldosterone
5. Adrenalin

9. Question
Inflammation of a patients eye was accompanied by accumulation of turbid
liquid with high protein at the bottom of anterior chamber that was called
hypopyon. What process underlies the changes under observation?

1. Secondary alteration
2. Proliferation
3. Disturbance of microcirculation
4. Primary alteration

10. Question
After an attack of bronchial asthma a patient had his peripheral blood tested.
What changes can be expected?

1. Thrombocytopenia
2. Leukopenia
3. Erythrocytosis
4. Eosinophilia
5. Lymphocytosis

11. Question
A group of mountain climbers went through the blood analysis at the height
of 3000 m. It revealed decrease of HCO3 to 15 micromole/l (standard is 22-26
micromole/l). What is the mechanism of HCO3 decrease?

1. Decrease of bicarbonate reabsorption in kidneys


2. Hypoventilation
3. Intensi cation of acidogenesis
4. Decrease of ammoniogenesis
5. Hyperventilation
LAKSHMAN

12. Question
Periodic renal colics attackes are observed in the woman with primary
hyperparathyroidizm. Ultrasonic examination revealed small stones in the
kidneys. What is the cause of the formation of the stones?

LAKSHMAN
1. Hyperkalemia
2. Hypercholesterinemia
3. Hyperphosphatemia
4. Hypercalcemia
5. Hyperuricemia

13. Question
A 50 year old patient suffers from essential hypertension. After a physical
stress he experienced muscle weakness, breathlessness, cyanosis of lips, skin
and face. Respiration was accompanied by distinctly heard bubbling rales.
What mechanism underlies the development of this syndrome?

1. Collapse
2. Cardiac tamponade
3. Chronic left-ventricular failure
4. Chronic right-ventricular failure
5. Acute left-ventricular failure

14. Question
A patient with a history of chronic glomerulonephritis presents with
azotemia, oliguria, hypo- and isosthenuria, proteinuria. What is the leading
factor in the pathogenesis of these symptoms development under chronic
renal failure?

1. Intensi cation of glomerular ltration


2. Tubular hyposecretion
3. Disturbed permeability of glomerular membranes
4. Intensi cation of sodium reabsorption
5. Mass decrease of active nephrons

15. Question
Patient with hypochromic anemia has splitting hair and loss of hair,
increased nail brittling and taste alteration. What is the mechanism of the
development of these symptoms?

1. De ciency of vitamin А
2. Decreased production of thyroid hormones
3. Decreased production of parathyrin
4. De ciency of iron-containing enzymes
5. De ciency of vitamin B12

16. Question
A 43-year-old patient has thrombopenia, reduction of fibrinogen, products of
degradation of fibrin presented in the blood, petechial haemorrhage along
with septic shock. What is the most likely cause of the changes?

1. DIC-syndrome
2. Autoimmune thrombocytopenia
LAKSHMAN

3. Disorder of thrombocytes production


4. Exogenous intoxication
5. Haemorrhagic diathesis

LAKSHMAN
17. Question
Substitution of the glutamic acid on valine was revealed while examining
initial molecular structure. For what inherited pathology is this typical?

1. Favism
2. Thalassemia
3. Minkowsky-Shauffard disease
4. Hemoglobinosis
5. Sickle-cell anemia

18. Question
A woman has been applying a new cosmetic preparation for a week that
resulted in eye-lid inflammation accompanied by hyperemia, infiltration and
painfulness. What type of allergic reaction was developed?

1. I
2. ІV
3. III
4. V
5. II

19. Question
Oval and round organelles with double wall are seen at the electron
micrograph. The outer membrane is smooth, the inner membrane folded into
cristae contain enzyme ATPase synthetase. These are:
1. Centrioles
2. Mitochondria
3. Ribosomes
4. Lysosomes
5. Golgi complex

20. Question
Person has stable HR, not more than 40 bpm. What is the pacemaker of the
heart rhythm in this person?

1. Branches of His bundle


2. His bundle
3. Sinoatrial node
4. Atrioventricular node
5. Purkinge bers

21. Question
Inflammation is characterised by increasing penetration of vessels of
microcirculation stream, increasing of their fluid dynamic blood pressure.
Increasing of the osmotic concentration and dispersity of protein structures
present in the intercellular fluid. What kind of edema will appear in this case?

1. Colloid-osmotic
LAKSHMAN

2. Mixed
3. Hydrodynamic
4. Membranogenic

LAKSHMAN
5. Lymphogenic

22. Question
In result of the damage of one of the Atomic Power Plant reactor the run-out
of radioelements happened. People in the increased radiation zone were
radiated with approximately 250-300 r. They were immediately hospitalized.
What changes in the blood count would be typical?

1. Leukopenia
2. Thrombopenia
3. Anemia
4. Lymphopenia
5. Neutropenia

23. Question
A 32 y.o. man is tall, he has gynecomastia, adult woman pattern of hair
distribution, high voice, mental deficiency, sterility. Provisional diagnosis is
Klinefelter’s syndrome. In order to specify diagnosis it is necessary to
analyze:

1. Blood group
2. Caryotype
3. Spermatogenesis
4. Leukogram
5. Genealogy
24. Question
Disorder of the airways passage in small and middle bronchi was revealed in
the patient. What disorder of the acid-base equilibrium can be detected in the
blood?

1. Metabolic acidosis
2. Respiratory alkalosis
3. Metabolic alkalosis
4. Respiratory acidosis

25. Question
A 57-year-old patient was admitted to the gastroenterological department
with suspicion on Zollinger-Ellison syndrom because of rapid increase of
gastrin level in the blood serum. What disorder of the secretory function of
the stomach is the most likely?

1. Achylia
2. Hyperacid hyposecretion
3. Hyperacid hypersecretion
4. Hypoacid hyposecretion
5. Hypoacid hypersecretion

26. Question
LAKSHMAN

A 34 year old woman was diagnosed with hereditary microspherocytic


hemolytic anemia (Minkowsky-Shauffard disease). What mechanism caused
haemolysis of erythrocytes?

LAKSHMAN
1. Bone marrow hypoploasia
2. Hemoglobinopathy
3. Enzymopathy
4. Membranopathy
5. Autoimmune disorder

27. Question
A patient is followed up in an endocrinological dispensary on account of
hyperthyreosis. Weight loss, tachycardia, finger tremor are accompanied by
hypoxia symptoms – headache, fatigue, eye flicker. What mechanism of
thyroid hormones action underlies the development of hypoxia?

1. Disjunction, oxydation and phosphorilation


2. Competitive inhibition of respiratory ferments
3. Speci c binding of active centres of respiratory ferments
4. Inhibition of respiratory ferment synthesis
5. Intensi cation of respiratory ferment synthesis

28. Question
From the group of children who were eating sweet sappy watermelon two kids
developed the signs of poisoning: rapid weakness, dizziness, headache,
vomiting, edema, tachycardia, cyanosis of mouth, ears, tips of the fingers
cyanosis. High concentration of nitrates was detected. What is the leading
mechanism of the pathogenesis of the poisoning in the two children?

1. Block cytochrome oxidase


2. Insuf ciency of catalase
3. Insuf ciency of met-Hb-reductase
4. Insuf ciency of superoxiddismutase
5. Insuf ciency glutathione pyroxidase

29. Question
A 12-year-old teenager has significantly put off weight within 3 months;
glucose concentration rose up to 50 millimole/l. He fell into a coma. What is
the main mechanism of its development?

1. Ketonemic
2. Hypoxic
3. Lactacidemic
4. Hyperosmolar
5. Hypoglycemic

30. Question
A 56 year old patient suffering from cardiac insufficiency has edema of feet
and shins, edematous skin is pale and cold. What is the leading mechanism of
edema pathogenesis?

1. Positive water balance


2. Disorder of lymph out ow
LAKSHMAN

3. Decrease of oncotic pressure in capillaries


4. Increase of capillary permeability
5. Rise of hydrostatic pressure in venules

LAKSHMAN
31. Question
A patient who suffers from severe disorder of water-salt metabolism
experienced cardiac arrest in diastole. What is the most probable mechanism
of cardiac arrest in diastole?

1. Hyperkaliemia
2. Organism dehydratation
3. Hyponatremia
4. Hypokaliemia
5. Hypernatremia

32. Question
Transmural myocardial infarction in the patient was complicated with
progressive acute left ventricle insufficiency. What is the most typical for this
state?

1. Arterial hypertension
2. Cyanosis
3. Edema of the lungs
4. Ascites
5. Edema of the extremities

33. Question
Having helped to eliminate consequences of a failure at a nuclear power
plant, a worker got an irradiation doze of 500 roentgen. He complains of
headache, nausea, dizziness. What changes in leukocytes quantity can be
expected 10 hours after irradiation?

1. Neutrophilic leukocytosis
2. Lymphocytosis
3. Agranulocytosis
4. Leukopenia
5. Leukemia

34. Question
Examination of a miner revealed pulmonary fibrosis accompanied by
disturbance of alveolar ventilation. What is the main mechanism of this
disturbance?

1. Disturbance of neural respiration control


2. Bronchi spasm
3. Constriction of superior respiratory tracts
4. Limitation of respiratory surface of lungs
5. Limitation of breast mobility

35. Question
LAKSHMAN

Chronic glomerulonephritis was diagnosed in a 34-year-old patient 3 years


ago. Edema has developed in the last 6 montes. What caused it?

1. Hyperosmolarity of plasma

LAKSHMAN
2. Hyperproduction of vasopressin
3. Hyperaldosteronism
4. Proteinuria
5. Disorder of albuminous kidneys function

36. Question
Shock and signs of acute renal failure (ARF) developed in the patient due to
permanent injury. What is the leading cause of development of ARF in the
case?

1. Increased pressure in the nephron capsule


2. Urine excretion violation
3. Decreased oncotic BP
4. Increased pressure in the renal arteries
5. Decreased arterial pressure

37. Question
A 12-year-old boy often suffers from virus and bacterial infections and
eczematous skin lesions. Enlargement of T-lymphocytes and IgM with normal
IgA and IgG was revealed on examination. What type of immune system
pathology is presented in the patient?

1. Composite immunedef ciency


2. Brutons hypogammaglobulinemia
3. Hypoplasia of thymus
4. Hereditary immunde ciency of the complement system
5. Turners syndrome

38. Question
ECG of a 44-year-old patient shows signs of hypertrophy of both ventricles and
the right atrium. The patient was diagnosed with the tricuspid valve
insufficiency. What pathogenetic variant of cardiac dysfunction is usually
observed in case of such insufficiency?

1. Coronary insuf ciency


2. Heart overload by volume
3. Primary myocardial insuf ciency
4. Cardiac tamponade
5. Heart overload by resistance

39. Question
A tissue sample of benign tumor was studied under the electron microscope. A
lot of small (15-20 nm) spherical bodies, consisting of 2 unequal subunits were
detected. These are:

1. Ribosomes
2. Mitochondria
3. Smooth endoplasmic reticulum
LAKSHMAN

4. Golgi complex
5. Microtubules

40. Question

LAKSHMAN
An animal with aortic valve insufficiency got hypertrophy of its left heart
ventricle. Some of its parts have local contractures. What substance
accumulated in the myocardiocytes caused these contractures?

1. Lactic acid
2. Sodium
3. Calcium
4. Carbon dioxide
5. Potassium

41. Question
A 68-year-old woman cant move by the upper and lower right extremities due
to insult. Muscle tone of these extremities and reflexes are increased. There
are pathological reflexes. What form of the paralysis is it?

1. Dissociation
2. Tetraplegia
3. Paraplegia
4. Hemiplegia
5. Monoplegia

42. Question
Pyruvate concentration in the patient’s urine has increased 10 times from
normal amount. What vitamin deficiency can be the reason of this change:

1. Vitamin E
2. Vitamin B1
3. Vitamin B6
4. Vitamin A
5. Vitamin C

43. Question
A 46-year-old patient suffering from the diffuse toxic goiter underwent
resection of the thyroid gland. After the surgery the patient presents with
appetite loss, dyspepsia, increased neuromuscular excitement. The body
weight remained unchanged. Body temperature is normal. Which of the
following has caused such a condition in this patient?

1. Increased production of thyroxin


2. Increased production of calcitonin
3. Reduced production of parathormone
4. Increased production of thyroliberin
5. Reduced production of thyroxin

44. Question
To prevent the transplant rejection after organ transplantation it is required
to administer hormonotherapy for the purpose of immunosuppression. What
hormones are used for this purpose?
LAKSHMAN

1. Glucocorticoids
2. Sexual hormones
3. Catecholamines
4. Thyroid

LAKSHMAN
5. Mineralocorticoids

45. Question
A woman who was infected with toxoplasmosis during the pregnancy has a
child with multiple congenital defects.This is a result of:

1. Teratogenesis
2. Biological mutogenesis
3. Recombination
4. Cancerogenesis
5. Chemical mutogenesis

46. Question
A patient has extrasystole. ECG shows no P wave, QRS complex is deformed,
there is a full compensatory pause. What extrasystoles are these?

1. Atrial
2. Ventricular
3. Sinus
4. Atrioventricular

47. Question
A patient caught a cold after which there appeared facial expression
disorder. He cannot close his eyes, raise his eyebrows, bare his teeth. What
nerve is damaged?
1. Vagus
2. Infraorbital
3. Facial
4. Glossopharyngeal
5. Trigeminus

48. Question
A 12 y.o. boy who suffers from bronchial asthma has an acute attack of
asthma: evident expiratory dyspnea, skin pallor. What type of alveolar
ventilation disturbance is it?

1. Central
2. Restrictive
3. Neuromuscular
4. Obstructive
5. Throraco-diaphragmatic

49. Question
While playing volleyball a sportsman made a jump and landed on the outside
edge of his foot. He felt acute pain in the talocrural joint, active movements
are limited, passive movements are unlimited but painful. A bit later there
appeared a swelling in the area of external ankle, the skin became red and
warm. What type of peripheral circulation disturbance is the case?
LAKSHMAN

1. Stasis
2. Embolism
3. Thrombosis

LAKSHMAN
4. Venous hyperemia
5. Arterial hyperemia

50. Question
A 62-year-old patient was admitted to the neurological department due to
cerebral haemorrage. Condition is grave. There is observed progression of
deepness and frequency of breath that turnes into reduction to apnoea,and
the cycle repeates. What respiration type has developed in the patient?

1. Cheyne-Stockes respiration
2. Kussmaul respiration
3. Gasping respiration
4. Biots respiration
5. Apneustic respiration

51. Question
A 27- year-old woman has dropped penicillin containing eye drops. In few
minutes there appeared feeling of itching, burning of the skin, lips and eyelids
edema, whistling cough, decreasing of BP. What antibodies take part in the
development of this allergic reaction?

1. IgG and IgD


2. IgA and IgM
3. IgM and IgG
4. IgE and IgG
5. IgM and IgD

52. Question
Arterial hypertention is caused by the stenosis of the renal arteries in the
patient. Activation of what system is the main link in the pathogenesys of this
form of hypertension?

1. Renin-angiotensin
2. Kallikrein-kinin
3. Parasympathetic
4. Sympathoadrenal
5. Hypothalamic-pituitary

53. Question
After a serious psychoemotional stress a 48 year old patient suddenly
developed acute heart ache irradiating to the left arm. Nitroglycerine relieved
pain after 10 minutes. What is the leading pathogenetic mechanism of this
process development?

1. Spasm of coronary arteries


2. Dilatation of peripheral vessels
3. Increase in myocardial oxygen consumption
4. Obstruction of coronary vessels
LAKSHMAN

5. Compression of coronary vessels

54. Question
After the traumatic tooth extraction a patient is complaining of acute, dull,

LAKSHMAN
poorly-localized pain in gingiva, body temperature rise up to 37,5°C. The
patient has been diagnosed with alveolitis. Specify the kind of pain in this
patient:

1. Visceral
2. Protopathic
3. Epicritic
4. Heterotopic
5. Phantom

55. Question
Dystrophic changes of the heart muscle are accompanied with cardiac cavity
enlargement, decrease of the strength of heart contraction, increased amount
of blood, which remains in the heart during systolic phase, overfilled veins.
For what state of heart is it characteristic?

1. Emergency stage of hyperfunction and hypertrophy


2. Tonogenic dilatation
3. Tamponage of the heart
4. Cardiosclerosis
5. Myogenic dilatation

56. Question
A patient who suffers from pneumonia has high body temperature. What
biologically active substance plays the leading part in origin of this
phenomenon?
1. Bradykinin
2. Interleukin-I
3. Histamine
4. Serotonin
5. Leukotrienes

57. Question
Necrosis focus appeared in the area of hyperemia and skin edema in few
hours after burn. What mechanism strengthens destructive events in the
inflammation area?

1. Diapedesis of erythrocytes
2. Secondary alteration
3. Primary alteration
4. Proliferation of broblasts
5. Emigration of lymphocytes

58. Question
A 48 y.o. patient was admitted to the hospital with complaints about
weakness, irritability, sleep disturbance. Objectively: skin and scleras are
yellow. In blood: conjugated bilirubin, cholalemia. Feces are acholic. Urine is
of dark colour (bilirubin). What jaundice is it?

Obstructive
LAKSHMAN

1. Hemolytic
2. Mechanic
3. Parenchymatous

LAKSHMAN
4. Gilberts syndrome
5. Crigler-Najjar syndrome

59. Question
Two weeks after lacunar tonsillitis a 20-year-old man started complaining
about general weakness, lower eyelid edemata. After examination the patient
was diagnosed with acute glomerulonephritis. What are the most likely
pathological changes in the urine formula?

Pink or cola-colored urine from red blood cells in y


our urine (hematuria) Foamy urine due to excess p
1. Natriuria

rotein (proteinuria) High blood pressure (hyperten


2. Pyuria
3. Presence of fresh erythrocytes sion)
4. Proteinuria
5. Cylindruria

60. Question
On simulation of inflammation of the lower extremity the animal experienced
raise of the temperature, increase of amount of antibodies and leucocytes in
the blood. What substances caused this general reaction of the organism on
inflammation?

1. Glucocorticoid
2. Leucotriens
3. Somatomedins
4. Interleukin
5. Mineralcorticoid
61. Question
A couple came for medical genetic counseling. The man has hemophilia, the
woman is healthy and there were no cases of hemophilia in her family. What
is the risk of having a sick child in this family?

1. 25%
2. 50%
3. 100%
4. 0
5. 75%

62. Question
During a prophylactic medical examination a 7-year-old boy was diagnosed
with daltonism. His parents are healthy and have normal colour vision, but
his grandfather on his mother’s side has the same abnormality. What is the
type of the abnormality inheritance?

1. Autosomal recessive
2. Semidominance
3. Autosomal dominant
4. Dominant, sex-linked
5. Recessive, sex-linked
LAKSHMAN

63. Question
2 years ago a patient underwent resection of pyloric part of stomach. He
complains of weakness, periodical dark shadows beneath his eyes, dyspnea.
In blood: Hb – 70 g/l, erythrocytes – 3,0×1012l, colour index – 0,7. What changes

LAKSHMAN
of erythrocytes in blood smears are the most typical for this condition?

1. Megalocytes
2. Macrocytes
3. Microcytes
4. Ovalocytes
5. Schizocytes

64. Question
A patient with tissue trauma was taken a blood sample for the determination
of blood clotting parameters. Specify the right sequence of extrinsic pathway
activation

1. III – VIII: TF – Xa
2. III – VIIa – Xa
3. IV – VIIa – Xa
4. IV – VIII: TF – Xa
5. III – IV – Xa

65. Question
A disaster fighter at a nuclear power plant developed hemorrhagic syndrome
on the background of acute radiation disease. What is the most important
factor of syndrome pathogenesis?

1. Decreased activity of coagulative factors


blood platelet count is low.
2. Thrombocytopenia
3. Increased activity of anticoagulative system factors
4. Increased activity of brinolysis factors
5. Vascular wall damage

66. Question
After a serious psycho-emotional stress a 45-year-old patient suddenly felt
constricting heart pain irradiating to the left arm, neck and left scapula. His
face turned pale, the cold sweat stood out on it. The pain attack was stopped
with nitroglycerine. What process has developed in this patient?

1. Myocardial infarction
2. Stomach ulcer perforation
3. Stenocardia
4. Psychogenic shock
5. Stroke

67. Question
A 32-year-old patient was admitted to the hospital with gross bloodloss due to
auto accident trauma. Ps – 110 Bpm, RR- 22 pm, BP- 100/60mm Hg. What
changes in the blood will occur in an hour after the bloodloss?

1. Hypochromia of erythrocytes
LAKSHMAN

2. Hypovolemia
3. Hypoproteinemia
4. Leukopenia

LAKSHMAN
5. Erythropenia

68. Question
A 49 y.o. woman consulted a doctor about heightened fatigue and dyspnea
during physical activity. ECG: heart rate is 50/min, PQ is extended, QRS is
unchanged, P wave quantity exceeds quantity of QRS complexes. What type of
arrhythmia does the patient have?

1. Sinoatrial block
2. Sinus bradycardia
3. Atrioventricular block
4. Extrasystole
5. Ciliary arhythmia

69. Question
A 27 y.o. patient put eye drops that contain penicillin. After a few minutes she
felt itching and burning of her body, there appeared lip and eye-lid edemata;
arterial pressure began to drop. What immunoglobulins took part in the
development of this allergic reaction?

1. IgG and IgD


2. IgM and IgG
3. IgA and IgM
4. IgM and IgD
5. IgE and IgG

70. Question
A 70-year-old patient suffers from atherosclerosis complicated by the lower
limb thrombosis that has caused gangrene on his left toes. What is the most
likely cause of the thrombosis origin?

1. Prothrombinase activation
2. Thrombocyte adhesion
3. Impaired heparin synthesis
4. Transformation of brinogen into brin
5. Transformation of prothrombin into thrombin

71. Question
Processes of repolarisation are disturbed in ventricular myocardium in
examined person. It will cause amplitude abnormalities of configuration and
duration of the wave:

1. S
2. P
3. Q
4. Т
5. R

72. Question
Prophylactic medical examination of a 36 year old driver revealed that his AP
LAKSHMAN

was 150/90 mm Hg. At the end of working day he usually hears ear noise, feels
slight indisposition that passes after some rest. He was diagnosed with
essential hypertension. What is the leading pathogenetic mechanism in this
case?

LAKSHMAN
1. Neurogenetic
2. Humoral
3. Endocrinal
4. Nephric
5. Re exogenic

73. Question
A healthy woman has three sons affected by color blindness who were born
after her two marriages. Children both of her husbands are healthy. What is
the most possible pattern of inheritance of this disease?

1. Autosomal dominant
2. X-linked dominant
3. Autosomal recessive
4. X-linked recessive
5. Y-linked

74. Question
As a result of increased permeability of the erythrocyte membrane in a
patient with microspherocytic anaemia (Minkowsky-Shauffard disease) cells
receive sodium ions and water. Erythrocytes take form of spherocytes and
can be easily broken down. What is the leading mechanism of erythrocyte
damage in this case?

1. Calcium
2. Electrolytic osmotic
3. Protein
4. Acidotic
5. Nucleic

75. Question
A patient who suffers from acute myocarditis has clinical signs of cardiogenic
shock. What of the under-mentioned pathogenetic mechanisms plays the main
part in shock development?

1. Decrease of vascular tone


2. Depositing of blood in organs
3. Disturbance of pumping ability of heart
4. Increase of peripheral vascular resistance
5. Reduction of diastolic ow to the heart

76. Question
A patient suffering from pheochromocytoma complains of thirst, dry mouth,
hunger. Blood test for sugar revealed hyperglycemia. What type of
hyperglycemia is it?

1. Adrenal
2. Somatotropic
LAKSHMAN

3. Hypoinsulinemic
4. Hypercorticoid
5. Alimentary

LAKSHMAN
77. Question
An infant has pylorospasm, weakness, hypodynamia, convulsions as a result
of frequent vomiting. What kind of acid-base disbalance is it?

1. Exogenous nongaseous acidosis


2. Excretory alkalosis
3. Gaseous alkalosis
4. Excretory acidosis
5. Metabolic acidosis

78. Question
A patient underwent a surgery for excision of a cyst on pancreas. After this he
developed haemorrhagic syndrome with apparent disorder of blood
coagulation. Development of this complication can be explained by:

1. Activation of anticoagulation system


2. Activation of brinolytic system
3. Activation of Christmas factor
4. Reduced number of thrombocytes
5. Insuf cient brin production

79. Question
A patient with the symptoms of acute alcoholic poisoning was brought to the
hospital. What carbohydrates metabolism changes are typical for this
condition?
1. The breakage of glycogen is increased in liver
2. The gluconeogenesis velocity in liver is decreased
3. The gluconeogenesis is increased in liver
4. The anaerobic glucose metabolism predominates in muscles
5. The anaerobic breakage of glucose is increased in muscles

80. Question
In a 45-year-old patient on ECG it was revealed: sinus rhythm, the number of
auricular complexesexceeds number of ventricular complexes; progressing
extension of the P-Q interval from complex to complex; fallout of some
ventricular complexes; Р waves and QRST complexes are without changes.
Name the type of heart rhythm disfunction

1. Intra-atrial block
2. Complete atrioventricular block
3. Synoauricular block
4. Atrioventricular block of the II degree
5. Atrioventricular blockade of the I degree

81. Question
Upper neck node of sympathetic trunk was removed from the rabbit on
experiment. Reddening and increased temperature of the skin of head is
observed. What form of peripheral circulation of the blood developed in the
LAKSHMAN

rabbit?

1. Stasis
2. Metabolic arterial hyperemia

LAKSHMAN
3. Venous hyperemia
4. Neurotonic arterial hyperemia
5. Neuroparalytic arterial hyperemia

82. Question
A woman who was sick with rubella during the pregnancy gave birth to a deaf
child with hare lip and cleft palate. This congenital defect is an example of:

1. Phenocopy

E-tri18
2. Edward’s syndrome
3. Patau’s syndrome
P-tri13
4. Genocopy D-tri21
5. Down’s syndrome Di george - deletion of 21 long arm
83. Question
Hepatitis has led to the development of hepatic failure. Mechanism of
edemata formation is activated by the impairment of the following liver
function:

1. Glycogen-synthetic
2. Protein-synthetic

Proteins in the blood tend to pull water into our blood vessels (acti
3. Chologenetic
4. Antitoxic ng like a "water magnet"). When the level of protein in the blood is
low, water may leave the blood vessels and collect in the tissues. W
ater in the tissues is called "edema". Critically ill patients develop e
5. Barrier

dema for many reasons.


84. Question
A 55 y.o. woman consulted a doctor about having continuous cyclic uterine
hemorrhages for a year, weakness, dizziness. Examination revealed skin
pallor. Hemogram: Hb- 70 g/l, erythrocytes – 3,2×1012/l, color index – 0,6,
leukocytes – 6,0×109/l, reticulocytes – 1%; erythrocyte hypochromia. What
anemia is it?

1. Hemolytic anemia
2. Aplastic anemia
3. Chronic post-hemorrhagic anemia
4. B12-folate-de ciency anemia
5. Iron-de ciency anemia

85. Question
A child was born with cleft palate. Examination revealed aorta defects and
reduced number of T-lymphocytes in blood. What immunodeficient syndrome
is it?

1. Chediak-Higashi
2. Louis-Bar
3. Swiss-type
4. Wiskott-Aldrich
5. DiGeorge
LAKSHMAN

86. Question
Violation of safety rules resulted in calomel intoxication. Two days later the
daily diuresis was 620 ml. A patient complained of headache, vomiting,

LAKSHMAN
convulsions, dyspnea, moist rales in lungs. What pathology is it?

Calomel, in high doses, led to mer


1. Pyelonephritis

cury poisoning
2. Glomerulonephritis
3. Chronic renal insuf ciency
4. Uraemic coma
5. Acute renal insuf ciency

87. Question
Examination of a 42 year old patient revealed a tumour of adenohypophysis.
Objectively: the patients weight is 117 kg, he has moon-like hyperemic face,
red-blue striae of skin distension on his belly. Osteoporosis and muscle
dystrophy are present. AP is 210/140 mm Hg. What is the most probable
diagnosis?

1. Diabetes mellitus
anterior pituitary (or adenohypop
2. Essential hypertension hysis
3. Cushings disease
4. Conns disease
5. Cushings syndrome

88. Question
A rabbits nerve that innervates the right ear was cut and its right superior
cervical ganglion was removed. Immediately after operation the temperature
of ear skin was measured. It was revealed that the temperature of the rabbits
ear skin on the side of denervation was by 1,50C higher than on the opposite
intact side. What of the following is the most probable explanation of the
above-mentioned effects?

1. Arterial neurotopical hyperemia


2. Atrerial hyperemia induced by metabolic factors
3. Arterial neuroparalytic hyperemia
4. Reactive arterial hyperemia
5. Physiological arterial hyperemia

89. Question
Rats being under stress have muscular hypertonia and high arterial pressure,
high glucose concentration in blood and intensified secretion of corticotropin
and corticosteroids. In what stress phase are these animals?

1. Shock phase Antishock phase: When the threat or stressor is identified


or realized, the body starts to respond and is in a state of
alarm. During this stage, the locus coeruleus and sympat
2. Erectile
3. Exhaustion hetic nervous system activate the production of catechola
4. Terminal mines
5. Antishock phase

90. Question
A child is pale, pastose, muscular tissue is bad developed, lymph nodes are
LAKSHMAN

enlarged. He often suffers from angina and pharyngitis, blood has signs of
lymphocytosis. The child is also predisposed to autoallergic diseases. What
type of diathesis can be presumed in this case?

LAKSHMAN
1. Lymphohypoplastic
2. Hemorrhagic elegant term for a predisposition or tendency. For e
xample, hemorrhagic diathesis means a tendency t
o bleed
3. Exudative

Gouty=urinary stones
4. Asthenic

Exudative=def of vit e
5. Gouty

91. Question
Blood test of a patient suffering from atrophic gastritis gave the following
results: RBCs – 2,0×1012/l, Hb- 87 g/l, colour index – 1,3, WBCs – 4,0×109/l,
thrombocytes – 180×109/l. Anaemia migh have been caused by the following
substance deficiency:

1. Iron
2. Zinc
3. Vitamin B12
4. Vitamin A Vitamin B12 or B9 (commonly called folate) deficie
5. Vitamin K ncy anaemia occurs when a lack of vitamin B12 or f
olate causes the body to produce abnormally large
92. Question red blood cells
A married couple came to the genetic counseling. The husband suffers from
the insulin-dependant diabetes, the wife is healthy. What is the probability
that this couple will have an insulin-dependant child?

1. The same as throughout the population


2. 100%
3. 50%
4. Higher than throughout the population
5. Lower than throughout the population

93. Question
A patient with obliterating atherosclerosis underwent sympathectomy of
femoral artery in the region of femoral trigone. What type of arterial
hyperemia was induced by the operation?

1. Neuroparalytic
2. Functional
3. Neurotonic
4. Reactive
5. Metabolic

94. Question
A patient suffering from stenocardia was taking nitroglycerine which caused
restoration of blood supply of myocardium and relieved pain in the cardiac
area. What intracellular mechanism provides restoration of energy supply of
insulted cells?

1. Intensi cation of RNA generation


2. Increased permeability of membranes
3. Reduction of ATP resynthesis
LAKSHMAN

4. Intensi cation of oxygen transporting into the cell


5. Intensi cation of ATP resynthesis

95. Question

LAKSHMAN
A patient was admitted to the infectious department. His symptoms: dry skin,
decreased skin turgor, rice-water stool. The patient was diagnosed with
cholera. What disorder of water-electrolytic balance is most often observed in
this disease?

1. Hypoosmotic hyperhydration
2. Isoosmotic hypohydration
3. Hyperosmotic hypohydration
4. Hyperosmotic hyperhydration
5. Hypoosmotic hypohydration

96. Question
In course of a preventive examination of a miner a doctor revealed changes of
cardiovascular fitness which was indicative of cardiac insufficiency at the
compensation stage. What is the main proof of cardiac compensation?

1. Rise of arterial pressure


2. Myocardium hypertrophy
Heart failure means that your heart muscle doesn't
p as much blood as your body needs. Because you
3. Cyanosis

cannot pump well, your heart and your body try to mak
4. Dyspnea
5. Tachycardia
up for it. This is called compensation. Your body has
97. Question markable ability to compensate for heart failure.
Examination of patients with periodontitis revealed the interdependence
between the rate of affection of periodontal tissues and the amount of
lysozymes in saliva and gingival liquid. These results can be obtained during
studying the following protection system of an organism:
Periodontitis also called gum disease, is a serious g
nfection that damages the soft tissue and, without tre
ment, can destroy the bone that supports your teeth
1. Humoral immunity
2. Autoresponsiveness

Nonspecific Resistance (Innate Immunity) The second l


3. Tolerance
4. Cellular immunity defense is nonspecific resistance that destroys invaders
generalized way without targeting specific individuals: Ph
5. Non-speci c resistance ocytic cells ingest and destroy all microbes that pass into b
y tissues.
98. Question
A patient was ill with burn disease that was complicated by DIC syndrome.
What stage of DIC syndrome can be suspected if it is known that the patients
blood coagulates in less than 3 minutes?
Disseminated intravascular coagulation (DIC) is a conditio
Transition phase n which blood clots form throughout the body, blocking s
ll blood vessels. Symptoms may include chest pain, s
1.
2. Fibrinolysis s of breath, leg pain, problems speaking, or problems
3. Terminal g parts of the body.
4. Hypercoagulation
5. Hypocoagulation

99. Question
Arterial pressure of a surgeon who performed a long operation raised up to
140/110 mm Hg. What changes of humoral regulation could have caused the
rise of arterial pressure in this case?
LAKSHMAN

70 and 100 mmHg to be normal. A


1. Activation of renin angiotensive system
2. Inhibition of sympathoadrenal system

LAKSHMAN
3. Activation of formation and excretion of aldosterone
4. Activation of sympathoadrenal system*
5. Activation of kallikrein kinin system

100. Question
A patient consulted a dentist about itching and burning in the oral cavity;
high temperature. The patient was diagnosed with trichomonal
gingivostomatitis. What drug should be chosen for his treatment?

1. Metronidazole
2. Nystatin Metronidazole, marketed under the brand
name Flagyl among others, is an antibiotic
3. Ampicillin and antiprotozoal medication. It is used eit
her alone or with other antibiotics to treat p
elvic inflammatory disease, endocarditis, a
4. Gentamicin sulfate
5. Doxycycline hydrochloride nd bacterial vaginosis. It is effective for dra
cunculiasis, giardiasis, trichomoniasis, and
101. Question amebiasis
A patient suffers from the haemorrhagic syndrome that shows itself in
frequent nasal bleedings, posttraumatic and spontaneous intracutaneous
and intra-articular haemorrhages. After a laboratory study a patient was
diagnosed with the type B haemophilia. This disease is provoked by the deficit
of the following factor of blood coagulation:

1. V
2. XI
3. IX
4. VII
5. VIII

102. Question
Tuberculine was injected intraperitoneally to the animal sensibilized with
tuberculine. Venous hyperemia and peritonial edema were detected on the
laparotomy in 24 hours. Increased amount of lymphocytes and monocytes
were in the smear-print from the peritonium. What pathological process is in
the animal?

1. Suppurative in ammation
2. Allergic in ammation
3. Aseptic in ammation
4. Fibrinous in ammation
5. Serous in ammation

103. Question
As a result of a road accident a 26-year-old man is in the torpid phase of
shock. Blood count: leukocytes – 3,2×109/l. What is the leading mechanism of
leukopenia development?

1. Faulty release of mature leukocytes from the bone marrow into the blood
2. Leukopoiesis inhibition
3. Leukocyte redistribution in the bloodstream
LAKSHMAN

4. Increased excretion of the leukocytes from the organism


5. Leukocyte destruction in the hematopietic organs

104. Question

LAKSHMAN
A 30 year old woman has face edemata. Examination revealed proteinuria
(5,87 g/l), hypoproteinemia, dysproteinemia, hyperlipidemia. What condition
is the set of these symptoms typical for?

1. Nephritic syndrome
2. Nephrotic syndrome
3. Acute renal failure Nephrotic syndrome is a condition that causes th
4. Chronic pyelonephritis e kidneys to leak large amounts of protein into th
e urine. This can lead to a range of problems, incl
uding swelling of body tissues and a greater chan
5. Chronic renal failure

105. Question ce of catching infections


A patient presents with icteritiousness of skin, scleras and mucous
membranes. Blood plasma the total bilirubin is increased, stercobilin is
increased in feces, urobilin is increased in urine. What type of jaundice is it?

1. Haemolytic
2. Cholestatic
3. Obturational
4. Gilberts disease
5. Parenchymatous

106. Question
A 59 year old patient is a plant manager. After the tax inspection of his plant
he felt intense pain behind his breastbone irradiating to his left arm. 15
minutes later his condition came to normal. Which of the possible
mechanisms of stenocardia development is the leading in this case?
1. Intravascular aggregation of blood corpuscles
2. High catecholamine concentration in blood*
3. Coronary atherosclerosis
4. Functional heart overload
5. Coronary thrombosis

107. Question
A newborn child with pylorostenosis has often repeating vomiting
accompanied by apathy, weakness, hypertonicity, sometimes convulsions.
What disorder form of acid-base balance is it?

1. Metabolic acidosis
2. Nongaseous alkalosis
3. Gaseous acidosis
4. Gaseous alkalosis
5. Excretory acidosis

108. Question
A 26 year old pregnant woman is treated at an in-patient hospital. After a
continuous attack of vomiting it was found reduced volume of her circulating
blood. What kind of changes in general blood volume will appear?

1. Oligocythemic hypervolemia
LAKSHMAN

2. Polycythemic hypovolemia
3. Simple hypovolemia condition characterized by an abn
ormally high number of red blood
cells in the blood.
4. Oligocythemic hypovolemia

LAKSHMAN
5. Polycythemic hypervolemia

109. Question
A 25 year old man has spent a long time in the sun under high air humidity. As
a result of it his body temperature rose up to 39 oC What pathological process
is it?

1. Hypothermia
2. Infectious fever
3. Hyperthermia
4. Noninfectious fever
5. Burn disease

110. Question
A chemical burn caused esophagus stenosis. Difficulty of ingestion led to the
abrupt loss of weight. In blood: 3,0×1012/l, Hb – 106 g/l, crude protein – 57 g/l.
What type of starvation is it?

1. Absolute
2. Proteinic
3. Complete
4. Water
5. Incomplete

111. Question
An infectious disease unit admitted a patient with signs of jaundice caused by
hepatitis virus. Select an indicator that is specific only for parenchymatous
jaundice:

1. Urobilinuria
2. Increase in ALT and AST rate
3. Bilirubinuria
4. Hyperbilirubinemia
5. Cholaemia

112. Question
A 58-year-old patient suffers from the cerebral atherosclerosis. Examination
revealed hyperlipoidemia. What class of lipoproteins will most probably show
increase in concentration in this patient’s blood serum?

1. Low-density lipoproteins
2. Fatty acid complexes with albumins
3. High-density lipoproteins
4. Chylomicrons
5. Cholesterol

113. Question
A driver who got a trauma in a road accident and is shocked has reduction of
LAKSHMAN

daily urinary output down to 300 ml. What is the main pathogenetic factor of
such diuresis change?

1. Drop of oncotic blood pressure

LAKSHMAN
2. Drop of arterial pressure
3. Decreased number of functioning glomerules
4. Secondary hyperaldosteronism
5. Increased vascular permeability

114. Question
A 56 year old patient came to a hospital with complaints about general
weakness, tongue pain and burning, sensation of limb numbness. In the past
he underwent resection of fore stomach. In blood: Hb- 80 g/l; erythrocytes –
2,01×1012/l; colour index – 1,2, leukocytes – 3,5×109/l. What type anemia is it?

1. Aplastic
2. Post-hemorrhagic
3. Hemolytic
4. B-12-foliate de cient
5. Iron-de cient

115. Question
A patient staying in the pulmonological department was diagnosed with
pulmonary emphysema accompanied by reduced elasticity of pulmonary
tissue. What type of respiration is observed?

1. Super cial respiration


2. Inspiratory dyspnea
3. Periodic respiration
4. Infrequent respiration
5. Expiratory dyspnea Expiration difficult
116. Question
A patient ill with enteritis accompanied by massive diarrhea has low water
rate in the extracellular space, high water rate inside the cells and low blood
osmolarity. What is such disturbance of water-electrolytic metabolism called?

1. Hypo-osmolar hypohydration
2. Hypo-osmolar hyperhydration
3. Hyperosmolar hypohydration
4. Hyperosmolar hyperhydration
5. Osmolar hypohydration

117. Question
A 23 y.o. patient complains of weakness, temperature rise up to 38-40°C.
Objectively: liver and spleen are enlarged. Hemogram: Hb- 100 g/l,
erythrocytes – 2,9×1012/l, leukocytes – 4,4×109/l, thrombocytes – 48×109/l,
segmentonuclear neutrophils – 17%, lymphocytes – 15%, blast cells – 68%. All
cytochemical reactions are negative. Make a hematological conclusion:

1. Chronic myeloleukosis
2. Acute myeloblastic leukosis
LAKSHMAN

3. Undifferentiated leukosis
4. Acute erythromyelosis
5. Acute lymphoblastic leukosis

LAKSHMAN
118. Question
After taking poor-quality food a patient developed repeated episodes of
diarrhea. On the next day he presented with decreased arterial pressure,
tachycardia, extrasystole. Blood pH is 7,18. These abnormalities were caused
by the development of:

1. Nongaseous alkalosis
2. Nongaseous acidosis
3. Gaseous acidosis
4. Metabolic alkalosis
5. Gaseous alkalosis

119. Question
A 5 year old child is ill with measles. Blood analysis revealed increase of total
number of leukocytes up to 13×109/l. Leukogram: basophils – 0, eosinophils – 1,
myelocytes – 0, juvenile neutrophils – 0, band neutrophils – 2, segmented
neutrophils – 41, lymphocytes – 28, monocytes – 28. Name this phenomenon:

1. Monocytosis
2. Neutropenia
3. Lymphocytosis
4. Agranulocytosis
5. Eosinopenia

120. Question
A couple had a child with Downs disease. Mother is 42 years old. This disease
is most probably caused by the following impairment of prenatal
development:
Fetopathy=Disease in a fetus after the mother's t
1. Embryopathy hird month of pregnancy
2. Non-speci c fetopathy
3. Speci c fetopathy
4. Gametopathy
5. Blastopathy

121. Question
A 45 year old patient was admitted to the cardiological department. ECG
data: negative P wave overlaps QRS complex, diastolic interval is prolonged
after extrasystole. What type of extrasystole is it?

1. Atrioventricular
2. Bundle-branch
3. Ventricular
4. Sinus
5. Atrial

122. Question
The patient with acute miocardial infarction was given intravenously
LAKSHMAN

different solutions during 8 hours with medical dropper 1500 ml and oxygen
intranasally. He died because of pulmonary edema. What caused the
pulmonary edema?

LAKSHMAN
1. Inhalation of the oxygen
2. Volume overload of the left ventricular
3. Allergic reaction
4. Neurogenic reaction
5. Decreased oncotic pressure due to hemodilution

123. Question
A 47 year old man with myocardium infarction was admitted to the
cardiological department. What changes of cellular composition of peripheral
blood are induced by necrotic changes in the myocardium?

1. Eosinophilic leukocytosis
2. Lymphopenia
3. Monocytosis
4. Thrombocytopenia
5. Neutrophilic leukocytosis

124. Question
A 50-year-old patient complains of thirst, drinking of a lot of water, marked
polyuria. Blood glucose is 4,8 mmol/L, urine glucose and acetone bodies are
absent, urine is colorless, specific gravity is 1,002 – 1,004. What is the cause of
polyuria?

1. Thyrotoxicosis
2. Insulin insuf ciency
3. Aldosteronism
4. Vasopressin insuf ciency
5. Hypothyroidism

125. Question
On the 6th day of treatment a patient with acute renal insufficiency developed
polyuria. Diuresis intensification at the beginning of polyuria stage of acute
renal insufficiency is caused by:

1. Growth of natriuretic factor


2. Volume expansion of circulating blood
3. Renewal of ltration in nephrons
4. Reduction of vasopressin content in plasma
5. Reduction of aldosteron content in plasma

126. Question
As a result of a trauma a patient has developed traumatic shock that led to
the following disorders: AP is 140/90 mm Hg, Ps is 120 bpm. The patient is
fussy, talkative, pale. Such state relates to the following shock phase:
torpid= arterial blood pressure falls, increased blood deposited fra
1. Latent periodction decreased CBV, pulse pressure, is often "thready" pulse; decr
eases alveolar ventilation
2. Terminal erectile=activation of circulatory system: tachycardia, hyperten
sion, blood flow redistribution; observed rapid breathing and a
n increase in alveolar ventilation; polycythemia
3. Torpid
4. Erectile
LAKSHMAN

127. Question
A patient was stung by a bee. Examination revealed that his left hand was hot,
pink, edematic, there was a big red blister on the site of sting. What is the

LAKSHMAN
leading mechanism of edema development?

1. Reduced vessel lling


2. Injury of vessels caused by the sting
3. Increased vessel permeability
4. Drop of osmotic pressure in tissue
5. Drop of oncotic pressure in tissue

128. Question
A 15 year old girl has pale skin, glossitis, gingivitis. Blood count: erythrocytes
– 3,3×1012/l, hemoglobin – 70 g/l, colour index – 0,5. Examination of blood
smear revealed hypochromia, microcytosis, poikilocytosis. What type of
anemia is it?

1. Sickle-cell
2. B12-folic acid-de cient
3. Thalassemia
4. Iron-de cient
5. Hemolytic

129. Question
A patient has been diagnosed with acute glomerulonephritis that developed
after he had had streptococcal infection. It is most likely that the affection of
basal glomerular membrane is caused by an allergic reaction of the following
type:
1. Anaphylactic
2. Cytotoxic
3. Immune complex
4. Delayed
5. Stimulating

130. Question
Inflammatory processes cause synthesis of protein of acute phase in an
organism. What substances stumulate their synthesis?

1. Biogenic amins
2. Interferons
3. Immunoglobulins
4. Interleukin-1
5. Angiotensin

131. Question
A patient with massive burns developed acute renal insufficiency
characterized by a significant and rapid deceleration of glomerular filtration.
What is the mechanism of its development?

1. Rise of pressure of tubular uid


LAKSHMAN

2. Damage of glomerular lter


3. Renal artery embolism
4. Reduction of renal blood ow
5. Reduction of functioning nephron number

LAKSHMAN
132. Question
A patient was diagnosed with autoimmune hemolitic cytotoxic anemia. What
substances are antigens in II type allergic reactions?

1. Hormones
2. Antibiotics
3. Modi ed receptors of cell membranes
4. Serum proteins
5. In ammation modulators

133. Question
A patient with skin mycosis has disorder of cellular immunity. The most
typical characteristic of it is reduction of the following index:

1. T-lymphocytes
2. Immunoglobulin E
3. B-lymphocytes
4. Plasmocytes
5. Immunoglobulin G

134. Question
Examination of a child who frequently suffers from infectious diseases
revealed that IgG concentration in blood serum was 10 times less than
normal, IgA and IgM concentration was also significantly reduced. Analysis
showed also lack of B-lymphocytes and plasmocytes. What disease are these
symptoms typical for? Bruton agammaglobulinemia, also known as X-linked agammaglobuline
mia (XLA) or Bruton's agammaglobulinemia, is an inherited immunod
ciency disorder. It is characterized by the absence of mature B cells whic
1. Louis-Bar syndrome h in turn leads to severe antibody deficiency and recurrent infections
2. Swiss-type agammaglobulinemia
3. Dysimmunoglobulinemia
4. Brutons disease
5. Di George syndrome

135. Question
A 28 year old man had a gunshot wound of shin that resulted in an ulcer from
the side of the injury. What is the main factor of neurodystrophy pathogenesis
in this case?

1. Psychical stress
2. Infection
3. Tissue damage
4. Microcirculation disturbance
5. Traumatization of peripheral nerve

136. Question
A 62 year old patient who previously worked as stoker was admitted to a
hospital with complaints about general weakness, abrupt weight loss, hoarse
LAKSHMAN

voice, dyspnea, dry cough. Laryngoscopy revealed a tumour in the pharynx


that invaded vocal cords and epiglottis. What is the most probable cause of
tumour development?

LAKSHMAN
1. Retroviruses
2. Nitrosamines
3. Ionizing radiation
4. Polycyclic aromatic carbohydrates
5. Aromatic amines and amides

137. Question
An unconscious young man with signs of morphine poisoning entered
admission office. His respiration is shallow and infrequent which is caused by
inhibition of respiratory centre. What type of respiratory failure is it?

1. Ventilative dysregulatory
2. Diffusive
3. Ventilative restrictive
4. Ventilative obstructive
5. Perfusive

138. Question
A patient ill with essential arterial hypertension had a hypertensic crisis that
resulted in an attack of cardiac asthma. What is the leading mechanism of
cardiac insufficiency in this case?

1. Heart overload caused by increased blood volume


2. Heart overload caused by high pressure
3. Myocardium damage
4. Absolute coronary insuf ciency
5. Blood supply disturbance

139. Question
There are several groups of molecular mechanisms playing important part in
pathogenesis of insult to cells which contributes to the pathology
development. What processes are stimulated by proteinic damage
mechanisms?

1. Enzyme inhibition
2. Osmotic membrane distension
3. Acidosis
4. Lipid peroxidation
5. Phospholipase activation

140. Question
Two hours after an exam a student had a blood count done and it was
revealed that he had leukocytosis without significant leukogram
modifications. What is the most probable mechanism of leukocytosis
development?

1. Deceleration of leukocyte migration to the tissues


2. Leukopoiesis intensi cation and deceleration of leukocyte lysis
LAKSHMAN

3. Leukopoiesis intensi cation


4. Deceleration of leukocyte lysis
5. Redistribution of leukocytes in the organism

LAKSHMAN
141. Question
Examination of a patient admitted to the surgical department with symptoms
of acute appendicitis revealed the following changes in the white blood cells:
the total count of leukocytes is 16×109/l. Leukocyte formula: basophils – 0,
eosinophils – 2%, juvenile forms – 2%, stabnuclear – 8%, segmentonuclear –
59%, lymphocytes – 25%, monocytes- 4%. The described changes can be
classified as: presence of band (immature) neutrophils in circulation is termed a rege
nerative left shift and indicates a bone marrow response to an inflammat
ory stimulus. When band cells outnumber mature neutrophils in the per
1. Neutrophilia with right shift ipheral blood, the condition is referred to as a degenerative left shif

2. Neutrophilic leukemoid reaction


3. Neutrophilia with hyperregenerative left shift
4. Neutrophilia with degenerative left shift
5. Neutrophilia with regenerative left shift

142. Question
An experimental rat with extremity paralysis has no tendon and cutaneous
reflexes, muscle tone is decreased, but muscles of the affected extremity
maintain their ability to react with excitation to the direct action of
continious current. What type of paralysis is it?

1. Extrapyramidal
2. Flaccid peripheral
3. Spastic peripheral Flaccid paralysis causes your muscles to shrink and be
4. Flaccid central come flabby. It results in muscle weakness.
5. Spastic central Spastic paralysis involves tight and hard muscles. It ca
n cause your muscles to twitch uncontrollably, or spas
m
143. Question
A 25 year old Palestinian woman complains of weakness, dizziness, dyspnea.
In anamnesis: periodically exacerbating anemia. In blood: Hb – 60 g/l,
erythrocytes – 2,5×1012/l, reticulocytes – 35o/oo, anisocytosis and
poikilocytosis of erythrocytes, a lot of target cells and polychromatophils.
What type of anemia is it? polychromatophil is an immature red blood cell which ha
s already lost its nucleus. ... An elevated number of polyc
hromatophils suggests an increased production of erythr
ocytes by the bone marrow (erythroid hyperplasia) in res
1. Sickle-cell anemia
2. Addison-Biermer disease ponse to anemia
3. Glucose 6-phosphate dehydrogenase-de cient anemia
4. Minkowsky-Shauffard disease
5. Thalassemia

144. Question
Examination of a 12 year old boy with developmental lag revealed
achondroplasia: disproportional constitution with evident shortening of
upper and lower limbs as a result of growth disorder of epiphyseal cartilages
of long tubal bones. This disease is:

Achondroplasia is a form of short-limbed dwarfism. The


1. Inherited, recessive word achondroplasia literally means "without cartilage fo
Inherited, sex-linked rmation." Cartilage is a tough but flexible tissue that mak
es up much of the skeleton during early development.
2.
3. Acquired
4. Inherited, dominant
5. Congenital
LAKSHMAN

145. Question
Daltonism was diagnosed in a 7-year-old boy while prophylactic medical
examination. Parents are healthy, color vision is normal. Grandfather from
the mothers side has the same disorder. What is the type of inheriting of this

LAKSHMAN
anomaly?

1. Autosomal-recessive
2. Autosomal-dominant
3. Dominant, connected with sex
4. Recessive, connected with sex
5. Incomplete domination

146. Question
A patient who had been working hard under conditions of elevated
temperature of the environment, has now a changed quantity of blood plasma
proteins. What phenomenon is the case?

1. Absolute hyperproteinemia
2. Relative hyperproteinemia
3. Absolute hypoproteinemia
4. Paraproteinemia
5. Disproteinemia

147. Question
A patient with nephrotic syndrome has massive edemata of his face and
limbs. What is the leading pathogenetic mechanism of edemata development?
Nephrotic syndrome is a condition that causes the ki
dneys to leak large amounts of protein into the urine
. This can lead to a range of problems, including swe
1. Lymphostasis
2. Rise of hydrodynamic blood pressure lling of body tissues and a greater chance of catchin
g infections
3. Drop of oncotic blood pressure
4. Increase of vascular permeability Oncotic pressure, or colloid osmotic-pressure, i
s a form of osmotic pressure induced by the pro
teins, notably albumin, in a blood vessel's plas
5. Increase of lymph out ow

148. Question ma
24 hours after appendectomy patients blood analysis showed neutrophilic
leukocytosis with regenerative shift. What is the most probable mechanism of
leukocytosis development?

1. Ampli cation of leukopoiesis and decelerated emigration of leukocytes to the tissues


2. Redistribution of leukocytes in the organism
3. Decelerated leukocyte destruction
4. Deceleratied emigration of leukocytes to the tissues
5. Ampli cation of leukopoiesis

149. Question
A 42 year old woman with neuralgia of trifacial nerve complains about
periodical reddening of the right part of her face and neck, sense of warmth
gush, increased skin sensitivity. These effects can be explained by the
following type of arterial hyperemia:

1. Neurotonic
2. Reactive Leukopoiesis is the process through which leukocytes are
generated from haematopoietic stem cells in the bone ma
LAKSHMAN

rrow. ...
3. Functional
4. Neuroparalytic Myelopoiesis generates cells of the innate immune system
, whereas lymphopoiesis gives rise to the cells of the adapt
ive immune system.
5. Metabolic

LAKSHMAN
150. Question
A 26 year old man is in the torpid shock phase as a result of a car accident. In
blood: 3,2×109/l. What is the leading mechanism of leukopenia development?

1. Leikopoiesis inhibition
2. Redistribution of leukocytes in bloodstream
3. Lysis of leukocytes in the blood-forming organs
4. Intensi ed elimination of leukocytes from the organism
5. Disturbed going out of mature leukocytes from the marrow into the blood

151. Question
A patient being treated for viral hepatitis type B got symptoms of hepatic
insufficiency. What blood changes indicative of protein metabolism disorder
will be observed in this case?

1. Absolute hypoalbuminemia
2. Absolute hyperalbuminemia
3. Absolute hyper brinogenemia
4. Absolute hyperglobulinemia
5. Proteinic blood composition is unchanged

152. Question
A 67 year old patient complains of periodic heart ache, dyspnea during light
physical activities. ECG reveals extraordinary contractions of heart
ventricles. Such arrhythmia is called:
1. Flutter
2. Fibrillation
3. Bradycardia
4. Extrasystole
5. Tachycardia

153. Question
A patient has been diagnosed with influenza. His condition became
drastically worse after taking antipyretic drugs. His consciousness is
confused, AP is 80/50 mm Hg, Ps is 140/m, body temperature droped down to
35,8°C. What complication developed in this patient?
antipyretic is a substance that reduces fever. Antipyretics cause the hyp
othalamus to override a prostaglandin-induced increase in temperature.
The body then works to lower the temperature, which results in a reduc
1. Hypovolemia
2. Alkalosis tion in fever. Most antipyretic medications have other purposes
3. Collapse
4. Acidosis
5. Hyperthermia

154. Question
Parents of a 3 year old child have been giving him antibiotics with purpose of
preventing enteric infections for a long time. A month later the child’s
condition changed for the worse. Blood examination revealed apparent
leukopenia and granulocytopenia. What is the most probable mechanism of
LAKSHMAN

blood changes?

1. Myelotoxic
2. Autoimmune
Inhibitory, depressant, or destruc

LAKSHMAN
tive to a component of bone marr
3. Age-speci c

ow. Relating to, derived from, or


4. Redistributive
5. Hemolytic
having the features of diseased bo
155. Question ne marrow.
A 70 year old man is ill with vascular atherosclerosis of lower extremities and
coronary heart disease. Examination revealed disturbance of lipidic blood
composition. The main factor of atherosclerosis pathogenesis is the excess of
the following lipoproteins:

1. Cholesterol
2. Chylomicrons
3. Intermediate density lipoproteins
4. High-density lipoproteins
5. Low-density lipoproteins

156. Question
While having the dinner the child choked and aspirated the food. Heavy cough has started, skin and
mucosa are cyanotic, rapid pulse, rear breathing, expiration is prolonged. What disorder of the
external breathing developed in the child? Dyspnea, also called shortness of breath,
is a tig
ht feeling in your chest where you may not be a
1. Biots breathing ble to take a deep breath
2. Stage of expiratory dyspnea on asphyxia

Asphyxiation, also called asphyxia or suffocation, is when th


3. Stage of inspiratory dyspnea on asphyxia
4. Alternating breathinge body doesn't get enough oxygen. Without immediate inter
vention, it can lead to loss of consciousness, brain injury, or
death
5. Stenotic breathing

157. Question
A 45 year old woman is ill with breast cancer. Her left arm has symptoms of
lymphatic system insufficiency – limb edema, lymph node enlargement. What
form of lymphatic circulation insufficiency is it?

1. Combined insuf ciency


2. Resorption insuf ciency
3. Dynamic insuf ciency
4. Mechanic insuf ciency

Created by Eneutron Team

Kiev eneutron.info@gmail.com
LAKSHMAN

LAKSHMAN
Krok 1 – 2014 Biology Base
1. A man suffering from a hereditary disease married a healthy woman. They got
5 children, three girls and two boys. All the girls inherited their father’s disease.
What is the type of the disease inheritance?
A. Dominant, X-linked
B. Autosomal recessive
C. Asutosomal dominant
D. Y-linked
E. Recessive, X-linked

2. A couple has a son with haemophilia. The parents are healthy but the maternal
grandfather also has haemophilia. Specify the type of inheritance:
A. Recessive sex-linked
B. Recessive autosomal
C. Dominant sex-linked
D. Semidominance
E. Autosomal dominant

3. Examination of newborns in one of the Ukrainian cities revealed a baby with


phenylketonuria. The baby's parents don't suffer from this disease and have
two other healthy children. Specify the most likely parents' genotype with
phenylketonuria gene:
A. Аа х Аа
B. АА х аа
C. аа х аа
D. Аа х аа
E. Аа х АА

4. It was proved that a molecule of immature mRNA (precursor mRNA)


contained more triplets than amino acids found in the synthesized protein. The
reason for that is that translation is normally preceded by:
A. Processing
B. Initiation
C. Reparation
D. Mutation
E. Replication

5. Examination of a patient revealed reduced contents of magnesium ions that


are necessary for attachment of ribosomes to the granular endoplasmatic
reticulum. It is known that it causes disturbance of protein biosynthesis. What
stage of protein biosynthesis will be disturbed?
A. Translation
B. Transcription
C. Replication
D. Aminoacid activation
E. Termination

6. According to the data of WHO, for about 250 mln of Earth population fall ill
with malaria. This disease is mostly spread in tropical and subtropical regions.
Range of its spread falls into the areal of the following mosquitoes:
A. Anopheles
B. Culex
C. Aedes
D. Mansonia
E. Culiseta

7. Labeled aminoacids alanine and tryptophane were introducted to a mouse in


order to study localization of protein biosynthesis in its cells. Around what
organellas will the accumulation of labeled aminoacids be observed?
A. Ribosomes
B. Agranular endoplasmic reticulum
C. Cell centre
D. Lysosomes
E. Golgi apparatus

8. In some regions of South Africa there is a spread sickle-shaped cell anemia, in


which erythrocytes have shape of a sickle as a result of substitution of glutamin
by valine in the hemoglobin molecule. What is the cause of this disease?
A. Gene mutation
B. Disturbance of mechanisms of genetic information realization
C. Crossingover
D. Genomic mutations
E. Transduction

9. A female patient consulted a physician about digestive disorder, extended


abdominal pain. Examination revealed drastic decrease in hemoglobin
concentration. It is known from the anamnesis that while living in the Far East
the patient used to eat freshly -salted caviar. Some relatives living with her had
the similar condition. What is the most likely diagnosis?
A. Diphyllobothriasis
B. Echinococcosis
C. Teniasis
D. Trichiniasis
E. Ascaridiasis

10. In the perianal folds of a 5-year-old girl her mother has found some white
"worms" that caused itch and anxiety in the child. The "worms" were sent to
the laboratory. During examination the physician saw white filiform helminths
0,5-1 cm long, with pointed ends, some helminths had twisted ends. What is
the most likely diagnosis?
A. Enterobiasis
B. Diphyllobothriasis
C. Teniasis
D. Ascaridiasis
E. Opisthorchiasis

11. A pregnant woman had been having toxicosis with severe repeated vomiting
for 24 hours. In the end of the day there appeared tetanic convulsions and fluid
loss. What shift of acid-base state caused these changes?
A. Excretory alkalosis
B. Gaseous alkalosis
C. Gaseous acidosis
D. Metabolic acidosis
E. Excretory acidosis

12. Medical examination at the military registration and enlistment office revealed
that a 15-year-old boy was high, with eunuchoid body proportions,
gynecomastia, female pattern of pubic hair distribution. The boy had also fat
deposits on the thighs, no facial hair, high voice, subnormal intelligence
quotient. Which karyotype corresponds with this disease?
A. 47, XXY
B. 45, XO
C. 46, XX
D. 46, XY
E. 47, XXX

13. A 26-year-old female consulted a doctor about having stool with white flat
moving organisms resembling noodles. Laboratory analysis revealed
proglottids with the following characteristics: long, narrow, with a longitudinal
channel of the uterus with 17-35 lateral branches on each side. What kind of
intestinal parasite was found?
A. Taeniarhynchus saginatus
B. Taenia solium
C. Hymenolepis nana
D. Diphyllobothrium latum
E. Echinococcus granulosus

14. RNA that contains AIDS virus penetrated into a leukocyte and by means of
reverse transcriptase forced a cell to synthetize a viral DNA. This process is
based upon:
A. Reverse transcription
B. Operon repression
C. Reverse translation
D. Operon depression
E. Convariant replication

15. According to the model of double DNA helix that was suggested by Watson and
Creek, it was established that one of chains would not be lost during replication
and the second chain would be synthesized complementary to the first one.
What way of replication is it?
A. Semiconservative
B. Analogous
C. Identical
D. Dispersed
E. Conservative

16. A teenager was irradiated with high radiation dose that resulted in serious
damages of lymphoid system, lysis of many lymphocytes. Restoration of
normal hemogram is possible due to the functioning of the following gland:
A. Thymus
B. Thyroid
C. Liver
D. Pancreas
E. Adrenal

17. It was revealed that T-lymphocytes were affected by HIV. Virus enzyme -
reverse transcriptase (RNA-dependent DNA polymerase) - catalyzes the
synthesis of:
A. DNA on the matrix of virus mRNA
B. Virus informational RNA on the matrix of DNA
C. DNA on virus ribosomal RNA
D. Viral DNA on DNA matrix
E. mRNA on the matrix of virus protein

18. Autopsy of a newborn boy revealed polydactylia, microcephalia, cheiloschisis


and uranoschisis as well as hypertrophy of parenchimatous organs. These
defects correspond with the description of Patau's syndrome. What is the most
probable cause of this pathology?
A. Trisomy of the 13th chromosome
B. Trisomy of the 18th chromosome
C. Trisomy of the 21st chromosome
D. Nondisjunction of sex chromosomes
E. Partial monosomy

19. An ovary specimen stained by hematoxylin-eosin presents a follicle, where


cells of follicular epithelium are placed in 1 -2 layers and have cubic form, there
is a bright-red membrane around the ovocyte. What follicle is it?
A. Primary
B. Primordial
C. Secondary
D. Mature
E. Atretic
20. A patient has symptoms of inflammation of urogenital tracts. Examination of
a vaginal smear revealed big monocellular, pear-shaped organisms with the
pointed spike at the posterior end of body, big nucleus and undulating
membrane. What protozoa were found in the smear?
A. Trichomonas vaginalis
B. Trichomonas hominis
C. Trichomonas buccalis
D. Trypanosoma gambiense
E. Lamblia intestinalis

21. During regular examination of schoolchildren, it was revealed that a 10-year-


old girl had asymmetric oval eggs with a larva in the scrape from her perianal
folds. What diagnosis should be made?
A. Enterobiasis
B. Ascariasis
C. Amebiasis
D. Trichocephalosis
E. Ankylostomiasis

22. A woman delivered a dead child with multiple developmental defects. What
protozoan disease might have caused the intrauterine death?
A. Toxoplasmosis
B. Leishmaniasis
C. Malaria
D. Amebiasis
E. Lambliasis

23. Tuberculosis can be treated by means of combined chemotherapy that includes


substances withdifferent mechanisms of action. What antituberculous
medication inhibits transcription of RNA into DNA in mycobacteria?
A. Rifampicin
B. Isoniazid
C. Streptomycin
D. Ethionamide
E. Para-aminosalicylic acid

24. A patient experienced a sudden temperature rise up to 39 0 С. After 6 hours the


temperature normalized. On the 2-nd day the attack recurred: in the period of
paroxysm the temperature reached 41 0С, apyrexial period began after 8 hours.
What type of temperature profile is it?
A. Intermitting
B. Recurrent
C. Hectic
D. Septic
E. Continued
25. Larvae were detected occasionally on the microscopic examination of the
sputum of the patient with pneumonia. Eosinophiles were detected on the
blood examination. What helminthiasis can be diagnosed?
A. Ascariasis
B. Enterobiosis
C. Trichocephaliasis
D. Paragonimiasis
E. Opistorchis

26. You are studying functioning of a bacteria operon. The operator gene has been
released from the repressor gene. Immediately after this the following process
will start in the cell:
A. Transcription
B. Translation
C. Replication
D. Processing
E. Repression

27. While studying maximally spiralized chromosomes of human karyotype the


process of cell division was stopped in the following phase:
A. Metaphase
B. Prophase
C. Interphase
D. Anaphase
E. Telophase

28. It is known that the gene responsible for development of blood groups
according to AB0 system has three allele variants. If a man has IV blood group,
it can be explained by the following variability form:
A. Combinative
B. Mutational
C. Phenotypic
D. Genocopy
E. Phenocopy

29. A patient complains of pain in the area of his liver. Duodenal intubation
revealed yellowish, oval, narrowed at the poles eggs with an operculum at the
end. Size of these eggs is the smallest among all helminth eggs. What is the
most probable diagnosis?
A. Opisthorchosis
B. Teniasis
C. Beef tapeworm infection
D. Echinococcosis
E. Diphyllobothriasis

30. A patient consulted a urologist about pain during urination. Analysis of his
urine taken in the daytime revealed eggs with a characteristic sharp point. It is
known from the anamnesis that the patient has recently returned from
Australia. What is the most likely diagnosis?
A. Urogenital schistosomiasis
B. Intestinal schistosomiasis
C. Japanese schistosomiasis
D. Opisthorchiasis
E. Dicroceliasis

31. A child complains of general weakness, loss of appetite, a troubled sleep,


itching in the perianal area. The provisional diagnosis is enterobiasis. In order
to specify this diagnosis, it is necessary to perform:
A. Scraping from perianal folds
B. Roentgenoscopy
C. Biopsy of muscle tissue
D. Immune diagnostics
E. Duodenal contents analysis

32. In course of practical training students studied a stained blood smear of a


mouse with bacteria phagocyted by leukocytes. What cell organella completes
digestion of these bacteria?
A. Lisosomes
B. Mytochondrions
C. Granular endoplasmic reticulum
D. Golgi apparatus
E. Ribosomes

33. A woman with ІІІ (B), Rh- blood group born a child with ІІ (A) blood group.
The childis diagnosed with hemolytic disease of newborn as a result of rhesus
incompatibility. What blood group is the child's father likely to have?
A. ІІ (A), Rh+
B. I (0), Rh+
C. III (B), Rh+
D. I (0), Rh-
E. II (A), Rh-

34. According to the phenotypic diagnosis a female patient has been provisionally
diagnosed with X-chromosome polysomia. This diagnosis can be confirmed by
a cytogenetic method. What karyotype will allow to confirm the diagnosis?
A. 47(ХХХ)
B. 48(XXXY)
C. 48(XXYY)
D. 47(XXY)
E. 46(XX)

35. Two days after consumption of smoked pork a patient got face and eye-lid
edemata, gastrointestinal disturbances, abrupt temperature rise, muscle pain.
Blood analysis showed full-blown eosinophilia. What helminth could the
patient be infected with?
A. Trichina
B. Pinworm
C. Ascarid
D. Whipworm
E. Hookworm

36. A lymph node punctate of a patient with suspected protozoal disease was
examined. Examination of the stained specimen (Romanovsky's stain)
revealed some crescent bodies withpointed end, blue cytoplasm and red
nucleus. What protozoan were revealed in the smears?
A. Toxoplasms
B. Malarial plasmodiums
C. Dermotropic leishmania
D. Viscerotropic leishmania
E. Trypanosomes

37. A patient complains of skin itch, especially between fingers, in the inguinal
creases, on the lower abdomen. Examination of these regions revealed there
some small vesicles. Laboratory diagnostics allowed to establish that this
condition had been caused by a representative of Arthropoda. Specify the
disease caused by this arthropod:
A. Scabies
B. Demodicosis
C. Myiasis
D. Pediculosis
E. Dermatotropic leishmaniasis

38. A patient has acne on his face. Microspcopic examination of scrapings from the
affected areas revealed living porrect vermiform arthropoda 0,2-0,5 mm large
with four pairs of short extremities in the front part of their bodies. What is the
laboratory diagnosis?
A. Demodicosis
B. Scabies
C. Myiasis
D. Pediculosis
E. Phthiriasis

39. A family of students who came from Africa got a child with anemia signs. The
child died soon. Examination revealed that the child's erythrocytes have
abnormal semilunar shape. Specify genotypes of the child's parents:
A. Аа х Аа
B. Аа х аа
C. АА х АА
D. аа х аа
E. Аа х АА

40. Examination of cell culture got from a patient with lysosomal pathology
revealed accumulation of great quantity of lipids in the lysosomes. What of the
following diseases is this disturbance typical for?
A. Tay-Sachs disease
B. Gout
C. Phenylketonuria
D. Wilson disease
E. Galactosemia

41. A 39-year-old woman has madescence in the region of mammilla, a small ulcer
with inflammatory hyperemia and cutaneous edema. Histologic examination
of tissue sampling from this area revealed in the malpighian layer of thickened
epidermis atypical cells with light and optically empty cytoplasm, with no
intracellular bridges. Such cells were also found in the orifice of big mammal
gland ducts. What is the most probable diagnosis?
A. Paget's disease
B. Intraductal cancer
C. Basal cell carcinoma
D. Epidermoid cancer
E. Melanocarcinoma

42. Golgi complex exports substances from a cell due to the fusion of the
membrane saccule with the cell membrane. The saccule contents flows out.
What process is it?
A. Exocytosis
B. Endocytosis
C. Active transport
D. Facilitated diffusion
E. All answers are false

43. A patient working at a pig farm complains about paroxysmal abdominal pain,
liquid feces with admixtures of mucus and blood, headache, weakness, fever.
Examination of large intestine revealed ulcers from 1 mm up to several cm
large, feces contained oval unicellular organisms with cilia. What disease
should be suspected?
A. Balantidiasis
B. Amebiasis
C. Toxoplasmosis
D. Lambliasis
E. Trichomoniasis

44. A boy found a spider with the following morphological characteristics: it is 2


cm long, has roundish black abdomen with two rows of red spots on its dorsal
side; four pairs of jointed limbs are covered with small black hairs. What
arthropod is it?
A. Karakurt spider
B. Scorpion
C. Solpuga
D. Mite
E. Tarantula

45. Normal, actively dividing cells of human red bone marrow are analyzed. What
number of cells' chromosomes is typical for G1 period?
A. 46
B. 48
C. 47
D. 45
E. 23

46. A man has worked in an African country for 3 years. A month after his return
to Ukraine he consulted an ophthalmologist and complained about eye ache,
eyelid edema, lacrimation and temporary visual impairment. Underneath the
eye conjunctiva the doctor revealed helminths 30-50 mm long with elongated
filiform body. What diagnosis might be suspected?
A. Filariasis
B. Diphyllobothriasis
C. Ascaridiasis
D. Enterobiasis
E. Trichocephaliasis

47. Life cycle of a cell includes the process of DNA autoreduplication. As a result
of it monochromatid chromosomes turn into bichromatid ones. What period
of cell cycle does this phenomenon fall into?
A. S
B. Go
C. G1
D. G2
E. M

48. An 18-year-old male has been diagnosed with Marfan syndrome. Examination
revealed a developmental disorder of connective tissue and eye lens structure,
abnormalities of the cardiovascular system, arachnodactylia. What genetic
phenomenon has caused the development of this disease?
A. Pleiotropy
B. Complementarity
C. Codominance
D. Multiple allelism
E. Incomplete dominance

49. A 28-year-old female patient consulted a gynecologist about sterility.


Examination revealed underdeveloped ovaries and uterus, irregular menstrual
cycle. Analysis of the sex chromatin revealed 2 Barr’s bodies in most somatic
cells. What chromosome disease is most likely?
A. Triple X syndrome
B. Edwards' syndrome
C. Patau's syndrome
D. Klinefelter's syndrome
E. Turner's syndrome

50. A cell at the stage of mitosis anaphase was stimulated by colchicine that
inhibits chromosome separation to the poles. What type of mutation will be
caused?
A. Polyploidy
B. Inversion
C. Deletion
D. Duplication
E. Translocation

51. Examination of duodenal contents revealed some pyriform protozoa with twin
nuclei and four pairs of flagella. There were two supporting filaments between
the nuclei and a suctorial disc on the ventral side. What representative of
protozoa was revealed in this patient?
A. Lamblia
B. Toxoplasma
C. Leishmania
D. Intestinal trichomonad
E. Trypanosome

52. A shepherd who has tended sheep together with dogs consulted a doctor about
pain in his right subcostal area, nausea, vomiting. Roentgenoscopy revealed a
tumour-like formation. What kind of helminthiasis might be suspected?
A. Echinococcosis
B. Ascaridiasis
C. Enterobiasis
D. Taeniarhynchosis
E. Taeniasis

53. A patient in a transplantation centre underwent heart transplantation. The


organ was taken from a donor who died in a road accident. Foreign heart can
be rejected as a result of development of transplantation immunity. It is
usually prevented by means of:
A. Immunosuppressors
B. Chemotherapy
C. Ultrasound
D. Enzymes
E. X-ray therapy
54. Following exposure to radiation a lot of mutant cells appeared in a patient.
Some time later most of them were detected and destroyed by the following
cells of the immune system:
A. T-lymphocytes-killers
B. Plasmoblasts
C. T-lymphocytes-supressors
D. B-lymphocyte
E. Stem cells

55. A gynaecologist was examining a patient and revealed symptoms of genital


tract inflammation. A smear from vagina contains pyriform protozoa with a
spine, flagella at their front; there is also an undulating membrane. What
disease can be suspected?
A. Urogenital trichomoniasis
B. Lambliasis
C. Intestinal trichomoniasis
D. Toxoplasmosis
E. Balantidiasis

56. A boy has I (I 0 I0) blood group and his sister has IV (I A IB) blood group. What
blood groups do their parents have?
A. II (I А I 0) and III (I ВI 0)
B. II (I А I А ) and III (I ВI 0 )
C. I (I 0 I 0 ) and IV (I A I B)
D. III (I ВI 0 ) and IV (I A I B)
E. I (I 0 I 0 ) and III (I ВI 0 )

57. For the purpose of myocardium infarction treatment, a patient was injected
with embryonal stem cells derived from this very patient by means of
therapeutic cloning. What transplantation type is it?
A. Autotransplantation
B. Allotransplantation
C. Xenotransplantation
D. Isotransplantation
E. Heterotransplantation

58. An alcoholic woman has born a girl with mental and physical developmental
lag. Doctors diagnosed the girl with fetal alcohol syndrome. What effect is the
cause of the girl's state?
A. Teratogenic
B. Mutagenic
C. Malignization
D. Carcinogenic
E. Mechanic
59. Sex chromosomes of a woman didn't separate and move to the opposite poles
of a cell during gametogenesis (meiosis). The ovum was impregnated with a
normal spermatozoon. Which chromosomal disease can be found in her child?
A. Turner's syndrome
B. Down's syndrome
C. Patau's syndrome
D. Edwards' syndrome
E. Cat cry syndrome

60. Hartnup disease is caused by point mutation of only one gene which results in
disturbance of tryptophane absorption in the bowels and its resorption in the
renal tubules. It is the reason for disorder of both digestive and urination
systems. What genetic phenomenon is observed in this case?
A. Pleiotropy
B. Complementary interaction
C. Polymery
D. Codominance
E. Semidominance

61. Cytogenetic examination of a patient with reproductive dysfunction revealed


normal karyotype 46 ХY in some cells, but most cells have karyotype of
Klinefelter's syndrome – 47 ХХY. Such cell heterogenity is called:
A. Mosaicism
B. Inversion
C. Transposition
D. Duplication
E. Monomorphism

62. In one of Polessye regions there was an outbreak of helminthiasis manifested


by cramps and facial edmata. The developed preventive measures in particular
included ban for eating infested pork even after heat processing. What
helminthiasis was the case?
A. Trichinosis
B. Taeniarhynchosis
C. Teniasis
D. Echinococcosis
E. Alveococcosis

63. As a result of prophylactic medical examination a 7-year-old boy was


diagnosed with Lesch-Nyhan syndrome (only boys fall ill with it). The boy's
parents are healthy but his grandfather by his mother's side suffers from the
same disease. What type of disease inheritance is it?
A. Recessive, sex-linked
B. Dominant, sex-linked
C. Autosomal recessive
D. Autosomal dominant
E. Semidominance
64. A female patient saught medical-genetic consultation. Physical examination
revealed pterygium colli deformity (webbed neck), broad chest,
underdeveloped breasts. Study of buccal epithelium cells revealed no X-
chromatin in the nuclei. This indicates that the patient has the follow ing
syndrome:
A. Turner's
B. Klinefelter's
C. Patau's
D. Down's
E. Edwards'

65. A doctor revealed tissues injury on patient's scalp with localized suppurations
and diagnosed his disease as myiasis. This infestation is caused by larvae of the
following insect:
A. Wohlfahrt fly
B. Kissing bug
C. Stable fly (Stomoxys calcitrans)
D. Malarial mosquito
E. Mosquito

66. A female suffered rubella during pregnancy. The child was born with
developmental abnormalities, namely cleft lip and palate. The child's genotype
is normal. These malformations are a manifestation of:
A. Modification variability
B. Polyploidy
C. Combinative variability
D. Chromosomal mutation
E. Aneuploidy

67. During cell division, DNA replication occurs by a signal from the cytoplasm,
and a certain portion of the DNA helix unwinds and splits into two individual
strains. What enzyme facilitates this process?
A. Helicase
B. RNA polymerase
C. Ligase
D. Restrictase
E. DNA polymerase

68. As an example of specific human parasites one can name Plasmodium


falciparum, human pinworm and some others. The source of parasite invasion
is always a human. Such specific human parasites cause the diseases that are
called:
A. Anthroponoses
B. Zoonoses
C. Anthropozoonoses
D. Infections
E. Multifactorial diseases

69. A patient who has recently arrived from an endemic area presents with
elevated body temperature, headache, chills, malaise, that is with the
symptoms which are typical for a common cold. What laboratory tests are
necessary to prove or to disprove the diagnosis of malaria?
A. Microscopy of blood smears
B. Study of lymph node punctate
C. Urinalysis
D. Study of cerebrospinal fluid
E. Microscopy of bone marrow punctate

70. Examination of the duodenal contents revealed some pear-shaped protozoa


with two nuclei and four pairs of flagella. The organisms had also two axostyles
between the nuclei and a ventral adhesive disc. What protozoan representative
was found in the patient?
A. Lamblia
B. Toxoplasma
C. Leishmania
D. Intestinal trichomonad
E. Trypanosome
Krok 1 – 2014 Anatomy Base
1. A patient has a right-sided fracture in the region of the frontal third of
mandible accompanied by a haematoma in the region of chin. It is caused by
the injury of the following artery:
A. Mental
B. Inferior labial
C. Lingual
D. Facial
E. Palatine

2. Children often have heavy nasal breathing resulting from excessive


development of lymphoid tissue of pharyngeal mucous membrane. What
tonsils growth may cause this effect?
A. Tonsilla pharyngea
B. Tonsilla palatina
C. Tonsilla lingualis
D. Tonsilla tubaria
E. All above mentioned tonsils

3. In course of laparotomy a surgeon revealed gangrenous lesion of descending


colon. It was caused by thrombosis of the following artery:
A. Sinister colic
B. Median colic
C. Dexter colic
D. Ileocolic
E. Superior mesenteric artery

4. As a result of a cold a patient has the abnormal pain and temperature

Anterior two-thirds: Lingual nerve (a branch of the


sensitivity of the frontal 2/3 of his tongue. Which nerve must have been
damaged?
A. Trigeminus mandibular division of the trigeminal nerve – V3)
B. Sublingual Posterior one-third: Glossopharyngeal nerve (cranial
C. Accessory
nerve IX), plus a small branch of the internal laryng
eal nerve (branch of the vagus nerve, cranial nerve X
D. Vagus
E. Glossopharyngeal

5. A patient with a stab wound of the anterior stomach wall is in surgical care.
What formation of abdominal cavity did the stomach contents get into?
A. Antegastrial bursa
B. Omental bursa
C. Hepatic bursa
D. Left mesenteric sinus
E. Right mesenteric sinus
6. A patient has tissue ischemia below the knee joint accompanied with
intermittent claudication. What artery occlusion should be suspected?
A. Popliteal artery
B. Peroneal artery
C. Posterior tibial artery
D. Anterior tibial artery
E. Proximal part of femoral artery

7. After a trauma of soft tissues in the region of the posterior sorface of medial
condyle of humerus a patient has got a skin prickle of medial forearm surface.
Which of the listed nerves is located in the affected region?
A. N.ulnaris
B. N.musculocutaneu
C. N.dorsalis scapularis
D. N.subscapularis
E. N.radialis

8. A female patient with a tumour of pancreas has developed mechanic jaundice


resulting from compression of a bile-excreting duct. Which duct is
compressed?
A. Ductus choledochus
B. Ductus cysticus
C. Ductus hepaticus communis
D. Ductus hepaticus dexter
E. Ductus hepaticus sinister

9. A 28-year-old woman has been diagnosed with extrauterine pregnancy


complicated by tha fallopian tube rupture. The blood is most likely to penetrate
the following peritoneal space:
A. Rectouterine
B. Vesicouterine
C. Right mesenteric sinus
D. Left mesenteric sinus
E. Intersigmoid sinus

10. Examination of a 2-year-old child revealed physical developmental lag, the


child often has pneumonias. The child was diagnosed with nonclosure of
ductus arteriosus. Haemodynamics disorder was caused by the
intercommunication of the following vessels:
A. Aorta and pulmonary trunk
B. Pulmonary trunk and pulmonary veins
C. Superior cava and aorta
D. Superior cava and pulmonary trunk
E. Aorta and pulmonary veins

11. Inflammation of the tympanic cavity (purulent otitis media) was complicated
by inflammation of mammillary process sockets. What wall of tympanic cavity
did the pus penetrate into the sockets through?
A. Posterior
B. Anterior
C. Medial
D. Lateral
E. Superior

12. It is necessary to take the cerebrospinal fluid from a patient with suspected
inflammation of brain tunics. Diagnostic puncture was performed between the
arches of the lumbar vertebras. During the puncture the needle went through
the following ligament:
A. Yellow (flaval)
B. Iliolumbar
C. Anterior longitudinal
D. Posterior longitudinal
E. Intertransverse

13. Nowadays about 50 minor bases have been found in the t-RNA structure
besides the main four nitrogenous bases. Choose the minor nitrogenous base:
A. Dihydrouracil
B. Uracil
C. Cysteine
D. Adenine
E. Cytosine

14. A patient operated on complicated appendicitis has the following changes of


blood count: erythrocytes - 4,0* 101 2/l, Нb - 120 g/l, color index - 0,9,
leukocytes – 18* 109 /l, basophils - 0, eosinophils - 0, myelocytes - 0, juvenile -
0, stab neutrophils - 20, segmentonuclear neutrophils - 53, lymphocytes - 21,
monocytes - 5. How is such nuclear shift of leukocytic formula called?
A. Degenerative left shift
B. Right shift
C. Regenerative left shift
D. Hyperregenerative
E. Regeneratively-degenerative

15. An old woman was hospitalized with acute pain, edema in the right hip joint;
the movements in the joint are limited. Which bone or part of it was broken?
A. The neck of the thigh
B. The body of the thigh bone
C. Condyle of the thigh
D. Pubic bone
E. Ischial bone

16. A 53-year-old female patient was diagnosed with liver rupture resulting from
a blunt abdominal injury. The escaped blood will be assembled in the following
anatomic formation:
A. Rectouterine pouch
B. Vesicouterine pouch
C. Right mesenteric sinus
D. Omental bursa
E. Left mesenteric sinus

17. A patient complains about edemata of legs, skin cyanosis, small ulcers on one
side of the lateral condyle. Examination revealed a swelling, enlarged veins,
formation of nodes. The pathological process has started in the following vein:
A. V. saphena parva
B. V. saphena magna
C. V. femoralis
D. V. profunda femoris
E. V. iliaca externa

18. A 70-year-old female patient was diagnosed with fracture of left femoral neck
accompanied by disruption of ligament of head of femur. The branch of the
following artery is damaged:
A. Obturator
B. Femoral
C. External iliac
D. Inferior gluteal
E. Internal pudendal

19. A woman underwent an operation on account of extrauterine (tubal)


pregnancy. In course of the operation the surgeon should ligate the branches
of the following arteries:
A. Uterine and ovarian
B. Superior cystic and ovarian
C. Inferior cystic and ovarian
D. Uterine and superior cystic
E. Uterine and inferior cystic

20. A 6-month old baby ill with bronchitis was taken for an X-ray of chest. Apart
of changes associated with bronchi the X-ray film showed a shadow of thymus
gland. What might have caused such changes?
A. The above-mentioned condition is a normal variant for this age
B. It's the effect of bronchitis
C. It is caused by abnormal position
D. It is caused by thymus inflammation
E. It is caused by neoplastic process

21. A patient suffering from chronic myeloleukemia has got the following
symptoms of anemia: decreased number of erythrocytes and low haemoglobin
concentration, oxyphilic and polychromatophilic normocytes, microcytes.
What is the leading pathogenetic mechanism of anemia development?
A. Substitution of haemoblast
B. Intravascular hemolysis of erythrocytes
C. Deficiency of vitamin B1 2
D. Reduced synthesis of erythropoietin
E. Chronic haemorrhage

22. A 7-year-old girl has signs of anemia. Laboratory examination revealed


pyruvate kinase deficiency in erythrocytes. What process disturbance plays the
main role in anemia development?
A. Anaerobic glycolysis
B. Oxidative phosphorylation
C. Tissue respiration
D. Peroxide decomposition
E. Aminoacids desamination

23. A patient complained about being unable to adduct and abduct fingers in the
metacarpophalangeal articulations towards and away from the 3rd finger.
Which muscles' function is impaired?
A. Interosseous muscles
B. Lumbrical muscles
C. Breviflexors of fingers
D. Long flexors of fingers
E. Extensors

24. While examining foot blood supply a doctor checks the pulsation of a large
artery running in the separate fibrous channel in front of articulatio
talocruralis between the tendons of long extensor muscles of hallux and toes.
What artery is it?
A. a dorsalis pedis
B. a tibialis anterior
C. a tarsea medialis
D. a tarsea lateralis
E. a fibularis

25. Microspecimen of spinal cord contains a nucleus that should be analyzed. Its
neurons form motor endings in the skeletal muscles. What nucleus of spinal
cord is meant?
A. Proper nucleus of the anterior horn
B. Thoracic nucleus
C. Intermediate lateral nucleus
D. Proper nucleus of the posterior horn
E. Proper nucleus of gray substance

26. A man with cut wound of his right foot sole was admitted to the hospital ward.
The patient has limited elevation of the lateral foot edge. In course of wound
management the injury of a muscle tendon was revealed. What muscle is
injured?
A. Long peroneal
B. Anterior tibial
C. Long extensor muscle of toes
D. Triceps muscle of crus
E. Short peroneal

27. A comminuted fracture of infraglenoid tubercle caused by shoulder joint injury


has been detected during X-ray examination of a patient. What muscle tendon
attached at this site has been damaged?
A. Long head of m. triceps brachii
B. Long head of m. biceps brachii
C. Medial head of m. triceps brachii
D. Lateral head of m. triceps brachii
E. Short head of m. biceps brachii

28. A 35-year-old man with a trauma of his left hand was admitted to the
traumatology department. Objectively: cut wound of palmar surface of left
hand; middle phalanxes of II–V fingers don't bend. What muscles are
damaged?
A. Superficial finger flexor
B. Profound finger flexor
C. Lumbrical muscles
D. Palmar interosseous muscles
E. Dorsal interosseous muscles

29. A 38-year-old patient came to a traumatology centre and complained about an


injury of his right hand. Objectively: the patient has a cut wound in the region
of the thenar eminence on the right hand; distal phalanx of the I finger cannot
be flexed. What muscle was injured?
A. Long flexor muscle of thumb
B. Short flexor muscle of thumb
C. Short abductor muscle of thumb
D. Opposer muscle of thumb
E. Abductor muscle of thumb

30. A patient with neuritis of femoral nerve has disturbed flexion of thigh as well
as disturbed crus extension in the knee joint. What muscle's function is
disturbed?
A. Quadriceps muscle of thigh
B. Biceps muscle of thigh
C. Triceps muscle of thigh
D. Semitendinous muscle
E. Semimembranous muscle
31. A 42-year-old male with a lesion of the ulnar nerve is unable to flex the II and
V fingers to the midline. Which muscle function is impaired in this case?
A. Palmar interosseous muscles
B. Dorsal interosseous muscle
C. Fidicinales
D. Short palmar muscle
E. Abductor muscle of little finger

32. A patient has difficulties with hand movement. Examination revealed


inflammation of common synovial sheath of flexor muscles. It is known from
the patient's anamnesis that he got a stab wound of finger a week ago. Which
finger was most probably damaged?
A. Digitus minimus
B. Pollex
C. Digitus medius
D. Index
E. Digitus anularis

33. An injured man has bleeding from branches of carotid artery. For a temporary
arrest of bleeding it is necessary to press the carotid artery to the tubercle of
cervical vertebra. Which vertebra is it?
A. VI
B. V
C. IV
D. III
E. II

34. During cytoscopy mucous membrane of urinary bladder normally makes folds
except for a single triangular area with smooth mucosa. This triangle is located
in the following part of urinary bladder:
A. Bladder floor
B. Bladder cervix
C. Bladder apex
D. Bladder body
E. Bladder isthmus

35. As a result of an accident a patient has intense painfullness and edema of the
anterior crus surface; dorsal flexion of foot is hindered. Function of which crus
muscle is most likely to be disturbed?
A. M.tibialis anterior
B. M.flexor digitorum longus
C. M.flexor hallucis longus
D. M.peroneus longus
E. M.peroneus brevis
36. A 40-year-old patient complains of intensive heartbeats, sweating, nausea,
visual impairment, arm tremor, hypertension. From his anamnesis: 2 years
ago he was diagnosed with pheochromocytoma. Hyperproduction of what
hormones causes the given pathology?
A. Catecholamines
B. Aldosterone
C. Glucocorticoids
D. ACTH
E. Thyroidal hormones

37. A patient who suffers from cancer of back of tongue has an intense bleeding as
a result of affection of dorsal lingual artery by the tumour. What vessel should
be ligated to stop bleeding?
A. Lingual artery
B. Dorsal lingual artery
C. Deep lingual artery
D. Facial artery
E. Ascending pharyngeal artery

38. A 58-year-old patient with acute cardiac insufficiency has decreased volume of
daily urine - oliguria. What is the mechanism of this phenomenon?
A. Decreased glomerular filtration
B. Decreased number of functioning glomerules
C. Drop of oncotic blood pressure
D. Rise of hydrostatic blood pressure in capillars
E. Reduced permeamility of renal filter

39. An 18-year-old man was delivered to the hospital after a road accident.
Examination at the traumatological department revealed multiple injuries of
soft tissues of face in the region of the medial eye angle. The injuries caused
massive haemorrhage. What arterial anastomosis might have been damaged
in this region?
A. a carotis externa et a. carotis interna
B. a carotis externa et a. subclavia
C. a carotis interna et a. subclavia
D. a subclavia et a. ophthalmica
E. a carotis interna et a. ophthalmica

40. After a 2 y.o. child has had flu, there appeared complaints about ear ache. A
doctor revealed hearing impairment and inflammation of the middle ear. How
did the infection penetrate into the middle ear?
A. Through the auditory tube
B. Through foramen jugularis
C. Through canalis caroticus
D. Through atrium mastoideum
E. Through canalis nasolacrimalis
41. A 40-year-old woman was admitted to the infectious diseases department with
high body temperature. Objectively: marked meningeal symptoms. A spinal
cord punction was made. What anatomic formation was puncturated?
A. Spatium subarachnoideum
B. Spatium subdurale
C. Spatium epidurale
D. Cavum trigeminale
E. Cisterna cerebellomedullaris posterior

42. After resection of the middle third of femoral artery obliterated by a thromb
the lower extremity is supplied with blood due to the surgical bypass. Name an
artery that plays the main role in reestablishment of blood flow:
A. Deep femoral artery
B. Superficial circumflex artery of hip bone
C. Descending genicular artery
D. Superficial epigastric artery
E. Deep external pudendal artery

43. A patient's knee joint doesn't extend, there is no knee-jerk reflex, skin
sensitivity of the anterior femoral surface is disturbed. What nerve structures
are damaged?
A. Femoral nerve
B. Superior gluteal nerve
C. Big fibular nerve
D. Obturator nerve
E. Inferior gluteal nerve

44. A 50 y.o. patient was admitted to the hospital with complaints about pain
behind his breastbone, asphyxia during physical activity. Angiography
revealed pathological changes in the posterior interventricular branch of the
right coronary artery. What heart parts are affected?
A. Posterior wall of the right and left ventricles
B. Left atrium
C. Anterior wall of the right and left ventricles
D. Right atrium
E. Right atrioventricular valve

45. A patient was admitted to the surgical department with suspected


inflammation of Meckel's diverticulum. What part of bowels should be
examined in order to discover the diverticulum in course of an operation?
A. Ileum
B. Duodenum
C. Jejunum
D. Caecum
E. Colon ascendens
46. Examination of a newborn boy's genitals revealed a cleft of urethra that opens
on the inferior surface of his penis. What developmental anomaly is meant?
A. Hypospadia
B. Hermaphroditism
C. Epispadia
D. Monorchism
E. Cryptorchism

47. A patient complains about impaired evacuatory function of stomach (long-


term retention of food in stomach). Examination revealed a tumour of initial
part of duodenum. Specify localization of the tumour:
A. Pars superior
B. Pars inferior
C. Pars descendens
D. Pars ascendens
E. Flexura duodeni inferior

48. A young man consulted a doctor about disturbed urination. Examination of his
external genitals revealed that urethra is split on top and urine runs out of this
opening. What anomaly of external genitals development is the case?
A. Epispadia
B. Phimosis
C. Hermaphroditism
D. Paraphimosis
E. Hypospadia

49. As a result of a continuous chronic encephalopathy, a patient has developed


spontaneous motions and a disorder of torso muscle tone. These are the
symptoms of the disorder of the following conduction tract:
A. Tractus rubrospinalis
B. Tractus corticospinalis
C. Tractus corticonuclearis
D. Tractus spinothalamicus
E. Tractus tectospinalis

50. A 35-year-old patient applied to a doctor with complaints about having intense
rhinitis and loss of sense of smell for a week. Objectively: nasal cavity contains
a lot of mucus that covers mucous membrane and blocks olfactory receptors.
In what part of nasal cavity are these receptors situated?
A. Superior nasal turbinate
B. Median nasal turbinate
C. Inferior nasal turbinate
D. Common nasal meatus
E. Vestibule of nose
51. As a result of a craniocerebral injury, a patient has a decreased skin sensitivity.
What area of the cerebral cortex is likely to be damaged?
A. Posterior central gyrus
B. Occipital region
C. Cingulate gyrus
D. Frontal cortex
E. Anterior central gyrus

52. A 40-year-old patient has ulcer perforation in the posterior wall of stomach.
What anatomical structure will blood and stomach content leak to?
A. Bursa omentalis
B. Bursa praegastrica
C. Right lateral channel (canalis lateralis dexter)
D. Left lateral channel (canalis lateralis sinister)
E. Bursa hepatica

53. Inflammatory process of modified subserous layer around cervix of the uterus
caused an intensive pain syndrome. In what region of genitals does the
pathological process take place?
A. Parametrium
B. Mesometrium
C. Myometrium
D. Endometrium
E. Perimetrium

54. In course of an operation surgeon removed a part of a lung that was ventilated
by a tertiary bronchus accompanied by branches of pulmonary artery and
other vessels. What part of a lung was removed?
A. Bronchopulmonary segment
B. Middle lobe
C. Inferior lobe
D. Superior lobe
E. Pulmonary lobule

55. Angiocardiography of a 60-year-old male patient revealed constriction of a


vessel located in the left coronary sulcus of the heart. What is the pathological
vessel called?
A. Ramus circumflexus
B. Ramus interventricularis posterior
C. A coronaria dextra
D. V cordis parva
E. Ramus interventricularis anterior

56. A patient complains of pain in the right lateral abdomen. Palpation revealed a
dense, immobile, tumor-like formation. A tumor is likely to be found in the
following part of the digestive tube:
A. Colon ascendens
B. Colon transversum
C. Colon descendens
D. Colon sigmoideum
E. Caecum

57. While examining the oral cavity a stomatologist revealed inflammation of


papillae on the border of the median and posterior third of the back of tongue.
What papillae are inflamed?
A. Papillae vallatae
B. Papillae fungiformes
C. Papillae foliatae
D. Papillae filiformes
E. Papillae conicae

58. Examination of a patient revealed an abscess of pterygopalatine fossa. Where


can the infection spread to unless the disease is managed in time?
A. To the orbit
B. To the interpterygoid space
C. To the frontal sinus
D. To the subgaleal temporal space
E. To the tympanic cavity

59. A patient with cholelithiasis fell ill with mechanic jaundice. Examination
revealed that the stone was in the common bile duct. What bile-excreting ducts
make up the obturated duct?
A. Ductus hepaticus communis et ductus cysticus
B. Ductus hepaticus dexter et sinister
C. Ductus hepaticus dexter et ductus cysticus
D. Ductus hepaticus sinister et ductus cysticus
E. Ductus hepaticus communis et ductus choledochus

60. Neurological examination of a 65 y.o. patient revealed a haemorrhage within


the superior temporal gyrus. In the blood supply area of which artery is it?
A. Middle cerebral artery
B. Anterior cerebral artery
C. Posterior cerebral artery
D. Anterior communicating artery
E. Basilar artery

61. A 70 y.o. man has cut an abscess off in the area of mammiform process during
shaving. Two days later he was admitted to the hospital with inflammation of
arachnoid membranes. How did the infection penetrate into the cavity of skull?
A. V.emissariaе mastoideaе
B. V.v.labyrinthi
C. V.v.tympanicae
D. V.facialis
E. V.v.auriculares

62. The cerebrospinal fluid is being examined for the purpose of diffrential
meningitis diagnostics. At what site is the lumbal puncture safe?
A. L III-L IV
B. L II-L III
C. L I-L II
D. Th XII-L I
E. L V-S I

63. A female patient has facial neuritis that has caused mimetic paralysis and
hearing impairment. Hearing impairment results from the paralysis of the
following muscle:
A. Stapedius muscle
B. Anterior auricular muscle
C. Superior auricular muscle
D. Posterior auricular muscle
E. Nasal muscle

64. Part of alveoles of a preterm infant didn't spread because of enhanced elastic
recoil of lungs. How can this recoil be reduced?
A. By surfactant introduction
B. By pure oxygene inhalation
C. By artificial pulmonary ventilation
D. By fluid suction from the respiratory tracts
E. By glycose introduction

65. Preventive examination of a patient revealed an enlarged lymph node of


metastatic origin on the medial wall of the left axillary crease. Specify the most
likely localization of the primary tumour:
A. Mammary gland
B. Submandibular salivary gland
C. Lung
D. Stomach
E. Thyroid gland

66. A man suffering from osteochondrosis got acute pain in the abdominal muscles
(lateral and anterior). During objective examination a physician
diagnosticated increased pain sensitivity of skin in the hypogastric region. This
pain might be caused by affection of the following nerve:
A. Iliohypogastric
B. Sciatic
C. Obturator
D. Femoral
E. Genitofemoral
67. A woman suffering from osteochondrosis felt acute pain in her humeral
articulation that became stronger when she abducted her shoulder. These
symptoms might be caused by damage of the following nerve:
A. Axillary nerve
B. Subscapular nerve
C. Dorsal scapular nerve
D. Subclavicular nerve
E. Throracodorsal nerve

68. Pyeloureterography X-ray photo showed a renal pelvis with minor calyces only
(major calyces were absent). What form of urinary tracts of a kidney was
revealed?
A. Embryonal
B. Fetal
C. Mature
D. Ampullar

69. A patient has a deep cut wound on the posterior surface of his shoulder in its
middle third. What muscle might be injured?
A. Triceps muscle of arm
B. Biceps muscle of arm
C. Anconeus muscle
D. Brachial muscle
E. Coracobrachial muscle

70. A patient has pain, edema and reddening of his skin in the anterosuperior area
of his thigh and his foot's thumb. What lymph nodes of his lower extremity
responded to the inflammatory process?
A. Superficial inguinal
B. Deep inguinal
C. Internal longitudinal
D. Superficial longitudinal
E. General longitudinal

71. A patient got a craniocerebral trauma that resulted in right-side convergent


strabismus. Damage of which craniocerebral nerve caused such consequences?
A. n. abducens
Eye adduction fixed
B. n. facialis
C. n. trigeminus
D. n. trochlearis
E. n. aculomotorius

72. In case of a penetrating wound of the anterior abdominal wall the wound tract
went above the lesser curvature of stomach. What peritoneum formation is

Liver
most likely to be injured?
A. Ligamentum hepatogastricum
B. Ligamentum gastrocolicum
C. Ligamentum hepatoduoduodenale
D. Ligamentum hepatorenale
E. Ligamentum triangulare sinistrum

73. After a trauma a patient lost ability of elbow extension. This might have been
caused by dysfunction of the following main muscle:
A. m. triceps brachii
B. m. subscapularis
C. m. teres major
D. m. infraspinatus
E. m. levator scapulae

74. While playing a child got a punch in the presternum region. As a result of this
trauma an organ located behind the presternum was damaged. Name this
organ:
A. Thymus
B. Thyroid gland
C. Heart
D. Pericardium
E. Larynx

75. A patient complains of acute pain attacks in the right lumbar region. During
examination the nephrolithic obturation of the right ureter in the region
between its abdominal and pelvic segments has been detected. What
anatomical boundary exists between those two segments?
A. Linea terminalis
B. Linea semilunaris
C. Linea arcuata
D. Linea transversa
E. Linea inguinalis

76. After a craniocerebral trauma a patient lost the ability to execute learned
purposeful movements (apraxia). The injury is most likely localized in the
following region of the cerebral cortex:
A. Gyrus supramarginalis
B. Gyrus angularis
C. Gyrus paracentralis
D. Gyrus lingualis
E. Gyrus parahippocampalis

77. A patient got an injury of spinal marrow in a road accident that caused loss of
tactile sensation, posture sense, vibration sense. What conduction tracts are
damaged?
A. Fascicle of Goll and cuneate fascicle
B. Anterior spinocerebellar tract
C. Rubrospinal tract
D. Reticulospinal tract
E. Tectospinal tract

78. Examination of a patient revealed hypertrophy and inflammation of lymphoid


tissue, edema of mucous membrane between palatine arches (acute tonsillitis).
What tonsil is normally situated in this area?
A. Tonsilla palatina
B. Tonsilla pharyngealis
C. Tonsilla tubaria
D. Tonsilla lingualis

79. While performing an operation in the area of axillary crease a surgeon has to
define an arterial vessel surrounded by fascicles of brachial plexus. What artery
is it?
A. A. axillaris
B. A. vertebralis
C. A. transversa colli
D. A. profunda brachii
E. A. subscapularis

80.Examination of a patient with impaired blood coagulation revealed thrombosis


of a branch of inferior mesenteric artery. What bowel segment is damaged?
A. Colon sigmoideum
B. Ileum
C. Caecum
D. Colon transversum
E. Colon ascendens

81. A patient was admitted to the surgical department with inguinal hernia.
During the operation the surgeon performs plastic surgery on posterior wall of
inguinal canal. What structure forms this wall?
A. Transverse fascia
B. Aponeurosis of abdominal external oblique muscle
C. Inguinal ligament
D. Loose inferior edge of transverse abdominal muscle
E. Peritoneum

82. In order to prevent massive haemorrhage in the region of oral cavity floor it is
required to ligate an artery which is located within Pirogov's triangle. What
artery is it?
A. Lingual artery
B. Superior thyroid artery
C. Facial artery
D. Ascending pharyngeal artery
E. Maxillary artery

83. A man with an injury of the dorsal area of his neck was admitted to the
resuscitation department. What muscle occupies this area?
A. M. trapezius
B. M. sternocleidomastoideus
C. M. latissimus dorsi
D. M. rhomboideus minor
E. M. scalenus anterior

84. A patient complains of dizziness and hearing loss. What nerve is damaged?
A. Vestibulocochlear
B. Trigeminus
C. Sublingual
D. Vagus
E. Trochlear

85. While palpating mammary gland of a patient a doctor revealed an induration


in form of a node in the inferior medial quadrant. Metastases may extend to
the following lymph nodes:
A. Parasternal
B. Posterior mediastinal
C. Profound lateral cervical
D. Bronchopulmonary
E. Superior diaphragmal

86. A patient got a trauma that caused dysfunction of motor centres regulating
activity of head muscles. In what parts of cerebral cortex is the respective
centre normally localized?
A. Inferior part of precentral gyrus

Sup=lower part of body


B. Superior part of precentral gyrus
C. Supramarginal gyrus
D. Superior parietal lobule
E. Angular gyrus
Inf.= Upper part of body

87. A patient was diagnosed with paralysis of facial and masticatory muscles. The
haematoma is inside the genu of internal capsule. What conduction tract is
damaged?
A. Tr. cortico-nuclearis
B. Tr. cortico-spinalis
C. Tr. cortico-thalamicus
D. Tr. cortico-fronto-pontinus
E. Tr. cortico-temporo-parieto-occipito-pontinus
88. A patient has lost skin sensitivity in the region of the medial surface of his
shoulder. This is the result of dysfunction of the following nerve:
A. Medial brachial cutaneous nerve
B. Medial antebrachial cutaneous nerve
C. Radial nerve
D. Ulnar nerve
E. Axillary nerve

89. A foreign body (a button) closed space of the right superior lobar bronchus.
What segments of the right lung won't be supplied with air?
A. Apical, posterior, anterior
B. Superior and inferior lingular
C. Apical and posterior basal
D. Apical and median basal
E. Medial and lateral

90. A patient was diagnosed with bartholinitis (inflammation of greater

Labia major
vulvovaginal glands). In which organ of urogenital system are these glands
localized?
A. Large lips of pudendum

Bartholin's glands supply a mucus for the lubrication of the e


B. Small lips of pudendum

ntrance to the vagina during sexual excitement, and that Cow


C. Clitoris
D. Vagina
E. Uterus per's glands secrete a mucous fluid which acts in part as a ve
hicle for the seminal discharge has long been noted.
91. While performing an inguinal canal operation on account of hernia a surgeon
damaged the canal's contents. What exactly was damaged?
A. Funiculus spermaticus
B. Urarchus
C. Lig. teres uteri
D. Lig. inguinalе

92. Ultrasonic examination of a patient revealed aneurism in the area of aortic


arch that caused alteration of vocal function of larynx. What nerve was
constricted?
A. Recurrent laryngeal
B. Diaphragmatic
C. Superior laryngeal
D. Mandibular
E. Sublingual

93. Surgical approach to the thyroid gland from the transverse (collar) approach
involves opening of interaponeurotic suprasternal space. What anatomic
structure localized in this space is dangerous to be damaged?
A. Jugular venous arch
B. External jugular vein
C. Subclavicular vein
D. Inferior thyroid arthery
E. Superior thyroid arthery

94. In course of a small pelvis operation it became necessary to ligate an ovarian


artery. What formation may be accidentally ligated together with it?
A. Ureter
B. Uterine tube
C. Round ligament of uterus
D. Internal iliac vein
E. Urethra

95. An injured person was delivered to the hospital with a penetrating wound in
the left lateral region of abdomen. What part of the large intestine is most likely
damaged?
A. Colon descendens
B. Colon ascendens
C. Colon transverses
D. Caecum
E. Rectum

96. After a road accident a driver was delivered to the hospital with an injury of
the medial epicondyle of humerus. What nerve might be damaged in this case?
A. n. ulnaris
B. n. radialis
C. n. axillaris
D. n. muscolocutaneus
E. n. medianus

97. A patient with a knife wound in the left lumbal part was delivered to the
emergency hospital. In course of operation a surgeon found that internal
organs were not damaged but the knife injured one of muscles of renal pelvis.
What muscle is it?
A. Greater psoas muscle
B. Iliac muscle
C. Erector muscle of spine
D. Abdominal internal oblique muscle
E. Abdominal external oblique muscle

98. A 25-year-old patient was examined by a medical board. Examination revealed


pathology of chest. Transverse dimensions were to small and the sternum was
strongly protruding. What chest type is it?
A. Keeled chest
B. Funnel chest
C. Flat chest
D. Cylindrical chest
E. Barrel chest

99. A man with a stab wound in the area of quadrilateral foramen applied to a
doctor. Examination revealed that the patient was unable to draw his arm aside
from his body. What nerve is most probably damaged?
A. N. axillaris
B. N. medianus
C. N. radialis
D. N. ulnaris
E. N. subclavius

100. Brain tomography revealed a tumour in the region of red nucleus. What
part of brain is damaged?
A. Midbrain
B. Medulla oblongata
C. Cerebellum
D. Interbrain
E. Pons cerebelli
Krok 1 – 2014 Biochemistry Base
1. After consumption of rich food, a patient has nausea and heartburn,
steatorrhea. This condition might be caused by:
A. Bile acid deficiency
B. Increased lipase secretion
C. Disturbed tripsin synthesis
D. Amylase deficiency
E. Disturbed phospholipase synthesis

2. Characteristic sign of glycogenosis is muscle pain during physical work. Blood


examination reveals usually hypoglycemia. This pathology is caused by
congenital deficiency of the following enzyme:
A. Glycogen phosphorylase
B. Glucose 6-phosphate dehydrogenase
C. Alpha amylase
D. Gamma amylase
E. Lysosomal glycosidase

3. An infant has apparent diarrhea resulting from improper feeding. One of the
main diarrhea effects is plentiful excretion of sodium bicarbonate. What form
of acid-base balance disorder is the case?
A. Metabolic acidosis
B. Metabolic alkalosis
C. Respiratory acidosis
D. Respiratory alkalosis
E. No disorders of acid-base balance will be observed

4. Examination of a patient suffering from cancer of urinary bladder revealed


high rate of serotoninand hydroxyanthranilic acid. It is caused by excess of the
following amino acid in the organism:
A. Tryptophan
B. Alanine
C. Histidine
D. Methionine
E. Tyrosine

5. A mother consulted a doctor about her 5-year-old child who develops


erythemas, vesicular rash and skin itch under the influence of sun. Laboratory
studies revealed decreased iron concentration in the blood serum, increased
uroporphyrinogen I excretion with the urine. What isthe most likely inherited
pathology in this child?
A. Erythropoietic porphyria
B. Methemoglobinemia
C. Hepatic porphyria
D. Coproporphyria
E. Intermittent porphyria

6. A 3-year-old child with fever was given aspirin. It resulted in intensified


erythrocyte haemolysis. Hemolytic anemia might have been caused by
congenital insufficiency of the following enzyme:
A. Glucose 6-phosphate dehydrogenase
B. Glucose 6-phosphatase
C. Glycogen phosphorylase
D. Glycerol phosphate dehydrogenase
E. γ-glutamiltransferase

7. Blood of a 12-year-old boy presents low concentration of uric acid and


accumulation of xanthine and hypoxanthine. This child has genetic defect of
the following enzyme:
A. Xanthine oxidase
B. Arginase
C. Urease
D. Ornithine carbamoyltransferase
E. Glycerylkinase

8. A patient is ill with diabetes mellitus that is accompanied by hyperglycemia of


over 7,2 millimole/l on an empty stomach. The level of what blood plasma
protein allows to estimate the glycemia rate retrospectively (4-8 weeks before
examination)?
A. Glycated hemoglobin
B. Albumin
C. Fibrinogen
D. C-reactive protein
E. Ceruloplasmin

9. A 4 y.o. child with signs of durative proteinic starvation was admitted to the
hospital. The signs were as follows: growth inhibition, anemia, edemata,
mental deficiency. Choose a cause of edemata development:
A. Reduced synthesis of albumins
B. Reduced synthesis of globulins
C. Reduced synthesis of hemoglobin
D. Reduced synthesis of lipoproteins
E. Reduced synthesis of glycoproteins

10. Researchers isolated 5 isoenzymic forms of lactate dehydrogenase from the


human blood serum and studied their properties. What property indicates
that the isoenzymic forms were isolated from the same enzyme?
A. Catalyzation of the same reaction
B. The same molecular weight
C. The same physicochemical properties
D. Tissue localization
E. The same electrophoretic mobility

11. On some diseases it is observed aldosteronism with hypertension and edema


due to sodium retention in the organism. What organ of the internal secretion
is affected on aldosteronism?
A. Adrenal glands
B. Testicle
C. Ovaries
D. Pancreas
E. Hypophysis

12. An experiment proved that UV-radiated cells of patients with xeroderma


pigmentosum restore the native DNA structure slower than cells of healthy
individuals as a result of reparation enzyme defection. What enzyme helps this
process?
A. Endonuclease
B. RNA ligase
C. Primase
D. DNA polymerase III
E. DNA gyirase

13. A patient presents high activity of LDH1 ,2 , aspartate aminotransferase,


creatine phosphokinase. In what organ (organs) is the development of a
pathological process the most probable?
A. In the heart muscle (initial stage of myocardium infarction)
B. In skeletal muscles (dystrophy, atrophy)
C. In kidneys and adrenals
D. In connective tissue
E. In liver and kidneys

14. While examining the child the doctor revealed symmetric cheeks roughness,
diarrhea, disfunction of the nervous system. Lack of what food components
caused it?
A. Nicotinic acid, tryptophane
B. Lysine, ascorbic acid
C. Threonine, pantothenic acid
D. Methionine, lipoic acid
E. Phenylalanine, pangamic acid

15. A 62-year-old female patient has developed a cataract (lenticular opacity)


secondary to the diabetes mellitus. What type of protein modification is
observed in case of diabetic cataract?
A. Glycosylation
B. Phosphorylation
C. ADP-ribosylation
D. Methylation
E. Limited proteolysis

16. A 53-year-old male patient is diagnosed with Paget's disease. The


concentration of oxyproline in daily urine is sharply increased, which
primarily means intensified disintegration of:
A. Collagen
B. Keratin
C. Albumin
D. Hemoglobin
E. Fibrinogen

17. Examination of a patient revealed typical presentations of collagenosis. This


pathology is characterized by increase of the following urine index:
A. Hydroxyproline
B. Arginine
C. Glucose
D. Mineral salts
E. Ammonium salts

18. Untrained people often have muscle pain after sprints as a result of lactate
accumulation. This might be caused by intensification of the following
biochemical process:
A. Glycolysis
B. Gluconeogenesis
C. Pentose phosphate pathway
D. Lipogenesis
E. Glycogenesis

19. ATP synthesis is totaly blocked in a cell. How will the value of membrane rest
potential change?
A. It will disappear
B. It will be slightly increased
C. It will be considerably increased
D. First it will increase, then decrease
E. First it will decrease, then increase

20. Examination of a patient suffering from frequent haemorrhages in the inner


organs and mucous membranes revealed proline and lysine being included in
collagen fibers. Impairment of their hydroxylation is caused by lack of the
following vitamin:
A. C
B. E
C. K
D. A
E. D
21. A 20-year-old patient complains of general weakness, dizziness, quick
fatigability. Blood analysisresults: Hb- 80 g/l. Microscopical examination
results: erythrocytes are of modified form. This condition might be caused by:
A. Sickle-cell anemia
B. Hepatocellular jaundice
C. Acute intermittent porphyria
D. Obturative jaundice
E. Addison's disease

22. A 48-year-old patient complained about intense pain, slight swelling and
reddening of skin over the joints, temperature rise up to 38 oC. Blood analysis
revealed high concentration of urates. This condition might be caused by
disturbed metabolism of:
A. Purines
B. Collagen
C. Cholesterol
D. Pyrimidines
E. Carbohydrates

23. A patient has yellow skin colour, dark urine, dark-yellow feces. What
substance will have strengthened concentration in the blood serum?
A. Unconjugated bilirubin
B. Conjugated bilirubin
C. Mesobilirubin
D. Verdoglobin
E. Biliverdin

24. A patient has an increased pyruvate concentration in blood. A large amount of


it is excreted withthe urine. What vitamin is lacking in this patient?
A. B1
B. E
C. B3
D. B6
E. B2

25. Ammonia is a very toxic substance, especially for nervous system. What
substance takes the most active part in ammonia detoxication in brain tissues?
A. Glutamic acid
B. Lysine
C. Proline
D. Histidine
E. Alanine

26. A patient has pellagra. Interrogation revealed that he had lived mostly on
maize for a long time and eaten little meat. This disease had been caused by
the deficit of the following substance in the maize:
A. Tryptophan
B. Tyrosine
C. Proline
D. Alanine
E. Histidine

27. Cyanide is a poison that causes instant death of the organism. What enzymes
found in mitochondria are affected by cyanide?
A. Cytochrome oxidase (aa3)
B. Flavin enzymes
C. Cytochrome В5
D. NAD+-dependent dehydrogenase
E. Cytochrome P-450

28. A patient with high rate of obesity was advised to use carnitine as a food
additive in order to enhance "fat burning". What is the role of carnitine in the
process of fat oxidation?
A. Transport of FFA (free fatty acids) from cytosol to the
mitochondria
B. Transport of FFA from fat depots to the tissues
C. It takes part in one of reactions of FFA beta-oxidation
D. FFA activation
E. Activation of intracellular lipolysis

29. An experimantal animal that was kept on protein-free diet developed fatty
liver infiltration, in particular as a result of deficiency of methylating agents.
This is caused by disturbed generationof the following metabolite:
A. Choline
B. DOPA
C. Cholesterol
D. Acetoacetate
E. Linoleic acid

30. A patient consulted a doctor about symmetric dermatitis of open skin areas. It
was found out that the patient lived mostly on cereals and ate too little meat,
milk and eggs. What vitamin deficiency is the most evident?
A. Nicotinamide
B. Calciferol
C. Folic acid
D. Biotin
E. Tocopherol

31. A 46-year-old woman suffering from chololithiasis developed jaundice. Her


urine became dark-yellow and feces became colourless. Blood serum will have
the highest concentration of the following substance:
A. Conjugated bilirubin
B. Unconjugated bilirubin
C. Biliverdin
D. Mesobilirubin
E. Urobilinogen

32. A 46-year-old patient applied to a doctor complaining about joint pain that
becomes stronger the day before weather changes. Blood examination
revealed strengthened concentration of uric acid. The most probable cause of
the disease is the intensified disintegration of the following substance:
A. Adenosine monophosphate
B. Cytidine monophosphate
C. Uridine triphosphate
D. Uridine monophosphate
E. Thymidine monophosphate

33. It has been found out that one of pesticide components is sodium arsenate
that blocks lipoic acid. Which enzyme activity is impaired by this pesticide?
A. Pyruvate dehydrogenase complex
B. Microsomal oxidation
C. Methemoglobin reductase
D. Glutathione peroxidase
E. Glutathione reductase

34. A 38-year-old patient suffers from rheumatism in its active phase. What
laboratory characteristic of blood serum is of diagnostic importance in case of
this pathology?
A. C-reactive protein
B. Uric acid
C. Urea
D. Creatinine
E. Transferrin

35. A patient with serious damage of muscular tissue was admitted to the
traumatological department. What biochemical urine index will be increased
in this case?
A. Creatinine
B. Common lipids
C. Glucose
D. Mineral salts
E. Uric acid

36. 12 hours after an accute attack of retrosternal pain a patient presented a jump
of aspartate aminotransferase activity in blood serum. What pathology is this
deviation typical for?
A. Myocardium infarction
B. Viral hepatitis
C. Collagenosis
D. Diabetes mellitus
E. Diabetes insipidus

37. Donor skin transplantation was performed to a patient with extensive burns.
On the 8-th day the graft became swollen and changed colour; on the 11 -th day
graft rejection started. What cells take part in this process?
A. T-lymphocytes
B. Erythrocytes
C. Basophils
D. Eosinophils
E. B-lymphocytes

38. A 30 y.o. woman had been ill for a year when she felt pain in the area of joints
for the first time, they got swollen and skin above them became reddened.
Provisional diagnosis is rheumatoid arthritis. One of the most probable causes
of this disease is a structure alteration of a connective tissue protein:
A. Collagen
B. Mucin
C. Myosin
D. Ovoalbumin
E. Troponin

39. Autopsy of a 12-year-old girl revealed: multiple cutaneous hemmorhages


(mostly into the skin of buttocks, lower extremities), serous and mucous
memrane hemmorhages, cerebral hemmorhages. Adrenal glands show focal
necrosis and massive hemmorhages; kidneys show necrotic nephrosis,
suppurative arthritis, iridocyclitis, vasculitis. What is the most probable
diagnosis?
A. Meningococcemia
B. Epidemic typhus
C. Periarteritis nodosa
D. Systemic lupus erythematosus
E. Radiation sickness

40. Examination of a 27-year-old patient revealed pathological changes in liver


and brain. Blood plasma analysis revealed an abrupt decrease in the copper
concentration, urine analysis revealed an increased copper concentration. The
patient was diagnosed with Wilson’s degeneration. To confirm the diagnosis it
is necessary to study the activity of the following enzyme in blood serum:
A. Ceruloplasmin
B. Carbonic anhydrase
C. Xanthine oxidase
D. Leucine aminopeptidase
E. Alcohol dehydrogenase
41. A patient complains about dyspnea provoked by the physical activity. Clinical
examination revealed anaemia and presence of the paraprotein in the zone of
gamma-globulins. To confirm the myeloma diagnosis, it is necessary to
determine the following index in the patient’s urine:
A. Bence Jones protein
B. Bilirubin
C. Haemoglobin
D. Ceruloplasmin
E. Antitrypsin

42. Examination of a child who hasn't got fresh fruit and vegetables during winter
revealed numeroussubcutaneous hemorrhages, gingivitis, carious cavities in
teeth. What vitamin combination should be prescribed in this case?
A. Ascorbic acid and rutin
B. Thiamine and pyridoxine
C. Folic acid and cobalamin
D. Riboflavin and nicotinamide
E. Calciferol and ascorbic acid

43. A drycleaner's worker has been found to have hepatic steatosis. This pathology
can be caused by the disruption of synthesis of the following substance:
A. Phosphatidylcholine
B. Tristearin
C. Urea
D. Phosphatidic acid
E. Cholic acid

44. As a result of exhausting muscular work a worker has largely reduced buffer
capacity of blood. What acidic substance that came to blood caused this
phenomenon?
A. Lactate
B. Pyruvate
C. 1,3-bisphosphoglycerate
D. 3-phosphoglycerate

45. A patient was delivered to the hospital by an emergency team. Objectively:


grave condition, unconscious, adynamy. Cutaneous surfaces are dry, eyes are
sunken, face is cyanotic. There is tachycardia and smell of acetone from the
mouth. Analysis results: blood glucose - 20,1 micromole/l (standard is 3,3-5,5
micromole/l), urine glucose - 3,5% (standard is - 0). What is the most
probable diagnosis?
A. Hyperglycemic coma
B. Hypoglycemic coma
C. Acute heart failure
D. Acute alcoholic intoxication
E. Anaphylactic shock
46. A 62 y.o. woman complains of frequent pains in the area of her chest and
backbone, rib fractures. A doctor assumed myelomatosis (plasmocytoma).
What of the following laboratory characteristics will be of the greatest
diagnostical importance?
A. Paraproteinemia
B. Hyperalbuminemia
C. Proteinuria
D. Hypoglobulinemia
E. Hypoproteinemia

47. A newborn child has convulsions that have been observed after prescription of
vitamin B6 . This most probable cause of this effect is that vitamin B6 is a
componet of the following enzyme:
A. Glutamate decarboxylase
B. Pyruvate dehydrostase
C. Netoglubarate dehydromine
D. Aminolevulinate synthase
E. Glycogen phosphorylase

48. Nappies of a newborn have dark spots that witness of formation of


homogentisic acid. Metabolic imbalance of which substance is it connected
with?
A. Thyrosine
B. Galactose
C. Methionine
D. Cholesterine
E. Tryptophane

49. Parodontitis is treated with calcium preparations and a hormone that


stimulates tooth mineralization and inhibits tissue resorption. What hormone
is it?
A. Calcitonin
B. Parathormone
C. Adrenalin
D. Aldosterone
E. Thyroxine

50. A child has an acute renal failure. What biochemical factor found in saliva can
confirm this diagnosis?
A. Increase in urea concentration
B. Increase in glucose concentration
C. Decrease in glucose concentration
D. Increase in concentration of higher fatty acids
E. Decrease in nucleic acid concentration
51. After implantation of a cardiac valve a young man constantly takes indirect
anticoagulants. His state was complicated by hemorrhage. What substance
content has decreased in blood?
A. Prothrombin
B. Haptoglobin
C. Heparin
D. Creatin
E. Ceruloplasmin

52. After severe viral hepatitis a 4-year-old boy presents with vommiting,
occasional loss of consciousness, convulsions. Blood test revealed
hyperammoniemia. Such condition is caused by a disorder of the following
biochemical hepatic process:
A. Disorder of ammonia neutralization
B. Disorder of biogenic amines neutralization
C. Protein synthesis inhibition
D. Activation of amino acid decarboxylation
E. Inhibition of transamination enzymes

53. A 50-year-old patient complains about general weakness, appetite loss and
cardiac arrhythmia. The patient presents with muscle hypotonia, flaccid
paralyses, weakened peristaltic activity of the bowels. Such condition might be
caused by:
A. Hypokaliemia
B. Hypoproteinemia
C. Hyperkaliemia
D. Hypophosphatemia
E. Hyponatremia

54. An 18-year-old patient has enlarged inguinal lymphnodes, they are painless,
thickened on palpation. In the area of genital mucous membrane there is a
small-sized ulcer with thickened edges and "laquer" bottom of greyish colour.
What is the most probable diagnosis?
A. Syphilis
B. Tuberculosis
C. Lepra
D. Trophic ulcer
E. Gonorrhea

55. A patient with suspected diphtheria went through bacterioscopic examination.


Examination of throat swab revealed rod-shaped bacteria with volutin
granules. What etiotropic preparation should be chosen in this case?
A. Antidiphtheric antitoxic serum
B. Bacteriophage
C. Diphtheria antitoxin
D. Eubiotic
E. Interferon

56. A woman who has been keeping to a clean-rice diet for a long time was
diagnosed with polyneuritis (beriberi). What vitamin deficit results in
development of this disease?
A. Thiamine
B. Ascorbic acid
C. Pyridoxine
D. Folic acid
E. Riboflavin

57. Removal of gall bladder of a patient has disturbed processes of Ca absorption


through the intestinal wall. What vitamin will stimulate this process?
A. D3
B. PP
C. C
D. B1 2
E. K

58. A 1,5-year-old child presents with both mental and physical lag, decolorizing
of skin and hair, decrease in catecholamine concentration in blood. When a
few drops of 5% solution of trichloroacetic iron had been added to the child’s
urine it turned olive green. Such alteration are typical for the following
pathology of the amino acid metabolism:
A. Phenylketonuria
B. Alkaptonuria
C. Tyrosinosis
D. Albinism
E. Xanthinuria

59. Feces of a patient contain high amount of undissociated fats and have grayish-
white color. Specify the cause of this phenomenon:
A. Obturation of bile duct
B. Hypoactivation of pepsin by hydrochloric acid
C. Hypovitaminosis
D. Enteritis
E. Irritation of intestinal epithelium

60. A patient complains of frequent diarrheas, especially after consumption of


fattening food, and ofbody weight loss. Laboratory examination revealed
steatorrhea; hypocholic feces. What can bethe cause of this condition?
A. Obturation of biliary tracts
B. Mucous membrane inflammation of small intestine
C. Lack of pancreatic lipase
D. Lack of pancreatic phospholipase
E. Unbalanced diet
61. A child is languid, apathetic. Liver is enlarged and liver biopsy revealed a
significant excess of glycogene. Glucose concentration in the blood stream is
below normal. What is the cause of lowglucose concentration?
A. Low (absent) activity of glycogene phosphorylase in liver
B. Low (absent) activity of hexokinase
C. High activity of glycogen synthetase
D. Low (absent) activity of glucose 6-phosphatase
E. Deficit of a gene that is responsible for synthesis of glucose 1 -phosphaturidine
transferase

62. A 46-year-old female patient consulted a doctor about pain in the small joints
of the upper and lower limbs. The joints are enlarged and shaped like
thickened nodes. Serum test revealed an increase in urate concentration. This
might be caused by a disorder in metabolism of:
A. Purines
B. Carbohydrates
C. Lipids
D. Pyrimidines
E. Amino acids

63. A 65-year-old man suffering from gout complains of kidney pain. Ultrasound
examination evealed renal calculi. The most probable cause of calculi
formation is the strengthened concentration of the following substance:
A. Uric acid
B. Cholesterol
C. Bilirubin
D. Urea
E. Cystine

64. A 5-year-old child who often fells ill with respiratory diseases has eczematous
appearances after consumption of some food products, tendency to prolonged
course of inflammatory processes. What kind of diathesis can be suspected in
this case?
A. Exudative-catharral
B. Hemmorhagic
C. Arthritism
D. Lymphohypoplastic
E. Asthenic

65. The greater amount of nitrogen is excreted from the organism in form of urea.
Inhibition of urea synthesis and accumulation of ammonia in blood and
tissues are induced by the decreased activity of the following liver enzyme:
A. Carbamoyl phosphate synthetase
B. Aspartate aminotransferase
C. Urease
D. Amylase
E. Pepsin

66. A 35 y.o. patient who often consumes alcohol was treated with diuretics. There
appeared serious muscle and heart weakness, vomiting, diarrhea, AP- 100/60
mm Hg, depression. This condition is caused by intensified excretion with
urine of:
A. Potassium
B. Sodium
C. Chlorine
D. Calcium
E. Phosphates

67. After intake of rich food, a patient feels nausea and sluggishness; with time
there appeared signs of steatorrhea. Blood cholesterine concentration is 9,2
micromole/l. This condition was caused by lack of:
A. Bile acids
B. Triglycerides
C. Fatty acids
D. Phospholipids
E. Chylomicrons

68. Examination of a man who hadn't been consuming fats but had been getting
enough carbohydrates and proteins for a long time revealed dermatitis, poor
wound healing, vision impairment. What is the probable cause of metabolic
disorder?
A. Lack of linoleic acid, vitamins A, D, E, K
B. Lack of palmitic acid
C. Lack of vitamins PP, H
D. Low caloric value of diet
E. Lack of oleic acid

69. A 44-year-old woman complains of common weakness, heart pain,


considerable increase of body weigt. Objectively: moon-like face, hirsutism,
AP- 165/100 mm Hg, height - 164 cm, weight - 103 kg; fat is mostly
accumulated in the region of neck, upper shoulder girdle, stomach. What is
the main pathogenetic mechanism of obesity?
A. Increased production of glucocorticoids
B. Decreased production of thyroidal hormones
C. Increased production of insulin
D. Decreased production of glucagon
E. Increased production of mineralocorticoids

70. An experimental animal has been given excessive amount of carbon-labeled


glucose for a week. What compound can the label be found in?
A. Palmitic acid
B. Methionine
C. Vitamin A
D. Choline
E. Arachidonic acid

71. After a serious viral infection, a 3-year-old child has repeated vomiting, loss of
consciousness, convulsions. Examination revealed hyperammoniemia. What
may have caused changes of biochemical blood indices of this child?
A. Disorder of ammonia neutralization in ornithinic cycle
B. Activated processes of aminoacids decarboxylation
C. Disorder of biogenic amines neutralization
D. Increased purtefaction of proteins in intestines
E. Inhibited activity of transamination enzymes

72. Examination of a patient with frequent hemorrhages from internals and


mucous membranes revealed proline and lysine being a part of collagene
fibers. What vitamin absence caused disturbance of their hydroxylation?
A. Vitamin C
B. Vitamin K
C. Vitamin A
D. Thiamine
E. Vitamin E

73. Albinos can't stand sun impact - they don't aquire sun-tan but get sunburns.
Disturbed metabolism of what aminoacid underlies this phenomenon?
A. Phenilalanine
B. Methionine
C. Tryptophan
D. Glutamic acid
E. Histidine

74. A nurse accidentally injected a nearly double dose of insulin to a patient with
diabetes mellitus. The patient lapsed into a hypoglycemic coma. What drug
should be injected in order to help him out of coma?
A. Glucose
B. Lidase
C. Insulin
D. Somatotropin
E. Noradrenaline

75. A biochemical urine analysis has been performed for a patient with
progressive muscular dystrophy. In the given case muscle disease can be
confirmed by the high content of the following substance in urine:
A. Creatine
B. Porphyrin
C. Urea
D. Hippuric acid
E. Creatinine

76. A patient with continious bronchopneumonia was admitted to the therapeutic


department. Antibiotic therapy didn't give much effect. What medication for
improvement of immune state should be added to the complex treatment of
this patient?
A. Timaline
B. Analgin
C. Sulfocamphocaine
D. Benadryl
E. Paracetamol

77. A patient suffers from hepatic cirrhosis. Examination of which of the following
substances excreted by urine can characterize the state of antitoxic function of
liver?
A. Hippuric acid
B. Ammonium salts
C. Kreatinine
D. Uric acid
E. Aminoacids

78. Malaria is treated with structural analogs of vitamin B2 (riboflavin). These


drugs disrupt the synthesis of the following enzymes in plasmodium:
A. FAD-dependent dehydrogenase
B. Cytochrome oxidase
C. Peptidase
D. NAD-dependent dehydrogenase
E. Aminotransferase

79. Vitamin A together with specific cytoreceptors penetrates through the nuclear
membranes, induces transcription processes that stimulate growth and
differentiation of cells. This biological function is realized by the following
form of vitamin A:
A. Trans-retinoic acid
B. Trans-retinal
C. Cis-retinal
D. Retinol
E. Carotin

80.Products of some proteins hydrolysis and modification are the biologically


active substances called hormones. Lipotropin, corticotropin, melanotropin
and endorphins are synthesized in the hypophysis of the following protein:
A. Proopiomelanocortin (POMC)
B. Neuroalbumin
C. Neurostromin
D. Neuroglobulin
E. Thyreoglobulin

81. In patients with the biliary tract obstruction the blood coagulation is
inhibited; the patients have frequent haemorrhages caused by the subnormal
assimilation of the following vitamin:
A. K
B. A
C. D
D. E
E. C

82. A newborn child suffers from milk curdling in stomach, this means that
soluble milk proteins (caseins) transform to insoluble proteins (paracaseins)
by means of calcium ions and a certain enzyme. What enzyme takes part in
this process?
A. Renin
B. Pepsin
C. Gastrin
D. Secretin
E. Lipase

83. A sportsman was recommended to take a medication that contains carnitine


in order to improve his results. What process is activated by carnitine the
most?
A. Fatty acids transport to mitochondrions
B. Synthesis of steroid hormones
C. Synthesis of ketone bodies
D. Synyhesis of lipids
E. Tissue respiration

84. Human red blood cells do not contain mitochondria. What is the main
pathway for ATP production in these cells?
A. Anaerobic glycolysis
B. Aerobic glycolysis
C. Oxidative phosphorylation
D. Creatine kinase reaction
E. Cyclase reaction

85. To prevent postoperative bleeding a 6 y.o. child was administered vicasol that
is a synthetic analogue of vitamin K. Name post-translational changes of blood
coagulation factors that will be activated by vicasol:
A. Carboxylation of glutamin acid
B. Phosphorylation of serine radicals
C. Partial proteolysis
D. Polymerization
E. Glycosylation

86. A 4 y.o. boy has had recently serious viral hepatitis. Now there are such
clinical presentations as vomiting, loss of consciousness, convulsions. Blood
analysis revealed hyperammoniemia. Disturbunce of which biochemical
process caused such pathological condition of the patient?
A. Disturbed neutralization of ammonia in liver
B. Disturbed neutralization of biogenic amines
C. Increased putrefaction of proteins in bowels
D. Activation of aminoacid decarboxylation
E. Inhibition of transamination enzyms

87. During examination of an 11-month-old infant a pediatrician revealed


osteoectasia of the lower extremities and delayed mineralization of cranial
bones. Such pathology is usually provoked by the deficit of the following
vitamin:
A. Cholecalciferol
B. Thiamin
C. Pantothenic acid
D. Bioflavonoids
E. Riboflavin

88. Examination of a patient suffering from chronic hepatitis revealed a


significant decrease in the synthesis and secretion of bile acids. What process
will be mainly disturbed in the patient’s bowels?
A. Fat emulsification
B. Protein digestion
C. Carbohydrate digestion
D. Glycerin absorption
E. Amino acid absorption

89. Glutamate decarboxylation results in formation of inhibitory transmitter in


CNS. Name it:
A. GABA
B. Glutathione
C. Histamine
D. Serotonin
E. Asparagine

90. In course of histidine catabolism a biogenic amin is formed that has powerful
vasodilatating effect. Name it:
A. Histamine
B. Serotonin
C. Dioxyphenylalanine
D. Noradrenalin
E. Dopamine
91. Utilization of arachidonic acid via cyclooxigenase pathway results in
formation of some bioactive substances. Name them:
A. Prostaglandins
B. Thyroxine
C. Biogenic amins
D. Somatomedins
E. Insulin-like growth factors

92. After a sprint an untrained person develops muscle hypoxia. This leads to the
accumulation of the following metabolite in muscles:
A. Lactate
B. Ketone bodies
C. Acetyl CoA
D. Glucose 6-phosphate
E. Oxaloacetate

93. Myocyte cytoplasm contains a big number of dissolved metabolites of glucose


oxidation. Name one of them that turns directly into a lactate:
A. Pyruvate
B. Oxaloacetate
C. Glycerophosphate
D. Glucose 6-phosphate
E. Fructose 6-phosphate

94. Emotional stress causes activation of hormon-sensitive triglyceride lipase in


the adipocytes. What secondary mediator takes part in this process?
A. Cyclic adenosine monophosphate
B. Cyclic guanosine monophosphate
C. Adenosine monophosphate
D. Diacylglycerol
E. Ions of Са2 +

95. A patient diagnosed with carcinoid of bowels was admitted to the hospital.
Analysis revealed high production of serotonin. It is known that this substance
is formed of tryptophane aminooacid. What biochemical mechanism underlies
this process?
A. Decarboxylation
B. Desamination
C. Microsomal oxydation
D. Transamination
E. Formation of paired compounds

96. A genetics specialist analyzed the genealogy of a family and found that both
males and females may have the illness, not across all the generations, and
that healthy parents may have ill children. What is the type of illness
inheritance?
A. Autosomal recessive
B. Autosomal dominant
C. X-linked dominant
D. X-linked recessive
E. Y-linked

97. Analysis of amniotic fluid that was obtained as a result of amniocentesis


(puncture of amniotic sac) revealed cells the nuclei of which contain sex
chromatin (Barr's body). What can it be evidence of?
A. Development of female fetus
B. Development of male fetus
C. Genetic disorders of fetus development
D. Trisomy
E. Polyploidy

98. Vitamin B1 deficiency results in disturbance of oxidative decarboxylation of α-


ketoglutaric acid. This will disturb synthesis of the following coenzyme:
A. Thiamine pyrophosphate
B. Nicotinamide adenine dinucleotide (NAD)
C. Flavine adenine dinucleotide (FAD)
D. Lipoic acid
E. Coenzyme A

99. A child's blood presents high content of galactose, glucose concentration is


low. There are suchpresentations as cataract, mental deficiency, adipose
degeneration of liver. What disease is it?
A. Galactosemia
B. Diabetes mellitus
C. Lactosemia
D. Steroid diabetes
E. Fructosemia

100. According to clinical indications a patient was administered pyridoxal


phosphate. What processes is this medication intended to correct?
A. Transamination and decarboxylation of aminoacids
B. Oxidative decarboxylation of ketonic acids
C. Desamination of purine nucleotide
D. Synthesis of purine and pyrimidine bases
E. Protein synthesis

101. A 45 y.o. woman suffers from Cushing's syndrome - steroid diabetes.


Biochemical examination revealed: hyperglycemia, hypochloremia. Which of
the under-mentioned processes is the first tobe activated?
A. Gluconeogenesis
B. Glycogenolysis
C. Glucose reabsorption
D. Glucose transport to the cell
E. Glycolysis

102. Autopsy of a 46-year-old man revealed multiple brown-and-green


layers and hemmorhages on the mucous membrane of rectum and sigmoid
colon; slime and some blood in colon lumen; histologically - fibrinous colitis.
In course of bacteriological analysis of colon contents S. Sonne were found.
What is the most probable diagnosis?
A. Dysentery
B. Cholera
C. Salmonellosis
D. Yersiniosis
E. Crohn's disease

103. A patient had been ill with bronchial asthma for many years and died
from asthmatic fit. Histologic lung examination revealed: lumen of
bronchioles and small bronches contain a lot of mucus with some eosinophils,
there is sclerosis of alveolar septums, dilatation of alveole lumen. What
mechanism of development of hypersensibility reaction took place?
A. Reagin reaction
B. Cytotoxic reaction
C. Immunocomplex reaction
D. Cytolysis determined by lymphocytes
E. Granulomatosis

104. Those organisms which in the process of evolution failed to develop


protection from H2 O2 can exist only in anaerobic conditions. Which of the
following enzymes can break hydrogen peroxide down?
A. Peroxidase and catalase
B. Oxygenase and hydroxylase
C. Cytochrome oxidase, cytochrome B5
D. Oxygenase and catalase
E. Flavin-dependent oxidase

105. Desulfiram is widely used in medical practice to prevent alcocholism. It


inhibits aldehyde dehydrogenase. Increased level of what metabolite causes
aversion to alcochol?
A. Acetaldehyde
B. Ethanol
C. Malonyl aldehyde
D. Propionic aldehyde
E. Methanol

106. A 1-year-old child with symptoms of muscle involvement was admitted


to the hospital. Examination revealed carnitine deficiency in his muscles.
What process disturbance is the biochemical basis of this pathology?
A. Transporting of fatty acids to mitochodrions
B. Regulation of Ca2 + level in mitochondrions
C. Substrate phosphorylation
D. Lactic acid utilization
E. Actin and myosin synthesis

107. A man got poisoned with mushrooms. They contain muscarine that
stimulates muscarinic cholinoreceptors. What symptom is typical for
poisoning with inedible mushrooms?
A. Miosis
B. Mydriasis
C. Bronchi dilation
D. Heart rate rise
E. Arterial pressure rise

108. Prolonged fasting causes hypoglycemia which is amplified by alcohol


consumption, as the following process is inhibited:
A. Gluconeogenesis
B. Glycolysis
C. Glycogenolysis
D. Lipolysis
E. Proteolysis

109. As a result of posttranslative modifications some proteins taking part in


blood coagulation, particularly prothrombin, become capable of calcium
binding. The following vitamin takes part in this process:
A. K
B. C
C. A
D. B1
E. B2

110. Objective examination of a patient revealed: slender figure, big skull,


highly developed frontal region of face, short extremities. What constitutional
type is it characteristic for?
A. Respiratory
B. Muscular
C. Digestive
D. Cerebral
E. Mixed

111. A 38-year-old patient died during intractable attack of bronchial


asthma. Histologic examination revealed mucus accumulation in bronchial
lumen, a lot of fat cells (labrocytes) in the wall of bronches, many of them are
in the state of degranulation, there are also a lot of eosinophils. What
pathogenesis of bronchial changes is it?
A. Atopy
B. Cytotoxic, cytolytic action of antibodies
C. Immunocomplex mechanism
D. Cellular cytolysis
E. Granulomatosis

112. Diabetes mellitus causes ketosis as a result of activated oxidation of


fatty acids. What disorders of acid-base equilibrium may be caused by
excessive accumulation of ketone bodies in blood?
A. Metabolic acidosis
B. Metabolic alcalosis
C. Any changes woun't happen
D. Respiratory acidosis
E. Respiratory alcalosis

113. A woman with 0 (I) bllod group has born a child with AB blood group.
This woman's husband has A blood group. What genetic interaction explains
this phenomenon?
A. Recessive epistasis
B. Codominance
C. Polymery
D. Incomplete dominance
E. Complementation

114. Osteolaterism is charcterized by a decrease in collagen strength caused


by much less intensiveformation of cross-links in collagen fibrils. This
phenomenon is caused by the low activity of the following enzyme:
A. Lysyl oxidase
B. Monoamino-oxidase
C. Prolyl hydroxylase
D. Lysyl hydroxylase
E. Collagenase

115. Depressions and emotional insanities result from the deficit of


noradrenalin, serotonin and other biogenic amines in the brain. Their
concentration in the synapses can be increased by means of the
antidepressants that inhibit the following enzyme:
A. Monoamine oxidase
B. Diamine oxidase
C. L-amino-acid oxidase
D. D-amino-acid oxidase
E. Phenylalanine-4-monooxygenase

116. A 3-year-old child with symptoms of stomatitis, gingivitis and


dermatitis of open skin areas was delivered to a hospital. Examination
revealed inherited disturbance of neutral amino acid transporting in the
bowels. These symptoms were caused by the deficiency of the following
vitamin:
A. Niacin
B. Pantothenic acid
C. Vitamin A
D. Cobalamin
E. Biotin

117. A 5-month-old boy was hospitalized for tonic convulsions. He has a


life-time history of this disease. Examination revealed coarse hair, thinned
and fragile nails, pale and dry skin. In blood: calcium - 1,5 millimole/l,
phosphor - 1,9 millimole/l. These changes are associated with:
A. Hypoparathyroidism
B. Hyperparathyroidism
C. Hyperaldosteronism
D. Hypoaldosteronism
E. Hypothyroidism

118. During hypersensitivity test a patient got subcutaneous injection of an


antigen which caused reddening of skin, edema, pain as a result of histamine
action. This biogenic amine is generatedas a result of transformation of the
following histidine amino acid:
A. Decarboxylation
B. Methylation
C. Phosphorylation
D. Isomerization
E. Deaminization

119. A patient with suspected diagnosis "progressing muscular dystrophy"


got his urine tested. What compound will confirm this diagnosis if found in
urine?
A. Kreatine
B. Collagen
C. Porphyrin
D. Myoglobin
E. Calmodulin

120. Cytogenetic examination of a patient with dysfunction of the


reproductive system revealed normal karyotype 46, ХУ in some cells, but most
cells have Klinefelter's syndrome karyotype - 47, ХХУ. Such phenomenon of
cell inhomogeneity is called:
A. Mosaicism
B. Inversion
C. Transposition
D. Duplication
E. Heterogeneity

121. An oncological patient had been administered methotrexate. With time


target cells of the tumour lost sensitivity to this drug. At the same time the
change in gene expression of the following enzyme is observed:
A. Dehydropholate reductase
B. Thiaminase
C. Deaminase
D. Pholate oxidase
E. Pholate decarboxylase

122. A patient complained about dizziness, memory impairment, periodical


convulsions. It was revealed that these changes were caused by a product of
decarboxylation of glutamic acid. Name this product:
A. GABA
B. Pyridoxal phosphate
C. TDP
D. ATP
E. THFA

123. A sportsman needs to improve his sporting results. He was


recommended to take a preparation that contains carnitine. What process is
activated the most by this compound?
A. Fatty acids transporting
B. Amino acids transporting
C. Calcium ions transporting
D. Glucose transporting
E. Vitamin K transporting

124. A doctor examined a child and revealed symptoms of rachitis.


Development of this desease was caused by deficiency of the following
compound:
A. 1,25 [ОН]-dichydroxycholecalciferol
B. Biotin
C. Tocopherol
D. Naphtaquinone
E. Retinol

125. Laboratory examination of a child revealed increased concentration of


leucine, valine, isoleucine and their ketoderivatives in blood and urine. Urine
smelt of maple syrup. This disease is characterized by the deficit of the
following enzyme:
A. Dehydrogenase of branched amino acids
B. Aminotransferase
C. Glucose-6-phosphatase
D. Phosphofructokinase
E. Phosphofructomutase

126. A 9-month-old infant is fed with artificial formulas with unbalanced


vitamin B6 concentration. The infant presents with pellagral dermatitis,
convulsions, anaemia. Convulsion development might be caused by the
disturbed formation of:
A. GABA
B. Histamine
C. Serotonin
D. DOPA
E. Dopamine

127. It was found out that some compounds, for instance fungi toxins and
some antibiotics can inhibit activity of RNA-polymerase. What process will be
disturbed in a cell in case of inhibition of this enzyme?
A. Transcription
B. Processing
C. Replication
D. Translation
E. Reparation

128. When blood circulation in the damaged tissue is restored, then lactate
accumulation comes to a stop and glucose consumption decelerates. These
metabolic changes are caused by activation of the following process:
A. Aerobic glycolysis
B. Anaerobic glycolysis
C. Lipolysis
D. Gluconeogenesis
E. Glycogen biosynthesis

129. During starvation muscle proteins break up into free amino acids.
These compounds will be the most probably involved into the following
process:
A. Gluconeogenesis in liver
B. Gluconeogenesis in muscles
C. Synthesis of higher fatty acids
D. Glycogenolysis
E. Decarboxylation

130. Examination of a patient revealed II grade obesity. It is known that he


consumes a lot of sweets and rich food, has sedentary way of life. That's why
anabolic metabolism has the priority in his organism. Which of the following
pathways is amphibolic?
A. Cycle of tricarboxylic acids
B. Glyconeogenesis
C. Lipolysis
D. Glycolysis
E. Fatty acids oxidation

131. Surgical removal of a part of stomach resulted in disturbed absorption


of vitamin B12, it is excreted with feces. The patient was diagnosed with
anemia. What factor is necessary for absorption of this vitamin?
A. Gastromucoprotein
B. Gastrin
C. Hydrochloric acid
D. Pepsin
E. Folic acid

132. A newborn develops dyspepsia after the milk feeding. When the milk is
substituted by the glucose solution the dyspepsia symptoms disappear. The
newborn has the subnormal activity of the following enzyme:
A. Lactase
B. Invertase
C. Maltase
D. Amylase
E. Isomaltase

133. Patients who suffer from severe diabetes and don't receive insulin have
metabolic acidosis. This is caused by increased concentration of the following
metabolites:
A. Ketone bodies
B. Fatty acids
C. Unsaturated fatty acids
D. Triacylglycerols
E. Cholesterol

134. A 4-year-old child with hereditary renal lesion has signs of rickets,
vitamin D concentration in blood is normal. What is the most probable cause
of rickets development?
A. Impaired synthesis of calcitriol
B. Increased excretion of calcium
C. Hyperfunction of parathyroid glands
D. Hypofunction of parathyroid glands
E. Lack of calcium in food

135. A patient present with dysfunction of cerebral cortex accompanied by


epileptic seizures. He has been administered a biogenic amine synthetized
from glutamate and responsible for central inhibition. What substance is it?
A. Gamma-amino butyric acid
B. Serotonin
C. Dopamine
D. Acetylcholine
E. Histamine

136. Toxic affection of liver results in dysfunction of protein synthesis. It is


usually accompanied by the following kind of dysproteinemia:
A. Absolute hypoproteinemia
B. Relative hypoproteinemia
C. Absolute hyperproteinemia
D. Relative hyperproteinemia
E. Paraproteinemia

137. A 6-year-old child was delivered to a hospital. Examination revealed


that the child couldn't fix his eyes, didn't keep his eyes on toys, eye ground had
the cherry-red spot sign. Laboratory analyses showed that brain, liver and
spleen had high rate of ganglioside glycometide. What congenital disease is
the child ill with?
A. Tay-Sachs disease
B. Wilson's syndrome
C. Turner's syndrome
D. Niemann-Pick disease
E. MacArdle disease

138. In clinical practice tuberculosis is treated with izoniazid preparation -


that is an antivitamin able to penetrate into the tuberculosis bacillus.
Tuberculostatic effect is induced by the interference with replication processes
and oxidation-reduction reactions due to the buildup of pseudo-coenzyme:
A. NAD
B. FAD
C. FMN
D. TDP
E. CoQ

139. A newborn child was found to have reduced intensity of sucking,


frequent vomiting, hypotonia. Urine and blood exhibit increased
concentration of citrulline. What metabolic process is disturbed?
A. Ornithinic cycle
B. Tricarboxylic acid cycle
C. Glycolysis
D. Glyconeogenesis
E. Cori cycle

140. Dietary intake of a 30-year-old nursing woman contains 1000 mg of


calcium, 1300 mg of phosphorus and 20 mg of iron per day. It is necessary to
change content of these mineral substances in the following way:
A. To increase phosphorus content
B. To increase calcium content
C. To reduce fluorine content
D. To increase iron content
E. To reduce iron content

141. Cardinal symptoms of primary hyperparathyroidism are osteoporosis


and renal lesion along with development of urolithiasis. What substance
makes up the basis of these calculi in this disease?
A. Calcium phosphate
B. Uric acid
C. Cystine
D. Bilirubin
E. Cholesterol

142. Study of conversion of a food colouring agent revealed that


neutralization of this xenobiotic takes place only in one phase - microsomal
oxydation. Name a component of this phase:
A. Cytochrome Р-450
B. Cytochrome B
C. Cytochrome C
D. Cytochrome A
E. Cytochrome oxidase

143. A patient had hemorrhagic stroke. Blood examination revealed


strengthened kinin concentration. The patient was prescribed contrical. It was
administered in order to inhibit the following proteinase:
A. Kallikrein
B. Pepsin
C. Trypsin
D. Chemotrypsin
E. Collagenase

144. A 49-year-old driver complains about unbearable constricting pain


behind the breastbone irradiating to the neck. The pain arose 2 hours ago.
Objectively: the patient’s condition is grave, he is pale, heart tones are
decreased. Laboratory studies revealed high activity of creatine kinase and
LDH1 . What disease are these symptoms typical for?
A. Acute myocardial infarction
B. Acute pancreatitis
C. Stenocardia
D. Cholelithiasis
E. Diabetes mellitus

145. Plasmic factors of blood coagulation are exposed to post-translational


modification with the participation of vitamin K. It is necessary as a cofactor
in the enzyme system of γ-carboxylation of protein factors of blood
coagulation due to the increased affinity of their molecules with calcium ions.
What amino acid is carboxylated in these proteins?
A. Glutamic
B. Valine
C. Serine
D. Phenylalanine
E. Arginine

146. Pharmacological effects of antidepressants are connected with


inhibition of an enzyme catalyzing biogenic amines noradrenaline and
serotonine in the mitochondrions of cerebral neurons. What enzyme
participates in this process?
A. Monoamine oxidase
B. Transaminase
C. Decarboxylase
D. Peptidase
E. Lyase

147. An oncological patient was prescribed methotrexate. With the lapse of


time target cells of the tumour lost susceptibility to this drug. There is change
of gene expression of the folowing enzyme:
A. Dehydrofolate reductase
B. Thiaminase
C. Deaminase
D. Folate oxidase
E. Folate decarboxylase

148. A 67-year-old male patient consumes eggs, pork fat, butter, milk and
meat. Blood test results: cholesterol - 12,3 mmol/l, total lipids - 8,2 g/l,
increased low-density lipoprotein fraction (LDL). What type of
hyperlipoproteinemia is observed in the patient?
A. Hyperlipoproteinemia type IIa
B. Hyperlipoproteinemia type I
C. Hyperlipoproteinemia type IIb
D. Hyperlipoproteinemia type IV
E. Cholesterol, hyperlipoproteinemia

149. A patient with signs of osteoporosis and urolithiasis has been admitted
to the endocrinology department. Blood test has revealed hypercalcemia and
hypophosphatemia. These changes are associated with abnormal synthesis of
the following hormone:
A. Parathyroid hormone
B. Calcitonin
C. Cortisol
D. Aldosterone
E. Calcitriol

150. Increased HDL levels decrease the risk of atherosclerosis. What is the
mechanism of HDL anti-atherogenic action?
A. They remove cholesterol from tissues
B. They supply tissues with cholesterol
C. They are involved in the breakdown of cholesterol
D. They activate the conversion of cholesterol to bile acids
E. They promote absorption of cholesterol in the intestine
Krok 1 – 2014 Histology Base
1. An electronic microphotograph shows a macrophagic cell with erythrocytes at
different stages of differentiation located along its processes. This is the cell of
the following organ:
A. Red bone marrow
B. Thymus
C. Spleen
D. Tonsil
E. Lymph node

2. Decreased blood supply to the organs causes hypoxia that activates fibroblasts
function. Volume of what elements is increased in this case?
A. Intercellular substance
B. Vessels of microcircular stream
C. Nerve elements
D. Parenchymatous elements of the organ
E. Lymphatic vessels

3. Lung of premature infant is presented on electronic photomicrography of


biopsy material. Collapse of the alveolar wall caused by the deficiency of
surfactant was revealed. Disfunction of what cells of the alveolar wall caused it?
A. Alveocytes type II
B. Alveocytes type I
C. Alveolar macrophages
D. Secretory cells
E. Fibroblasts

4. During histological examination of the stomach it was found out that glands
contain very small amount of pariental cells or they are totally absent. Mucose
membrane of what part of the stomach was studied?
A. Pyloric part
B. Fundus of stomach
C. Cardiak part
D. Body of stomach
E. -

5. In the blood of a 26-year-old man it was revealed 18% of erythrocytes of the


spherical, ball-shaped, flat and thorn-like shape. Other eritrocytes were in the
form of the concavo-concave disks. How is such phenomenon called?
A. Physiological poikilocytosis
B. Pathological poikilocytosis
C. Physiological anisocytosis
D. Pathological anisocytosis
E. Erytrocytosis
6. Hypertrychosis of auricles is caused by a gene that is localized in Y-
chromosome. Father has this feature. What is the probability to give birth to a
boy with such anomaly?
A. 100%
B. 0%
C. 25%
D. 35%
E. 75%

7. During postembryonal haemopoiesis in the red bone marrow the cells of one of
the cellular differons demonstrate a gradual decrease in cytoplasmic basophilia
as well as an increase in oxyphilia, the nucleus is being forced out. Such
morphological changes are typical for the following haemopoiesis type:
A. Erythropoiesis
B. Lymphopoiesis
C. Neutrophil cytopoiesis
D. Eosinophil cytopoiesis
E. Basophil cytopoiesis

8. In the microspecimen of red bone marrow there were revealed multiple


capillares through the walls of which mature blood cells penetrated. What type
of capillares is it?
A. Sinusoidal
B. Fenestrational
C. Somatical
D. Visceral
E. Lymphatic

9. Autopsy of a man who died from chronic cardiacvascular collapse revealed


"tiger heart". Sidewards of endocardium a yellowish-white banding can be seen;
myocardium is dull, dark-yellow. What process caused this pathology?
A. Fatty parenchymatous degeneration
B. Carbohydrate degeneration
C. Hyaline degeneration
D. Fatty vascular-stromal degeneration
E. Amyloidosis

10. Most participants of Magellan expedition to America died from avitominosis.


This disease declared itself by general weakness, subcutaneous hemmorhages,
falling of teeth, gingival hemmorhages. What is the name of this avitiminosis?
A. Scurvy
B. Pellagra
C. Rachitis
D. Polyneuritis (beriberi)
E. Biermer's anemia
11. A histological specimen presents a receptor zone of a sensoepithelial sense
organ. Cells of this zone are placed upon the basal membrane and include the
following types: external and internal receptor cells, external and internal
phalangeal cell, stem cells, external limiting cells and external supporting cell.
The described receptor zone belongs to the following sense organ:
A. Acoustic organ
B. Visual organ
C. Gustatory organ
D. Equilibrium organ
E. Olfactory organ

12. A patient was admitted to the hospital with an asphyxia attack provoked by a
spasm of smooth muscles of the respiratory tracts. This attack was mainly
caused by alterations in the following parts of the airways:
A. Small bronchi
B. Median bronchi
C. Large bronchi
D. Terminal bronchioles
E. Respiratory part

13. Examination of a 43 y.o. patient revealed that his stomach has difficulties with
digestion of protein food. Gastric juice analysis revealed low acidity. Function
of which gastric cells is disturbed in this case?
A. Parietal exocrinocytes
B. Main exocrinocytes
C. Mucous cells (mucocytes)
D. Endocrinous cells
E. Cervical mucocytes

14. A patient has been given high doses of hydrocortisone for a long time. This
caused atrophy of one of the adrenal cortex zones. Which zone is it?
A. Fascial
B. Glomerular
C. Reticular
D. Glomerular and reticular
E. -

15. In a histological specimen parenchyma of an organ is represented by lymphoid


tissue that forms lymph nodes; the latter are arranged in a diffuse manner and
enclose a central artery. What anatomic formation has such morphological
structure?
A. Spleen
B. Tonsil
C. Lymph node
D. Thymus
E. Red bone marrow
16. As a result of a trauma a patient has damaged anterior roots of spinal cord.
What structures have been affected?
A. Axons of motoneurons and axons of neurons of lateral horns
B. Central processes of sensitive neurons of spinal ganglions
C. Peripheral processes of sensitive spinal ganglions
D. Axons of neurons of lateral horns
E. Dendrites of neurons of spinal ganglions

17. A histological specimen of a kidney shows a part of the distal tubule going
between the afferent and efferent arteriole. The cells building the tubule wall
have dense nuclei; basal membrane is absent. Such structural formation is
called:
A. Macula densa
B. Juxtaglomerular cells
C. Mesangial cells
D. Juxtavascular cells
E. -

18. In course of an experiment a big number of stem cells of red bone marrow was
in some way destructed. Regeneration of which cell populations in the loose
connective tissue will be inhibited?
A. Of macrophags
B. Of fibroblasts
C. Of pigment cells
D. Of lipocytes
E. Of pericytes

19. Histological examination of a 40 y.o. man's thymus revealed decreased share of


parenchymatous gland elements, increased share of adipose and loose
connective tissue, its enrichment with thymus bodies. The organ's mass was
unchanged. What phenomenon is it?
A. Age involution
B. ccidental involution
C. Hypotrophy
D. Dystrophy
E. Atrophy

20. A histological specimen shows a blood vessel. Its inner coat is composed by
endothelium, subendothelium and internal elastic membrane. The middle coat
is enriched with smooth myocytes. Such morphological characteristics are
typical for the following vessel:
A. Muscular-type artery
B. Elastic-type artery
C. Capillary
D. Non-muscular vein
E. Muscular-type vein

21. In course of indirect histogenesis of tubular bone tissue a plate is formed


between epiphyseal and diaphyseal ossification centres that provides further
lengthwise growth of bones. What structure is it?
A. Metaphyseal plate
B. Osseous cuff
C. Osseous plate
D. Osteon
E. Layer of interior general plates

22. A 2-year-old child has got intestinal dysbacteriosis, which results in


hemorrhagic syndrome. What is the most likely cause of hemorrhage of the
child?
A. Vitamin K insufficiency
B. Activation of tissue thromboplastin
C. PP hypovitaminosis
D. Fibrinogen deficiency
E. Hypocalcemia

23. Histological specimen presents a vessel the wall of which consists of


endothelium, basal membrane and loose connective tissue. What type of vessel
is it?
A. Vein of non-muscular type
B. Artery
C. Vein of muscular type
D. Hemocapillary
E. Lymphocapillary

24. Vitamin A deficit results in the impairment of twilight vision. Name the cells
that have the above-mentioned photoreceptor function:
A. Rod receptor cell
B. Horizontal neurocytes
C. Cone receptor cells
D. Bipolar neurons
E. Ganglion neurocytes

25. Kidneys of a man under examination show increased resorbtion of calcium ions
and decreased resorbtion of phosphate ions. What hormone causes this
phenomenon?
A. Parathormone
B. Thyrocalcitonin
C. Hormonal form D3
D. Aldosterone
E. Vasopressin
26. A 22-year-old female student consulted a physician about fever up to 38oC,
weakness, sore throat. Objectively: there is white coating of the tongue. What
histological structures of the tongue are involved in the formation of this
coating?
A. Epithelium of the filiform papillae
B. Epithelium of the foliate papillae
C. Epithelium of the fungiform papillae
D. Epithelium of the circumvallate papillae
E. Connective-tissue base of all the lingual papillae

27. During pubescence the cells of male sexual glands begin to produce male sex
hormon testosterone that calls forth secondary sexual characters. What cells of
male sexual glands produce this hormone?
A. Leidig cells
B. Sustentocytes
C. Sertoli's cells
D. Supporting cells
E. Spermatozoa

28. A patient with thrombophlebitis of lower extremities had got chest pains, blood
spitting, growing respiratory failure that caused his death. Autopsy revealed
multiple pulmonary infarctions. What is the most probable reason of their
development?
A. Pulmonary artery embolism
B. Pulmonary artery thrombosis
C. Bronchial artery thrombosis
D. Bronchial artery embolism
E. Pulmonary venous thrombosis

29. A patient complains of dryness of head skin, itching, fragility and loss of hair.
After examination he was diagnosed with seborrhea. Disturbed activity of which
cells caused this condition?
A. Cells of sebaceous glands
B. Cells of sudoriferous glands
C. Epithelial cells
D. Adipocytes
E. Melanocytes

30. A histological specimen of kidney shows a structure consisting of a glomerulus


of fenestrated capillaries and a bilayer epithelial capsule. Specify this structure:
A. Renal corpuscle
B. Proximal tubule
C. Distal tubule
D. Henle's loop
E. Receiving tube
31. A sensitive neural ganglion consists of roundish neurocytes with one extension
that divides into axon and dendrite at some distance from the perikaryon. What
are these cells called?
A. Pseudounipolar
B. Unipolar
C. Bipolar
D. Multipolar
E. Apolar

32. An embryo displays disturbed process of dorsal mesoderm segmentation and


somite formation. What part of skin will have developmental abnormalities?
A. Derma
B. Hair
C. Sebaceous glands
D. Epidermis
E. Sudoriferous glands

33. A microslide contains the preparation of a gland composed of several secretory


saccule-shaped parts that open in the common excretory duct. What gland is it?
A. Simple branched alveolar gland
B. Compound branched alveolar gland
C. Simple unbranched alveolar gland
D. Compound unbranched alveolar gland
E. Simple branched tubular gland

34. A scheme presents an exocrinous gland that has unbranched excretory duct
with a terminal part in form of a saccule openining into the duct. How is this
gland called according to the morphological classification of exocrinous glands?
A. Simple unbranched alveolar
B. Compound branched alveolar
C. Simple branched tubular
D. Compound unbranched alveolar
E. Compound unbranched alveolar tubular

35. Roentgenological examination of skull base bones revealed enlargement of


sellar cavity, thinning of anterior clinoid processes, destruction of different
parts, destruction of different parts of sella turcica. Such bone destruction
might be caused by a tumour of the following endocrinous gland:
A. Hypophysis
B. Epiphysis
C. Thymus gland
D. Adrenal glands
E. Thyroid gland

36. Electronic microphotography of pulmonary alveole's wall presents a big cell. Its
cytoplasm has a lot of mitochondria, developed Golgi apparatus, osmiophil
lamellated corpuscles. What is the main function of this cell?
A. It produces surfactant
B. It is a component of blood-air barrier
C. It warms the air
D. It purifies the air
E. It absorbs microorganisms

37. A pathological process in bronchi resulted in epithelium desquamation. What


cells will regenerate bronchial epithelium?
A. Basal
B. Intercalary
C. Ciliate
D. Endocrinal
E. Goblet

38. A viral infection has damaged cells that form walls of bile capillaries. This
stimulated conditions for inflow of bile into the blood of sinusoidal capillaries.
What cells are damaged?
A. Hepatocytes
B. Kupffer's cells
C. Ito cells
D. Pit-cells
E. Endotheliocytes

39. A histological specimen of spleen shows a vessel with a wall consisting of


endothelium and subendothelial layer, median membrane is absent, exterior
membrane inosculates with the layers of spleen connective tissue. What vessel
is it?
A. Vein of non-muscular type
B. Vein of muscular type
C. Artery of muscular type
D. Arteriole
E. Capillary

40. In course of a conditional experiment the development of mesenchyma cells


was completely inhibited. Development of the following muscular tissue will be
disturbed:
A. Smooth muscular tissue
B. Neural muscular tissue
C. Epidermal muscular tissue
D. Cardiac muscular tissue
E. Skeletal muscular tissue

41. A patient ill with chronic gastritis went for endogastric pH-metry that allowed
to reveal decreased acidity of gastric juice. It is indicative of diminished
function of the following cells:
A. Parietal exocrinocytes
B. Chief exocrinocytes
C. Endocrinocytes
D. Cervical cells
E. Accessory cells

42. Ultramicroscopical examination of "dark" hepatocyte population in the cell


cytoplasm detected a developed granular endoplasmic reticulum. What
function has this organella in these cells?
A. Synthesis of blood plasma proteins
B. Carbohydrate synthesis
C. Deintoxicative function
D. Bile production
E. Calcium ion depositing

43. An endocrinal gland with parenchyma consisting of epithelium and neural


tissue is under morphological examination. Epithelial trabecules have two types
of cells: chromophilic and chromophobic. Identify this organ:
A. Hypophysis
B. Adrenal glands
C. Hypothalamus
D. Thyroid gland
E. Parathyroid gland

44. A histological specimen presents an artery. One of the membranes of its wall
has flat cells lying on the basal membrane. What type of cells is it?
A. Endothelium
B. Mesothelium
C. Smooth myocytes
D. Fibroblasts
E. Macrophages

45. One of sections of central nervous system has layerwise arrangement of


neurocytes. Among them there are cells of the following forms: stellate,
fusiform, horizontal, pyramidal. What section of central nervous system is this
structure typical for?
A. Cortex of cerebrum
B. Spinal cord
C. Cerebellum
D. Medulla oblongata
E. Hypothalamus

46. As a result of an injury, the integrity of the anterior spinal cord root was broken.
Specify the neurons and their processes that had been damaged:
A. Axons of motor neurons
B. Motor neuron dendrites
C. Axons of sensory neurons
D. Dendrites of sensory neurons
E. Dendrites of association neurons

47. Study of fingerprints (dactylography) is used by criminalists for personal


identification as well as for diagnostics of genetic abnormalities, particularly
Dawn's disease. What layer of skin determines individuality of fingerprints?
A. Dermopapillary
B. Horny
C. Reticular
D. Clear (stratum lucidum epidermidis)
E. Basal

48. A microspecimen of the submandibular salivary gland shows some basket-


shaped cells concentrated around the acines and excretory ducts. These cells
surround bases of the serous cells and are called myoepitheliocytes. These cells
relate to the following tissue:
A. Muscular tissue
B. Epithelial tissue
C. Neural tissue
D. Special connective tissue
E. Loose fibrous connective tissue

49. An infectious disease caused contractive activity of muscles that contract and
dilate eye pupil (paralytic state). What functional eye system was damaged?
A. Accomodative
B. Dioptric
C. Ancillary
D. Photosensory
E. Lacrimal apparatus

50. An electron micrograph shows a cell-to-cell adhesion consisting, in each cell, of


an attachment plaque. The intercellular space is filled with electron-dense
substance including transmembrane fibrillar structures. Specify this adhesion:
A. Desmosome
B. Synapse
C. Tight junction
D. Nexus
E. Adherens junction
Krok 1 – 2014 Microbiology Base
1. Quite often the cause of secondary immunodeficiency is an infection
involvement, when the causative agents propagate directly in the cells of
immune system and destroy it. The following diseases are characterized by:
A. Infectious mononucleosis, AIDS
B. Tuberculosis, mycobacteriosis
C. Poliomyelitis, type A hepatitis
D. Dysentery, cholera
E. Q-febris, epidemic typhus

2. From the nasopharynx of a 5-year-old child it was excreted amicroorganism


which is identical to Corynebacterium diphtheriae dose according to
morphological and biochemical signs.Microorganism does not produce
exotoxin. As a result of what process can this microorganism become
toxigenic?
A. Phage conversion
B. Cultivation in the telluric environment
C. Passing through the organism of the sensative animals
D. Growing with antitoxic serum
E. Chromosome mutation

3. While studying a microslide obtained from the punctuate of a regional lymph


node and stained by Romanovsky -Giemsa method a physician revealed some
light-pink thin microorganisms with 12-14 regular spiral coils and pointed
ends, up to 10-13 micrometer long. This might be the causative agent of the
following disease:
A. Syphilis
B. Trypanosomiasis
C. Leptospirosis
D. Relapsing fever
E. Leishmaniasis

4. Sanitary bacteriological research on water by the membrane filter method


revealed two red colonies on a membrane filter (Endo agar) through which 500
ml of analyzed water were passed. Calculate the coli index and coli titer of the
analyzed water:
A. 4 and 250
B. 2 and 500
C. 250 and 4
D. 500 and 2
E. 250 and 2

5. While examining a patient an otolaryngologist noticed hyperaemia and


significantly edematous tonsils with a grayish film upon them. Microscopical
examination of this film revealed some gram-positive bacilli placed at an angle
with each other. What disease might be suspected?
A. Diphtheria
B. Angina
C. Scarlet fever
D. Meningococcal nasopharyngitis
E. Epidemic parotitis

6. Patient with vomiting, dizziness, and sensation of dubble vision, difficult


swallowing was admitted to the hospital. Doctor suspects botulism. What
diagnostic methods should be used for diagnosis approving?
A. Biological test, bacteriological
B. Allergic test, serological
C. Bacteriological, mycological
D. Protozoological, microscopical
E. -

7. The infectious diseases department of a hospital admitted a patient with


nausea, liquid stool with mucus and blood streaks, fever, and weakness.
Dysentery was suspected. What method of laboratory diagnostics should be
applied to confirm the diagnosis?
A. Bacteriological
B. Serological
C. Mycological
D. Microscopic
E. Protozoological

8. A man who was bitten by the unknown dog applied to the surgeon. Wide
ragged woundes were localised on the face. What curative-prophylactic aid
should be given to prevent rabies?
A. Start immunisation with rabies vaccine
B. Prescribe combine antibiotic therapy
C. Immediate injection of DPT (Diphtheria, Pertusis, Tetanus) vaccine
D. Hospitalize the patient and keep under the doctor's supervision
E. Immediately inject normal gamma globulin

9. In a patient with clinical signs of immunodeficiency the number and functional


activity of T and B lymphocytes are not changed. Defect with dysfunction of
antigen-presentation to the immunocompetent cells was found during
investigation on the molecule level. Defect of what cells is the most probable?
A. Macrophages, monocytes
B. Т-lymphocytes, В-lymphocytes
C. NK-cells
D. Fibroblasts, Т-lymphocytes, В-lymphocytes
E. 0-lymphocytes

10. On bacteriological study of rinsing water of the patient with food poisoning,
the pure bacterial culture was inoculated with the following properties: gram-
negative motile bacillus in the Endo environment grows like achromic colony .
Representative of what genus has caused the illness?
A. Salmonella
B. Shigella
C. Yersinia
D. Escherichia
E. Citrobacter

11. The person was selling "homemade pork" sausages on the market. State
sanitary inspector suspected falcification of the sausages.With help of what
serological immune reaction can food substance be identified?
A. Precipitation test
B. Indirect hemagglutination test
C. Agglutination test
D. Immunofluorescence test
E. Complement- fixation test

12. A 6-year-old child with suspected active tuberculous process underwent the
diagnostic Mantoux test. What immunobiological preparation was injected?
A. Tuberculin
B. BCG vaccine
C. DTP vaccine
D. Tularinum
E. Td vaccine

13. While registering the child to the school Mantu's test was made to define
whether revaccination was needed test result is negative. What does this result
of the test mean?
A. Absence of cell immunity to the tuberculosis
B. Presence of cell immunity to the tuberculosis
C. Absence of antibodies for tubercle bacillus
D. Absence of antitoxic immunity to the tuberculosis
E. Presence of antibodies for tubercle bacillus

14. The donor who for a long time didn't donate the blood was investigated with
IFA method. Anti-HBs antibodies were revealed. What does positive result of
IFA in this case mean?
A. Previous hepatitis B
B. Acute hepatitis B
C. Acute hepatitis C
D. Chronic hepatitis В
E. Chronic hepatitis С

15. Bacteriological examination of purulent discharges from the urethra revealed


gram-negative bacteria looking like coffee beans. They were localized in the
leukocytes and could decompose glucose and maltose to acid. These are the
causative agents of the following disease:
A. Gonorrhoea
B. Syphilis
C. Veneral lymphogranulomatosis
D. Soft chancre
E. Melioidosis

16. Scraps of the mycelium of a fungus, spores, air bubbles and fat drops were
discovered on microscopy of the patient's hair excluded from the infected
areas. For what fungus disease is this microscopic picture characteristic?
A. Favus
B. Microspory
C. Trichophytosis
D. Epidermophytosis
E. Sporotrichosis

17. In order to speed up healing of a wound of oral mucosa a patient was


prescribed a drug that is a thermostable protein occuring in tears, saliva, and
mother’s milk as well as in a new-laid hen's egg. It is known that this protein is
a factor of natural resistance of an organism. What is it called?
A. Lysozyme
B. Complement
C. Interferon
D. Interleukin
E. Imanine

18. Study of bacteriological sputum specimens stained by the Ziel-Neelsen method


revealed some bright-red acid-resistant bacilli that were found in groups or
singularly. When inoculated onto the nutrient media, the signs of their growth
show up on the 10-15 day. These bacteria relate to the following family:
A. Micobacterium tuberculosis
B. Yersinia pseudotuberculosis
C. Histoplasma dubrosii
D. Klebsiella rhinoscleromatis
E. Coxiella burnettii

19. A man was admitted to the hospital on the 5th day of disease that manifested
itself by jaundice, muscle aching, chill, nose bleedings. In course of laboratory
diagnostics a bacteriologist performed dark-field microscopy of the patient's
blood drop. Name a causative agent of this disease:
A. Leptospira interrogans
B. Borrelia dutlonii
C. Calymmatobacterium granulomatis
D. Bartonella bacilloformis
E. Rickettsia mooseri
20. Gramnegative bin-shaped diplococcus inside and outside of leucocytes were
detected on bacteriological examination of the purulent exudates from the
cervix of the uterus. Name the causative agent of purulent inflammation of the
cervix of the uterus.
A. Neisseria gonorroeae
B. Chlamidia trachomatis
C. Haemophilus vaginalis
D. Trichomonas vaginalis
E. Calymmatobacterium granulomatis

21. Patient with diarrhoea was admitted to the infection unit. Gramnegative
curved rod-like bacteria were founded on bacterioscopic examination of faecal
masses. What is the most likely disease in this patient?
A. Cholera
B. Typhoid fever
C. Salmonellosis gastroenteritis
D. Diphtheria
E. Intestinal form of plague

22. In a 2-year-old child with catarrhal presentations and skin rash a pediatrician
suspected scarlet fever. The child was given intracutaneously a small dose of
serum antibody to the streptococcal erythrogenic toxin; on the site of injection
the rash disappeared. What do the reaction results mean?
A. The clinical diagnosis was confirmed
B. The child has hypersensitivity to the erythrogenic toxin
C. The disease wasn't caused by haemolytic streptococcus
D. The whole serum dose may be injected intravenously
E. The child has very weak immune system

23. A man died from an acute infectious disease accompanied by fever, jaundice,
haemorrhagic rash on the skin and mucous membranes as well as by acute
renal insufficiency. Histological examination of renal tissue (stained by
Romanovsky-Giemsa method) revealed some convoluted bacteria looking like
C und S letters. What bacteria were revealed?
A. Leptospira
B. Treponema
C. Spirilla
D. Borrelia
E. Campilobacteria

24. Bacteriological examination of the urine of the patient with pyelonephritis


revealed microorganisms that produced yellow-green pigment and a
characteristic odor in meat-peptone agar. What are they called?
A. Pseudomonas
B. Escherichia
C. Proteas
D. Klebsiella
E. Azotobacter

25. A 16 y.o. boy from countryside entered an educational establishment.


Scheduled Manteux test revealed that the boy had negative reaction. What are
the most reasonable actions in this case?
A. To perform BCG vaccination
B. To repeat the reaction in a month
C. To perform serodiagnostics of tuberculosis
D. To isolate the boy temporarily from his mates
E. To perform rapid Price diagnostics

26. Examination of a patient with pustular skin lesions allowed to isolate a


causative agent that forms in the blood agar roundish yellow middle-sized
colonies surrounded by haemolysis zone. Smears from the colonies contain
irregular-shaped clusters of gram-positive cocci. The culture is oxidase- and
catalase-positive, ferments mannitol and synthesizes plasmocoagulase. What
causative agent was isolated?
A. Staphylococcus aureus
B. Streptococcus agalactiae
C. Streptococcus pyogenes
D. Staphylococcus epidermidis
E. Staphylococcus saprophyticus

27. Microscopic examination of a Gram-stained scrape from patient's tongue


revealed oval, round, elongated chains of dark-violet gemmating cells. What
disease can be caused by this causative agent?
A. Candidosis
B. Actinomycosis
C. Streptococcic infection
D. Staphylococcic infection
E. Diphtheria

28. From pharynx of a child with suspected diphtheria a pure culture of


microorganisms was isolated. Their morphological, tinctorial, cultural and
biochemical properties appeared to be typical for diphtheria causative agents.
What study should be conducted in order to drow a conclusion that this is a
pathogenic diphtheria bacillus?
A. Estimation of toxigenic properties
B. Estimation of proteolytic properties
C. Estimation of urease activity
D. Estimation of cystinous activity
E. Estimation of ability to decompose starch

29. Examination of a child revealed some whitish spots looking like coagulated
milk on the mucous membrane of his cheeks and tongue. Analysis of smears
revealed gram-positive oval yeast-like cells. What causative agents are they?
A. Candida
B. Staphylococci
C. Diphtheria bacillus
D. Actinomycetes
E. Fusobacteria

30. A duodenal content smear of a patient with indigestion contains protosoa 10-
18 mcm large. They have piriform bodies, 4 pairs of filaments, two
symmetrically located nuclei in the broadened part of body. What kind of the
lowest organisms is it?
A. Lamblia
B. Dysentery ameba
C. Trichomonas
D. Intestinal ameba
E. Balantidium

31. Blood of a patient with presumable sepsis was inoculated into sugar broth.
There appeared bottom sediment. Repeated inoculation into blood agar
caused growth of small transparent round colonies surrounded by hemolysis
zone. Examination of a smear from the sediment revealed gram-positive cocci
in form of long chains. What microorganisms are present in blood of this
patient?
A. Streptococci
B. Micrococci
C. Staphylococci
D. Tetracocci
E. Sarcina

32. Bacterioscopy of nasopharyngeal mucus taken from a 2,5-year-old child with


nasopharyngitis revealed gram-positive diplococci looking like coffee grains.
What organs of the child are most likely to be affected if these microorganisms
penetrate the blood?
A. Brain tunics
B. Cardiac valves
C. Renal glomeruli
D. Urogenital tracts
E. Lymph nodes

33. Analysis of sputum taken from a patient with suspected pneumonia revealed
rather elongated gram-positive diplococci with somewhat pointed opposite
ends. What microorganisms were revealed in the sputum?
A. Streptococcus pneumoniae
B. Staphylococcus aureus
C. Klebsiella pneumoniae
D. Neisseria meningitidis
E. Neisseria gonorrhoeae
34. Serological diagnostics of infectious diseases is based upon specific interaction
with antigenes. Specify the serological reaction that underlies adhesion of
microorganisms when they are affected by specific antibodies in presence of
an electrolyte:
A. Agglutination reaction
B. Precipitation reaction
C. Complement-binding reaction
D. Hemadsorption reaction
E. Neutralization reaction

35. The immunoblot detected gp120 protein in the blood serum. This protein is
typical for the following disease:
A. HIV-infection
B. Virus B hepatitis
C. Tuberculosis
D. Syphilis
E. Poliomyelitis

36. HIV has gp41 and gp120 on its surface interacts with target cells of an
organism. Which of the following human lymphocyte antigens is gp120
complementary bound with?
A. CD 4
B. CD 3
C. CD 8
D. CD 19
E. CD 28

37. On bacteriological examination of the defecation of a 4-months-old baby with


the symptoms of acute bowel infection there were revealed red colonies spread
in the large quantity in the Endo environment. What microorganism can it be?
A. Escherichia
B. Salmonella
C. Staphylococcus
D. Streptococcus
E. Shigell

38. The contents of vesicles that appeared on the mucous membrane of a patient
with variola was sent to a virological laboratory. Which of the listed changes
were revealed during the smear microscopy?
A. Paschen bodies
B. Babes-Negri bodies
C. Guarnieri bodies
D. Babes-Ernst bodies
E. Syncytium
39. Bacteriological examination of a patient with food poisoning required
inoculation of a pure culture of bacteria with the following properties: gram-
negative movable bacillus that grows in the Endo's medium in form of
colourless colonies. A representative of which species caused this disease?
A. Salmonella
B. Shigella
C. Iersinia
D. Esherichia
E. Citrobacter

40. Examination of a young man in the AIDS centre produced a positive result of
immune-enzyme assay with HIV antigens. Patient's complaints about state of
his health were absent. What can the positive result of immune-enzyme assay
be evidence of?
A. HIV infection
B. Being ill with AIDS
C. Being infected with HBV
D. Having had AIDS recently
E. HBV persistence

41. Microscopy of stained (Ziehl-Neelsen staining) smears taken from the sputum
of a patient with chronic pulmonary disease revealed red bacilli. What property
of tuberculous bacillus was shown up?
A. Acid resistance
B. Alkali resistance
C. Alcohol resistance
D. Capsule formation
E. Sporification

42. Reaction of passive hemagglutination conducted with erythrocytic typhoid Vi-


diagnosticum helped to reveal some antibodies in the dilution of the patient's
serum at a ratio of 1:80 that exceeds the diagnostic titer. Such result witnesses
of:
A. Being a potential carrier of typhoid bacilli
B. Being ill with acute typhoid fever
C. Typhoid fever recurrence
D. Incubation period of typhoid fever
E. Reconvalescence of a patient ill with typhoid fever

43. In order to determine toxigenicity of diphtheria bacilli a strip of filter paper


impregnated with antitoxic diphtherial serum was put on the dense nutrient
medium. There were also inoculated a microbal culture under examination
and a strain that is known to be toxigenic. If the microbal culture under
examination produces exotoxin, this wil result in formation of:
A. Precipitin lines
B. Haemolysis zones
C. Zones of diffuse opacification
D. Zones of lecithovitellinous activity
E. Precipitin ring

44. A patient underwent esophagogastroduodenoscopy. Analysis of the biopsy


material enabled doctors to diagnose him with helicobacteriosis. What
property of the bacteria found in this patient had to be obligatory taken into
account during their cultivation?
A. Microaerophilic ability
B. Presence of urease
C. Colonisation of gastral cells
D. Absence of spores and capsules
E. Presence of six polar flagella

45. In order to estimate toxigenity of diphtheria agents obtained from patients the
cultures were inoculated on Petri dish with nutrient agar on either side of a
filter paper strip that was put into the centre and moistened with
antidiphtheric antitoxic serum. After incubation of inoculations in agar the
strip-like areas of medium turbidity were found between separate cultures and
the strip of filter paper. What immunological reaction was conducted?
A. Precipitation gel reaction
B. Coomb's test
C. Agglutination reaction
D. Rings precipitation reaction
E. Opsonization reaction

46. A patient with clinical signs of encephalitis was delivered to the infectious
diseases hospital. Anamnesis registers a tick bite. Hemagglutination -
inhibition reaction helped to reveal antibodies to the causative agent of tick -
borne encephalitis in the dilution 1:20 which is not diagnostic. What actions
should the doctor take after he had got such result?
A. To repeat the examination with serum taken 10 days later
B. To examine the same serum
C. To apply more sensitive reaction
D. To repeat examination with another diagnosticum
E. To deny diagnosis of tick-borne encephalitis

47. A smear from the tonsillar coating of a patient with suspected diphtheria was
found to contain blue bacilli with a thickening at the poles. What method of
smear staining was used?
A. Leffler
B. Burri
C. Hins
D. Gram
E. Neisser
48. The first grade pupils were examined in order to sort out children for
tuberculosis revaccination. What test was applied for this purpose?
A. Mantoux test
B. Schick test
C. Supracutaneous tularin test
D. Burnet test
E. Anthraxine test

49. Clinical diagnosis of a female patient was gonorrhoea. What examination


method can be applied for confirmation of this diagnosis?
A. Microscopy of pathological material
B. Infection of laboratory animals
C. Test with bacteriophage
D. Hemagglutination reaction
E. Immobilization reaction

50. A patient suffering from periodical attacks caused by inhalation of different


flavoring substances was diagnosed with atopic bronchial asthma. IgE level
was increased. This is typical for the following type of reactions:
A. Anaphylactic reactions
B. Cytotoxic reactions
C. Immunocomplex reactions
D. Delayed-type hypersensitivity
E. Autoimmune reactions

51. A patient has been hospitalised with provisional diagnosis of virus B hepatitis.
Serological reaction based on complementation of antigen with antibody
chemically bound to peroxidase or alkaline phosphatase has been used for
disease diagnostics. What is the name of the applied serological reaction?
A. Immune-enzyme analysis
B. Radioimmunoassay technique
C. Immunofluorescence test
D. Bordet-Gengou test
E. Antigen-binding assay

52. Bacteriological laboratory examines canned meat whether it contains


botulinum toxin. For this purpose, an extract of test specimen and antitoxic
antibotulinic serum of A, B, E types were introducted to a group of mice under
examination; a control group of mice got the extract without antibotulinic
serum. What serological reaction was applied?
A. Neutralization
B. Precipitation
C. Complement binding
D. Opsono-phagocytic
E. Double immune diffusion
53. For the purpose of retrtospective diagnostics of recent bacterial dysentery it
was decided to perform serological examination of blood serum in order to
determine antibody titer towards Shiga bacilli. What of the following reactions
should be applied?
A. Passive hemagglutination
B. Bordet-Gengou test
C. Precipitation
D. Hemolysis
E. Bacteriolysis

54. During the repeated Widal's agglutination test it was noticed that the ratio of
antibody titers and O-antigens \emph{S.typhi} in the patient's serum had
increased from 1:100 to 1:400. How would you interpret these results?
A. The patient has typhoid fever
B. The patient is an acute carrier of typhoid microbes
C. The patient is a chronic carrier of typhoid microbs
D. The patient previously had typhoid fever
E. The patient was previously vaccinated against typhoid fever

55. A patient recovered from Sonne dysentery and was once more infected with
the same causative agent. What is such infection form called?
A. Reinfection
B. Recidivation
C. Superinfection
D. Persisting infection
E. Chronic infection

56. A 10-year-old child had the mantoux tuberculin test administered. 48 hours
later a papule up to 8 mm in diameter appeared on the site of the injection.
What type of hypersensitivity reaction developed after the tuberculin
injection?
A. Type IV hypersensitivity reaction
B. Arthus phenomenon
C. Seroreaction
D. Atopic reaction
E. Type II hypersensitivity reaction

57. A patient with clinical presentations of immunodeficiency went through


immunological examinations. They revealed significant loss of cells that form
rosettes with erythrocytes of a ram. What conclusion can be made according
to the analysis data?
A. Decrease of T-lymphocytes rate
B. Decrease of B-lymphocytes rate
C. Decrease of natural killer cell rate
D. Decrease of complement system rate
E. Insufficiency of effector cells of humoral immunity
58. As a result of durative antibiotic therapy a 37-year old patient developed
intestinal dysbacteriosis. What type of drugs should be used in order to
normalize intestinal microflora?
A. Eubiotics
B. Sulfanilamides
C. Bacteriophages
D. Autovaccines
E. Vitamins

59. Among junior children of an orphanage an outbreak of intestinal infection with


signs of colienteritis was registered. In order to identify isolated causative
agent, it is necessary to:
A. Study antigenic properties of the causative agent
B. To determine sensitivity to antibiotics
C. To study sensitivity to bacteriophages
D. To study biochemical properties of the causative agent
E. To study virulence of the causative agent

60. Urine examination of a patient with acute cystitis revealed leukocytes and a lot
of gram-negative bacilli. Inoculation resulted in growth of colonies of mucous
nature that formed green soluble pigment. What microorganism is the most
probable cause of the disease?
A. Pseudomonas aeruginosa
B. Escherihia coli
C. Klebsiella pneumoniae
D. Proteus mirabilis
E. Salmonella enteritidis

61. A laboratory received a material from a patient's wound. Ppreliminary


diagnosis is gaseous gangrene. What microbiological method should be
applied to determine species of causative agent?
A. Bacteriological
B. Allergic
C. Bacterioscopic
D. Serological
E. RIA

62. A virological laboratory obtained pathological material (mucous discharges


from nasal meatuses) taken from a patient with provisional diagnosis
"influenza". What quick test will allow to reveal specific viral antigen in the
material under examination?
A. Direct and indirect immunofluorescence test
B. Direct and indirect fluorescence immunoassay
C. Hemagglutination inhibition assay
D. Radioimmunoassay
E. -

63. In the surgical department of a hospital there was an outbreak of hospital


infection that showed itself in often postoperative wound abscesses.
Bacteriological examination of pus revealed aurococcus. What examination
shall be conducted to find out the source of this causative agent among the
department personnel?
A. Phagotyping
B. Microscopical examination
C. Serological identification
D. Estimation of antibiotic susceptibility
E. Biochemical identification

64. A 7-year-old child often suffers from streprococcic angina. Doctor suspected
development of rheumatism and administered serological examination. The
provisional diagnosis will be most probably confirmed by presence of
antibodies to the following streptococcic antigen:
A. O-streptolysin
B. C-carbohydrate
C. M-protein
D. Erythrogenic toxin
E. Capsular polysaccharide

65. A culture of monkey cells (Vero) and a group of mouse sucklings were infected
with an inoculums taken from a child with provisional diagnosis "enterovirus
infection". There was no cytopathic effect on the cell culture but mouse
sucklings died. What enteric viruses might have caused disease of this child?
A. Coxsackie A
B. Coxsackie B
C. ECHO virus
D. Polioviruses
E. Unclassified enteric viruses 68-71

66. A patient has been suffering from elevated temperature and attacks of typical
cough for 10 days. Doctor administered inoculation of mucus from the
patient's nasopharynx on the agar. What microorganism is presumed?
A. Pertussis bacillus
B. Pfeiffer's bacillus
C. Listeria
D. Klebsiella
E. Staphylococcus

67. A patient of surgical department complains about pain in the small of her back
and in the lower part of her belly; painful and frequent urination.
Bacteriological examination of urine revealed gram-negative oxidase-positive
rod-like bacteria forming greenish mucoid colonies with specific smell. What
causative agent can it be?
A. Pseudomonas aeruginosa
B. Proteus mirabilis
C. E. coli
D. Str.pyogenes
E. Mycoplasma pneumonie

68. There was a record of some anthrax cases among animals in a countryside. The
spread of disease can be prevented by means of immunization. What kind of
vaccine should be used?
A. STI live vaccine
B. BCG vaccine
C. Salk vaccine
D. Sabin's vaccine
E. Diphteria and tetanus toxoids and pertussis vaccine

69. A female patient underwent liver transplantation. 1,5 months after it her
condition became worse because of reaction of transplant rejection. What
factor of immune system plays the leading part in this reaction?
A. T-killers
B. Interleukin-1
C. Natural killers
D. B-lymphocytes
E. T-helpers

70. Microscopical examination of a microbal culture revealed fusiform spore-


forming microorganisms that get violet-blue Gram's stain. What
microorganisms were revealed?
A. Clostridia
B. Streptococci
C. Spirochaete
D. Actinomycete
E. Diplococci

71. A specimen stained by Ozheshko method contains rod-like microorganisms


stained blue with round terminal components stained red. What are these
components called?
A. Spores
B. Cilia
C. Flagella
D. Capsules
E. Mesosomas

72. During the regular sanitary-epidemiological inspection of a pharmacy, the


bacteriological analysis of air was performed. The air was found to have bacilli,
yeast fungi, hemolytic streptococci, micrococci. Which of the detected
microorganisms indicate the direct epidemic danger?
A. Haemolytic streptococci
B. Micrococci
C. Bacilli
D. Yeast fungi
E. -

73. A bacteriological laboratory received sputum sample of a patient suffering


from tuberculosis. Bacterioscopic examination of smears and detection of
tuberculosis bacillus can be realized by one of enrichment methods that
involves processing of sputum only with solution of caustic soda. What is this
method called?
A. Homogenization
B. Inactivation
C. Flotation
D. Filtration
E. Neutralization

74. A pregnant woman was registered in an antenatal clinic and underwent


complex examination for a number of infections. Blood serum contained IgM
to the rubella virus. What is this result indicative of?
A. Of primary infection
B. Of a chronic process
C. The woman is healthy
D. Of exacerbation of a chronic disease
E. Of recurring infection with rubella virus

75. A 65-year-old man has purulent abscess on his neck. Analyses revealed a
culture of gram-positive cocci with plasmocoagulase activity. This culture
relates most likely to:
A. Staphylococcus aureus
B. Streptococcus pyogenes
C. Staphylococcus epidermidis
D. Staphylococcus saprophyticus

76. Material taken from a patient with provisional diagnosis "influenza" was
referred to a laboratory. For virological examination the hemadsorption
reaction was applied. This reaction can be applied for detection of the follow ing
viruses:
A. Viruses containing hemagglutinins
B. All the simple viruses
C. All the complex viruses
D. DNA-genomic viruses
E. Any viruses

77. Inoculum from pharynx of a patient ill with angina was inoculated into blood-
tellurite agar. It resulted in growth of grey, radially striated (in form of
rosettes) colonies 4-5 mm in diameter. Gram-positive bacilli with clublike
thickenings on their ends placed in form of spread wide apart fingers are
visible by microscope. What microorganisms are these?
A. Diphtheria corynebacteria
B. Botulism clostridia
C. Diphtheroids
D. Streptococci
E. Streptobacilli

78. During examination of a patient a dentist revealed a lot of "white spots" - zones
of enamel demineralization. What microorganisms take part in the
development of this process?
A. Streptococcus mutans
B. Streptococcus salivarius
C. Streptococcus pyogenes
D. Veilonella parvula
E. Staphylococcus epidermidis

79. Planned mass vaccination of all newborn 5-7 day old children against
tuberulosis plays an important role in tuberculosis prevention. In this case the
following vaccine is applied:
A. BCG
B. Diphteria and tetanus toxoids and pertussis vaccine
C. Diphtheria and tetanus anatoxin vaccine
D. Adsorbed diphtheria vaccine

80.Stool culture test revealed in a 6-month-old bottle-fed baby the strain of


intestinal rod-shaped bacteria of antigen structure 0-111. What diagnosis can
be made?
A. Colienteritis
B. Gastroenteritis
C. Choleriform disease
D. Food poisoning
E. Dysentery-like disease
Krok 1 – 2014 Path Anatomy Base
1. Multiple oval ulcers along the intestine were revealed on autopsy of the person,
who died from diffuse of peritonitis in the distant part of the small intestine.
Bottom parts of the ulcers are clear, smooth, formed with muscular or serous
covering, edges of ulcers are flat, rounded. There are perforations up to 0,5 cm
in diameter in two ulcers. What diseases can be diagnosed?
A. Typhoid fever
B. Dysentery
C. Cholera
D. Tuberculosis
E. Typhus

2. Examination of a patient revealed a dense, movable skin tumor that is standing


out distinctly from the surrounding tissues. Its section is found to be white and
composed of fibrous tissue. Microscopic examination revealed interlacing
collagen fibers and few cells. What tumor is it?
A. Fibroma
B. Myoma
C. Histiocytoma
D. Dermatofibroma
E. Desmoid

3. A 46-year-old man complains of difficult nose breathing. Mikulich cells,


storage of epithelioid cells, plasmocytes, lymphocytes, hyaline balls are
discovered in the biopsy material of the nose thickening. What is the most
likely diagnosis?
A. Scleroma
B. Virus rhinitis
C. Allergic rhinitis
D. Rhinovirus infection
E. Meningococcal nasopharyngitis

4. Extensive thromboembolic infarction of the left cerebral hemispheres, large


septic spleen, immunocomplex glomerulonephritis, ulcers on the edges of the
aortic valves, covered with polypous thrombus with colonies of staphylococcus
were revealed on autopsy of the young man who died in coma. What disease
caused cerebral thromboemboly?
A. Septic bacterial endocarditis
B. Septicemia
C. Acute rheumatic valvulitis
D. Septicopyemia
E. Rheumatic thromboendocarditis

5. A patient ill with diabetes mellitus felt acute pain in his right foot. Objectively:
foot thumb is black, foot tissues are edematous, there are foci of epidermis
desquamation, stinking discharges. What clinicopathological form of necrosis
is it?
A. Moist gangrene
B. Bedsore
C. Sequestrum
D. Dry gangrene
E. Infarction

6. On autopsy it is revealed that kidneys are enlarged, surface is large-granular


because of multiple cavities with smooth wall, which are filled with clear fluid.
What kidney disease did the patient have?
A. Polycystic kidney
B. Necrotic nephrosis
C. Pyelonephritis
D. Glomerulonephritis
E. Infarction

7. On autopsy it is revealed enlarged dense right lung, fibrin layers on the pleura.
Lung tissue is light green color on incision with muddy liqued exudates. What
lung disease are these symptoms typical for?
A. Lung-fever
B. Bronchopneumonia
C. Interstitial pneumonia
D. Pulmonary gangreneі
E. Fibrosing alveolitis

8. On autopsy it is revealed: soft arachnoid membrane of the upper parts of


cerebral hemisphere is plethoric, it is of yellowish-green color, soaked with
purulent and fibrose exudate, it lookes like cap. For what disease is it
characteristical picture
A. Meningococcal meningitis
B. Tuberculous meningitis
C. Influenza meningitis
D. Meningitis at anthrax
E. Meningitis at typhus

9. A patient has hoarseness of voice. During laryngoscopy a gray-white larynx


tumor with papillary surface has been detected. Microscopic investigation has
shown the following: growth of connective tissue covered with multilayer,
strongly keratinized pavement epithelium, no cellular atypia. What is the most
likely diagnosis?
A. Papilloma
B. Fibroma
C. Polyp
D. Angioma
E. Angiofibroma
10. During autopsy approximately 2,0 liters of pus have been found in the
abdominal cavity of the corpse. Peritoneum is lustreless and has grayish shade,
serous tunic of intestines has grayish-colored coating that is easily removable.
Specify the most likely type of peritonitis in the patient:
A. Fibrinopurulent peritonitis
B. Hemorrhagic peritonitis
C. Serous peritonitis
D. Tuberculous peritonitis

11. For a long time, a 49-year-old woman was suffering from glomerulonephritis
which caused death. On autopsy it was revealed that kidneys size was 7х3х2.5
sm, weight is 65,0 g, they are dence and small-grained. Microscopically :
fibrinogenous inflammation of serous and mucous capsules, dystrophic
changes of parenchymatous organs, brain edema. What complication can
cause such changes of serous capsules and inner organs?
A. Uraemia
B. Anemia
C. Sepsis
D. DIC-syndrome
E. Thrombopenia

12. Autopsy of the dead patient who died from pulmonary edema revealed a large
yellow-grey nidus in the myocardium, and a fresh thrombus in the coronary
artery. What is the most likely diagnosis?
A. Myocardial infarction
B. Cardiosclerosis
C. Myocarditis
D. Amyloidosis
E. Cardiomyopathy

13. A patient died under conditions of cardiovascular insufficiency. Autopsy


results: postinfarction cardiosclerosis, myocardium hypertrophy and
dilatation of its cavities, especially of its right ventricle. Liver is enlarged, its
surface is smooth, and incision revealed that it was plethoric, with dark-red
specks against the background of brownish tissue. Histologically: plethora of
central parts of lobules; peritheral parts around portal tracts contain
hepatocytes in a state of adipose degeneration. How are these liver changes
called?
A. Nutmeg liver
B. Pseudonutmeg liver
C. Amyloidosis
D. Liver cirrhosis
E. Liver steatosis

14. A 59-year-old man has signs of the parenchymatous jaundice and portal
hypertension. On histological examination of the puncture of the liver
bioptate, it was revealed: beam-lobule structure is affected, part of hepatocytes
has signs of fat dystrophy, port-portal connective tissue septa with formation
of pseudo-lobules, with periportal lympho-macrophage infiltrations. What is
the most probable diagnosis?
A. Liver cirrhosis
B. Alcohol hepatitis
C. Chronic hepatosis
D. Viral hepatitis
E. Toxic dystrophy

15. On microscopic examination of the enlarged neck gland of a 14-year-old girl it


was revealed destruction of the tissue structure of the node, absence of the
lymph follicles, sclerotic and necrosis parts, cell constitution of the node is
polymorphous, lymphocites, eosinophiles, atypical cells of the large size with
multiple-lobule nuclei (Beresovsky-Shternberg cells) and onenucleus cells of
the large size are present. What is the most likely diagnosis?
A. Lymphogranulomatous
B. Acute lympholeucosis
C. Chronic lympholeucosis
D. Berkitt's lymphoma
E. Fungous mycosis

16. A female patient suffering from bronchial asthma had got a viral infection that
provoked status asthmaticus with fatal outcome. Histological examination of
lungs revealed spasm and edema of bronchioles, apparent infiltration of their
walls with lymphocytes, eosinophils and other leukocytes; labrocyte
degranulation. What mechanism of hypersensitivity underlies the described
alterations?
A. Reagin reaction
B. Inflammatory
C. Autoimmune
D. Immune complex
E. Immune cytolysis

17. Local lymphonodules enlarged near the infected wound. Increased amount of
macrophages, lymphocytes, lymphatic follicles in the cortical layer and large
amount of plasma cells were revealed on histological examination. What
process in the lymphatic nodules represents these histological changes?
A. Antigen stimulation
B. Acquired insufficiency of the lymphoid tissue
C. Innate insufficiency of the lymphoid tissue
D. Tumour transformation
E. Hypersensibility reaction

18. On autopsy of the man with alcohol abuse for a long time it was revealed:
dense, small-knobby, small size liver. Microscopically: small pseudo-lobules,
divided with thin layers of connective tissue with lymphomacrophagial
infiltrates; hepatocytes in the state of globular fatty dystrophy. What is the
most likely diagnosis?
A. Alcohol cirrhosis
B. Chronic active alcohol hepatitis
C. Chronic persistent alcohol hepatitis
D. Toxic liver dystrophy
E. Fatty hepatosis

19. An 8-year-old child was admitted to the infectious department with fever (up
to 38oC) and punctuate bright-red skin rash. The child was diagnosed as
having scarlet fever. Objectively: mucous membrane of pharynx is apparently
hyperaemic and edematic, the tonsils are enlarged and have dull yellowish-
grey foci with some black areas. What inflammation is the reason for the
pharynx alterations?
A. Purulent necrotic
B. Fibrinous
C. Haemorrhagic
D. Serous
E. Catarrhal

20. A patient who has been abusing tobacco smoking for a long time has got cough
accompanied by excretion of viscous mucus; weakness after minor physical
stress, pale skin. The patient has also lost 12,0 kg of body weight. Endoscopic
examination of biosy material his illness was diagnosed as squamous cell
carcinoma. Name a pathological process that preceded formation of the
tumour:
A. Metaplasia
B. Hypoplasia
C. Hyperplasia
D. Necrosis
E. Sclerosis

21. Diagnostic scraping was performed to the woman with dysfunctional uterine
bleeding. Multiple convoluted glands, ganglially dilated cavities of some glands
were revealed histologically in the scrape. Name the type of general
pathological process.
A. Glandulo-gangliac hyperplasia
B. Atrophy
C. Metaplasia
D. Displasia
E. Hypertrophic excrescence

22. Tuberculine was injected intracutaneously to the child for tuberculin test.
Marked hyperemia, tissue infiltration developed on the place of injection in 24
hours. What mechanism caused these modifications?
A. Cells cytotoxity
B. Reagin type cytotoxity
C. Antibody cytotoxity
D. Granuloma formation
E. Immunocomplex cytotoxity

23. The intraoperational biopsy of mammal gland has revealed the signs of
atypical tissue with disorder of parenchyma stroma proportion with
domination of the last, gland structures of the different size and shape, lined
with single-layer proliferative epithelium. What is the most appropriate
diagnosis?
A. Fibroadenoma
B. Papilloma
C. Noninfiltrative cancer
D. Infiltrative cancer
E. Mastitis

24. Arterial hypertension, hyperglycemia, glucosuria were observed clinically for


a long time in the patient with upper type of obesity. Death was due to the
cerebral haemorrhage. Basophilic hypophysis adenoma, hyperplasia of
adrenal gland cortex was revealed on pathomorphological examination. What
is the likely diagnosis?
A. Cushing disease
B. Diabetes mellitus
C. Acromegaly
D. Hypophysis nanism
E. Adiposogenitalis dystrophy

25. On autopsy it was revealed: large (1 -2 cm) brownish-red, easy crumbling


formations covering ulcerative defects on the external surface of the aortic
valve. What is the most likely diagnosis?
A. Polypus-ulcerative endocarditis
B. Recurrent warty endocarditis
C. Acute warty endocarditis
D. Fibroplastic endocarditis
E. Diffusive endocarditis

26. Autopsy of a woman with cerebral atherosclerosis revealed in the left cerebral
hemisphere a certain focus that is presented by flabby, anhistic, greyish and
yellowish tissue with indistinct edges. What pathological process is the case?
A. Ischemic stroke
B. Multifocal tumor growth with cystic degeneration
C. Multiple foci of fresh and old cerebral hemorrhage
D. Focal encephalitis
E. Senile encephalopathy

27. Autopsy of a 73-year-old man who had been suffering from the coronary heart
disease along with cardiac insufficiency for a long time revealed: nutmeg liver,
brown induration of lungs, cyanotic induration of kidneys and spleen. What
kind of circulation disorder was the cause of such effects?
A. General chronic venous congestion
B. Arterial hyperaemia
C. General acute venous congestion
D. Acute anaemia
E. Chronic anaemia

28. A male patient is 28 years old. Histological study of a cervical lymph node
revealed a change of its pattern due to the proliferation of epithelioid,
lymphoid cells and macrophages having nuclei inform of a horseshoe. In the
center of some cell clusters there were non-structured light-pink areas with
fragments of nuclei. What disease are these changes typical for?
A. Tuberculosis
B. Hodgkin's disease
C. Actinomycosis
D. Tumor metastasis
E. Syphilis

29. Autopsy of a man who had tuberculosis revealed a 3x2 cm large cavity in the
superior lobe of the right lung. The cavity was interconnected with a bronchus,
its wall was dense and consisted of three layers: the internal layer was
pyogenic, the middle layer was made by tuberculous granulation tissue and the
external one was made by connective tissue. What is the most likely diagnosis?
A. Fibrous cavernous tuberculosis
B. Fibrous focal tuberculosis
C. Tuberculoma
D. Acute focal tuberculosis
E. Acute cavernous tuberculosis

30. Autopsy of a man who died from ethylene glycol poisoning revealed that his
kidneys are a little bit enlarged, edematic; their capsule can be easily removed.
Cortical substance is broad and light-grey. Medullary substance is dark-red.
What pathology had this man?
A. Necrotic nephrosis
B. Acute pyelonephritis
C. Acute glomerulonephritis
D. Acute tubular-interstitial nephritis
E. Lipoid nephrosis

31. A man with a long-term history of bronchial asthma died from asphyxia.
Histological examination of his lungs revealed that the lumens of bronchioles
and minor bronchi contained a lot of mucus with some eosinophils. There was
also sclerosis of interalveolar septa, dilatation of alveole lumens. What
mechanism accounts for the development of hypersensitivity reaction?
A. Reagine reaction
B. Cytotoxic reaction
C. Immune complex reaction
D. Lymphocyte-mediated cytolysis
E. Granulomatosis

32. Histologic analysis of uterus mucous membrane revealed twisting glands,


serrated and spinned, they were extended by stroma growth with proliferation
of its cells. Formulate a diagnosis:
A. Glandular hyperplasia of endometrium
B. Acute endometritis
C. Leiomyoma
D. Cystic mole
E. Placental polyp

33. A 22-year-old patient from the West Ukraine complains of laboured nasal
breathing. Morphological examination of biopsy material of nasal mucous
membrane revealed lymphoid, epithelioid, plasma cells as well as Mikulicz's
cells. What is the most probable diagnosis?
A. Rhinoscleroma
B. Glanders
C. Tuberculosis
D. Leprosy
E. Syphilis

34. During autopsy the following has been revealed: the meninges of the upper
cerebral hemispheres are extremely plethoric, of yellow-green color and are
soaked with purulent effluent. What kind of meningitis is characterised by
such clinical presentations?
A. Meningococcal meningitis
B. Tuberculous meningitis
C. Grippal meningitis
D. Anthrax-induced
E. Epidemic typhus-induced

35. Autopsy of a man who had been working as a miner for many years and died
from cardiopulmonary decompensation revealed that his lungs were airless,
sclerosed, their apexex had emphysematous changes, the lung surface was
greyish-black, the incised lung tissue was coal-black. What disease caused
death?
A. Anthracosis
B. Silicosis
C. Talcosis
D. Asbestosis
E. Aluminosis

36. Examination of coronary arteries revealed atherosclerotic calcific plaques that


close vessel lumen by 1/3. The muscle has multiple whitish layers of connective
tissue. What process was revealed in myocardium?
A. Diffuse cardiosclerosis
B. Tiger heart
C. Postinfarction cardiosclerosis
D. Myocarditis
E. Myocardium infarction

37. A 63-year-old male patient who had been suffering from chronic diffuse
obstructive disease, pulmonary emphysema, for 15 years died from cardiac
insufficiency. Autopsy revealed nutmeg liver cirrhosis, cyanotic induration of
kidneys and spleen, ascites, edemata of lower limbs. These changes of internal
organs are typical for the following disease:
A. Chronic right-ventricular insufficiency
B. Acute right-ventricular insufficiency
C. Chronic left-ventricular insufficiency
D. Acute left-ventricular insufficiency
E. General cardiac insufficiency

38. Microscopical examination of an enlarged cervical lymph node revealed


blurring of its structure, absence of lymphoid follicles; all the microscopic
fields showed cells with roundish nuclei and thin limbus of basophil cytoplasm.
It is known from the clinical data that other groups of lymph nodes are also
enlarged as well as spleen and liver. What disease might be suspected?
A. Lymphoid leukosis
B. Lymphogranulomatosis
C. Lymphosarcoma
D. Myeloid leukosis
E. Multiple myeloma

39. As a result of careless handling of an iron, a 34-year-old female patient has got
acute pain, redness, swelling of her right index finger. A few minutes later,
there appeared a blister filled with a transparent liquid of straw-yellow color.
The described changes are a manifestation of the following pathological
process:
A. Exudative inflammation
B. Traumatic edema
C. Alternative inflammation
D. Proliferative inflammation
E. Vacuolar degeneration

40. A worker of a cattle farm fell acutely ill and then died from the progressing
intoxication. Autopsy revealed enlarged, hyposthenic spleen of dark-cherry
colour when dissected; excessive pulp scraping. At the base and fornix of brain
pia maters are edematous, soaked with blood, dark-red ("scarlet hat").
Microscopic examination revealed serous haemorrhagic inflammation of brain
tissues and tunics along with destruction of small vessel walls. What is the
most likely diagnosis?
A. Anthrax
B. Tularemia
C. Brucellosis
D. Plaque
E. Cholera

41. Histological examination of a skin tissue sampling revealed granulomas


consisting of macrophagal nodules with lymphocytes and plasmatic cells.
There are also some big macrophages with fatty vacuoles containing causative
agents of a disease packed up in form of spheres (Virchow's cells). Granulation
tissue is well vascularized. What disease is this granuloma typical for?
A. Lepra
B. Tuberculosis
C. Syphilis
D. Rhinoscleroma
E. Glanders

42. A 40-year-old man noticed a reddening and an edema of skin in the area of his
neck that later developed into a small abscess. The incised focus is dense,
yellowish-green. The pus contains white granules. Histological examination
revealed drusen of a fungus, plasmatic and xanthome cells, macrophages.
What type of mycosis is the most probable?
A. Actinomycosis
B. Aspergillosis
C. Candidosis
D. Sporotrichosis
E. Coccidioidomycosis

43. A physician examined a patient and found inguinal hernia. Through what
anatomic formation does it penetrate into the skin?
A. Hiatus saphenus
B. Anulus femoralis
C. Canalis adductorius
D. Lacuna musculorum
E. Anulus inguinalis superficialis

44. Autopsy of a man who died from burn disease revealed brain edema, liver
enlargement as well as enlargement of kidneys with wide light-grey cortical
layer and plethoric medullary area. Microscopic examination revealed necrosis
of tubules of main segments along with destruction of basal membranes,
intersticium edema with leukocytic infiltration and haemorrhages. What is the
most probable postmortem diagnosis?
A. Necrotic nephrosis
B. Tubulointerstitial nephritis
C. Pyelonephritis
D. Gouty kidney
E. Myeloma kidney

45. A 30-year-old man had been suffering from acute respiratory disease and died
from cardiopulmonary decompensation. Autopsy revealed fibrinous-
haemorrhagic inflammation in the mucous membrane of larynx and trachea,
destructive panbronchitis, enlarged lungs that look black due to the multiple
abcesses, haemorrhages, and necrosis. What is the most probable postmortem
diagnosis?
A. Influenza
B. Parainfluenza
C. Respiratory syncytial infection
D. Measles
E. Adenoviral infection

46. A man with a wound of his limb that had been suppurating for a long time died
from intioxication. Autopsy revealed extreme emaciation, dehydration, brown
atrophy of liver, myocardium, spleen and cross-striated muscles as well as
renal amyloidosis. What diagnosis corresponds with the described picture?
A. Chroniosepsis
B. Septicopyemia
C. Septicemia
D. Chernogubov's syndrome
E. Brucellosis

47. 6 months after delivery a woman had uterine bleeding. Gynecological


examination revealed in the uterine cavity a dark-red tissue with multiple
cavities that resembled of "sponge". Microscopic examination of the tumour
revealed some atypic light epithelial Langhans cells and giant cells of
cyncytiotrophoblast in blood lacunas. What tumour is it?
A. Chorioepithelioma
B. Squamous cell nonkeratinous carcinoma
C. Adenocarcinoma
D. Fibromyoma
E. Vesicular mole

48. A patient with android-type obesity had been suffering from arterial
hypertension, hyperglycemia, glycosuria for a long time and died from the
cerebral haemorrhage. Pathologic examination revealed pituitary basophil
adenoma, adrenal cortex hyperplasia. What is the most likely diagnosis?
A. Itsenko-Cushing's syndrome
B. Diabetes mellitus
C. Acromegalia
D. Pituitary nanism
E. Adiposogenital dystrophy

49. A 40-year-old patient with the progressing staphylococcal purulent


periodontitis developed purulent inflammation of bone marrow spaces of the
alveolar process, and then of the body of mandible. Microscopy revealed
thinning of bone trabeculae, foci of necrosis, bone sequesters surrounded by
the connective tissue capsule. What is the most likely diagnosis?
A. Chronic osteomyelitis
B. Acute osteomyelitis
C. Parodontome
D. Chronic fibrous periostitis
E. Purulent abscess

50. A 40-year-old female patient has undergone thyroidectomy. Histological study


of thyroid gland found the follicles to be of different size and contain foamy
colloid, follicle epithelium is high and forms papillae, and there is focal
lymphocytic infiltration in stroma. Diagnose the thyroid gland disease:
A. Basedow's disease
B. Hashimoto's thyroiditis
C. Riedel's thyroiditis
D. De Quervain's disease
E. Nodular goiter

51. Mucous membrane of the right palatine tonsil has a painless ulcer with smooth
lacquer fundus and regular cartilagenous edges. Microscopically :
inflammatory infiltration that consists of lymphocytes, plasmocytes, a small
number of neutrophils and epithelioid cells; endovasculitis and perivasculitis.
What disease is it?
A. Syphilis
B. Actinomycosis
C. Tuberculosis
D. Pharyngeal diphtheria
E. Ulcerous necrotic Vincent's angina

52. Autopsy of a man with a malignant stomach tumour who had died from cancer
intoxication revealed in the posteroinferior lung fields some dense, grayish-
red irregular foci protruding above the section surface. Microscopic
examination revealed exudate containing a large amount of neutrophils in the
lumen and walls of small bronchi and alveoles. Such pulmonary alterations
indicate the following disease:
A. Acute purulent bronchopneumonia
B. Acute bronchitis
C. Croupous pneumonia
D. Intermittent pneumonia
E. Acute serous bronchopneumonia

53. Microscopical examination of a removed appendix revealed an edema, diffuse


neutrophilic infiltration of appendix wall along with necrosis and defect of
mucous membrane with affection of its muscle plate. What appendicitis form
was developed?
A. Ulcerophlegmonous
B. Phlegmonous
C. Gangrenous
D. Superficial
E. Apostematous

54. A 39 y.o. woman went through an operation in course of which surgeons


removed her uterine tube that was enlarged and a part of an ovary with a big
cyst. Histological examination of a tube walls revealed decidual cells, chorion
villi. What was the most probable diagnosis made after examination of the
uterine tube?
A. Tubal pregnancy
B. Placental polyp
C. Choriocarcinoma
D. Papyraceous fetus
E. Lithopedion

55. Autopsy of a 1,5-year-old child revealed haemorrhagic skin rash, moderate


hyperaemia and edema of nasopharyngeal mucous membrane, small
haemorrhages in the mucous membranes and internal organs; dramatic
dystrophic alterations in liver and myocardium; acute necrotic nephrosis;
massive haemorrhages in the adrenal glands. What disease are these
alterations the most typical for?
A. Meningococcal infection
B. Scarlet fever
C. Diphtheria
D. Measles
E. Epidemic typhus

56. 48 hours after performing tuberculin test (Mantoux test) to a child a 10 mm


papule appeared on the spot of tuberculin introduction. What hypersensitivity
mechanism underlies these changes?
A. Cellular cytotoxicity
B. Anaphylaxis
C. Antibody-dependent cytotoxicity
D. Immune complex cytotoxicity
E. Granulomatosis

57. Colonoscopy of a patient ill with dysentery revealed that mucous membrane of
his large intestine is hyperemic, edematic, its surface was covered with grey-
and-green coats. Name the morphological form of dysenteric collitis:
A. Fibrinous
B. Catarrhal
C. Ulcerous
D. Purulent
E. Necrotic
58. A patient has been syffering from diarrhea for 5 days. On the fith day
colonoscopy revealed that membrane of rectum was inflamed, there were
greyish-green films closely adhering to the subjacent tissue. What is the most
probable diagnosis?
A. Dysentery
B. Typhoid fever
C. Nonspecific ulcerous colitis
D. Salmonellosis
E. Crohn's disease

59. A 10-year-old child has painful swallowing, neck edema, temperature rise up
to 39.0 C; the whole body is covered with bright-red petechial rash. Back of the
throat and tonsils are hyperemic, the tongue is crimson-colored. Tonsillar
surface is covered with isolated grayish-colored necrosis nidi. What disease is
it?
A. Scarlet fever
B. Meningococcal nasopharyngitis
C. Diphtheria
D. Influenza
E. Measles

60. Autopsy of a 48 y.o. man revealed a round formation 5 cm in diameter with


clear-cut outlines in the region of the 1st segment of his right lung. This
formation was encircled with a thin layer of connective tissue full of white
brittle masses. Make a diagnosis of the secondary tuberculosis form:
A. Tuberculoma
B. Caseous pneumonia
C. Acute cavernous tuberculosis
D. Acute focal tuberculosis
E. Fibrous cavernous tuberculosis

61. A man had worked in a coal mine for over 20 years. After his death autopsy
revealed that his lungs were dense, grayish-black and had large areas of
neogenic connective tissue containing a lot of microphages with black pigment
in the cytoplasm. What is the most likely diagnosis?
A. Anthracosis
B. Anthracosilicosis
C. Silicoanthracosis
D. Talcosis
E. Siderosis

62. Skin of a man who died from cardiac insufficiency has an eruption in form of
spots and specks. There are also bedsores in the area of sacrum and spinous
vertebral processes. Microscopical examination of CNS, skin, adrenal glands
revealed in the vessels of microcirculatory bed and in small arteries
destructive-proliferative endothrombovasculitis with Popov's granulomas;
interstitial myocarditis. What diagnosis corresponds with the described
picture?
A. Spotted fever
B. Q fever
C. Enteric fever
D. Nodular periarteritis
E. HIV

63. Autopsy of a 17-year-old girl who died from pulmonary failure revealed a small
area of caseous necrosis in the inferior lobe of the right lung, and occurences
of caseous necrosis in the bronchopulmonary, bronchial and bifurcational
lymph nodes. What is the most probable postmortem diagnosis?
A. Primary tuberculosis
B. Hematogenous progression of primary tuberculosis
C. Hematogenous tuberculosis with predominant lung affection
D. Tuberculoma
E. Caseous pneumonia under secondary tuberculosis

64. Autopsy of a man who died from the sepsis in his femoral bone revealed
phlegmonous inflammation that affected the marrow, haversian canals and
periosteum. Under the periosteum there are multiple abscesses; adjoining soft
tissues of thigh also have signs of phlegmonous inflammation. What
pathological process was described?
A. Acute hematogenous osteomyelitis
B. Osteoporosis
C. Chronic hematogenous osteomielitis
D. Osteopetrosis

65. An experimental animal was first sensibilized whereupon an antigen dose was
introduced subcutaneously. This injection resulted in the development of a
fibrinous inflammation with alteration of vessel walls, basal substance and
fibrous structures of connective tissue in form of mucoid and fibrinoid swelling
and necrosis. What immunological reaction took place?
A. Immediate hypersensitivity
B. Delayed-type hypersensitivity
C. Reaction of transplantation immunity
D. Normergic reaction
E. Granulomatosis

66. Examination of a 55-year-old woman revealed under the skin of


submandibular area a movable slowly growing pasty formation with distinct
borders 1, 0x0, 7 cm large. Histological examination revealed lipocytes that
form segments of diffrent forms and sizes separated from each other by thin
layers of connective tissue with vessels. What is the most probable diagnosis?
A. Lipoma
B. Fibroma
C. Angioma
D. Liposarcoma
E. Fibrosarcoma

67. A 4-year-old child complained of pain during deglutition, indisposition.


Objectively: palatine arches and tonsils are moderately edematic and
hyperemic, there are greyish-white films up to 1 mm thick closely adhering to
the subjacent tissues. What pathological process are these changes typical for?
A. Inflammation
B. Dystrophy
C. Necrosis
D. Metaplasia
E. Organization

68. A 9 m.o. child has delayed dentition; it is also out of order. Upper jaw
configuration is horizontal ("high" palate); microscopically - irregular
mineralization of tooth enamel, wrinkled enamel prisms, some of them are
vacuolized. Predentin zone is extended; there are solitary denticles. What
disease is it?
A. Early rickets
B. Late rickets
C. Osteomalacia
D. Gout
E. Hypervitaminosis D

69. Microscopical renal examination of a 36 y.o. woman who died from renal
insufficiency revealed in the glomerules proliferation of capsule
nephrothelium as well as of podocytes and phagocytes accompanied by
formation of "crescents", capillary loop necrosis, fibrinous thrombs in their
lumens; sclerosis and hyalinosis of glomerules, atrophy of tubules and fibrosis
of renal stroma. What is the most probable diagnosis?
A. Subacute glomerulonephritis
B. Acute glomerulonephritis
C. Chronic glomerulonephritis
D. Focal segmentary sclerosis
E. Membranous nephropathy

70. A forensic medical expert examines the body of a 58 y.o. man who had been
consuming large amounts of alcochol for a long time and died at home.
Microscopicaly: the right lung is dense and enlarged, its incision revealed that
the tissue is greyish and homogenous, pleura is covered with greyish layers.
Microscopically - alveolar cavities contain fibrin, hemolyzed erythrocytes.
Make a diagnosis:
A. Croupous pneumonia
B. Focal pneumonia
C. Interstitial pneumonia
D. Primary pulmonary tuberculosis
E. Caseous pneumonia
71. Autopsy of a 50-year-old man revealed the following changes: his right lung
was moderately compact in all parts, the dissected tissue was found to be
airless, fine-grained, dryish. Visceral pleura had greyish-brown layers of fibrin.
What is the most likely diagnosis?
A. Croupous pneumonia
B. Tuberculosis
C. Bronchopneumonia
D. Interstitial pneumonia
E. Pneumofibrosis

72. Autopsy of a 56 y.o. man revealed in the right temporal part of brain a big focus
of softened grey matter that was semi-liquid and light grey. Arteries of cerebral
tela contain multiple whitish-yellow thickenings of intima that abruptly
narrow the lumen. What is your diagnosis?
A. Ischemic stroke
B. Brain abscess
C. Hemorrhage
D. Hemorrhagic infarction
E. Brain edema

73. A 22 y.o. woman has enlarged lymph nodes. Histologically: a lymph node
contains lymphocytes, histiocytes, reticular cells, small and big Hodgkin's
cells, multinucleated Sternberg cells, isolated foci of caseous necrosis. What
disease are these changes typical for?
A. Lymphogranulomatosis
B. Lymphosarcoma
C. Chronic leukosis
D. Acute leukosis
E. Lung cancer metastasis

74. Analysis of a punction biopsy material of liver revealed hepatocyte dystrophy


with necroses as well as sclerosis with disorder of beam and lobulous structure,
with formation of pseudolobules and regenerative nodes. What is the most
probable diagnosis:
A. Liver cirrhosis
B. Chronic hepatosis
C. Chronic hepatitis
D. Progressive massive liver necrosis
E. Acute hepatitis

75. Autopsy of a man, who had been suffering from the multiple bronchiectasis for
5 years and died from chronic renal insufficiency, revealed that kidneys were
dense and enlarged, with thickened cortical layer of white colour with greasy
lustre. What renal disease might be suspected?
A. Secondary amyloidosis
B. Glomerulonephritis
C. Chronic pyelonephritis
D. Necrotic nephrosis

76. Autopsy of a 49-year-old woman who died from chronic renal insufficiency ,
revealed: kidneys were dense, reduced, multicoloured, with haemorrhagic
areas. Microscopic examination revealed some hematoxylin bodies in the
nuclei of the renal tubule epithelium; "wire-loop" thickening of the glomerular
capillary basement membrane; here and there in the capillaries some hyaline
thrombi and foci of fibrinoid necrosis were present. What is the most likely
diagnosis?
A. Systemic lupus erythematosus
B. Rheumatism
C. Arteriosclerotic pneumosclerosis
D. Amyloidosis
E. Atherosclerotic nephrosclerosis

77. A 39-year-old man who had been operated for the stomach ulcer died 7 days
after the surgery. Autopsy revealed that peritoneal leaves were dull, plephoric,
covered with massive yellow-greenish films, the peritoneal cavity contained for
about 300 ml of thick yellow-greenish liquid. What pathologic process was
revealed in the peritoneal cavity?
A. Fibrinous suppurative peritonitis
B. Serous peritonitis
C. Fibrinous serous peritonitis
D. Peritoneal commissures
E. Fibrinous haemorrhagic peritonitis

78. A 45 y.o. patient consulted a doctor about plaque-shaped formation on his


neck. Histological examination of biopsy skin material revealed tumourous
cells of round and oval form with thin ring of basophilic cytoplasma that
resemble of cells of basal epidermal layer. What tumour is it?
A. Basalioma
B. Epidermal cancer
C. Hydradenoma
D. Trichoepithelioma
E. Syringoadenoma

79. A 71-year-old man had been presenting with diarrhea for 10 days. The feces
had admixtures of blood and mucus. He was delivered to a hospital in grave
condition and died 2 days later. Autopsy of the body revealed the following:
diphtheritic colitis with multiple irregularly-shaped ulcers of different depth
in both sigmoid colon and rectus. Bacteriological analysis revealed Shigella.
What was the main disease?
A. Dysentery
B. Typhoid fever
C. Salmonellosis
D. Nonspecific ulcerous colitis
E. Yersiniosis

80.A 63 y.o. man fell ill with acute tracheitis and bronchitis accompanied by
bronchial pneumonia. On the 10th day the patient died from cardiopulmonary
insufficiency. Autopsy revealed fibrinous hemorrhagic
laryngotracheobronchitis; lungs were enlarged, their incision revealed the
"coal-miner's" effect caused by interlacing of sections of bronchial pneumonia,
hemorrhages into the pulmonary parenchyma, acute abscesses and
atelectases. Internal organs have discirculatory and dystrophic changes. What
is the most probable diagnosis?
A. Influenza, severe form
B. Moderately severe influenza
C. Parainfluenza
D. Respiratory syncytial infection
E. Adenoviral infection

81. Autopsy of a man who died from influenza revealed that his heart was slightly
enlarged, pastous, myocardium was dull and had specks. Microscopical
examination of myocardium revealed signs of parenchymatous adipose and
hydropic dystrophy; stroma was edematic with poor macrophagal and
lymphocytic infiltration, vessels were plethoric; perivascular analysis revealed
petechial hemorrhages. What type of myocarditis was developed in this case?
A. Serous diffuse
B. Interstitial proliferative
C. Serous focal
D. Purulent
E. Granulomatous

82. A boy is 7 y.o. Objectively: against the background of hyperemic skin there is
knobby bright-pink rash on his forehead, neck, at the bottom of abdomen, in
the popliteal spaces; nasolabial triangle is pale. Examination of oropharyngeal
surface revealed localized bright-red hyperemia; tonsils are swollen, soft,
lacunas contain pus, tongue is crimson. Cervical lymph nodes are enlarged,
dense and painful. What is the most probable diagnosis?
A. Scarlet fever
B. Rubella
C. Whooping cough
D. Diphtheria
E. Infectious mononucleosis

83. Autopsy of a 75-year-old patient who had been suffering from disseminated
atherosclerosis and died under chronic cardiac failure revealed constriction
and deformation of coronary arteries, tuberous intima whose section appeared
to be white and petrosal. Specify the stage of atherosclerosis morphogenesis:
A. Atherocalcinosis
B. Lipoidosis
C. Liposclerosis
D. Bilipid
E. Atheromatosis

84. Examination of a bronchial tissue sample revealed atrophy of mucous


membrane, cystic degeneration of glands, focal metaplastic changes of lining
prismatic epithelial cells into multilayer squamous cells; increase in goblet cell
number; in some parts of bronchial wall and especially in the mucous
membrane there was marked cellular inflammatory infiltration and growth of
granulation tissue bulging into the bronchial lumen in form of a polyp. What
is the most likely diagnosis?
A. Chronic bronchitis
B. Lobar pneumonia
C. Acute bronchitis
D. Bronchopneumonia
E. Interstitial pneumonia

85. Gynecological examination of the uterine cervix in a 30-year-old woman


revealed some bright-red lustrous spots that easily bleed when touched. Biopsy
showed that a part of the uterine cervix was covered with cylindrical epithelium
with papillary outgrowths; in the depth of tissue the growth of glands was
present. What pathology of the uterine cervix was revealed?
A. Pseudoerosion
B. True erosion
C. Endocervicitis
D. Glandular hyperplasia
E. Leukoplakia

86. A stillborn child was found to have thickened skin resembling of the tortoise
shell, underdeveloped auricles. Histological examination of skin revealed
hyperkeratosis, atrophy of the granular epidermis layer; inflammatory
changes were not present. What is the most likely diagnosis?
A. Ichthyosis
B. Leukoplakia
C. Xerodermia
D. Erythroplakia
E. Dermatomyositis

87. A 3-year-old child with meningeal symptoms died. Postmortem macroscopy of


the pia matter revealed miliary nodules which were microscopically
represented by a focus of caseous necrosis with masses of epithelioid and
lymphoid cells with some crescent-shaped large cells inbetween having
peripheral nuclei. Specify the type of meningitis in the child:
A. Tuberculous
B. Syphilitic
C. Brucellar
D. Grippal
E. Meningococcal

88. A pathology-histology laboratory received a vermiform appendix up to 2,0 cm


thick. Its serous membrane was pale, thick and covered with yellowish-green
films. The wall was flaccid, of grayish-red colour. The appendix lumen was
dilated and filled with yellowish-green substance. Histological examination
revealed that the appendix wall was infiltrated with neutrophils. Specify the
appendix disease:
A. Acute phlegmonous appendicitis
B. Acute gangrenous appendicitis
C. Acute superficial appendicitis
D. Acute simple appendicitis
E. Chronic appendicitis

89. A 46-year-old patient who had been suffering from tuberculosis for 6 years
died from massive pulmonary haemorrhage. Autopsy revealed different-sixed
foci of sclerosis and caseous necrosis in lungs, in the upper part of the right
lung there was a cavity 5 cm in diameter with dense grey walls, the cavity
contained liquid blood and blood clots. What type of tuberculosis is it?
A. Fibrocavernous
B. Acute cavernous
C. Infiltrative
D. Fibrous focal
E. Acute focal

90. A patient died from cardiopulmonary decompensation. Histological


examination revealed diffused pulmonary lesion together with interstitial
edema, infiltration of tissue by limphocytes, macrophages, plasmocytes;
pulmonary fibrosis, panacinar emphysema. What disease corresponds with
the described picture?
A. Fibrosing alveolitis
B. Chronic bronchitis
C. Bronchopneumonia
D. Pulmonary atelectasis
E. Bronchial asthma

91. A 50-year-old patient has been taking treatment thrice for the last 6 months
because of fractures caused by domestic accidents. Microscopical examination
of bony tissue revealed foci of lacunar resolution, giant-cell granulomas in the
tumour-like formations, cysts. Bony tissuewas substituted by fibrous
connective tissue. Examination revealed also adenoma of parathyroid gland
and hypercalcemia. What is the most probable diagnosis?
A. Parathyroid osteodystrophy
B. Myelomatosis
C. Osteomyelitis
D. Osteopetrosis
E. Paget's disease
92. A patient who abuses smoking has chronic bronchitis. Biopsy of his primary
bronchus revealed multilayer pavement epithelium. What pathological
process was revealed in the bronchus?
A. Metaplasia
B. Physiological regeneration
C. Reparative regeneration
D. Hyperplasia
E. Dysplasia

93. 2 hours after a skeletal extension was performed to a 27-year-old patient with
multiple traumas (closed injury of chest, closed fracture of right thigh) his
condition abruptly became worse and the patient died from acute
cardiopulmonary decompensation. Histological examination of pulmonary
and cerebral vessels stained with Sudan III revealed orange drops occluding
the vessel lumen. What complication of polytrauma was developed?
A. Fat embolism
B. Gaseous embolism
C. Microbal embolism
D. Thromboembolism
E. Air embolism

94. A 50-year-old patient underwent resection of tumour of large intestine wall.


Microscopically it presents itself as fascicles of divergent collagen fibers of
different thickness and form and some monomorphous fusiform cells that are
irregularly distributed among the fibers. Cellular atypia is not evident. What
tumour is it?
A. Hard fibroma
B. Fibromyoma
C. Soft fibroma
D. Desmoma
E. Fibrosarcoma

95. Autopsy of a 5-year-old child revealed in the area of vermis of cerebellum a soft
greyish-pink node 2 cm in diameter with areas of haemorrhage. Histologically
this tumour consisted of atypical monomorphous small roundish cells with big
polymorphous nuclei. What tumour is it?
A. Medulloblastoma
B. Meningioma
C. Glioblastoma
D. Astrocytoma
E. Oligodendroglioma

96. In course of severe respiratory viral infection there appeared clinical signs of
progressing cardiac insufficiency that caused death of a patient in the 2nd week
of disease. Autopsy revealed that the heart was sluggish, with significant cavity
dilatation. Histological examination of myocardium revealed plephora of
microvessels and diffuse infiltration of stroma by lymphocytes and histiocytes.
What disease corresponds with the described picture?
A. Myocarditis
B. Stenocardia
C. Acute coronary insufficiency
D. Myocardium infarction
E. Cardiomyopathy

97. A patient with high-titer antinuclear antibodies died from progressing renal
impairment. Autopsy revealed mesangioproliferative glomerulonephritis and
abacterial polypous endocarditis. There was periarterial bulbar sclerosis in
spleen and productive proliferative vasculitis in skin. What is the most likely
diagnosis?
A. Systemic lupus erythematosus
B. Nephrotic syndrome
C. Rheumatism
D. Dermatomyositis
E. Periarteritis nodosa

98. A 38-year-old patient with full-blown jaundice, small cutaneous hemorrhages,


general weakness and loss of appetite underwent puncture biopsy of liver.
Histological examination revealed disseminated dystrophy, hepatocyte
necrosis, and Councilman’s bodies. Lobule periphery has signs of significant
infiltration by lymphocytes; there are also individual multinuclear
hepatocytes. What is the most probable diagnosis?
A. Acute viral hepatitis
B. Acute alcoholic hepatitis
C. Miliary hepatic cirrhosis
D. Toxic degeneration of liver
E. Chronic hepatitis

99. A 20-year-old patient died from intoxication 8 days after artificial illegal
abortion performed in her 14-15th week of pregnancy. Autopsy of the corpse
revealed yellowish colour of eye sclera and of skin, necrotic suppurative
endometritis, multiple pulmonary abscesses, spleen hyperplasia with a big
number of neutrophils in its sinuses. What complication after abortion was
developed?
A. Septicopyemia
B. Septicemia
C. Hemorrhagic shock
D. Chroniosepsis
E. Viral hepatitis type A

100. A section of the left lung was found to have an area of dense red tissue.
The area was cone-shaped, stood out distinctly from the healthy tissue, with
its base directed to the pleura. The dissected tissue was granular, dark -red.
What is the most likely diagnosis?
A. Haemorrhagic infarction
B. Lung abscess
C. Lung gangrene
D. Primary tuberculous affection
E. Croupous pneumonia

101. A patient has a cluster of matted together dense lymph nodes on his
neck. Histological examination of a removed lymph node revealed
proliferation of reticular cells, presense of Reed-Sternberg cells. What disease
is meant?
A. Lymphogranulomatosis
B. Lymphoblastic leukosis
C. Myeloblastic leukosis
D. Myelocytic leukosis
E. Lymphocytic leukosis

102. A patient had been suffering from profuse diarrhea and vomiting for 2
days. He died from acute dehydration. Autopsy revealed that the intestinal wall
was edematic and hyperemic, with multiple haemorrhages in the mucous
membrane. Intestine lumen contains whitish fluid resembling of rice water.
What disease caused death?
A. Cholera
B. Dysentery
C. Salmonellosis
D. Typhoid fever
E. Enterocolitis

103. Examination of a 66-year-old patient revealed a lytic tumour in the locus


of pathological rib fracture. Histologically this tumour consists of atypical
plasmoblasts. Further examination revealed osteoporosis in the bones of
vertebral column and pelvis. These changes are typical for:
A. Myelomatosis
B. Tuberculous osteomyelitis
C. Ewing's osteosarcoma
D. Neuroblastoma
E. Metastatic lung cancer

104. A patient died from acute cardiac insufficiency, among clinical


presentations there was gastrointestinal haemorrhage. Examination of
mucous membrane of sromach revealed some defects reaching myenteron;
their edges and bottom were mostly even and loose, some of them contained
dark-red blood. What pathological process was revealed?
A. Acute ulcers
B. Chronic ulcers
C. Erosions
D. Thrombosis
E. Inflammation

105. A 33-year-old man died from uraemia. Autopsy revealed enlarged


kidneys weighing 500, 0 each and consisting of multiple cavities 0,5 -2 cm in
diameter. The cavities were full of light-yellow transparent liquid. Renal pelvis
and ureters had no pecularities. What renal disease caused uraemia?
A. Bilateral polycystic renal disease
B. Chronic pyelonephritis
C. Renal tumour
D. Renal tuberculosis
E. Rapidly progressing glomerulonephritis

106. A patient ill with tuberculosis died from progressing cardiopulmonary


decompensation. Autopsy in the area of the right lung apex revealed a cavity 5
cm in diameter communicating with lumen of a segmental bronchus. On the
inside cavity walls are covered with caseous masses with epithelioid and
Langhans cells beneath them. What morphological form of tuberculosis is it?
A. Acute cavernous tuberculosis
B. Tuberculoma
C. Caseous pneumonia
D. Infiltrative tuberculosis
E. Acute focal tuberculosis

107. A 45-year-old male died from disseminated tuberculosis. On autopsy the


symptoms of tuberculosis were confirmed by both microscopical and
histological analyses. All the affected organs had epithelioid cell granulomas
with caseous necrosis in the centre. What kind of hypersensitivity reaction
underlies the process of granuloma development?
A. Delayed
B. Antibody-dependent cytotoxicity
C. Complement-dependent cytotoxicity
D. Anaphylactic
E. Immune complex

108. A 30-year-old woman has applied a lipstick with a fluorescent substance


for a long time. Then she got a limited erythema and slight peeling on her lip
border, later there appeared transversal striae and cracks. Special methods of
microscopic examination of the affected area helped to reveal sensibilized
lymphocytes and macrophages in the connective tissue; cytolysis. What typeof
immunological hypersensitivity was developed?
A. IV type (cellular cytotoxicity)
B. I type (reaginic)
C. II type (antibody cytotoxicity)
D. III type (immune complex cytotoxicity)
E. Granulomatosis
109. Examination of a young woman revealed a node-like, soft and elastic
homogenous tumour of pinkish-white colour along the acoustic nerve. The
tumour contains cell bundles with oval nuclei. Cellular fibrous bundles form
rhythmic structures made up by parallel rows of regularly oriented cells
arranged in form of a palisade with cell-free homogenous zone (Verocay
bodies) between them. What tumour is it?
A. Neurinoma
B. Malignant neurinoma
C. Ganglioneurinoma
D. Neuroblastoma
E. Ganglioneuroblastoma

110. A 23-year-old man has perforation of hard palate. In the area of this
perforation there was a compact well-defined formation. Microscopic
examination of the resected formation revealed a large focus of caseous
necrosis surrounded by granulation tissue with endovasculitis, cellular
infiltration composed of lymphocytes, epithelioid cells (mainly plasmocytes).
What is the most probable diagnosis?
A. Syphilis
B. Tuberculosis
C. Scleroma
D. Sarcoma
E. Leprosy

111. A 50-year-old man who was referred to the hospital for treatment of
cervical lymphadenitis underwent test for induvidual sensitivity to penicillin.
30 seconds after he went hot all over, AP dropped down to 0 mm Hg that led
to cardiac arrest. Resuscitation was unsuccessful. Autopsy results: acute
venous plethora of internal organs; histological examination of skin (from the
site of injection) revealed degranulation of mast cells (tissue basophils).
Degranulation was also revealed in myocardium and lungs. What type of
hypersensitivity reaction is it?
A. Anaphylactic
B. Delayed-type hypersensitivity
C. Complement-mediated cytotoxic
D. Immunocomplex-mediated

112. During blood transfusion a patient has developed intravascular


erythrocyte hemolysis. What kind of hypersensitivity does the patient have?
A. II type (antibody-dependent)
B. I type (anaphylactic)
C. III type (immune complex)
D. IV type (cellular cytotoxicity)
E. IV type (granulomatosis)

113. A 2-year-old child had acute respiratory viral infection and died from
cardiopulmonary decompensation. Autopsy revealed that his right lung was
hyperemic; in the 2nd, 6th and 10th segments and on the incision there were
airless yellowish foci of irregular form, from several mm up to 1 cm large.
Microscopical examination revealed exudate consisting mainly of neutrophils
in the given areas of pulmonary tissue in the alveoles, bronchioles and
bronchial tubes. What is the most probable diagnosis?
A. Focal pneumonia
B. Interstitial pneumonia
C. Croupous pneumonia
D. Acute bronchitis
E. Pulmonary abscess

114. The upper lobe of the right lung is enlarged, grey and airless, the
inscision surface is dripping with turbid liquid, the pleura has many
fibrinogenous films; microscopical examination of alveoles revealed exudate
containing neutrophils, desquamated alveolocytes and fibrin fibers. The
bronchus wall is intact. What is the most probable diagnosis?
A. Croupous pneumonia
B. Interstitial pneumonia
C. Pulmonary abscess
D. Focal pneumonia
E. Influenzal pneumonia

115. A 28-year-old patient had high arterial pressure, hematuria and facial
edemata. In spite of treatment renal insufficiency was progressing. 6 months
later the patient died from uremia. Microscopic examination of his kidneys and
their glomerules revealed proliferation of capsule nephrothelium and of
podocytes with "demilune" formation, sclerosis and hyalinosis of glomerules.
What disease corresponds with the described picture?
A. Subacute glomerulonephritis
B. Acute pyelonephritis
C. Nephrotic syndrome
D. Chronic glomerulonephritis
E. Acute glomerulonephritis

116. Autopsy of a man ill with severe hypothyroidism revealed that


connective tissue, organ stroma, adipose and cartilaginous tissues were
swollen, semitransparent, mucus-like. Microscopic examination of tissues
revealed stellate cells having processes with mucus between them. What type
of dystrophy is it?
A. Stromal-vascular carbohydrate
B. Stromal-vascular adipose
C. Stromal-vascular proteinaceous
D. Parenchymatous proteinaceous
E. Parenchymatous adipose

117. Examination of the anterior abdominal wall of a pregnant woman


revealed a tumour-like formation that arose on the spot of a tumour that was
removed two years ago. The neoplasm was well-defined, dense, 2х1 cm large.
Histological examination revealed that the tumour was composed of
differentiated connective tissue with prevailing collagen fibres. What tumour
might be suspected?
A. Desmoid
B. Lipoma
C. Fibrosarcoma
D. Hibernoma
E. Leiomyoma

118. In the course of puncture biopsy of transplanted kidney, the following


has been revealed: diffuse infiltration of stroma by lymphocytes and
plasmocytes and necrotic arteritis. What pathological process has developed
in the transplant?
A. Immune rejection
B. Ischemic kidney failure
C. Glomerulonephritis
D. Tubular necrosis
E. Pyelonephritis

119. A 35-year-old female patient has undergone biopsy of the breast


nodules. Histological examination has revealed enhanced proliferation of the
small duct and acini epithelial cells, accompanied by the formation of
glandular structures of various shapes and sizes, which were located in the
fibrous stroma. What is the most likely diagnosis?
A. Fibroadenoma
B. Adenocarcinoma
C. Cystic breast
D. Invasive ductal carcinoma
E. Mastitis

120. Autopsy of a dead patient revealed bone marrow hyperplasia of tubular


and flat bones (pyoid marrow), splenomegaly (6 kg) and hepatomegaly (5 kg),
enlargement of all lymph node groups. What disease are the identified changes
typical for?
A. Chronic myelogenous leukemia
B. Chronic lymphocytic leukemia
C. Multiple myeloma
D. Polycythemia vera
E. Hodgkin's disease
Krok 1 – 2014 Path Physiology Base
1. A 27- year-old woman has dropped penicillin containing eye drops. In few
minutes there appeared feeling of itching, burning of the skin, lips and eyelids
edema, whistling cough, decreasing of BP. What antibodies take part in the
development of this allergic reaction?
A. IgE and IgG
B. IgM and IgG
C. IgA and IgM
D. IgM and IgD
E. IgG and IgD

2. A patient underwent a surgery for excision of a cyst on pancreas. After this he


developed haemorrhagic syndrome with apparent disorder of blood
coagulation. Development of this complication can be explained by:
A. Activation of fibrinolytic system
B. Insufficient fibrin production
C. Reduced number of thrombocytes
D. Activation of anticoagulation system
E. Activation of Christmas factor

3. A 43-year-old patient has thrombopenia, reduction of fibrinogen, products of


degradation of fibrin presented in the blood, petechial haemorrhage along with
septic shock. What is the most likely cause of the changes?
A. DIC-syndrom
B. Autoimmune thrombocytopenia
C. Haemorrhagic diathesis
D. Disorder of thrombocytes production
E. Exogenous intoxication

4. Person has stable HR, not more than 40 bpm. What is the pacemaker of the
heart rhythm in this person?
A. Atrioventricular node
B. Sinoatrial node
C. His' bundle
D. Branches of His' bundle
E. Purkinye' fibers

5. A 32-year-old patient was admitted to the hospital with gross bloodloss due to
auto accident trauma. Ps - 110Bpm, RR- 22 pm, BP- 100/60mm Hg. What
changes in the blood will occur in an hour after the bloodloss?
A. Hypovolemia
B. Erythropenia
C. Hypochromia of erythrocytes
D. Leukopenia
E. Hypoproteinemia
6. A 70-year-old patient suffers from atherosclerosis complicated by the lower
limb thrombosis that has caused gangrene on his left toes. What is the most
likely cause of the thrombosis origin?
A. Thrombocyte adhesion
B. Prothrombinase activation
C. Transformation of prothrombin into thrombin
D. Transformation of fibrinogen into fibrin
E. Impaired heparin synthesis

7. ECG of a 44-year-old patient shows signs of hypertrophy of both ventricles and


the right atrium. The patient was diagnosed with the tricuspid valve
insufficiency. What pathogenetic variant of cardiac dysfunction is usually
observed in case of such insufficiency?
A. Heart overload by volume
B. Heart overload by resistance
C. Primary myocardial insufficiency
D. Coronary insufficiency
E. Cardiac tamponade

8. Shock and signs of acute renal failure (ARF) developed in the patient due to
permanent injury. What is the leading cause of development of ARF in the
case?
A. Decreased arterial pressure
B. Urine excretion violation
C. Increased pressure in the nephron capsule
D. Increased pressure in the renal arteries
E. Decreased oncotic BP

9. Substitution of the glutamic acid on valine was revealed while examining initial
molecular structure. For what inherited pathology is this typical?
A. Sickle-cell anemia
B. Thalassemia
C. Minkowsky-Shauffard disease
D. Favism
E. Hemoglobinosis

10. Inflamation is characterised by increasing penetration of vessels of


microcirculation stream, increasing of their fluid dynamic blood pressure.
Increasing of the osmotic concentration and dispersity of protein structures
present in the intercellular fluid. What kind of edema will appear in this case?
A. Mixed
B. Hydrodynamic
C. Colloid-osmotic
D. Lymphogenic
E. Membranogenic
11. Disorder of the airways passage in small and middle bronchi was revealed in
the patient. What disorder of the acid-base equilibrium can be detected in the
blood?
A. Respiratory acidosis
B. Metabolic acidosis
C. Respiratory alkalosis
D. Metabolic alkalosis
E. -

12. A 62-year-old patient was admitted to the neurological department due to


cerebral haemorrage. Condition is grave. There is observed progression of
deepness and frequency of breath that turnes into reduction to apnoea, and the
cycle repeates. What respiration type has developed in the patient?
A. Cheyne-Stockes respiration
B. Kussmaul respiration
C. Biot's respiration
D. Gasping respiration
E. Apneustic respiration

13. In a 45-year-old patient on ECG it was revealed: sinus rhythm, the number of
auricular complexesexceeds number of ventricular complexes; progressing
extension of the P-Q interval from complex to complex; fallout of some
ventricular complexes; Р waves and QRST complexes are without changes.
Name the type of heart rhythm disfunction.
A. Atrioventricular block of the II degree
B. Synoauricular block
C. Atrioventricular blockade of the I degree
D. Intraatrial block
E. Complete atrioventricular block

14. Upper neck node of sympathetic trunk was removed from the rabbit on
experiment. Reddening and increased temperature of the skin of head is
observed. What form of peripheral circulation of the blood developed in the
rabbit?
A. Neuroparalytic arterial hyperemia
B. Neurotonic arterial hyperemia
C. Metabolic arterial hyperemia
D. Venous hyperemia
E. Stasis

15. In result of the damage of one of the Atomic Power Plant reacttor the run-out
of radioelements happened. People in the increased radiation zone were
radiated with approximately 250-300 r. They were immediately hospitalized.
What changes in the blood count would be typical?
A. Lymphopenia
B. Leukopenia
C. Anemia
D. Thrombopenia
E. Neutropenia

16. Dystrophic changes of the heart muscle are accompanied with cardiac cavity
enlargement, decrease of the strength of heart contraction, increased amount
of blood, which remains in the heart during systolic phase, overfilled veins. For
what state of heart is it characteristic?
A. Myogenic dilatation
B. Tonogenic dilatation
C. Emergency stage of hyperfunction and hypertrophy
D. Cardiosclerosis
E. Tamponage of the heart

17. Transmural myocardial infarction in the patient was complicated with


progressive acute left ventricle insufficiency. What is the most typical for this
state?
A. Edema of the lungs
B. Edema of the extremities
C. Cyanosis
D. Ascites
E. Arterial hypertension

18. Arterial hypertention is caused by the stenosis of the renal arteries in the
patient. Activation of what system is the main link in the pathogenesys of this
form of hypertension?
A. Renin-angiotensin
B. Sympathoadrenal
C. Parasympathetic
D. Kallikrein-kinin
E. Hypothalamic-pituitary

19. A 12-year-old boy often suffers from virus and bacterial infections and
eczematous skin lesions. Enlargement of T -lymphocytes and IgM with normal
IgA and IgG was revealed on examination. What type of immune system
pathology is presented in the patient?
A. Composite immunedefficiency
B. Hypoplasia of thymus
C. Bruton's hypogammaglobulinemia
D. Turner's syndrome
E. Hereditary immundeficiency of the complement system

20. Patient with diabetes didn't get insulin injection in time that caused
hyperglycemic coma (glucose in the blood 50mmol/L). What mechanism is
prevalent in the development of the coma?
A. Hyperosmia
B. Hypokaliemia
C. Hypoxia
D. Hyponatremia
E. Acidosis

21. Necrosis focus appeared in the area of hyperemia and skin edema in few hours
after burn. What mechanism strengthens destructive events in the
inflammation area?
A. Secondary alteration
B. Primary alteration
C. Emigration of lymphocytes
D. Diapedesis of erythrocytes
E. Proliferation of fibroblasts

22. Processes of repolarisation are disturbed in ventricular myocardium in


examined person. It will cause amplitude abnormalities of configuration and
duration of the wave:
A. Т
B. Q
C. R
D. S
E. P

23. A boy referred to a genetics clinic was found to have 1 drumstick in blood
neutrophils. The boy is likely to have the following syndrome:
A. Klinefelter's
B. Down's
C. Turner's
D. Edwards'
E. Trisomy X

24. After a serious psycho-emotional stress a 45-year-old patient suddenly felt


constricting heart pain irradiating to the left arm, neck and left scapula. His
face turned pale, the cold sweat stood out on it. The pain attack was stopped
with nitroglycerine. What process has developed in this patient?
A. Stenocardia
B. Myocardial infarction
C. Stroke
D. Psychogenic shock
E. Stomach ulcer perforation

25. While having the dinner the child choked and aspirated the food. Heavy cough
has started, skin and mucose are cyanotic, rapid pulse, rear breathing,
expiration is prolonged. What disorder of the external breathing developed in
the child?
A. Stage of expiratory dyspnea on asphyxia
B. Stage of inspiratory dyspnea on asphyxia
C. Stenotic breathing
D. Alternating breathing
E. Biot's breathing

26. Chronic glomerulonephritis was diagnosed in a 34-year-old patient 3 years


ago. Edema has developed in the last 6 monthes. What caused it?
A. Proteinuria
B. Hyperproduction of vasopressin
C. Disorder of albuminous kidneys function
D. Hyperosmolarity of plasma
E. Hyperaldosteronism

27. While playing volleyball a sportsman made a jump and landed on the outside
edge of his foot. He felt acute pain in the talocrural joint, active movements are
limited, passive movements are unlimited but painful. A bit later there
appeared a swelling in the area of external ankle; the skin became red and
warm. What type of peripheral circulation disturbance is the case?
A. Arterial hyperemia
B. Stasis
C. Embolism
D. Venous hyperemia
E. Thrombosis

28. Having helped to eliminate consequences of a failure at a nuclear power plant,


a worker got an irradiation doze of 500 roentgens. He complains of headache,
nausea, dizziness. What changes in leukocytes quantity can be expected 10
hours after irradiation?
A. Neutrophilic leukocytosis
B. Lymphocytosis
C. Leukopenia
D. Agranulocytosis
E. Leukemia

29. A 12 y.o. boy who suffers from bronchial asthma has an acute attack of asthma:
evident expiratory dyspnea, skin pallor. What type of alveolar ventilation
disturbance is it?
A. Obstructive
B. Restrictive
C. Throracodiaphragmatic
D. Central
E. Neuromuscular

30. A 46-year-old patient suffering from the diffuse toxic goiter underwent
resection of the thyroid gland. After the surgery the patient presents with
appetite loss, dyspepsia, increased neuromuscular excitement. The body
weight remained unchanged. Body temperature is normal. Which of the
following has caused such a condition in this patient?
A. Reduced production of parathormone
B. Increased production of thyroxin
C. Increased production of calcitonin
D. Increased production of thyroliberin
E. Reduced production of thyroxin

31. To prevent the transplant rejection after organ transplantation it is required to


administer hormonotherapy for the purpose of immunosuppression. What
hormones are used for this purpose?
A. Glucocorticoids
B. Mineralocorticoids
C. Sexual hormones
D. Catecholamines
E. Thyroid

32. A patient caught a cold after which there appeared facial expression disorder.
He cannot close his eyes, raise his eyebrows, and bare his teeth. What nerve is
damaged?
A. Facial
B. Vagus
C. Trigeminus
D. Glossopharyngeal
E. Infraorbital

33. A patient with a history of chronic glomerulonephritis presents with azotemia,


oliguria, hypo- and isosthenuria, proteinuria. What is the leading factor in the
pathogenesis of these symptoms development under chronic renal failure?
A. Mass decrease of active nephrons
B. Intensification of glomerular filtration
C. Tubular hyposecretion
D. Disturbed permeability of glomerular membranes
E. Intensification of sodium reabsorption

34. After an attack of bronchial asthma, a patient had his peripheral blood tested.
What changes can be expected?
A. Eosinophilia
B. Leukopenia
C. Lymphocytosis
D. Thrombocytopenia
E. Erythrocytosis

35. After the traumatic tooth extraction, a patient is complaining of acute, dull,
poorly-localized pain in gingiva, body temperature rises up to 37,5 oC. The
patient has been diagnosed with alveolitis. Specify the kind of pain in this
patient:
A. Protopathic
B. Epicritic
C. Visceral
D. Heterotopic
E. Phantom

36. A 50-year-old patient suffers from essential hypertension. After a physical


stress he experienced muscle weakness, breathlessness, cyanosis of lips, skin
and face. Respiration was accompanied by distinctly heard bubbling rales.
What mechanism underlies the development of this syndrome?
A. Acute left-ventricular failure
B. Chronic right-ventricular failure
C. Chronic left-ventricular failure
D. Collapse
E. Cardiac tamponade

37. A patient with obliterating endarteritis underwent ganglionic sympathectomy.


What type of arterial hyperaemia should have developed as a result of the
surgery?
A. Neuroparalytic
B. Neurotonic
C. Metabolic
D. Functional
E. Reactive

38. A group of mountain climbers went through the blood analysis at the height of
3000 m. It revealed decrease of HCO 3 to 15 micromoles/l (standard is 22-26
micromole/l). What is the mechanism of HCO 3 decrease?
A. Hyperventilation
B. Intensification of acidogenesis
C. Hypoventilation
D. Decrease of ammoniogenesis
E. Decrease of bicarbonate reabsorption in kidneys

39. ECG of a patient shows such alterations: P-wave is normal, P-Q-interval is


short, ventricular QRST complex is wide, R-wave is double-peak or two-phase.
What form of arrhythmia is it?
A. WPW syndrome (Wolff-Parkinson-White)
B. Frederick's syndrome (atrial flutter)
C. Atrioventricular block
D. Ventricular fibrillation
E. Ciliary arrhythmia

40. A patient who suffers from pneumonia has high body temperature. What
biologically active substance plays the leading part in origin of this
phenomenon?
A. Interleukin-I
B. Histamine
C. Bradykinin
D. Serotonin
E. Leukotrienes

41. A 34-year-old woman was diagnosed with hereditary microspherocytic


hemolytic anemia (Minkowsky-Shauffard disease). What mechanism caused
haemolysis of erythrocytes?
A. Membranopathy
B. Enzymopathy
C. Hemoglobinopathy
D. Autoimmune disorder
E. Bone marrow hypoploasia

42. 2 years ago a patient underwent resection of pyloric part of stomach. He


complains of weakness, periodical dark shadows beneath his eyes, dyspnea. In
blood: Hb - 70 g/l, erythrocytes - 3, 0* 1012/l, colour index - 0,7. What changes
of erythrocytes in blood smears are the most typical for this condition?
A. Microcytes
B. Megalocytes
C. Schizocytes
D. Ovalocytes
E. Macrocytes

43. During a prophylactic medical examination, a 7-year-old boy was diagnosed


with daltonism. His parents are healthy and have normal colour vision, but his
grandfather on his mother’s side has the same abnormality. What is the type
of the abnormality inheritance?
A. Recessive, sex-linked
B. Dominant, sex-linked
C. Semidominance
D. Autosomal recessive
E. Autosomal dominant

44. A 27 y.o. patient put eye drops that contain penicillin. After a few minutes she
felt itching and burning of her body, there appeared lip and eye-lid edemata;
arterial pressure began to drop. What immunoglobulins took part in the
development of this allergic reaction?
A. lgE and lgG
B. IgM and IgG
C. IgA and IgM
D. IgM and IgD
E. IgG and IgD
45. Inflammation of a patient's eye was accompanied by accumulation of turbid
liquid with high protein at the bottom of anterior chamber that was called
hypopyon. What process underlies the changes under observation?
A. Disturbance of microcirculation
B. Primary alteration
C. Secondary alteration
D. Proliferation
E. -

46. A 48 y.o. patient was admitted to the hospital with complaints about weakness,
irritability, and sleep disturbance. Objectively: skin and scleras are yellow. In
blood: conjugated bilirubin, cholalemia. Feces are acholic. Urine is of dark
colour (bilirubin). What jaundice is it?
A. Mechanic
B. Hemolytic
C. Parenchymatous
D. Gilbert's syndrome
E. Crigler-Najjar syndrome

47. A 41-year-old male patient has a history of recurrent attacks of heartbeats


(paroxysms), profuse sweating, and headaches. Examination revealed
hypertension, hyperglycemia, increased basal metabolic rate, and tachycardia.
These clinical presentations are typical for the following adrenal pathology:
A. Hyperfunction of the medulla
B. Hypofunction of the medulla
C. Hyperfunction of the adrenal cortex
D. Hypofunction of the adrenal cortex
E. Primary aldosteronism

48. A patient who suffers from severe disorder of water-salt metabolism


experienced cardiac arrest in diastole. What is the most probable mechanism
of cardiac arrest in diastole?
A. Hyperkaliemia
B. Hypernatremia
C. Organism dehydratation
D. Hypokaliemia
E. Hyponatremia

49. A patient who suffers from heart failure has enlarged liver, edemata of lower
extremities, ascites. What is the leading mechanism in the development of this
edema?
A. Hydrodynamic
B. Colloid osmotic
C. Lymphogenous
D. Membranogenic
50. Two weeks after lacunar tonsillitis a 20-year-old man started complaining
about general weakness, lower eyelid edemata. After examination the patient
was diagnosed with acute glomerulonephritis. What are the most likely
pathological changes in the urine formula?
A. Proteinuria
B. Cylindruria
C. Presence of fresh erythrocytes
D. Pyuria
E. Natriuria

51. A 32 y.o. man is tall; he has gynecomastia, adult woman pattern of hair
distribution, high voice, mental deficiency, sterility. Provisional diagnosis is
Klinefelter's syndrome. In order to specify diagnosis, it is necessary to analize:
A. Caryotype
B. Leukogram
C. Spermatogenesis
D. Blood group
E. Genealogy

52. Examination of a miner revealed pulmonary fibrosis accompanied by


disturbance of alveolar ventilation. What is the main mechanism of this
disturbance?
A. Limitation of respiratory surface of lungs
B. Constriction of superior respiratory tracts
C. Disturbance of neural respiration control
D. Limitation of breast mobility
E. Bronchi spasm

53. After transfusion of 200 ml of blood a patient presented with body


temperature rise up to 37,9oC. Which of the following substances is the most
likely cause of temperature rise?
A. Interleukin-1
B. Interleukin-2
C. Tumour necrosis factor
D. Interleukin-3
E. Interleukin-4

54. A 49 y.o. woman consulted a doctor about heightened fatigue and dyspnea
during physical activity. ECG: heart rate is 50/min, PQ is extended, QRS is
unchanged, P wave quanity exceeds quantity of QRS complexes. What type of
arrhythmia does the patient have?
A. Atrioventricular block
B. Extrasystole
C. Sinus bradycardia
D. Ciliary arhythmia
E. Sinoatrial block
55. A woman has been applying a new cosmetic preparation for a week that
resulted in eye-lid inflammation accompanied by hyperemia, infiltration and
painfulness. What type of allergic reaction was developed?
A. ІV
B. I
C. II
D. III
E. V

56. A patient is followed up in an endocrinological dispensary on account of


hyperthyreosis. Weight loss, tachycardia, finger tremor are accompanied by
hypoxia symptoms - headache, fatigue, eye flicker. What mechanism of thyroid
hormones action underlies the development of hypoxia?
A. Disjunction, oxydation and phosphorilation
B. Inhibition of respiratory ferment synthesis
C. Competitive inhibition of respiratory ferments
D. Intensification of respiratory ferment synthesis
E. Specific binding of active centres of respiratory ferments

57. A 56 y.o. patient has been suffering from thyreotoxicosis for a long time. What
type of hypoxia can be developed?
A. Tissue
B. Hemic
C. Circulatory
D. Respiratory
E. Mixed

58. An animal with aortic valve insufficiency got hypertrophy of its left heart
ventricle. Some of its parts have local contractures. What substance
accumulated in the myocardiocytes caused these contractures?
A. Calcium
B. Potassium
C. Lactic acid
D. Carbon dioxide
E. Sodium

59. A girl is diagnosed with adrenogenital syndrome (pseudohermaphroditism).


This pathology was caused by hypersecretion of the following adrenal
hormone:
A. Androgen
B. Estrogen
C. Aldosterone
D. Cortisol
E. Adrenalin
60. A patient has extrasystole. ECG shows no P wave, QRS complex is deformed,
there is a full compensatory pause. What extrasystoles are these?
A. Ventricular
B. Atrial
C. Atrioventricular
D. Sinus
E. -

61. After a serious psychoemotional stress a 48-year-old patient suddenly


developed acute heart ache irradiating to the left arm. Nitroglycerine relieved
pain after 10 minutes. What is the leading pathogenetic mechanism of this
process development?
A. Spasm of coronary arteries
B. Dilatation of peripheral vessels
C. Obstruction of coronary vessels
D. Compression of coronary vessels
E. Increase in myocardial oxygen consumption

62. A 12-year-old teenager has significantly put off weight within 3 months;
glucose concentration rose up to 50 millimole/l. He fell into a coma. What is
the main mechanism of its development?
A. Hyperosmolar
B. Hypoglycemic
C. Ketonemic
D. Lactacidemic
E. Hypoxic

63. A 43-year-old-patient has arterial hypertension caused by an increase in


cardiac output and general peripheral resistance. Specify the variant of
hemodynamic development of arterial hypertension in the given case:
A. Eukinetic
B. Hyperkinetic
C. Hypokinetic
D. Combined

64. As a result of increased permeability of the erythrocyte membrane in a patient


with
microspherocytic anaemia (Minkowsky-Shauffard disease) cells receive
sodium ions and water. Erythrocytes take form of spherocytes and can be
easily broken down. What is the leading mechanism of erythrocyte damage in
this case?
A. Electrolytic osmotic
B. Calcium
C. Acidotic
D. Protein
E. Nucleic
65. Examination of a 52-year-old female patient has revealed a decrease in the
amount of red blood cells and an increase in free hemoglobin in the blood
plasma (hemoglobinemia). Color index is 0.85. What type of anemia is being
observed in the patient?
A. Acquired hemolytic
B. Hereditary hemolytic
C. Acute hemorrhagic
D. Chronic hemorrhagic
E. Anemia due to diminished erythropoiesis

66. A 56-year-old patient suffering from cardiac insufficiency has edema of feet
and shins; edematous skin is pale and cold. What is the leding mechanism of
edema pathogenesis?
A. Rise of hydrostatic pressure in venules
B. Drop of oncotic pessure in capillaries
C. Increase of capillary permeability
D. Disorder of lymph outflow
E. Positive water balance

67. Prophylactic medical examination of a 36-year-old driver revealed that his AP


was 150/90 mm Hg. At the end of working day he usually hears ear noise, feels
slight indisposition that passes after some rest. He was diagnosed with
essential hypertension. What is the leading pathogenetic mechanism in this
case?
A. Neurogenetic
B. Nephric
C. Humoral
D. Endocrinal
E. Reflexogenic

68. Blood analysis of a patient with jaundice reveals conjugated bilirubinemia,


increased concentration of bile acids. There is no stercobilinogen in urine.
What type of jaundice is it?
A. Obstructive jaundice
B. Hepatocellular jaundice
C. Parenchymatous jaundice
D. Hemolytic jaundice
E. Cythemolytic jaundice

69. Violation of safety rules resulted in calomel intoxication. Two days later the
daily diuresis was 620 ml. A patient experienced headache, vomiting,
convulsions, dyspnea, and moist rales in lungs. What pathology is it?
A. Acute renal insufficiency
B. Chronic renal insufficiency
C. Uraemic coma
D. Glomerulonephritis
E. Pyelonephritis

70. A newborn child with pylorostenosis has often repeating vomiting


accompanied by apathy, weakness, hypertonicity, sometimes convulsions.
What disorder form of acid-base balance is it?
A. Nongaseous alkalosis
B. Gaseous alkalosis
C. Gaseous acidosis
D. Metabolic acidosis
E. Excretory acidosis

71. The development of both immune and allergic reactions is based upon the
same mechanisms of the immune system response to an antigen. What is the
main difference between the immune and allergic reactions?
A. Development of tissue lesion
B. Amount of released antigen
C. Antigen structure
D. Routes by which antigens are delivered into the body
E. Hereditary predisposition

72. 24 hours after appendectomy blood of a patient presents neutrophilic


leukocytosis with regenerative shift. What is the most probable mechanism of
leukocytosis development?
A. Amplification of leukopoiesis
B. Redistribution of leukocytes in the organism
C. Decelerated leukocyte destruction
D. Deceleratied emigration of leukocytes to the tissues
E. Amplification of leukopoiesis and decelerated emigration of leukocytes to the
tissues

73. A 59-year-old patient is a plant manager. After the tax inspection of his plant
he felt intense pain behind his breastbone irradiating to his left arm. 15
minutes later his condition came to normal. Which of the possible mechanisms
of stenocardia development is the leading in this case?
A. High catecholamine concentration in blood
B. Coronary atherosclerosis
C. Intravascular aggregation of blood corpuscles
D. Coronary thrombosis
E. Functional heart overload

74. Arterial pressure of a surgeon who performed a long operation rised up to


140/110 mm Hg. What changes of humoral regulation could have caused the
rise of arterial pressure in this case?
A. Activation of sympathoadrenal system
B. Activation of formation and excretion of aldosterone
C. Activation of renin angiotensive system
D. Activation of kallikrein kinin system
E. Inhibition of sympathoadrenal system

75. A patient was ill with burn disease that was complicated by DIC syndrome.
What stage of DIC syndrome can be suspected if it is known that the patient's
blood coagulates in less than 3 minutes?
A. Hypercoagulation
B. Transition phase
C. Hypocoagulation
D. Fibrinolysis
E. Terminal

76. In a patient elevation of body temperature takes turns with drops down to
normal levels during the day. The rise in temperature is observed periodically
once in four days. Specify the type of temperature curve:
A. Febris internuttens
B. Febris continua
C. Febris reccurens
D. Febris hectica
E. Febris remitens

77. A 55 y.o. woman consulted a doctor about having continuous cyclic uterine
hemorrhages for a year, weakness, dizziness. Examination revealed skin
pallor. Hemogram: Hb- 70 g/l, erythrocytes - 3, 2* 101 2/l, color index - 0,6,
leukocytes - 6,0* 109 /l, reticulocytes - 1%; erythrocyte hypochromia. What
anemia is it?
A. Chronic posthemorrhagic anemia
B. Hemolytic anemia
C. Aplastic anemia
D. B12-folate-deficiency anemia
E. Iron-deficiency anemia

78. A 56-year-old patient came to a hospital with complaints about general


weakness, tongue pain and burning, sensation of limb numbness. In the past
he underwent resection of forestomach. In blood: Hb- 80 g/l; erythrocytes - 2,
0* 101 2 /l; colour index - 1, 2, leukocytes - 3,5 * 109 /l. What anemia type is it?
A. B12-folate deficient
B. Hemolytic
C. Posthemorrhagic
D. Aplastic
E. Iron-deficient

79. A 25-year-old Palestinian woman complains of weakness, dizziness, dyspnea.


In anamnesis: periodically exacerbating anemia. In blood: Hb - 60 g/l,
erythrocytes - 2, 5 * 101 2/l, reticulocytes - 35 o/ oo, anisocytosis and poikilocytosis
of erythrocytes, a lot of target cells and polychromatophils. What type of
anemia is it?
A. Thalassemia
B. Sickle-cell anemia
C. Minkowsky-Shauffard disease
D. Addison-Biermer disease
E. Glucose 6-phosphate dehydrogenase-deficient anemia

80.A 23 y.o. patient complains of weakness; temperature rise up to 38-40oC.


Objectively: liver and spleen are enlarged. Hemogram: Hb- 100 g/l,
erythrocytes - 2,9* 101 2/l, leukocytes - 4,4* 109 /l, thrombocytes – 48* 109 /l,
segmentonuclear neutrophils - 17%, lymphocytes - 15%, blast cells - 68%. All
cytochemical reactions are negative. Make a hematological conclusion:
A. Undifferentiated leukosis
B. Chronic myeloleukosis
C. Acute myeloblastic leukosis
D. Acute lymphoblastic leukosis
E. Acute erythromyelosis

81. Diabetic nephropathy with uremia has developed in a patient with pancreatic
diabetes. The velocity of glomerular filtration is 9 ml/min. What mechanism
of a decrease in glomerular filtration velocity and chronic renal failure
development is most likely in the case of this patient?
A. Reduction of active nephron mass
B. Decrease in systemic arterial pressure
C. Obstruction of nephron tubules with hyaline casts
D. Tissue acidosis
E. Arteriolar spasm

82. Inflammatory processes cause synthesis of protein of acute phase in an


organism. What substances stumulate their synthesis?
A. Interleukin-1
B. Immunoglobulins
C. Interferons
D. Biogenic amins
E. Angiotensin

83. A patient is diagnosed with hereditary coagulopathy that is characterised by


factor VIII deficiency. Specify the phase of blood clotting during which
coagulation will be disrupted in the given case:
A. Thromboplastin formation
B. Thrombin formation
C. Fibrin formation
D. Clot retraction
E. -
84. A chemical burn caused esophagus stenosis. Difficulty of ingestion led to the
abrupt loss of weight. In blood: 3, 0 * 101 2/l, Hb - 106 g/l, crude protein - 57 g/l.
What type of starvation is it?
A. Incomplete
B. Proteinic
C. Complete
D. Water
E. Absolute

85. A 42-year-old woman with neuralgia of trifacial nerve complains about


periodical reddening of the right part of her face and neck, sense of warmth
gush, increased skin sensitivity. These effects can be explained by the following
type of arterial hyperemia:
A. Neurotonic
B. Neuroparalytic
C. Metabolic
D. Functional
E. Reactive

86. A patient who suffers from acute myocarditis has clinical signs of cardiogenic
shock. What of the under-mentioned pathogenetic mechanisms plays the main
part in shock development?
A. Disturbance of pumping ability of heart
B. Depositing of blood in organs
C. Reduction of diastolic flow to the heart
D. Decrease of vascular tone
E. Increase of peripheral vascular resistance

87. On the 6th day of treatment a patient with acute renal insufficiency developed
polyuria. Diuresis intensification at the beginning of polyuria stage of acute
renal insufficiency is caused by:
A. Renewal of filtration in nephrons
B. Volume expansion of circulating blood
C. Growth of natriuretic factor
D. Reduction of aldosteron content in plasma
E. Reduction of vasopressin content in plasma

88. A 30-year-old woman has face edemata. Examination revealed proteinuria (5,
87 g/l), hypoproteinemia, dysproteinemia, hyperlipidemia. What condition is
the set of these symptoms typical for?
A. Nephrotic syndrome
B. Nephritic syndrome
C. Chronic pyelonephritis
D. Acute renal failure
E. Chronic renal failure
89. A patient with nephrotic syndrome has massive edemata of his face and limbs.
What is the leading pathogenetic mechanism of edemata development?
A. Drop of oncotic blood pressure
B. Increase of vascular permeability
C. Rise of hydrodynamic blood pressure
D. Lymphostasis
E. Increase of lymph outflow

90. A patient staying in the pulmonological department was diagnosed with


pulmonary emphysema accompanied by reduced elasticity of pulmonary
tissue. What type of respiration is observed?
A. Expiratory dyspnea
B. Inspiratory dyspnea
C. Superficial respiration
D. Infrequent respiration
E. Periodic respiration

91. An unconscious young man with signs of morphine poisoning entered


admission office. His respiration is shallow and infrequent which is caused by
inhibition of respiratory centre. What type of respiratory failure is it?
A. Ventilative dysregulatory
B. Ventilative obstructive
C. Ventilative restrictive
D. Perfusive
E. Diffusive

92. A patient complaining of pain in the left shoulder-blade region has been
diagnosed with miocardial infarction. What kind of pain does the patient have?
A. Radiating
B. Visceral
C. Phantom
D. Protopathic
E. Epicritic

93. A patient has been diagnosed with acute glomerulonephritis that developed
after he had had streptococcal infection. It is most likely that the affection of
basal glomerular membrane is caused by an allergic reaction of the following
type:
A. Immune complex
B. Anaphylactic
C. Cytotoxic
D. Delayed
E. Stimulating

94. Examination of patients with periodontitis revealed the interdependence


between the rate of affection of periodontal tissues and the amount of
lysozymes in saliva and gingival liquid. These results can be obtained during
studying the following protection system of an organism:
A. Non-specific resistance
B. Humoral immunity
C. Cellular immunity
D. Autoresponsiveness
E. Tolerance

95. A 62-year-old patient who previously worked as stoker was admitted to a


hospital with complaints about general weakness, abrupt weight loss, hoarse
voice, dyspnea, dry cough. Laryngoscopy revealed a tumour in the pharynx
that invaded vocal cords and epiglottis. What is the most probable cause of
tumour development?
A. Polycyclic aromatic carbohydrates
B. Nitrosamines
C. Aromatic amines and amides
D. Retroviruses
E. Ionizing radiation

96. A married couple came to the genetic counseling. The husband suffers from
the insulin-dependent diabetes; the wife is healthy. What is the probability that
this couple will have an insulin-dependent child?
A. Higher than throughout the population
B. The same as throughout the population
C. Lower than throughout the population
D. 100%
E. 50%

97. A patient, who had been working hard under conditions of elevated
temperature of the environment, has now a changed quantity of blood plasma
proteins. What penomenon is the case?
A. Relative hyperproteinemia
B. Absolute hyperproteinemia
C. Absolute hypoproteinemia
D. Disproteinemia
E. Paraproteinemia

98. An experimental rat with extremity paralysis has no tendon and cutaneous
reflexes, muscle tone is decreased, but muscles of the affected extremity
maintain their ability to react with excitation to the direct action of continious
current. What type of paralysis is it?
A. Flaccid peripheral
B. Flaccid central
C. Spastic peripheral
D. Spastic central
E. Extrapyramidal
99. A 28-year-old man had a gunshot wound of shin that resulted in an ulcer from
the side of the injury. What is the main factor of neurodystrophy pathogenesis
in this case?
A. Traumatization of peripheral nerve
B. Psychical stress
C. Microcirculation disturbance
D. Infection
E. Tissue damage

100. Blood test of a patient suffering from atrophic gastritis gave the following
results: RBCs - 2,0* 101 2/l, Hb- 87 g/l, colour index - 1,3, WBCs - 4,0* 109 /l,
thrombocytes - 180* 109 /l. Anaemia migh have been caused by the following
substance deficiency:
A. Vitamin B12
B. Vitamin A
C. Vitamin K
D. Iron
E. Zinc

101. A 45-year-old patient was admitted to the cardiological department.


ECG data: negative P wave overlaps QRS complex, diastolic interval is
prolonged after extrasystole. What type of extrasystole is it?
A. Atrioventricular
B. Sinus
C. Atrial
D. Ventricular
E. Bundle-branch

102. A 37-year-old female patient complains of headache, vertigo, troubled


sleep, numbness of limbs. For the last 6 years she has been working at the gas-
discharge lamp-producing factory in the lead-processing shop. Blood test
findings: low hemoglobin and RBC level, serum iron concentration exceeds the
norm by several times. Specify the type of anemia:
A. Iron refractory anemia
B. Iron-deficiency anemia
C. Minkowsky-Shauffard disease
D. Hypoplastic anemia
E. Metaplastic anemia

103. A rabbit's nerve that innervates the right ear was cut and its right
superior cervical ganglion was removed. Immediately after operation the
temperature of ear skin was measured. It was revealed that the temperature of
the rabbit's ear skin on the side of denervation was by 1,5 oC higher than on the
opposite intact side. What of the following is the most probable explanation of
the above-mentioned effects?
A. Arterial neuroparalytic hyperemia
B. Arterial neurotopical hyperemia
C. Atrerial hyperemia induced by metabolic factors
D. Reactive arterial hyperemia
E. Physiological arterial hyperemia

104. Two hours after an exam a student had a blood count done and it was
revealed that he had leukocytosis without significant leukogram modifications.
What is the most probable mechanism of leukocytosis development?
A. Redistribution of leukocytes in the organism
B. Leukopoiesis intensification
C. Deceleration of leukocyte lysis
D. Deceleration of leukocyte migration to the tissues
E. Leukopoiesis intensification and deceleration of leukocyte lysis

105. A patient present with icteritiousness of skin, scleras and mucous


membranes. Blood plasma the total bilirubin is increased, stercobilin is
increased in feces, urobilin is increased in urine. What type of jaundice is it?
A. Haemolytic
B. Gilbert's disease
C. Parenchymatous
D. Obturational
E. Cholestatic

106. Hepatitis has led to the development of hepatic failure. Mechanism of


edemata formation is activated by the impairment of the following liver
function:
A. Protein-synthetic
B. Barrier
C. Chologenetic
D. Antitoxic
E. Glycogen-synthetic

107. As a result of a road accident a 26-year-old man is in the torpid phase of


shock. Blood count: leukocytes - 3, 2* 109 /l. What is the leading mechanism of
leukopenia development?
A. Leukocyte redistribution in the bloodstream
B. Leukopoiesis inhibition
C. Faulty release of mature leukocytes from the bone marrow into the blood
D. Leukocyte destruction in the hematopietic organs
E. Increased excretion of the leukocytes from the organism

108. As a result of a trauma a patient has developed traumatic shock that led
to the following disorders: AP is 140/90 mm Hg, Ps is 120 bpm. The patient is
fussy, talkative, and pale. Such state relates to the following shock phase:
A. Erectile
B. Latent period
C. Terminal
D. Torpid
E. -

109. Examination of a patient admitted to the surgical department with


symptoms of acute appendicitis revealed the following changes in the white
blood cells: the total count of leukocytes is 16 * 109 /l. Leukocyte formula:
basophils - 0, eosinophils - 2%, juvenile forms - 2%, stabnuclear - 8%,
segmentonuclear - 59%, lymphocytes - 25%, monocytes- 4%. The described
changes can be classified as:
A. Neutrophilia with regenerative left shift
B. Neutrophilia with right shift
C. Neutrophilia with degenerative left shift
D. Neutrophilic leukemoid reaction
E. Neutrophilia with hyperregenerative left shift

110. A patient being treated for viral hepatitis type B got symptoms of hepatic
insufficiency. What blood changes indicative of protein metabolism disorder
will be observed in this case?
A. Absolute hypoalbuminemia
B. Absolute hyperalbuminemia
C. Absolute hyperfibrinogenemia
D. Proteinic blood composition is unchanged
E. Absolute hyperglobulinemia

111. A patient was stung by a bee. Examination revealed that his left hand
was hot, pink, edematic, there was a big red blister on the site of sting. What is
the leading mechanism of edema development?
A. Increased vessel permeability
B. Reduced vessel filling
C. Injury of vessels caused by the sting
D. Drop of oncotic pressure in tissue
E. Drop of osmotic pressure in tissue

112. Amniocentesis revealed two sex chromatin bodies (Barr bodies) in each
cell of the sample. What disease is this character typical for?
A. Trisomy X
B. Klinefelter syndrome
C. Turner's syndrome
D. Down's syndrome
E. Patau syndrome

113. A patient consulted a dentist about itching and burning in the oral cavity;
high temperature. The patient was diagnosed with trichomonal
gingivostomatitis. What drug should be chosen for his treatment?
A. Metronidazole
B. Ampicillin
C. Doxycycline hydrochloride
D. Gentamicin sulfate
E. Nystatin

114. A patient suffering from pheochromocytoma complains of thirst, dry


mouth, hunger. Blood test for sugar revealed hyperglycemia. What type of
hyperglycemia is it?
A. Adrenal
B. Hypercorticoid
C. Alimentary
D. Somatotropic
E. Hypoinsulinemic

115. A patient suffering from stenocardia was taking nitroglycerine which


caused restoration of blood supply of myocardium and relieved pain in the
cardiac area. What intracellular mechanism provides restoration of energy
supply of insulted cells?
A. Intensification of ATP resynthesis
B. Reduction of ATP resynthesis
C. Increased permeability of membranes
D. Intensification of oxygen transporting into the cell
E. Intensification of RNA generation

116. A patient is diagnosed with iron-deficiency sideroachrestic anemia,


progression of which is characterised by skin hyperpigmentation, pigmentary
cirrhosis, and heart and pancreas affection. Iron level in the blood serum is
increased. What disorder of iron metabolism causes this disease?
A. Failure to assimilate iron leading to iron accumulation in tissues
B. Excessive iron intake with food
C. Disorder of iron absorption in bowels
D. Increased iron assimilation by body

117. A couple had a child with Down's disease. Mother is 42 years old. This
disease is most probably caused by the following impairment of prenatal
development:
A. Gametopathy
B. Blastopathy
C. Embryopathy
D. Non-specific fetopathy
E. Specific fetopathy

118. There are several groups of molecular mechanisms playing important


part in pathogenesis of insult to cells which contributes to the pathology
development. What processes are stimulated by proteinic damage
mechanisms?
A. Enzyme inhibition
B. Lipid peroxidation
C. Phospholipase activation
D. Osmotic membrane distension
E. Acidosis

119. A child was born with cleft palate. Examination revealed aorta defects
and reduced number of T-lymphocytes in blood. What immunodeficient
syndrome is it?
A. DiGeorge
B. Wiskott-Aldrich
C. Chediak-Higashi
D. Louis-Bar
E. Swiss-type

120. Examination of a child who frequently suffers from infectious diseases


revealed that IgG concentration in blood serum was 10 times less than normal;
IgA and IgM concentration was also significantly reduced. Analysis showed
also lack of B-lymphocytes and plasmocytes. What disease are these symptoms
typical for?
A. Bruton's disease
B. Swiss-type agammaglobulinemia
C. Dysimmunoglobulinemia
D. Louis-Bar syndrome
E. Di George syndrome

121. A driver who got a trauma in a road accident and is shocked has
reduction of daily urinary output down to 300 ml. What is the main
pathogenetic factor of such diuresis change?
A. Drop of arterial pressure
B. Drop of oncotic blood pressure
C. Increased vascular permeability
D. Decreased number of functioning glomerules
E. Secondary hyperaldosteronism

122. Examination of a 42-year-old patient revealed a tumour of


adenohypophysis. Objectively: the patient's weight is 117 kg; he has moon-like
hyperemic face, red-blue striae of skin distension on his belly. Osteoporosis
and muscle dystrophy are present. AP is 210/140 mm Hg. What is the most
probable diagnosis?
A. Cushing's disease
B. Cushing's syndrome
C. Conn's disease
D. Diabetes mellitus
E. Essential hypertension
123. Examination of a 12-year-old boy with developmental lag revealed
achondroplasia: disproportional constitution with evident shortening of upper
and lower limbs as a result of growth disorder of epiphyseal cartilages of long
tubal bones. This disease is:
A. Inherited, dominant
B. Inherited, recessive
C. Inherited, sex-linked
D. Congenital
E. Acquired

124. A patient was diagnosed with autoimmune hemolitic cytotoxic anemia.


What substances are antigens in II type allergic reactions?
A. Modified receptors of cell membranes
B. Antibiotics
C. Hormones
D. Serum proteins
E. Inflammation modulators

125. A patient suffers from the haemorrhagic syndrome that shows itself in
frequent nasal bleedings, posttraumatic and spontaneous intracutaneous and
intra-articular haemorrhages. After a laboratory study a patient was diagnosed
with the type B haemophilia. This disease is provoked by the deficit of the
following factor of blood coagulation:
A. IX
B. VIII
C. XI
D. V
E. VII

126. A 58-year-old patient suffers from the cerebral atherosclerosis.


Examination revealed hyperlipoidemia. What class of lipoproteins will most
probably show increase in concentration in this patient’s blood serum?
A. Low-density lipoproteins
B. High-density lipoproteins
C. Fatty acid complexes with albumins
D. Chylomicrons
E. Cholesterol

127. A 67-year-old patient complains of periodic heart ache, dyspnea during


light physical activities. ECG reveals extraordinary contractions of heart
ventricles. Such arrhythmia is called:
A. Extrasystole
B. Bradycardia
C. Tachycardia
D. Flutter
E. Fibrillation
128. In course of a preventive examination of a miner a doctor revealed
changes of cardiovascular fitness which was indicative of cardiac insufficiency
at the compensation stage. What is the main proof of cardiac compensation?
A. Myocardium hypertrophy
B. Tachycardia
C. Rise of arterial pressure
D. Dyspnea
E. Cyanosis

129. A 47-year-old man with myocardium infarction was admitted to the


cardiological department. What changes of cellular composition of peripheral
blood are induced by necrotic changes in the myocardium?
A. Neutrophilic leukocytosis
B. Monocytosis
C. Eosinophilic leukocytosis
D. Thrombocytopenia
E. Lymphopenia

130. An infectious disease unit admitted a patient with signs of jaundice


caused by hepatitis virus. Select an indicator that is specific only for
parenchymatous jaundice:
A. Increase in ALT and AST rate
B. Hyperbilirubinemia
C. Bilirubinuria
D. Cholaemia
E. Urobilinuria

131. A patient with skin mycosis has disorder of cellular immunity. The most
typical characteristic of it is reduction of the following index:
A. T-lymphocytes
B. Immunoglobulin G
C. Immunoglobulin E
D. B-lymphocytes
E. Plasmocytes

132. A patient with massive burns developed acute renal insufficiency


characterized by a significant and rapid deceleration of glomerular filtration.
What is the mechanism of its development?
A. Reduction of renal blood flow
B. Damage of glomerular filter
C. Reduction of functioning nephron number
D. Rise of pressure of tubular fluid
E. Renal artery embolism

133. A child is pale, pastose, muscular tissue is bad developed, and lymph
nodes are enlarged. He often suffers from angina and pharyngitis; blood has
signs of lymphocytosis. The child is also predisposed to autoallergic diseases.
What type of diathesis can be presumed in this case?
A. Lymphohypoplastic
B. Exudative
C. Gouty
D. Asthenic
E. Hemorrhagic

134. A 26-year-old pregnant woman is under treatment at an in-patient


hospital. After a continuous attack of vomiting she was found to have reduced
volume of circulating blood. What kind of change in general blood volume is
the case?
A. Polycythemic hypovolemia
B. Simple hypovolemia
C. Oligocythemic hypovolemia
D. Polycythemic hypervolemia
E. Oligocythemic hypervolemia

135. Parents of a 3-year-old child have been giving him antibiotics with
purpose of preventing enteric infections for a long time. A month later the
child's condition changed for the worse. Blood examination revealed apparent
leukopenia and granulocytopenia. What is the most probable mechanism of
blood changes?
A. Myelotoxic
B. Autoimmune
C. Redistributive
D. Age-specific
E. Hemolytic

136. A patient ill with enteritis accompanied by massive diarrhea has low
water rate in the extracellular space, high water rate inside the cells and low
blood osmolarity. What is such disturbance of water-electrolytic metabolism
called?
A. Hypo-osmolar hypohydration
B. Hyperosmolar hypohydration
C. Osmolar hypohydration
D. Hypo-osmolar hyperhydration
E. Hyperosmolar hyperhydration

137. A patient with obliterating atherosclerosis underwent sympathectomy of


femoral artery in the region of femoral trigone. What type of arterial hyperemia
was induced by the operation?
A. Neuroparalytic
B. Reactive
C. Metabolic
D. Neurotonic
E. Functional

138. A 15-year-old girl has pale skin, glossitis, and gingivitis. Blood count:
erythrocytes - 3,3* 101 2 /l, hemoglobin - 70 g/l, colour index - 0,5. Examination
of blood smear revealed hypochromia, microcytosis, poikilocytosis. What type
of anemia is it?
A. Iron-deficient
B. B1 2 -folic acid-deficient
C. Sickle-cell
D. Hemolytic
E. Thalassemia

139. A 70-year-old man is ill with vascular atherosclerosis of lower


extremities and coronary heart disease. Examination revealed disturbance of
lipidic blood composition. The main factor of atherosclerosis pathogenesis is
the excess of the following lipoproteins:
A. Low-density lipoproteins
B. Cholesterol
C. High-density lipoproteins
D. Intermediate density lipoproteins
E. Chylomicrons

140. A patient ill with essential arterial hypertension had a hypertensic crisis
that resulted in an attack of cardiac asthma. What is the leading mechanism of
cardiac insufficiency in this case?
A. Heart overload caused by high pressure
B. Heart overload caused by increased blood volume
C. Absolute coronary insufficiency
D. Myocardium damage
E. Blood supply disturbance

141. A 5-year-old child is ill with measles. Blood analysis revealed increase of
total number of leukocytes up to 13 * 109 /l. Leukogram: basophils - 0,
eosinophils - 1, myelocytes - 0, juvenile neutrophils - 0, band neutrophils - 2,
segmented neutrophils - 41, lymphocytes - 28, monocytes - 28. Name this
phenomenon:
A. Monocytosis
B. Agranulocytosis
C. Lymphocytosis
D. Eosinopenia
E. Neutropenia

142. Rats being under stress have muscular hypertonia and high arterial
pressure, high glucose concentration in blood and intensified secretion of
corticotropin and corticosteroids. In what stress phase are these animals?
A. Antishock phase
B. Exhaustion
C. Shock phase
D. Erectile
E. Terminal

143. After taking poor-quality food a patient developed repeated episodes of


diarrhea. On the next day he presented with decreased arterial pressure,
tachycardia, extrasystole. Blood pH is 7,18. These abnormalities were caused
by the development of:
A. Nongaseous acidosis
B. Gaseous acidosis
C. Nongaseous alkalosis
D. Gaseous alkalosis
E. Metabolic alkalosis

144. A patient was admitted to the infectious department. His symptoms: dry
skin, decreased skin turgor, rice-water stool. The patient was diagnosed with
cholera. What disorder of water-electrolytic balance is most often observed in
this disease?
A. Isoosmotic hypohydration
B. Hyperosmotic hyperhydration
C. Hypoosmotic hypohydration
D. Hyperosmotic hypohydration
E. Hypoosmotic hyperhydration

145. A 45-year-old woman is ill with breast cancer. Her left arm has
symptoms of lymphatic system insufficiency - limb edema, lymph node
enlargement. What form of lymphatic circulation insufficiency is it?
A. Mechanic insufficiency
B. Dynamic insufficiency
C. Resorption insufficiency
D. Combined insufficiency

146. A 25-year-old man has spent a long time in the sun under high air
humidity. As a result of it his body temperature rose up to 39 oC. What
pathological process is it?
A. Hyperthermia
B. Infectious fever
C. Hypothermia
D. Noninfectious fever
E. Burn disease

147. A 26-year-old man is in the torpid shock phase as a result of a car


accident. In blood: 3, 2* 109 /l. What is the leading mechanism of leukopenia
development?
A. Redistribution of leukocytes in bloodstream
B. Leikopoiesis inhibition
C. Disturbed going out of mature leukocytes from the marrow into the blood
D. Lysis of leukocytes in the blood-forming organs
E. Intensified elimination of leukocytes from the organism

148. Since a patient has had myocardial infarction, his atria and ventricles
contract independently from each other with a frequency of 60-70 and 35-40
per minute. Specify the type of heart block in this case:
A. Complete atrioventricular
B. Partial atrioventricular
C. Sino-atrial
D. Intra-atrial
E. Intraventricular

149. A 54-year-old female was brought to the casualty department after a car
accident. A traumatologist diagnosed her with multiple fractures of the lower
extremities. What kind of embolism is most likely to develop in this case?
A. Fat
B. Tissue
C. Thromboembolism
D. Gaseous
E. Air

150. A 53-year-old male patient complains of acute pain in the right


hypochondrium. Objective examination revealed scleral icterus. Laboratory
tests revealed increased ALT activity, and stercobilin was not detected in the
stool. What disease is characterized by these symptoms?
A. Cholelithiasis
B. Hemolytic jaundice
C. Hepatitis
D. Chronic colitis
E. Chronic gastritis
Krok 1 – 2014 Pharmacology Base
1. A 30-year-old patient complains about having abdominal pain and diarrhea
for five days; body temperature rises up to 37,5 oC along with chills. The day
before a patient had been in a forest and drunk from an open water reservoir.
Laboratory analyses enabled to make the following diagnosis: amebic
dysentery. What is the drug of choice for its treatment?
A. Metronidazole
B. Furazolidonum
C. Levomycetin
D. Phthalazol
E. Emetine hydrochloride

2. A woman works as railway traffic controller. She suffers from seasonal


vasomotor rhinitis and gets treatment in the outpatient setting. She was
prescribed an antihistamine that has no effect upon central nervous system.
What drug is it?
A. Loratadine
B. Dimedrol
C. Promethazine
D. Suprastin
E. Tavegil

3. During an operation a patient got injection of muscle relaxant dithylinum.


Relaxation of skeletal muscles and inhibition of respiration lasted two hours.
This condition was caused by absence of the following enzyme in blood
serum:
A. Butyrylcholin esterase
B. Catalase
C. Acetylcholinesterase
D. Glucose 6-phosphatase
E. Glutathione peroxidase

4. A 45-year-old woman suffers from allergic seasonal coryza caused by the


ambrosia blossoming. What medicine from the stabilizer of the adipose cells
group can be used for prevention of this disease?
A. Ketotifen
B. Diazoline
C. Phencarol
D. Tavegyl
E. Dimedrol

5. Systemic amebiasis with involvment of intestines, liver, lungs were diagnosed


in a 52-year-old patient. What drug should be prescribed?
A. Metronidasol
B. Quiniofone
C. Tetracycline
D. Quingamine
E. Enteroseptol

6. A 38-year-old man who poisoned himself with mercury dichloride was taken
to the admission room in grave condition. What antidote should be
immediately introduced?
A. Unithiol
B. Dipiroxim
C. Atropine
D. Nalorphine
E. Isonitrosine

7. A patient who suffers from insomnia caused by emotional disorder was


prescribed a hypnotic drug with tranquillizing effect. What hypnotic was
prescribed?
A. Nitrazepam
B. Phenobarbital
C. Chloral hydrate
D. Sodium ethaminal
E. Bromisoval

8. A patient had to go through an operation. Doctors introduced him dithylinum


(listenone) and performed intubation. After the end of operation and
cessation of anesthesia the independent respiration wasn't restored. Which
enzyme deficit prolongs the action of muscle relaxant?
A. Pseudocholinesterase
B. Succinate dehydrogenase
C. Carbanhydrase
D. N-acetyltransferase
E. K-Na-adenosine triphosphatase

9. The alternate usage of dichlotiazide, etacrin acid and lasex didn't cause
marked diuretic effect in patient with marked peripheral edema. Increased
amount of aldosterone is in the blood. Indicate the medicine to be prescribed.
A. Spironolacton
B. Mannit
C. Clopamid
D. Urea
E. Amilorid

10. A 56-year-old patient with complains of thirst and frequent urination was
diagnosed to have diabete mellitus and butamin was prescribed. What is the
mechanism of action of this medicine?
A. It stimulates beta-cells of Langergans’ islets
B. It helps to absorb the glucose by the cells of the organism tissues
C. It relieves transport of glucose through the cells' membranes
D. It inhibits alpha cells of Langergans’ islets
E. It inhibits absorption of glucose in the intestines
11. A 37-year-old patient suffering from obliterating vascular endarteritis of
lower limbs takes daily 60 micrograms/kilogram of phenylin. Because of
presentations of convulsive disorder (craniocerebral trauma in anamnesis) he
was prescribed phenobarbital. Withholding this drug caused nasal
hemorhage. What is this complication connected with?
A. Induction of enzymes of microsomal oxidation in liver caused by
phenobarbital
B. Aliphatic hydroxylation of phenobarbital
C. Conjugation of phenylin with glucuronic acid
D. Oxidative deamination of phenylin
E. Inhibition of microsomal oxidation in liver caused by phenobarbital

12. Patient with complaints of dryness in the mouth, photophobia and vision
violation was admitted to the reception-room. Skin is hyperemic, dry, pupils
are dilated, tachycardia. Poisoning with belladonna alkaloids was diagnosed
on further examination. What medicine should be prescribed?
A. Prozerin
B. Diazepam
C. Pilocarpine
D. Armine
E. Dipyroxim

13. Patient complaines of weakness, dyspnea, low extremities oedema.


Diagnosis: chronic cardiac insufficiency. What medicine should be prescribed
first of all?
A. Digitoxin
B. Caffeine
C. Papaverine
D. Propranolol
E. Raunatin

14. Signs of gastropathy develop in the patient with rheumatoid arthritis who was
treated with indometacin. With what activity of the drug can this
complication be connected?
A. Anticyclooxygenase
B. Antiserotonin
C. Antihistamine
D. Antikinine
E. Local irritating

15. Testosterone and its analogs increase the mass of skeletal muscles that allows
to use them for treatment of dystrophy. Due to interaction of the hormon with
what cell substance is this action caused?
A. Nuclear receptors
B. Membrane receptors
C. Ribosomes
D. Chromatin
E. Proteins- activators of transcription
16. A patient ill with neurodermatitis has been taking prednisolone for a long
time. Examination revealed high rate of sugar in his blood. This complication
is caused by the drug influence upon the following link of carbohydrate
metabolism:
A. Gluconeogenesis activation
B. Glycogenogenesis activation
C. Intensification of glucose absorption in the bowels
D. Inhibition of glycogen synthesis
E. Activation of insulin decomposition

17. Proserin increases skeletal muscle tone when given systematically. Halothane
induces relaxation of skeletal muscles and reduces proserin effects. What is
the nature of proserin and halothane interaction?
A. Indirect functional antagonism
B. Direct functional antagonism
C. Competitive antagonism
D. Independent antagonism
E. Noncompetitive antagonism

18. A patient with frequent attacks of stenocardia was prescribed sustak-forte to


be taken one tablet twice a day. At first the effect was positive but on the
second day stenocardia attacks resumed. What can explain inefficiency of the
prescribed drug?
A. Tachyphylaxis
B. Cumulation
C. Sensibilization
D. Idiosyncrasy
E. Dependence

19. Analeptical remedy of reflective type from the H-cholinomimetics group was
given to the patient for restoration of breathing after poisoning with carbon
monoxide. What medicine was prescribed to the patient?
A. Lobeline hydrochloride
B. Atropine sulphate
C. Adrenalin hydrochloride
D. Mesaton
E. Pentamin

20. An aged patient complains of headache, dizziness, quick tiredness, worsening


of memory. Anamnesis: craniocerebral injury. Medicine of what group should
be prescribed?
A. Nootropics
B. Somnific
C. Neuroleptics
D. Analgetics
E. Sedatives
21. The patient was treated medically for psychosis for 2 weeks. Patient's
condition improved but rigidity, tremor, hypokinesia developed. Which of the
drugs can cause such complications?
A. Aminazine
B. Diphenine
C. Sydnocarb
D. Imipramine
E. Chlordiazepoxide

22. Gonorrhoea was revealed in the patient on bacterioscopy of the smear from
urethra. Taking into account that medecines for gonorrhoea are
fluorquinolones, patient should be prescribed:
A. Ciprofloxacin
B. Furazolidone
C. Fluorouracil
D. Urosulfan
E. Cefazoline

23. Patient with bronchial asthma was taking tablets which caused insomnia,
headache, increased blood pressure. What medecine can cause such
complications?
A. Ephedrine
B. Adrenaline
C. Chromolin sodium
D. Euphyline
E. Izadrine

24. Diuretic drug was prescribed to the patient with hypertension in the course
of complex treatment. In a few days BP decreased but signs of hypokaliemia
developed. What drug could cause such complications?
A. Lasix
B. Spironolactone
C. Clophelin
D. Triamterene
E. Enalapril

25. Ascarid eggs have been detected during stool analysis. What drug should be
prescribed?
A. Mebendazole
B. Nystatin
C. Chloramphenicol
D. Tetracycline
E. Furazolidone

26. A patient suffering from myasthenia has been administered proserin. After
its administration the patient has got nausea, diarrhea, twitch of tongue and
skeletal muscles. What drug would help to eliminate the intoxication?
A. Atropine sulfate
B. Physostigmine
C. Pyridostigmine bromide
D. Isadrine
E. Mesatonum

27. A patient who had myocardial infarction was administered 75 mg of


acetylsalicinic acid a day. What is the purpose of this administration?
A. Reduction of thrombocyte aggregation
B. Inflammation reduction
C. Pain relief
D. Temperature reduction
E. Coronary vessel dilatation

28. A patient suffering from chronic hyperacidic gastritis takes an antacid drug
for heartburn elimination. After its ingestion the patient feels better but at the
same time he has a sensation of stomach swelling. Which of the following
drugs might be the cause of such side effect?
A. Sodium hydrocarbonate
B. Magnesium oxide
C. Magnesium trisilicate
D. Aluminium hydrooxide
E. Pepsin

29. A patient consulted a doctor about bowels disfunction. The doctor established
symptoms of duodenitis and enteritis. Laboratory examination helped to
make the following diagnosis: lambliosis. What medication should be
administered?
A. Metronidazole
B. Erythromycin
C. Monomycin
D. Chingamin
E. Tetracycline

30. Introduction of a pharmaceutical substance to an experimental animal


resulted in reduction of salivation, pupil mydriasis. Next intravenous
introduction of acetylcholine didn't lead to any significant changes of heart
rate. Name this substance:
A. Atropine
B. Adrenaline
C. Propranolol
D. Proserin
E. Salbutamol

31. Continious taking of a drug can result in osteoporosis, erosion of stomach


mucous membrane, hypokaliemia, retention of sodium and water, reduced
content of corticotropin in blood. Name this drug:
A. Prednisolone
B. Hydrochlorothiazide
C. Digoxin
D. Indometacin
E. Reserpine

32. A 63 y.o. man with collapse symptoms was delivered to the emergency
hospital. A doctor chose noradrenaline in order to prevent hypotension. What
is the action mechanism of this medication?
A. Activation of α1 -adrenoreceptors
B. Activation of serotonin receptors
C. Activation of β-adrenoreceptors
D. Activation of dopamine receptors
E. Block of M-cholinoreceptors

33. A patient suffering from syphilis has been treated with bismuth preparations.
As a result of it some grey spots turned up on the mucous membrane of the
oral cavity; nephropathy symptoms were also present. What drug should be
used for treatment of bismuth intoxication?
A. Unithiol
B. Methylene blue
C. Naloxone
D. Bemegride
E. Nalorphine

34. A 65-year-old patient with chronic heart failure has been taking digitoxin in
self-administered dosages for a long time. She was admitted to the hospital
for general health aggravation, arrhythmia, nausea, reduced diuresis,
insomnia. What is the primary action to be taken?
A. To withhold digitoxin
B. To reduce digitoxin dosage
C. To administer strophanthine intravenously
D. To administer digoxin
E. To give an intravenous injection of calcium gluconate solution

35. A 20-year-old patient complains of morbid thirst and huperdiuresis (up to 10


l daily). Glucose concentration in blood is normal but it is absent in urine. The
patient has been diagnosed with diabetes insipidus. What hormonal drug is
the most appropriate for management of this disorder?
A. Vasopressin
B. Cortisol
C. Thyroxin
D. Oxytocin
E. Insulin

36. A patient has myocardial infarction with thrombosis of the left coronary
artery. What pharmocological preparation group should be used to
reestablish blood flow?
A. Fibrinolysis activators
B. Narcotic analgesics
C. beta-adrenergic blockers
D. Angiotensin-converting enzyme inhibitors
E. Glucocorticoids

37. A patient has a slowly healing fracture. What medicine can be used to
accelerate formation of connective tissue matrix?
A. Methyluracil
B. Prednisolone
C. Cyclophosphan
D. Methotrexate
E. Cyclosporine

38. A 4-year-old child was admitted to the orthopaedic department with shin
fracture together with displacement. Bone fragments reposition requires
preliminary analgesia. What preparation should be chosen?
A. Promedol
B. Analgin
C. Morphine hydrochloride
D. Panadol

39. A patient with chronic heart failure with edema has increased level of blood
aldosterone. What diuretic would be most effective in this case?
A. Spironolactone
B. Triamterene
C. Acetazolamide
D. Hydrochlorothiazide
E. Furosemide

40. A patient was diagnosed with active focal pulmonary tuberculosis. What drug
should be prescribed in the first place?
A. Isoniazid
B. Sulfalen
C. Cyclocerine
D. Ethionamide
E. Ethoxide

41. What drug will be most appropriate for the patient who has chronic gastritis
with increased secretion?
A. Pirenzepine
B. Pancreatine
C. Pepsin
D. Aprotinin
E. Chlorphentermine

42. A patient taking clonidine for essential hypertension treatment was using
alcohol that caused intense inhibition of central nervous system. What may it
be connected with?
A. Effect potentiating
B. Effect summation
C. Cumulation
D. Intoxication
E. Idiosyncrasy

43. Examination of a patient revealed extremely myotic pupils, sleepiness,


infrequent Chain-Stoke's respiration, urinary retention, slowing-down of
heart rate, enhancement of spinal reflexes. What substance caused the
poisoning?
A. Morphine
B. Atropine
C. Phosphacole
D. Caffeine
E. Barbital

44. A patient suffers from diabetes melitus. After the regular insulin injection his
condition grew worse: there appeared anxiety, cold sweat, tremor of limbs,
general weakness, and dizziness. What preparation can eliminate these
symptoms?
A. Adrenaline hydrochloride
B. Butamide
C. Caffeine
D. Noradrenaline
E. Glibutide

45. Continuous taking of some drugs foregoing the pregnancy increase the risk of
giving birth to a child with genetic defects. What is this effect called?
A. Mutagenic effect
B. Embryotoxic effect
C. Teratogenic effect
D. Fetotoxical effect
E. Blastomogenic effect

46. A doctor administered a patient with allergic dermatitis a H1 -histamine


blocker as a part of complex treatment. Name this medication:
A. Loratadine
B. Cromolyn sodium
C. Prednisolone
D. Adrenaline
E. Hydrocortisone

47. A 19-year-old woman suffers from primary syphilis. Doctor administered her
complex therapy that includes benzylpenicillin sodium salt. What is the
mechanism of action of this drug?
A. It blocks synthesis of peptidoglycan of microbal membrane
B. It blocks synthesis of cytoplasm proteins
C. It blocks thiol enzymes
D. It blocks RNA synthesis
E. It blocks DNA synthesis

48. A 65-year-old female patient suffers from chronic renal insufficiency


accompanied by evident edemata caused by chronic glomerulonephritis.
What diuretic should be administered for forced diuresis?
A. Furosemide
B. Hydrochlorothiazide
C. Chlorthalidone
D. Cyclometazide
E. Acetazolamide

49. A patient suffering from coronary artery disease had taken a certain drug
many times a day in order to arrest stenocardia attacks. Overdose of this drug
finally caused intoxication. Objectively: cyanotic skin and mucous
membranes, dramatic fall in the arterial pressure, tachycardia, and
respiration inhibition. Blood has increased concentration of methemoglobin.
The drug the patient had taken relates to the following group:
A. Organic nitrates
B. α-adrenoceptor blockers
C. Calcium channel blockers
D. Adenosine drugs
E. Myotropic spamolytics

50. A child was born asphyxiated. What drug must be administered to the
newborn to stimulate breathing?
A. Aethimizolum
B. Lobeline
C. Prazosin
D. Atropine
E. Proserine

51. A patient with II stage hypertension has been taking one of hypotensive
medications for the purpose of treatment. After a time, arterial pressure
decreased, but the patient started complaining of flaccidity, sleepiness,
indifference. A bit later he felt stomach pain. He was diagnosed with ulcer.
What hypotensive medication has the patient been taking?
A. Reserpine
B. Dibazole
C. Furosemide
D. Verapamil
E. Captopril

52. A child suffers from drug idiosyncrasy. What is the cause of such reaction?
A. Hereditary enzymopathy
B. Exhaustion of substrate interacting with pharmaceutical substance
C. Accumulation of pharmaceutical substance
D. Inhibition of microsomal liver enzymes
E. Associated disease of target organ

53. Examination of a 70-year-old patient rrevealed insulin-dependent diabetes.


What drug should be administered?
A. Glibenclamid
B. Insulin
C. Mercazolilum
D. Parathyroidin
E. Cortisone

54. For the preparation of a patient's burn skin surface a certain medication was
used. Its antiseptic action is provided by free oxygen that segregates in
presence of organic substances. Choose the right answer:
A. Potassium permanganate
B. Furacilin
C. Chlorhexidine
D. Boric acid
E. Sodium bicarbonate

55. A patient suffering from chronic cardiac insufficiency was recommended to


undergo a prophylactic course of treatment with a cardiological drug from the
group of cardiac glycosides that is to be taken enterally. What drug was
recommended?
A. Digoxin
B. Strophanthine
C. Corglycon
D. Cordiamin
E. Cordarone

56. A 50 y.o. patient with chronic cardiac insufficiency and tachyarrythmia was
prescribed a cardiotonic drug. What drug was prescribed?
A. Digoxin
B. Dopamine
C. Dobutamine
D. Amyodarone
E. Mildronate

57. An ophthalmologist used a 1% mesaton solution for the diagnostic purpose


(pupil dilation for eye-ground examination). What is the cause of mydriasis
induced by the drug?
A. Activation of α1 adrenoreceptors
B. Activation of α2 adrenoreceptors
C. Block of α1 adrenoreceptors
D. Activation of β1 adrenoreceptors
E. Activation of M-cholinoreceptors
58. Mother of a 2-year-old child consulted a stomatologist. In the period of
pregnancy, she was irregularly taking antibiotics for an infectious disease.
Examination of the child revealed incisor destruction, yellow enamel, and
brown rim around the dental cervix. What drug has apparent teratogenic
effect?
A. Doxacycline
B. Furosemide
C. Ampiox
D. Xantinol nicotinate
E. Octadine

59. A 36 y.o. man has a craniocerebral trauma. Objectively: diminished breath


sounds, thready pulse, no reflexes. What way of pyracetam introduction will
be the most apropriate in this case?
A. Intravenous
B. Rectal
C. Subcutaneous
D. Peroral
E. Inhalation

60. A patient complaining of rapid pulse, dyspnea and bluish color of mucosa has
been admitted to the cardiological department. The objective symptoms are
as follows: edema of lower extremities, ascites. Which of the given medicines
should be prescribed for intravenous administration to improve the patient's
general state?
A. Corglyconum
B. Cordiamin
C. Adrenalin hydrochloride
D. Digitoxin
E. Drotaverine

61. A patient suffering from initial hypertension has been taking an


antihypertensive preparation for a long time. Suddenly he stopped taking this
preparation. After this his condition grew worse, this led to development of
hypertensive crisis. This by -effect can be classified as:
A. Abstinence syndrome
B. Cumulation
C. Tolerance
D. Sensibilization
E. Dependence

62. A patient ill with bronchial asthma didn't inform his doctor that he had
attacks of stenocardia. Doctor administered him a medication, which taking
resulted in less frequent attacks of bronchial asthma, but stenocardia attacks
became more frequent. What medication was administered?
A. Isadrin
B. Salbutamol
C. Aminophylline
D. Cromolyn sodium
E. Phenotherol

63. A patient who has been suffering from cardiac insufficiency for several
months has been taking digoxin on an outpatient basis. At a certain stage of
treatment there appeared symptoms of drug overdose. What phenomenon
underlies the development of this complication?
A. Material cumulation
B. Habituation
C. Sensibilization
D. Functional cumulation
E. Tachyphylaxis

64. After the second abortion a 23-year-old woman has been diagnosed with
toxoplasmosis. Which drug should be used for toxoplasmosis treatment?
A. Co-trimoxazole
B. Itraconazole
C. Mebendazole
D. Azidothimidine
E. Acyclovir

65. After 4 months of treatment for tuberculosis the patient began complaining
of toes and fingers numbness, sensation of creeps. He was diagnosed with
polyneuritis. What antituberculous drug might have caused these
complications?
A. Isoniazid
B. Rifampicin
C. Ciprofloxacin
D. Sodium salt of benzylpenicillin
E. Alcohol iodine solution

66. A patient suffers from vision impairment - hemeralopy (night blindness).


What vitamin preparation should be administered the patient in order to
restore his vision?
A. Retinol acetate
B. Vicasol
C. Pyridoxine
D. Thiamine chloride
E. Tocopherol acetate

67. A patient suffers from severe postoperative pseudomonadous infection. What


of the following antibiotics should be administered in this case?
A. Amicacin sulfate
B. Benzylpenicillin
C. Cephazolin
D. Erythromycin
E. Doxycycline
68. After a tooth extraction a patient felt persistent pain behind his breast bone.
After sublingual intake of an antianginal drug the pain behind the breast bone
disappeared, but the patient complained of headache and dizziness. What
drug are these properties typical for?
A. Nitroglycerin
B. Propranolol
C. Metoprolol
D. Validol
E. Verapamil

69. A patient with fracture of his lower jaw was admitted to the maxillofacial
department. It was decided to fix his bones surgically under anaesthetic. After
intravenous introduction of muscle relaxant there arose short fibrillar
contractions of the patient's facial muscles. What muscle relaxant was
applied?
A. Dithylinum
B. Tubocurarin chloride
C. Pipecuronium bromide
D. Diazepam
E. Melictine

70. An elderly patient has chronic constipations induced by large intestine


hypotonia. What drug should be administered?
A. Bisacodyl
B. Sodium sulphate
C. Castor oil
D. Atropine
E. Novocaine amide

71. A patient with epilepsy and depressive reaction has been administered a drug
that reduced epilepsy manifestations and improved the patient's psychic
condition.
A. Sodium valproate
B. Ethosuxemide
C. Amitriptyline
D. Phenytoin
E. Phenobarbital

72. A patient diagnosed with morphinism has been admitted to the narcological
department. A doctor noted a decrease in pharmacological activity of
morphine. Repetitive use of a drug may result in tolerance to its effect, and
this phenomenon is called:
A. Addiction
B. Cumulation
C. Tachyphylaxis
D. Antagonism
E. Allergy
73. Examination of a 60 y.o. patient revealed hyperglycemia and glucosuria. A
doctor administered him a medication for internal use. What medication is
it?
A. Glibenclamid
B. Furosemide
C. Oxytocin
D. Pancreatine
E. Corglycon

74. An elderly female patient suffers from the type 2 diabetes mellitus
accompanied by obesity, atherosclerosis, and coronary artery disease. Basal
hyperinsulinemia is also present. What treatment would be the most
appropriate?
A. Glibenclamid
B. Insulin
C. Retabolil
D. Lovastatin
E. Amlodipine

75. A patient suffers from stenocardia and takes isosorbide mononitrate. He was
prescribed a complementary drug with disaggregating effect. What drug is it?
A. Acetylsalicinic acid
B. Nitroglycerine
C. Propranolol
D. Nifedipine
E. Validol

76. A patient in postoperative period was prescribed an anticholinesterase drug


for stimulation of intestinal peristalsis and tonus of urinary bladder. What
drug is it?
A. Proserin
B. Dichlothiazide
C. Reserpine
D. Mannitol
E. Propanolol

77. A liquidator of a breakdown at a nuclear power plant who was irradiated


complained about vomiting that occurs all of a sudden. What medication
should be prescribed?
A. Metoclopramide
B. Reserpine
C. Atropine
D. Aeron
E. De-Nol

78. A patient with chronic cardiac insufficiency has been treated with cardiotonic
drugs and a thiazide diuretic, but in spite of it there are still edemata and risk
of ascites. What medication should be prescribed to amplify diuretic effect of
the applied drugs?
A. Spironolactone
B. Furosemide
C. Amyloride
D. Clopamide
E. Manitole

79. A patient ill with collagenesis has been taking prednisolone for a long time.
Hypokaliemia development caused spastic pain of skeletal muscles. What
medication should be used in order to correct potassium exchange?
A. Panangin
B. Dithylinum
C. Diazepam
D. Noshpa
E. Thyrocalcitonin

80.A patient ill with essential hypertension was recommended a drug that
prevents thrombosis. It is to be taken parenterally. What drug is it?
A. Heparin
B. Amben
C. Protamine sulfate
D. Neodicumarin
E. Syncumar

81. A patient presents with twilight vision impairment. Which of the following
vitamins should be administered?
A. Retinol acetate
B. Cyanocobalamin
C. Pyridoxine hydrochloride
D. Ascorbic acid
E. Nicotinic acid

82. A female patient consulted a doctor about pain and limited movements in the
knee joints. Which of the following nonsteroid anti-inflammatory drugs
should be administered taking into consideration that the patient has a
history of chronic gastroduodenitis?
A. Celecoxib
B. Diclofenac sodium
C. Promedol
D. Acetylsalicilic acid
E. Butadiounum

83. A 66-year-old female patient got intravenous injection of magnesium sulfate


solution for the purpose of elimination of hypertensive crisis. But arterial
pressure didn't go down and after repeated introduction of the same
preparation there appered sluggishness, slow response, inhibition of
consciousness and respiration. What preparation is antagonist of magnesium
sulfate and can eliminate symptoms of its overdose?
A. Calcium chloride
B. Potassium chloride
C. Sodium chloride
D. Activated carbon
E. Potassium permanganate

84. A 64-year-old woman has impairment of twilight vision (hemeralopy). What


vitamin should be recommended in the first place?
A. Vitamin A
B. Vitamin B2
C. Vitamin E
D. Vitamin C
E. Vitamin B6

85. A patient underwent appendectomy. In the postoperative period he has been


taking an antibiotic. The patient complains about hearing impairment and
vestibular disorders. What group of antibiotics has such by -effects?
A. Aminoglycosides
B. Penicillins
C. Tetracyclines
D. Macrolides
E. Cephalosporins

86. A student came to see a doctor and asked to administer him a drug for
treatment of allergic rhinitis that occurs in the period of linden flowering.
What drug may be used?
A. Loratadine
B. Noradrenaline hydrotartrate
C. Propanolol
D. Ambroxol
E. Losartan

87. A 7-year-old child is ill with bronchitis. It is necessary to administer him an


antibacterial drug. What drug of fluoroquinolone group is CONTRA-
INDICATED at this age?
A. Cyprofloxacin
B. Ampicillin
C. Amoxicillin
D. Sulfadimethoxine
E. Ampiox

88. A child patient has dry cough. What non-narcotic antitussive drug will relieve
the patient's condition?
A. Glaucine hydrochloride
B. Codeine phosphate
C. Morphine hydrochloride
D. Potassium iodide
E. Althaea officinalis root extract
89. A patient has been diagnosed with transmural myocardial infarction. What
drug should be given in order to prevent cardiogenic shock?
A. Promedol
B. Reserpin
C. Octadine
D. Phentolamine
E. Analgin

90. A patient consulted a physician about muscle rigidity, constrained


movements, permanent arm tremor. The patient was diagnosed with
Parkinson's disease. What preparation should be administered?
A. Levodopa
B. Phenytoin
C. Phenobarbital
D. Diazepam
E. Ethosuximide

91. A patient takes digoxin for treatment of cardiac insufficiency. What diuretic
may increase digoxin toxicity due to the intensified excretion of K + ions?
A. Hydrochlorothiazide
B. Spironolactone
C. Panangine
D. Siliborum
E. Lisinopril

92. A patient with coronary artery disease was admitted to the cardiological
department. For stenocardia prevention a drug from the group of β-
adrenoceptor blockers was administered. What drug is it?
A. Metoprolol
B. Atropine sulfate
C. Morphine hydrochloride
D. Oxytocin
E. Furosemide

93. A woman was delivered to a hospital for trachea intubation. What of the
following drugs should be applied in this case?
A. Dithylinum
B. Nitroglycerine
C. Metronidazole
D. Atropine sulfate
E. Gentamycin sulfate

94. A patient suffers from pulmonary tuberculosis. During treatment neuritis of


visual nerve arose. What drug has caused this by -effect?
A. Isoniazid
B. Ethambutol
C. Kanamycin
D. Rifampicin
E. Streptomycin

95. A patient ill with amebiasis was prescribed a certain drug. The use of alcohol
together with this drug is contra-indicated because the drug inhibits
metabolism of ethyl alcohol. What drug is it?
A. Metronidazole
B. Reserpine
C. Clonidine
D. Diazepam
E. Aminazine

96. A patient suffers from chronic left-ventricular insufficiency. What medication


should be administered?
A. Digoxin
B. Bemegride
C. Etimizole
D. Vinpocetine
E. Pyracetam

97. A patient ill with chronic cardiac insufficiency was prescribed an average
therapeutic dose of digoxin. Two weeks after begin of its taking there
appeared symptoms of drug intoxication (bradycardia, extrasystole, nausea).
Name the phenomenon that caused accumulation of the drug in the
organism?
A. Material cumulation
B. Functional cumulation
C. Tolerance
D. Tachyphylaxis
E. Idiosyncrasy

98. A patient with drug intoxication presented with the dryness of oral mucous
membrane and mydriatic pupils. Such action of this drug is associated with
the following effect:
A. Muscarinic cholinoreceptor block
B. Muscarinic cholinoreceptor stumulation
C. Nicotinic cholinoreceptor stumulation
D. Adrenoreceptor stimulation
E. Adrenoreceptor block

99. A patient with ischemic heart disease has been administered an anti-anginal
drug that reduces the myocardial oxygen consumption and improves blood
supply of myocardium. What drug is it?
A. Nitroglycerine
B. Validol
C. Propranolol
D. Promedol
E. Retabolil

100. An patient with insomnia induced by allergic rash and itch has been
prescribed the drug that has antihistamine and hypnotic effect. Specify this
drug:
A. Benadryl
B. Loratadine
C. Prednisolone
D. Acetylsalicylic acid
E. Analgin

101. A patient with a limb fracture must be administered a depolarizing drug


from the myorelaxant group for the purpose of a short-time surgery. What
drug is it?
A. Dithylinum
B. Tubocurarine chloride
C. Cytitonum
D. Atropine sulfate
E. Pentaminum

102. A patient suffering from chronic bronchitis takes a synthetic mucolytic


drug that facilitates the sputum thinning. What drug is it?
A. Acetylcysteine
B. Diazepam
C. Heparin
D. Furosemide
E. Enalapril

103. For relief of hypertensive crisis, a doctor administered a patient a drug


that apart from antihypertensive effect has also sedative, spasmolytic and
anticonvulsive effect. The drug was taken parenterally. When it is taken
enterally it acts as a laxative and cholagogue. What drug was administered?
A. Magnesium sulfate
B. Dibasolum
C. Reserpine
D. No-spa
E. Apressin

104. A patient with diabetes mellitus complicated by angiopathy has been


recommended a drug which is a sulphonyl urease derivate of the second
generation. It improves microcirculation and is known for its relatively good
tolerance. What drug is it?
A. Glibenclamide
B. Glibutidum
C. Insulin
D. Acarbose
E. Adrenalin
105. Burned skin surface was treated with a certain preparation. Its
antiseptic properties are provided by atomic oxygen that is formed in
presence of organic substances. What preparation was applied?
A. Potassium permanganate
B. Furacillin
C. Chlorhexidine bigluconate
D. Alcoholic iodine solution
E. Sodium hydrocarbonate

106. On the 2-3rd day after stomach resection intestinal peristalsis wasn't
restored. What is to be administered for stimulation of gastrointestinal tract?
A. Proserin
B. Prasosin
C. Cyclodole
D. Atropine sulfate
E. Acetylcholine

107. A stomatologist injected a patient with a certain drug in order to reduce


salivation during tooth filling. What drug is it?
A. Atropine sulfate
B. Adrenaline hydrochloride
C. Proserin
D. Pilocarpine hydrochloride
E. Mesaton

108. A patient with chronic cardiac insufficiency has been taking foxglove
(Digitalis) preparations for a long time. Due to the violation of intake
schedule the woman got symptoms of intoxication. These symptoms result
from:
A. Material cumulation
B. Tachyphylaxis
C. Idiosyncrasy
D. Antagonism
E. Sensibilization

109. During local anesthetization the patient has gone into anaphylactic
shock. What drug must be administered to the patient?
A. Epinephrine hydrochloride
B. Diazepam
C. Atropine sulfate
D. Propranolol
E. Nitroglycerin

110. Before tooth extraction a patient was advised to take a certain drug for
haemorrhage prevention. What drug was advised?
A. Vicasolum
B. Heparin
C. Asparcam
D. Magnesium sulfate
E. Dimedrol

111. A man who has been taking a drug for a long time cannot withhold it
because this causes impairment of psychic, somatic and vegetative functions.
Name the syndrome of different disturbances caused by drug
discontinuation:
A. Abstinence
B. Sensibilization
C. Idiosyncrasy
D. Tachyphylaxis
E. Cumulation

112. A patient consulted a stomatologist about purulent inflammation of his


gums. What drug will be the most effective if it is suspected that a causative
agent is an anaerobe?
A. Metronidazole
B. Gentamicin
C. Oxacillin sodium
D. Co-trimoxazole
E. Nitroxoline

113. A 45-year-old patient suffers from neurosis characterized by irritability,


sleeplessness, motiveless anxiety. What drug would eliminate all the
symptoms?
A. Diazepam
B. Valerian extract
C. Pyracetam
D. Caffeine sodium benzoate
E. Levodopa

114. A 12-year-old child has intolerance to some foodstuffs. Their


consumption causes an allergic reaction in form of itching skin eruptions.
What antihistaminic drug should be admistered so that the child could attend
school?
A. Loratadine
B. Dimedrol
C. Diclofenac
D. Aminophylline
E. Ephedrine

115. A patient that entered the admission office had the following signs of
acute cardiac insufficiency: paleness, acrocyanosis, and frequent shallow
respiration. What drug is indicated in this case?
A. Corglycon
B. Digitoxin
C. Cordiamin
D. Nitroglycerine
E. Adrenaline hydrochloride

116. A patient with myocardial infarction was admitted to the cardiological


department. For pain relief it was decided to potentiate fentanyl action with
a neuroleptic. Which of the following neuroleptics is the most suitable for
neuroleptanalgesia?
A. Droperidol
B. Aminazine
C. Triftazine
D. Haloperidol
E. Sulpiride

117. A patient with bacterial pneumonia was prescribed benzylpenicillin.


What is the mechanism of its antibacterial effect?
A. Inhibition of synthesis of microorganism wall
B. Inhibition of intracellular protein synthesis
C. Abnormal permeability of cytoplasmic membrane
D. Inhibition of SH-groups of microorganism enzymes
E. Antagonism with p-amino-benzoic acid

118. A patient has herpetic rash. What medication should be administered?


A. Acyclovir
B. Gentamycin
C. Clotrimazole
D. Benzylpenicillin sodium salt
E. Biseptol

119. A patient who has been treated in a neural clinic and has been taking a
sedative for a long time got the following complication: cough, rhinitis,
epiphora. What drug caused these disturbances?
A. Sodium bromide
B. Diazepam
C. Valerian
D. Phenazepam
E. Reserpine

120. In order to accelerate healing of a radiation ulcer a vitamin drug was


administered. What drug is it?
A. Retinol acetate
B. Retabolil
C. Prednisolone
D. Levamisole
E. Methyluracil

121. A patient who had been taking diclofenac sodium for arthritis of
mandibular joint developed an acute condition of gastric ulcer. Such side
effect of this medicine is caused by inhibition of the following enzyme:
A. Cyclooxygenase-1 (COX-1)
B. Cyclooxygenase-2 (COX-2)
C. Lipoxygenase
D. Phosphodiesterase
E. Monoamine oxidase

122. A patient with hip fracture was prescribed a narcotic analgetic. Its
anesthetic action is determined by interaction with the following receptors:
A. Opiate receptors
B. Adrenoreceptors
C. Cholinoreceptors
D. Benzodiazepine receptors
E. GABA-ergic receptors

123. A patient with hypertensic crisis was admitted to the cardiological


department, he was injected intravenously with an antihypertensive drug -
salt of an alkaline-earth metal. What drug was injected?
A. Magnesium sulfate
B. Potassium chloride
C. Sodium hydrocarbonate
D. Calcium lactate
E. Benzohexamethonium

124. A patient with acute morphine poisoning was delivered to a hospital.


What specific narcotic antagonist should be chosen in this case?
A. Naloxone
B. Paracetamol
C. Methacin
D. Digoxin
E. Unithiol

125. A 55-year-old male had been delivered to the resuscitation unit


unconscious. Relatives reported him to have mistakenly drunk an alcoholic
solution of unknown origin. On examination the patient was diagnosed with
methanol intoxication. What antidote should be used in this case?
A. Ethanol
B. Teturamum
C. Naloxone
D. Protamine sulfate
E. Acetylcysteine

126. An 8-week-pregnant woman with acute respiratory disease and


temperature rise up to 39.0 C has called in a doctor. The doctor insisted on
her avoiding taking paracetamol, because in this period of pregnancy there is
a risk of its:
A. Teratogenicity
B. Embryotoxicity
C. Fetotoxicity
D. Hepatotoxicity
E. Allergenicity

127. A pneumonia patient has been administered acetylcysteine as a part of


complex therapy. What principle of therapy has been taken into consideration
when applying this drug?
A. Pathogenetic
B. Symptomatic
C. Etiotropic
D. Antimicrobial
E. Immunomodulatory

128. A 26-year-old female patient with bronchitis has been administered a


broad spectrum antibiotic as a causal treatment drug. Specify this drug:
A. Doxycycline
B. Interferon
C. BCG vaccine
D. Ambroxol
E. Dexamethasone

129. A patient has arterial hypertension. What long-acting drug from the
group of calcium channel blockers should be prescribed?
A. Amlodipine
B. Octadine
C. Pyrroxanum
D. Atenolol
E. Reserpine

130. Administration of doxycycline hydrochloride caused an imbalance of


the symbiotic intestinal microflora. Specify the kind of imbalance caused by
the antibiotic therapy:
A. Dysbacteriosis
B. Sensibilization
C. Idiosyncrasy
D. Superimposed infection
E. Bacteriosis
Krok 1 – 2014 Physiology Base
1. Patient with hypersecretion of the gastric juices was recommended to exclude
from the diet concentrated bouillons and vegetable decoctions because of their
stimulation of gastric secretion. What is dominating mechanism of stimulation
of secretion in this case?
A. Stimulation of gastrin production by G-cells
B. Irritation of taste receptors
C. Irritation of mechanoreceptors of the oral cavity
D. Irritation of mechanoreceptors of the stomach
E. Stimulation of excretion of secretin in the duodenum

2. Person felt thirsty after staying in heat for a long time. Signals of what
receptors caused it first of all?
A. Osmoreceptors of hypothalamus
B. Sodium receptors of hypothalamus
C. Osmoreceptorsof the liver
D. Glucoreceptors of hypothalamus
E. Baroreceptors of aortic arch

3. While emotional excitement the heart rate in a 30-year-old person run up to


112 Bpm. What part of the conducting system of the heart caused it?
A. Synoatrial node
B. Purkinje's fibers
C. His bundle branches
D. Intraventricular node
E. His bundle

4. A 38-year-old woman was admitted to the admission-diagnostic department


with uterine bleeding. What are the most likely changes of blood?
A. Reduction of haematocrite rate
B. Increase of haematocrite rate
C. Leukopenia
D. Leucocytosis
E. Polycythemia

5. Due to action of electric current on the exitable cell there appeared


depolarization of it's membrane. Movement of what ions through the
membrane caused depolarisation?
A. Na +
B. НСО3 -
C. Са2 +
D. Сl-
E. К+
6. On experiment on the dog the peripheral part of nervus vagus of the neck was
irritated. What changes of the heart function would be observed?
A. Decreased contraction rate
B. Increased contraction force
C. Increased atrioventricular conduction
D. Increased contraction force and rate
E. Increased myocardial excitability

7. Power inputs of a boy increased from 500 to 2000 kJ pro hour. What can be
the cause of it?
A. Physical exercise
B. Raise of outer temperatute
C. Mental activity
D. Food intake
E. Transition from sleep to wakefulness

8. In a young man during exercise, the minute oxygen uptake and carbon dioxide
emission equalled to 1000 ml. What substrates are oxidized in the cells of his
body?
A. Carbohydrates
B. Proteins
C. Fats
D. Carbohydrates and fats
E. Carbohydrates and proteins

9. In the experiment on the animal the part of the cerebral cortex hemispheres
was removed. It caused elimination of previously formed conditioned reflex to
the light irritation. What part of the cortex was removed?
A. Occipital cortex
B. Precentral convolution
C. Postcentral convolution
D. Limbic cortex
E. Temporal lobe

10. Inhibition of alpha-motoneuron of the extensor muscles was noticed after


stimulation of alpha-motoneuron of the flexor muscles during the experiment
on the spinal column. What type of inhibition can this process cause?
A. Reciprocal
B. Presynaptic
C. Depolarizational
D. Recurrent
E. Lateral

11. Respiratory coefficient was studied in the patient who strictly kept certain diet
for 10 days. It was determined that it is 1. What diet does the patient follow?
A. With domination of carbohydrates
B. With domination of proteins and fat
C. With domination of fat and carbohydrates
D. Mixed
E. With domination of proteins and carbohydrates

12. A sportsman spontaneously held breath for 40 seconds, which resulted in an


increase in heart rate and systemic arterial pressure. Changes of these
indicators are due to activation of the following regulatory mechanisms:
A. Unconditioned sympathetic reflexes
B. Unconditioned parasympathetic reflexes
C. Conditioned sympathetic reflexes
D. Conditioned parasympathetic reflexes
E. -

13. Punctata hemorrhage was found out in the patient after application of a
tourniquet. With disfunction of what blood cells is it connected?
A. Platelets
B. Eosinophiles
C. Monocytes
D. Lymphocytes
E. Neutrophiles

14. Students who are taking examinations often have dry mouth. The mechanism
that causes this state is the realization of the following reflexes:
A. Conditioned sympathetic
B. Unconditioned parasympathetic
C. Conditioned parasympathetic
D. Unconditioned sympathetic
E. Unconditioned peripheral

15. Middle part of cochlear of internal ear was destroyed in animal while
experiment. It will cause abnormalities of the sound perception of the
following frequencies:
A. Middle
B. Low
C. High
D. High and low
E. No abnormalities

16. The temperature of the ambient environment is 38oC and relative air humidity
is 50%. What ways of heat emission provide maintaining a constant
temperature of the human body?
A. Evaporation
B. Radiation
C. Heat conduction
D. Convection
E. Convection and conduction

17. The minute blood volume in a patient with transplanted heart has increased as
a result of physical activity. What regulative mechanism is responsible for
these changes?
A. Catecholamines
B. Sympathetic unconditioned reflexes
C. Parasympathetic unconditioned reflexes
D. Sympathetic conditioned reflexes
E. Parasympathetic conditioned reflexes

18. Isolated muscle of a frog is rhythmically irritated with electric impulses. Every
next impulse is in a period of relaxation from the previus contraction. What
contraction of the muscle appears?
A. Waved tetanus
B. Single
C. Asynchronous
D. Continuous(smooth) tetanus
E. Tonic

19. A 30-year-old woman has subnormal concentration of enzymes in the


pancreatic juice. This might be caused by the hyposecretion of the following
gastrointestinal hormone:
A. Cholecystokinin-pancreozymin
B. Somatostatin
C. Secretin
D. Gastro-inhibiting peptide
E. Vaso-intestinal peptide

20. A patient has a trauma of sternocleidomastoid muscle. This caused a decrease


in value of the following indicator of external respiration:
A. Inspiratory reserve volume
B. Expiratory reserve volume
C. Respiratory capacity
D. Residual volume
E. Functional residual lung capacity

21. A man has normal sensitivity of his finger skin, however he doesn’t sense his
wedding ring around the finger. What process induced by wearing of the ring
has caused this phenomenon?
A. Receptor adaptation
B. Development of the fibrous tissue
C. Abnormality of the epidermis structure
D. Impaired circulation
E. Abnormality of the receptor structure
22. An aged man had raise of arterial pressure under a stress. It was caused by
activation of:
A. Sympathoadrenal system
B. Parasympathetic nucleus of vagus
C. Functions of thyroid gland
D. Functions of adrenal cortex
E. Hypophysis function

23. A month after surgical constriction of rabbit's renal artery the considerable
increase of systematic arterial pressure was observed. What of the following
regulation mechanisms caused the animal's pressure change?
A. Angiotensin-II
B. Vasopressin
C. Adrenaline
D. Noradrenaline
E. Serotonin

24. A child has abnormal formation of tooth enamel and dentin as a result of low
concentration of calcium ions in blood. Such abnormalities might be caused by
deficiency of the following hormone:
A. Parathormone
B. Thyrocalcitonin
C. Thyroxin
D. Somatotropic hormone
E. Triiodothyronine

25. A sportsman was examined after an intensive physical activity. The


examination revealed disorder of movement coordination but the force of
muscle contractions remained the same. It can be explained by retarded speed
of excitement conduction through:
A. Central synapses
B. Neuromuscular synapses
C. Efferent nerves
D. Afferent nerves
E. Conduction tracts

26. After a long training session, a sportsman has developed fatigue accompanied
by abrupt performance decrement. What link of the reflex arch was the fatigue
initiated in?
A. Nerve centres
B. Afferent conductor
C. Receptors
D. Efferent conductor
E. Muscles
27. ECG study showed that the $T$-waves were positive in the standard extremity
leads, their amplitude and duration were normal. The right conclusion would
be that the following process runs normally in the heart ventricles:
A. Repolarization
B. Depolarization
C. Excitement
D. Contraction
E. Relaxation

28. Blood minute volume of a 30-year-old woman at rest is 5 l/m. What blood
volume is pumped through the pulmonary vessels per minute?
A. 5 l
B. 3,75 l
C. 2,5 l
D. 2,0 l
E. 1,5 l

29. As a result of long-term starvation the glomerular filtration of a man was


accelerated by 20\%. The most probable cause of filtration changes under such
conditions is:
A. Fall of oncotic pressure of blood plasma
B. Rise of systemic arterial pressure
C. Increased permeability of renal filter
D. Growth of filtration coefficient
E. Increase of renal plasma flow

30. In course of an experiment a skeletal muscle is being stimulated by a series of


electric impulses. What type of muscle contraction will arise, if every
subsequent impulse comes in the period of shortening of the previous single
muscle contraction?
A. Holotetanus
B. Partial tetanus
C. Asynchronous tetanus
D. A series of single contractions
E. Muscle contracture

31. A patient under test was subjected to a moderate physical stress. His minute
blood volume amounted 10 l/min. What blood volume was pumped through
his lung vessels every minute?
A. 10 l/min
B. 5 l/min
C. 4 l/min
D. 6 l/min
E. 7 l/min

32. A patient presents with the following motor activity disturbances: tremor,
ataxia and asynergia movements, dysarthria. The disturbances are most likely
to be localized in:
A. Cerebellum
B. Basal ganglions
C. Limbic system
D. Brainstem
E. Medulla oblongata

33. A man has a considerable decrease in diuresis as a result of 1,5 l blood loss. The
primary cause of such diuresis disorder is the hypersecretion of the following
hormone:
A. Vasopressin
B. Corticotropin
C. Natriuretic
D. Cortisol
E. Parathormone

34. An animal experiment is aimed at studying the cardiac cycle. All the heart
valves are closed. What phase of the cycle is characterized by this status?
A. Isometric contraction
B. Asynchronous contraction
C. Protodiastolic period
D. Rapid filling
E. Reduced filling

35. While shifting the gaze to the closely situated object the refracting power of
eye's optical mediums will increase by 10 diopters. It results from changing of
such eye structure:
A. Lens
B. Cornea
C. Vitreous body
D. Liquid of the anterior chamber of eye
E. Muscle that dilatates pupil

36. Spasm of smooth muscle of bronchi developed in the patient. Usage of


activators of what membrane cytoreceptors is fisiologically valid to decrease
attack?
A. β-adrenoreceptors
B. α-аdrenoreceptors
C. α- та β-аdrenoreceptors
D. Н-cholinoreceptors
E. М-cholinoreceptors

37. On examination of the person it was revealed that minute volume of heart is
3500mL, systolic volume is 50 mL. What is the frequency of cardiac
contraction?
A. 70 bpm
B. 60 bpm
C. 50 bpm
D. 80 bpm
E. 90 bpm

38. Due to activation of ion channels of external membrane of excitable cell it’s rest
potential has significantly increased. What channels were activated?
A. Potassium channels
B. Natrium channels
C. Fast calcium channels
D. Slow calcium channels
E. Natrium and calcium channels

39. The ventral roots of 5 frontal segment of spinal cord were cut during
experiment in the animal. What changes will take place in the innervation
region?
A. Loss of movements
B. Loss of touch sensitivity
C. Loss of temperature sensitivity
D. Loss of proprioceptive sensitivity
E. Hypersensitivity

40. Glomerular filtration rate (GFR) increased for 20% due to prolonged
starvation of the person. The most likely cause of filtration changes under this
conditions is:
A. Decrease of oncotic pressure of blood plasma
B. Increase of systemic blood pressure
C. Increase of penetration of the renal filter
D. Increase of filtration coefficient
E. Increase of renal plasma stream

41. A patient has a transverse disruption of spinal cord below the IV thoracic
segment. What changes of respiration will it cause?
A. Respiration will stay unchanged
B. Respiration will stop
C. Respiration will become less frequent
D. Respiration will become deeper
E. Respiration will become more frequent

42. A lightly dressed man is standing in a room, air temperature is +$14^0C$,


windows and doors are closed. In what way does he emit heat the most
actively?
A. Heat radiation
B. Heat conduction
C. Convection
D. Evaporation
E. Perspiration

43. ECG of a patient with hyperfunction of thyroid gland showed heart hurry. It is
indicated by depression of the following ECG element:
A. R-R interval
B. P-Q segment
C. P-Q interval
D. P-T interval
E. QRS complex

44. A peripheral segment of vagus nerve on a dog's neck was being stimulated in
course of an experiment. The following changes of cardiac activity could be
meanwhile observed:
A. Heart rate fall
B. Heart hurry
C. Enhancement of atrioventricular conduction
D. Heart rate and heart force amplification
E. Increased excitability of myocardium

45. ECG of a patient shows prolongation of T -wave. This is caused by deceleration


in ventricles of:
A. Repolarization
B. Depolarization and repolarization
C. Depolarization
D. Contraction
E. Relaxation

46. In a healthy adult speed of the excitement conduction through the


atrioventricular node is 0,02-0,05 m/sec. Atrioventricular delay enables:
A. Sequence of atrial and ventricular contractions
B. Simultaneity of both atria contractions
C. Simultaneity of both ventricles contractions
D. Sufficient force of atrial contractions
E. Sufficient force of ventricular contractions

47. A 2 y.o. child has convulsions as a result of lowered concentration of calcium


ions in blood plasma. It is caused by reduced function of:
A. Parathyroid glands
B. Hypophysis
C. Adrenal cortex
D. Pineal gland
E. Thymus

48. During preparation of a patient to a heart surgery it was necessary to measure


pressure in heart chambers. In one of them pressure varied from 0 mm Hg up
to 120 mm Hg within one cardiac cycle. What heart chamber is it?
A. Left ventricle
B. Right ventricle
C. Right atrium
D. Left atrium

49. Heart rate of a man permanently equals 40 beats pro minute. What is the
pacemaker?
A. Atriventricular node
B. Sinoatrial node
C. His' bundle
D. His' bundle branches
E. Purkinje's fibers

50. Stimulation of an excitable cell by the electric current has led to the
depolarization of its membrane. The depolarization has been caused mainly by
the following ions penetrating into the cell through its membrane:
A. Na +
B. HCO3 -
C. Ca2 +
D. Cl-
E. K+

51. Parents of a 10 y.o. boy consulted a doctor about extension of hair-covering,


growth of beard and moustache, low voice. Intensified secretion of which
hormone must be assumed?
A. Of testosterone
B. Of somatotropin
C. Of oestrogen
D. Of progesterone
E. Of cortisol

52. Lung ventilation in a person is increased as a result of physical activity. Which


of the following indices of the external respiration is much higher than in a
state of rest?
A. Respiratory volume
B. Vital capacity of lungs
C. Inspiratory reserve volume
D. Expiratory reserve volume
E. Total lung capacity

53. A man took a quiet expiration. Name an air volume that is meanwhile
contained in his lungs:
A. Functional residual capacity
B. Residual volume
C. Expiratory reserve volume
D. Respiratory volume
E. Vital lung capacity

54. Examination of an isolated cardiomyocyte revealed that it didn't generate


excitation impulses automatically. This cardiomyocyte was obtained from:
A. Ventricles
B. Sinoatrial node
C. Atrioventricular node
D. His' bundle
E. Purkinje's fibers

55. Examination of a man established that cardiac output equaled 3500 ml,
systolic output - 50 ml. What is the man's heart rate pro minute?
A. 70
B. 60
C. 50
D. 80
E. 90

56. As a result of continuous starvation the glomerular filtration rate has increased
by 20\%. The most probable cause of the glomerular filtration alteration under
the mentioned conditions is:
A. Decrease in the oncotic pressure of blood plasma
B. Increase in the systemic arterial pressure
C. Increase in the permeability of the renal filter
D. Increase of the filtartion quotient
E. Increase of the renal blood flow

57. A man who went for a ride on a roundabout had amplification of heart rate,
sweating and nausea. What receptors stimulation is it primarily connected
with?
A. Vestibular
B. Proprioceptors
C. Tactors
D. Auditory
E. Visual

58. A man's intrapleural pressure is being measured. In what phase did the man
hold his breath, if his pressure is 7,5 cm Hg?
A. Quiet inspiration
B. Quiet expiration
C. Forced inspiration
D. Forced expiration
59. Atria of an experimental animal were superdistended by blood that resulted in
decreased reabsorption of Na+ and water in renal tubules. This can be
explained by the influence of the following factor upon kidneys:
A. Natriuretic hormone
B. Aldosterone
C. Renin
D. Angiotensin
E. Vasopressin

60. A middle-aged man went to a foreign country because he had been offered a
job there. However, he had been unemployed for quite a long time. What
endocrine glands were exhausted most of all in this man?
A. Adrenal glands
B. Parathyroid glands
C. Seminal glands
D. Substernal gland
E. Thyroid gland

61. A 42-year-old patient complains of pain in the epigastral area, vomiting; vomit
masses have the colour of "coffee-grounds", the patient has also melena.
Anamnesis records gastric ulcer. Blood formula: erythrocytes - 2,8* 101 2/l,
leukocytes – 8* 109 /l, Hb- 90 g/l. What complication is it?
A. Haemorrhage
B. Penetration
C. Perforation
D. Canceration
E. Pyloric stenosis

62. A human body cools in water much faster that in the air. What way of heat
emission in water is much more efficient?
A. Heat conduction
B. Convection
C. Heat radiation
D. Sweat evaporation
E. -

63. As a result of spinal-cord trauma a 33 y.o. man has a disturbed pain and
temperature sensitivity that is caused by damage of the following tract:
A. Spinothalamic
B. Medial spinocortical
C. Posterior spinocerebellar
D. Lateral spinocortical
E. Anterior spinocerebellar

64. After a surgery a 36-year-old woman was given an intravenous injection of


concentrated albumin solution. This has induced intensified water movement
in the following direction:
A. From the intercellular fluid to the capillaries
B. From the intercellular fluid to the cells
C. From the cells to the intercellular fluid
D. From the capillaries to the intercellular fluid
E. No changes of water movement will be observed

65. A clinic observes a 49-year-old patient with significant prolongation of


coagulation time, gastrointestinal haemorrhages, subcutaneous hematomas.
These symptoms might be explained by the deficiency of the following vitamin:
A. K
B. B1
C. B6
D. H
E. E

66. Examination of a patient revealed hyperkaliemia and hyponatremia. Low


secretion of which hormone may cause such changes?
A. Aldosteron
B. Vasopressin
C. Cortisol
D. Parathormone
E. Natriuretic

67. Examination of a 43 y.o. anephric patient revealed anemia symptoms. What is


the cause of these symptoms?
A. Reduced synthesis of erythropoietins
B. Enhanced destruction of erythrocytes
C. Iron deficit
D. Vitamin B1 2 deficit
E. Folic acid deficit

68. While determining power inputs of a patient’s organism it was established that
the respiratory coefficient equaled 1,0. This means that in the cells of the
patient the following substances are mainly oxidized:
A. Carbohydrates
B. Proteins
C. Fats
D. Proteins and carbohydrates
E. Carbohydrates and fats

69. A patient has a disturbed absorbtion of fat hydrolysates. It might have been
caused by a deficit in the small intestine cavity:
A. Of bile acids
B. Of bile pigments
C. Of lipolytic enzymes
D. Of sodium ions
E. Of liposoluble vitamins

70. Inhabitants of territories with cold climate have high content of an adaptive
thermoregulatory hormone. What hormone is meant?
A. Thyroxin
B. Insulin
C. Glucagon
D. Somatotropin
E. Cortisol

71. A concentrated solution of sodium chloride was intravenously injected to an


animal. This caused decreased reabsorption of sodium ions in the renal
tubules. It is the result of the following changes of hormonal secretion:
A. Aldosterone reduction
B. Aldosterone increase
C. Vasopressin reduction
D. Vasopressin increase
E. Reduction of atrial natriuretic factor

72. People adapted to high external temperatures have such pecularity: profuse
sweating isn't accompanied by loss of large volumes of sodium chloride. This
is caused by the effect of the following hormone upon the perspiratory glands:
A. Aldosterone
B. Vasopressin
C. Cortisol
D. Tgyroxin
E. Natriuretic

73. The processes of heat transfer in a naked person at room temperature have
been studied. It was revealed that under these conditions the greatest amount
of heat is transferred by:
A. Heat radiation
B. Heat conduction
C. Convection
D. Evaporation

74. During an experiment the dorsal roots of the spinal cord of an animal have
been cut. What changes will be observed in the innervation zone?
A. Sensitivity loss
B. Loss of motor functions
C. Decrease in muscle tone
D. Increase in muscle tone
E. Sensitivity loss and loss of motor functions
75. As a result of destruction of certain brainstem structures an animal has lost its
orientative reflexes in response to strong photic stimuli. What structures were
destroyed?
A. Anterior tubercles of quadrigeminal plate
B. Posterior tubercles of quadrigeminal plate
C. Red nuclei
D. Vestibular nuclei
E. Black substance

76. As a result of damage to certain structures of brainstem an animal lost


orientation reflexes. What structures were damaged?
A. Quadritubercular bodies
B. Medial nuclei of reticular formation
C. Red nuclei
D. Vestibular nuclei
E. Black substance

77. Urine analysis has shown high levels of protein and erythrocytes in urine. This
can be caused by the following:
A. Renal filter permeability
B. Effective filter pressure
C. Hydrostatic blood pressure in glomerular capillaries
D. Hydrostatic primary urine pressure in capsule
E. Oncotic pressure of blood plasma

78. Osmotic pressure of a man's blood plasma is 350 mosmole/l (standard


pressure is 300 mosmole/l). First of all, it will result in high secretion of the
following hormone:
A. Vasopressin
B. Aldosteron
C. Cortisol
D. Adrenocorticotropin
E. Natriuretic

79. After a craniocerebral injury a patient is unable to recognize objects by touch.


What part of brain has been damaged?
A. Postcentral gyrus
B. Occipital lobe
C. Temporal lobe
D. Precentral gyrus
E. Cerebellum

80.A hypertensive glucose solution was introduced to a patient. It will intensify


water movement:
A. From the cells to the intercellular liquid
B. From the intercellular liquid to the capillaries
C. From the intercellular liquid to the cells
D. From the capillaries to the intercellular liquid
E. There will be no changes of water movement

81. To prevent long-term effects of 4-day malaria a 42-year-old patient was


prescribed primaquine. On the 3-rd day from the begin of treatment there
appeared stomach and heart pains, dyspepsia, general cyanosis,
hemoglobinuria. What caused side effects of the preparation?
A. Genetic insufficiency of glucose 6-phosphate dehydrogenase
B. Cumulation of the preparation
C. Decreased activity of microsomal liver enzymes
D. Delayed urinary excretion of the preparation
E. Drug potentiation by other preparations

82. According to audiometry data a patient has a disturbed perception of medium-


frequency sounds. It might have been caused by a damage of:
A. Middle part of helix
B. Cochlear nuclei
C. Spiral ganglion
D. Quadritubercular structure
E. Lateral geniculate bodies

83. A 17-year-old boy fell seriously ill, body temperature rose up to 38,5 oС, there
is cough, rhinitis, lacrimation, nasal discharges. What kind of inflammation is
it?
A. Catarrhal inflammation
B. Serous inflammation
C. Fibrinous inflammation
D. Suppurative inflammation
E. Hemorrhagic inflammation

84. A patient with disturbed cerebral circulation has problems with deglutition.
What part of brain was damaged?
A. Brainstem
B. Cervical part of spinal cord
C. Forebrain
D. Interbrain
E. Midbrain

85. A patient who has been treated with diazepam on account of neurosis
complains of toothache. Doctor administered him an analgetic, but its dose
was lower than average therapeutic dose. What phenomenon did the doctor
take into account while prescribing the patient an underdose?
A. Potentiation
B. Summation
C. Cumulation
D. Drug dependence
E. Tolerance

86. Long-term starvation cure of a patient resulted in diminished ratio of


albumines and globulines in plasma. What of the following will be result of
these changes?
A. Increase of ESR
B. Decrease of ESR
C. Increase of hematocrit
D. Decrease of hematocrit
E. Hypercoagulation

87. A patient has a decreased vasopressin synthesis that causes polyuria and as a
result of it evident organism dehydratation. What is the mechanism of polyuria
development?
A. Reduced tubular reabsorption of water
B. Reduced tubular reabsorption of Na ions
C. Reduced tubular reabsorption of protein
D. Reduced glucose reabsorption
E. Acceleration of glomerular filtration

88. A 35-year-old man consulted a dentist about reduced density of dental tissue,
high fragility of teeth during eating solid food. This patient suffers the most
probably from the deficiency of the following mineral element:
A. Calcium
B. Potassium
C. Sodium
D. Magnesium
E. Iron

89. A patient is 44 years old. Laboratory examination of his blood revealed that
content of proteins in plasma was 40 g/l. What influence will be exerted on the
transcapillary water exchange?
A. Filtration will be increased, reabsorption - decreased
B. Both filtration and reabsorption will be increased
C. Both filtration and reabsorption will be decreased
D. Filtration will be decreased, reabsorption - increased
E. Exchange will stay unchanged

90. A patient has a critical impairment of protein, fat and hydrocarbon digestion.
Most likely it has been caused by low secretion of the following digestive juice:
A. Pancreatic juice
B. Saliva
C. Gastric juice
D. Bile
E. Intestinal juice
91. After destruction of CNS structures an animal lost orientative reflexes. What
structure was destroyed?
A. Quadrigeminal plate
B. Red nucleus
C. Lateral vestibular nuclei
D. Black substance
E. Medial reticular nuclei

92. An isolated cell of human heart automatically generates excitation impulses


with frequency 60 times pro minute. What structure does this cell belong to?
A. Sinoatrial node
B. Atrium
C. Ventricle
D. Atrioventricular node
E. His' bundle

93. A patient has a traumatic injury of sternocleidomastoid muscle. This has


resulted in a decrease in the following value:
A. Inspiratory reserve volume
B. Expiratory reserve volume
C. Respiratory volume
D. Residual volume
E. Functional residual lung capacity

94. Examination of a patient revealed a strong, balanced, inert type of higher


nervous activity according to Pavlov. What temperament type does the patient
have (according to Hippocrates classification)?
A. Phlegmatic
B. Sanguine
C. Choleric
D. Melancholic

95. The receptors under study provide transfer of information to the cortex
without thalamic involvement. Specify these receptors:
A. Olfactory
B. Tactile
C. Gustatory
D. Visual
E. Auditory

96. Examination of a patient revealed overgrowth of facial bones and soft tissues,
tongue enlargement, wide interdental spaces in the enlarged dental arch. What
changes of the hormonal secretion are the most likely?
A. Hypersecretion of the somatotropic hormone
B. Hyposecretion of the somatotropic hormone
C. Hypersecretion of insulin
D. Hyposecretion of thyroxin
E. Hyposecretion of insulin

97. A patient has a haemorrhage into the posterior central gyrus. What type of
sensitivity on the opposite side will be disturbed?
A. Skin and proprioceptive
B. Visual
C. Auditory
D. Olfactory
E. Auditory and visual

98. A 32-year-old patient consulted a doctor about the absence of lactation after
parturition. Such disorder might be explained by the deficit of the following
hormone:
A. Prolactin
B. Somatotropin
C. Vasopressin
D. Thyrocalcitonin
E. Glucagon

99. During influenza epidemic 40% of pupils who didn't go in for sports were
affected by the disease, and among the pupils who regularly did physical
exercises this index was only 20\%. What adaptative mechanisms determined
such a low sickness rate of pupils participating in the sports?
A. Cross adaptation
B. Specific adaptation
C. Physiological adaptation
D. Biochemical adaptation
E. Genetic adaptation

100. A 60-year-old patient was found to have a dysfunction of main digestive


enzyme of saliva. This causes the disturbance of primary hydrolysis of:
A. Carbohydrates
B. Fats
C. Proteins
D. Cellulose
E. Lactose

101. A 49-year-old woman spent a lot of time standing. As a result of it she


got leg edema. What is the most likely cause of the edema?
A. Increase in hydrostatic pressure of blood in veins
B. Decrease in hydrostatic pressure of blood in veins
C. Decrease in hydrostatic pressure of blood in arteries
D. Increase in oncotic pressure of blood plasma
E. Increase in systemic arterial pressure

102. A patient presented to a hospital with complaints about quick fatigability


and significant muscle weakness. Examination revealed an autoimmune
disease that causes functional disorder of receptors in the neuromuscular
synapses. This will result in the disturbed activity of the following mediator:
A. Acetylcholine
B. Noradrenaline
C. Dopamine
D. Serotonin
E. Glycine

103. A 30-year-old woman was diagnosed with insufficiency of exocrinous


function of pancreas. Hydrolisis of what nutrients will be disturbed?
A. Proteins, fats, carbohydrates
B. Proteins, fats
C. Proteins, carbohydrates
D. Fats, carbohydrates
E. Proteins

104. During an animal experiment, surgical damage of certain brain


structures has caused deep prolonged sleep. What structure is most likely to
cause such condition, if damaged?
A. Reticular formation
B. Basal ganglion
C. Red nuclei
D. Hippocampus
E. Cerebral cortex

105. Short-term physical activity resulted in reflex amplification of heart rate


and raise of systemic arterial pressure. What receptors activation was the main
cause of pressor reflex realization?
A. Proprioreceptors of active muscles
B. Vascular chemoreceptors
C. Vascular volume receptors
D. Vascular baroceptors
E. Hypothalamus thermoreceptors

106. In course of an experiment a skeletal muscle is being stimulated by a


series of electric impulses. What type of muscle contraction will arise, if every
subsequent impulse comes in the period of relaxation of single muscle
contraction?
A. Partial tetanus
B. Holotetanus
C. A series of single contractions
D. Muscle contructure
E. Asynchronous tetanus

107. A patient takes cholagogues. What other process besides biliary


excretion will be stimulated?
A. Intestinal motility
B. Gastric juice secretion
C. Pancreatic juice secretion
D. Gastric motor activity
E. Water absorption

108. Packed cell volume of a man was 40% before the trauma. What packed
cell volume will be observed 24 hours after blood loss of 750 ml?
A. 30%
B. 40%
C. 55%
D. 45%
E. 50%

109. A patient staggers and walks astraddle. He has hypomyotonia of arm and
leg muscles, staccato speech. In what brain section is this affection localized?
A. Cerebellum
B. Putamen
C. Caudate nucleus
D. Motor cortex
E. Red nucleus

110. A pregnant woman had her blood group identified. Reaction of


erythrocyte agglutination with standard serums of 0αβ (I), Bα (III) groups
didn't proceed with standard serum of Aβ (II) group. The blood group under
examination is:
A. Аβ (II)
B. 0αβ (I)
C. Вα (III)
D. АВ (IV)

111. Blood group of a 30-year-old man was specified before an operation. His
blood is Rh-positive. Reaction of erythrocyte agglutination was absent with
standard sera of 0αβ (I), Аβ (II), Вα (III) groups. The blood under examination
is of the following group:
A. 0αβ (I)
B. Аβ (II)
C. Вα (III)
D. АВ (IV)

112. During an experiment the myotatic reflex has been studied in frogs. After
extension in a skeletal muscle its reflectory contraction was absent. The reason
for it might be a dysfunction of the following receptors:
A. Muscle spindles
B. Nociceptors
C. Articular
D. Golgi tendon organs
E. Tactile

113. Vagus branches that innervate heart are being stimulated in course of an
experiment. As a result of it the excitement conduction from atria to the
ventricles was brought to a stop. It is caused by electrophysical changes in the
following structures:
A. Atrioventricular node
B. His' bundle
C. Sinoatrial node
D. Ventricles
E. Atria

114. If a man has an attack of bronchiospasm it is necessary to reduce the


effect of vagus on smooth muscles of bronchi. What membrane cytoreceptors
should be blocked for this purpose?
A. M-cholinoreceptors
B. N-cholinoreceptors
C. α-adrenoreceptors
D. β-adrenoreceptors
E. α- and β-adrenoreceptors

115. When water affects mucous membrane of the inferior nasal meatuses,
this causes "diver reflex" that provokes:
A. Reflectory apnea
B. Reflectrory dyspnea
C. Reflectory hyperpnea
D. Cough
E. Bronchospasm

116. A man weighs 80 kg, after long physical activity his circulating blood
volume is reduced down to 5,4 l, hematocrit makes up 50%, whole blood
protein is 80 g/l. These blood characteristics are determined first of all by:
A. Water loss with sweat
B. Increased number of erythrocytes
C. Increased protein concentration in plasm
D. Increased circulating blood volume
E. Increased diuresis

117. A 16-year-old boy after an illness has diminished function of protein


synthesis in liver as a result of vitamin $K$ deficiency. It will cause disturbance
of:
A. Blood coagulation
B. Erythrocyte sedimentation rate
C. Anticoagulant generation
D. Erythropoietin secretion
E. Osmotic blood pressure

118. In response to a change in body position from horizontal to vertical blood


circulation system develops reflectory pressor reaction. Which of the following
is its compulsory component?
A. Systemic constriction of the venous vessels
B. Systemic dilatation of the arterial resistive vessels
C. Decrease in the circulating blood volume
D. Increase in the heart rate
E. Weakening of the pumbing ability of heart

119. Examination of a pregnant woman revealed twice as much concentration


of fibrinogen in blood plasm. What ESR can this woman have?
A. 40-50 mm/h
B. 10-15 mm/h
C. 2-12 mm/h
D. 5-10 mm/h
E. 0-5 mm/h

120. Introduction of a big dose of histamine to an experimental animal caused


abrupt drop of arterial pressure as a result of:
A. Dilatation of resistance vessels
B. Constriction of resistance vessels
C. Increase of heart rate
D. Decrease of heart rate
E. Decrease of heart rate and force

121. Systemic arterial pressure of an adult dropped from 120/70 to 90/50


mm Hg that led to reflectory vasoconstriction. The vasoconstriction will be
maximal in the following organ:
A. Bowels
B. Heart
C. Brain
D. Kidneys
E. Adrenals

122. Microelectrode technique allowed to register a potential following "all-


or-none" law and being able of undecremental spreading. Specify this
potential:
A. Action potential
B. Excitatory postsynaptic potential
C. Rest potential
D. Inhibitory postsynaptic potential
E. Receptor potential

123. Vagus branches that innervate heart are being stimulated during an
experiment. This caused reduction of heart rate due to the intensification of
the following process (through the cell membrane of cardiac pacemaker):
A. Potassium ion yield
B. Potassium ion entry
C. Calcium ion entry
D. Calcium ion yield
E. Calcium and potassium ion yield

124. Rest potential of a cell equals -80 mV. At what stage of action potential
did the membrane potential equal +30 mV?
A. Reverse polarization
B. After hyperpolarization
C. After depolarization
D. Depolarization

125. A 35-year-old man got an injury that caused complete disruption of


spinal cord at the level of the first cervical segment. What respiration changes
will be observed?
A. It will come to a standstill
B. No changes will be observed
C. Diaphragmal respiration will be maintained, thoracic respiration will disappear
D. Thoracic respiration will be maintained, diaphragmal respiration will disappear
E. It will become infrequent and deep

126. A doctor asked a patient to breath out fully after taking a normal breath.
What muscles contract during such exhalation?
A. Abdominal muscles
B. External intercostal muscles
C. Diaphragm
D. Trapezius muscles
E. Pectoral muscles

127. A man was intoxicated with mushrooms. They contain muscarine that
stimulates muscarinic cholinoreceptors. What symptoms signalize
intoxication with inedible mushrooms?
A. Myotic pupils
B. Mydriatic pupils
C. Bronchi dilatation
D. Increased heart rate
E. Rise of arterial pressure
128. A man presents with increased heart rate, mydriatic pupils, dry mouth.
This condition results from the activation of the following system of function
regulation:
A. Sympathetic
B. Parasympathetic
C. Metasympathetic
D. Vago-insular
E. Hypothalamo-pituitary-adrenal

129. In course of an experiment a peripheral section of vagus of an


expiremental animal is being stimulated. What changes will be observed?
A. Heart rate fall
B. Heart hurry
C. Pupil dilation
D. Increase of respiration rate
E. Bronchi dilation

130. Voluntary breath-holding caused increase of respiration depth and


frequency. The main factor stimulating these changes of external respiration
is:
A. Increased tension of CO2 in blood
B. Increased tension of O 2 in blood
C. Decreased tension of O2 in blood
D. Decreased tension of CO 2 in blood
E. Decreased concentration of H+ in blood

131. A patient has delayed conduction of excitement through the


atrioventricular node. What changes of ECG will be observed?
A. Prolongation of P-Q interval
B. Prolongation of Q-S interval
C. Negative T wave
D. S-T-segment displacement
E. Prolongation of Q-T interval

132. Surface with an intact toad on it was inclined to the right. Tone of
extensor muscles became reflectory higher due to the activation of the
following receptors:
A. Vestibuloreceptors of utricle and saccule
B. Vestibuloreceptors of semicircular ducts
C. Mechanoreceptors of foot skin
D. Photoreceptors of retina
E. Proprioreceptors

133. In course of an experiment a toad's right labyrinth was destroyed. It will


cause amyotonia of the following muscles:
A. Right extensors
B. Left flexors
C. Left extensors
D. Right flexors
E. Right and left extensors

134. An animal has an increased tonus of extensor muscles. This the result of
intensified information transmission to the motoneurons of the spinal cord
through the following descending pathways:
A. Vestibulospinal
B. Medial corticospinal
C. Reticulospinal
D. Rubrospinal
E. Lateral corticospinal

135. Workers of a hothouse farm work under conditions of unfavourable


microclimate: air temperature is +37oC, relative humidity is 90%, air speed is
0,2 m/s. The way of heat emission under these conditions will be:
A. Evaporation
B. Heat conduction
C. Convection
D. Radiation
E. All the ways

136. Lungs of a preterm infant have areas of atelectasis (pulmonary collapse).


The main cause is:
A. Surfactant deficiency
B. Increased viscous resistance
C. Underdeveloped inspiration muscles
D. Diminished force of surface tension of lungs
E. Surfactant excess

137. Vagi of an experimental animal were cut on both sides. What respiration
changes will be observed?
A. It will become deep and infrequent
B. It will become shallow and frequent
C. It will become deep and frequent
D. It will become shallow and infrequent
E. No changes will be observed

138. A cardiac electric stimulator was implanted to a 75-year-old man with


heart rate of 40 bpm. Thereafter the heart rate rose up to 70 bpm. The electric
stimulator has undertaken the function of the following heart part:
A. Sinoatrial node
B. Atrioventricular node
C. His' bundle branches
D. His' bundle fibers
E. Purkinje's fibers

139. A patient came to the hospital complaining about quick fatigability and
apparent muscle weakness. Examination revealed an autoimmune disease that
causes disorder of functional receptor condition in neuromuscular synapses.
What transmitter will be blocked?
A. Acetylcholine
B. Noradrenalin
C. Dopamine
D. Serotonin
E. Glycine

140. Which muscle contraction will be observed in the upper extremity during
holding (not moving) a load in a certain position?
A. Isometric
B. Isotonic
C. Auxotonic
D. Concentric
E. Excentric

141. Examination of a 35-year-old patient revealed high acidity of gastric


juice. What receptors should be blocked in order to reduce it?
A. Histamine
B. α1 -adrenoreceptors
C. α2 -adrenoreceptors
D. β1 -adrenoreceptors
E. β2 -adrenoreceptors

142. A young woman who entered a production department where it strongly


smelt of paints and varnishes had a bronchospasm. This reflex was caused by
irritation of the following receptors:
A. Irritant
B. Juxtaglomerular
C. Pleura receptors
D. Central chemoreceptors
E. Peripheral chemoreceptors

143. A 60-year-old patient presents with weakened peristaltic activity of the


bowels. Which of the following foodstuffs would stimulate peristalsis most of
all?
A. Brown bread
B. White bread
C. Meat
D. Lard
E. Tea
144. An isolated muscle fiber is under examination. It was established that
the threshold of stimulation force became significantly lower. What is the
cause of this phenomenon?
A. Activation of sodium channels of membrane
B. Activation of potassium channels of membrane
C. Inactivation of sodium channels of membrane
D. Inactivation of potassium channels of membrane
E. Block of energy production in the cell

145. It was established that agglutination of the recipient’s blood erythrocytes


had been caused by the standard sera from the I and II groups. Serum from the
III group as well as anti-Rh serum hadn’t provoke any agglutination. Which
blood group and rhesus is allowed to be transfused this recipient?
A. B, α (III) Rh-
B. A, β (II) Rh-
C. 0, α, β, (I) Rh+
D. AB (IV), Rh+
E. AB (IV), Rh-

146. A patient consumed a lot of reach in proteins food that caused increase
of rate of proteolytic enzymes of pancreatic juice. It is also accompanied by
increase of rate of the following enzyme:
A. Tripsin
B. Pepsin
C. Enterokinase
D. Gastricsin
E. Renin

147. In course of an experiment thalamocortical tracts of an animal were cut.


What type of sensory perception remained intact?
A. Olfactory
B. Auditory
C. Exteroreceptive
D. Visual
E. Nociceptive

148. Due to the use of poor-quality measles vaccine for preventive


vaccination, a 1-year-old child has developed an autoimmune renal injury. The
urine was found to contain macromolecular proteins. What process of urine
formation has been disturbed?
A. Filtration
B. Reabsorption
C. Secretion
D. Reabsorption and secretion
E. Secretion and filtration
149. In course of an experiment there has been an increase in the nerve
conduction velocity. This may be caused by an increase in the concentration of
the following ions that are present in the solution around the cell:
A. Na +
B. K+ and Cl-
C. K+ and Na+
D. Ca2 + and Cl-
E. Ca2 +

150. A patient has increased thickness of alveolar-capillary membrane caused


by a pathologic process. The direct consequence will be reduction of the
following value:
A. Diffusing lung capacity
B. Oxygen capacity of blood
C. Respiratory minute volume
D. Alveolar ventilation of lungs
E. Expiratory reserve volume
Krok 1 – 2015 Biology Base
1. Woman applied to the medico-genetic consulting centre for information
about the risk of haemophilia in her son. Her husband has been suffering
from this disease since birth. Woman and her parents are healthy (don't have
haemophilia). Is the boy likely to have the disease in this family?
A. All boys will be healthy
B. All boys will be ill
C. 50 % of the boys will be ill
D. 25 % of the boys will be ill
E. 75 % of the boys will be ill

2. A man suffering from a hereditary disease married a healthy woman. They


got 5 children, three girls and two boys. All the girls inherited their father’s
disease. What is the type of the disease inheritance?
A. Dominant, X-linked
B. Autosomal recessive
C. Asutosomal dominant
D. Y-linked
E. Recessive, X-linked

3. Electrocardiogram of a 45-year-old man showed absence of P-wave in all


the leads. What part of the conducting system is blocked?
A. Sinu-atrial node
B. Atrioventricular node
C. Common branch of the bundle of His
D. Branches of the bundle of His
E. Purkinje’s fibres

4. When a patient with traumatic impairment of the brain was examined, it was
discovered that he had stopped to distinguish displacement of an object on
the skin. What part of the brain was damaged?
A. Posterior central gurus
B. Occipital zone of the cortex
C. Parietal zone of the cortex
D. Frontal central gurus
E. Frontal zone

5. Different functional groups can be presented in the structure of L-amino


acid’s radicals. Identify the group that is able to form ester bond:
A. -OH
B. -SH
C. -CONH2
D. -CH3
E. -NH2
6. The conjugated protein necessarily contains special component as a non-
protein part. Choose the substance that can’t carry out this function:
A. HNO3
B. АТP
C. Thiamine pyrophosphate
D. AMP
E. E Glucose

7. Moving of the daughter chromatids to the poles of the cell is observed


in the mitotically dividing cell. On what stage of the mitotic cycle is
this cell?
A. Anaphase
B. Metaphase
C. Telophase
D. Prophase
E. Interfase

8. The patient with diabetes mellitus has been delivered in hospital in the
state of unconsciousness. Arterial pressure is low. The patient has
acidosis. Point substances, which accumulation in the blood results in
these manifestations:
A. Ketone bodies
B. Amino acids
C. Monosaccharides
D. High fatty acids
E. Cholesterol esters

9. A 58-year-old female has undergone surgery for necrotic bowel. Despite


having been treated with antibiotics, on postoperative day 5, she develops
symptoms (fever, hypotension, tachycardia, declining urine output, and
confusion) consistent with septic shock. What hemodynamic support would
be helpful?
A. Fluids and Dobutamine infusion
B. Dobutamine infusion
C. Antibiotic administration
D. Fluid administration
E. Atropine administration

10. It was proved that a molecule of immature mRNA (precursor mRNA)


contained more triplets than amino acids found in the synthesized protein.
The reason for that is that translation is normally preceded by:
A. Processing
B. Initiation
C. Reparation
D. Mutation
E. Replication

11. Examination of a patient revealed reduced contents of magnesium ions that


are necessary for attachment of ribosomes to the granular endoplasmatic
reticulum. It is known that it causes disturbance of protein biosynthesis.
What stage of protein biosynthesis will be disturbed?
A. Translation
B. Transcription
C. Replication
D. Aminoacid activation
E. Termination

12. Patient 54 year-old, 5th day after surgical operation. Blood count:
Erythrocytes 3,6*1012/l, Hemoglobin 95 g/l, Erythrocyte’s hemoglobin
content (color index) 0,78; Leukocytes 16*109 /l, Platelets 450*10 9 /l Blood
picture: anizocytosis, poikilocytosis, reticulocytes-3,8%. What anemia does
this patient have?
A. Acute posthemorragic anemia
B. Acquired hemolytic anemia
C. Anemia from iron deficiency
D. Hypoplastic anemia
E. Chronic posthemorragic anemia

13. The patient with pneumonia was treated with antibiotics for a long
period. After treatment patient complains of frequent and watery stool,
abdomenal pain. What is the reason of intestine function disorder?
A. Intestinal disbacteriosis development
B. Antibiotics toxic influence on the GIT
C. Autoimmune reaction development
D. Bacteria toxins influence
E. Hereditary enzyme defect

14. A 16 year-old patient got numerous traumas in automobile accident.


Now the patient is haning a shock. АP - 80/60 mm Hg. daily urine
volume 60-80 ml. What pathogenic mechanism leads to kidneys
function violation?
A. Decreased hydrostatic pressure in glomerular capillaries
B. Increased osmotic pressure in glomerular capillaries
C. Increased pressure in Bowman’s capsule
D. Increased vasopressin blood concentration
E. Trauma of the urinary bladder

15. A 46 year-old patient has complained of headache, fatigue, thirst, pains in


the spine and joints for the last 2 years. Clinically observed disproportional
enlargement of hands, feet, nose, superciliary arches. He notes that he
needed to buy bigger shoes three times. What is the main reason of such
disproportional enlargement of different parts of the body?
A. Cartilaginous tissue proliferation under growth hormone influence
B. Increased sensitivity of the tissues to growth hormone
C. Joints dystrophy development
D. Increased sensitivity of the tissues to insulin
E. Joints chronic inflammation development

16. A 55-year-old patient with continuing ventricular arrhythmias was admitted


to the hospital. The patient is taking timolol drops for glaucoma, daily insulin
injections for diabetes mellitus, and an ACE inhibitor for hypertension. You
have decided to use phenytoin instead of procainamide. What is the reason?
A. The anticholinergic effect of procainamide would aggravate
glaucoma
B. The local anesthetic effect of procainamide would potentiate diabetes
C. The hypertensive effects of procainamide would aggravate the hypertension
D. The local anesthetic effect of procainamide would aggravate the hypertension
E. The cholinergic effects of procainamide would aggravate the diabetes

17. A 25-year-old woman with red and itchy eczematoid dermatitis visits your
office. She had a dental procedure one day earlier with administration of a
local anesthetic. There were no other findings, although she indicated that
she had a history of allergic reactions. Which of the following drugs is most
likely involved?
A. Procaine
B. Cocaine
C. Lidocaine
D. Bupivacaine
E. Etidocaine

18. The CNS stimulation produced by methylxanthines, such as caffeine, is most


likely due to the antagonism of one of the following receptors:
A. Adenosine receptors
B. Glycine receptors
C. Glutamate receptors
D. GABA receptors
E. Cholinergic muscarinic receptors

19. Patients with similar complaints applied to the doctor: weakness, pain in the
intestines, disorder of GIT. Examination of the faeces revealed that one
patient with four nucleus cysts should be hospitalized immidiately. For what
protozoa are such cysts typical?
A. Dysenteric amoeba
B. Intestinal amoeba
C. Balantidium
D. Trichomonas
E. Lamblia
20. According to the data of WHO, for about 250 mln of Earth population fall ill
with malaria. This disease is mostly spread in tropical and subtropical
regions. Range of its spread falls into the areal of the following mosquitoes:
A. Anopheles
B. Culex
C. Aedes
D. Mansonia
E. Culiseta

21. Labeled aminoacids alanine and tryptophane were introducted to a mouse


in order to study localization of protein biosynthesis in its cells. Around
what organellas will the accumulation of labeled aminoacids be observed?
A. Ribosomes
B. Agranular endoplasmic reticulum
C. Cell centre
D. Lysosomes
E. Golgi apparatus

22. Highly injured person gradually died. Please choose the indicator of
biological death:
A. Autolysis and decay in the cells
B. Disarray of chemical processes
C. Loss of consciousness
D. Absence of palpitation
E. Absence of movements

23. After breathing with poisonous steams there is an increased quantity of


slime in respiratory passages of a chemical production worker. What of
respiratory tract epithelial cells participate in mucousa moistening?
A. Goblet cells
B. Fibroblasts
C. Endocrine cells
D. Langergans cells
E. Intercalated cells

24. A patient suffering from thyrotoxicosis symptoms of vegetoasthenic


syndrome was revealed. What of the following would show the histological
appearance of a thyroid gland being stimulated by thyroid-stimulating
hormone (TSH)?
A. Columnar-shaped follicular cells
B. Decreased numbers of follicular cells
C. Increased numbers of parafollicular cells
D. An abundance of colloid in the lumen of the follicle
E. Decreased numbers of parafollicular capillaries
25. The reason of occurrence of some diseases of an oral cavity is connected
with structural peculiarities of its mucous membrane. What morphological
attributes characterize these features?
A. No muscularis mucosa, stratified squamous epithelium
B. Transitional epithelium, no submucosa
C. Simple columnar ciliated epithelium
D. Well developed muscularis, no submucosa
E. Transitional epithelium, no muscularis mucosa

26. There is the change of teeth at the 6-8-year-old children: deciduous are
replaced by permanent. What embrionic tissues are the sources of formation
of permanent teeth tissues?
A. Ectodermal epithelium of a tooth plate and mesenhime
B. Entodermal epithelium of a tooth plate and mesenhime
C. Mesodermal epithelium and mesenhime
D. I, II brachial arches
E. Entodermal epithelium and mesoderm

27. The B cells of endocrine portion of pancreas are selectively damaged by


alloxan poisoning. How will it be reflected in blood plasma?
A. The content of sugar increases
B. The content of fibrinogen decrease
C. The level of sugar decreases
D. The content of globulins decreases
E. The content of albumins decreases

28. In some regions of South Africa there is a spread sickle-shaped cell


anemia, in which erythrocytes have shape of a sickle as a result of
substitution of glutamin by valine in the hemoglobin molecule. What is
the cause of this disease?
A. Gene mutation
B. Disturbance of mechanisms of genetic information realization
C. Crossingover
D. Genomic mutations
E. Transduction

29. A female patient consulted a physician about digestive disorder, extended


abdominal pain. Examination revealed drastic decrease in hemoglobin
concentration. It is known from the anamnesis that while living in the Far
East the patient used to eat freshly -salted caviar. Some relatives living with
her had the similar condition. What is the most likely diagnosis?
A. Diphyllobothriasis
B. Echinococcosis
C. Teniasis
D. Trichiniasis
E. Ascaridiasis

30. On autopsy of a still-born infant it is revealed heart development


abnormalities: ventricles are not separated, originates from the right part
single arterial trunk. For what class of vertebrate is such heart construction
characteristic?
A. Amphibian
B. Fishes
C. Reptiles
D. Mammals
E. Birds

31. If strong oxidizers get into the bloodstream, a methemoglobin is formed.


It is a compound, where iron (II) becomes iron (III). What has to be done
to save the patient?
A. Interchangeable hemotransfusion has to be done
B. Patient has to be exposed to the fresh air
C. He has to be calmed down and put to bed
D. He has to be given pure oxygen
E. Respiratory centers have to be stimulated

32. A 10-year-old child complains of weakness, nausea, irritability. Helminthes of


white color and 5-10 mm long were found on the underwear. On microscopy
of the scrape from the perianal folds achromic ova of the unsymmetrical form
were revealed. Indicate what helminth is parasiting on the child?
A. Enterobins vermicularis
B. Ascaris lumbricoides
C. ancylostoma duodenalis
D. Trichina
E. Trichuris

33. Parents with ill child came to the infectionist. They worked in one of the
Asian countries for a long time. Child has eathy colored skin, loss of
appetite, laxity, enlarged liver, spleen, peripheral glands. What protozoan
illness can this child have?
A. Visceral leishmaniasis
B. Balantidiasis
C. Amebiasis
D. Toxoplasmosis
E. Lambliasis

34. White-haired, with blue eyes girl was born in healthy parents. Irritability,
anxiety, troubled sleep and feeding developed in the first months of life of
the infant. What method of genetic investigation should be used for the
exact diagnosis?
A. Biochemical
B. Cytological
C. Twin
D. Genealogical
E. Population-statistical

35. The calcium canals of cardiomyocytes have been blocked on an isolated


rabbit's heart. What changes in the heart's activity can happen as a result?
A. Decreased rate and force of heart beat
B. Decreased heart beat rate
C. Decreased force of the contraction
D. Heart stops in systole
E. Heart stops in diastole

36. After the trauma, the patient’s right n.vagus was damaged. Which
violation of the cardiac activity is possible in this case?
A. Violation of the automatism of a Kiss-Fleck node
B. Violation of the automatism of a atrio-ventricular node
C. Violation of a conductivity in the right auricle
D. Block of a conductivity in the atrio-ventricular node
E. Arrhythmia

37. In the ovary specimen colored with hematoxylin-eosin, follicle is


determined where cubic-shaped follicle epithelium cells are placed in 1 -2
layers, and scarlet covering is seen around ovocyte. Name this follicle:
A. Primary
B. Primordial
C. Secondary
D. Mature
E. Atretic

38. While studing of the family tree with history of hypertrichosis


(hyperhirsutism of the ear) this sign was founded only in the men and it was
inherited from father to the son. Define the type of hypertrichosis
inheritance?
A. Connected with Y-chromosome
B. Autosomal- recessive
C. Autosomal-dominant
D. Connected with Х-chromosome recessive
E. Connected with Х-chromosome dominant

39. In the perianal folds of a 5-year-old girl her mother has found some
white "worms" that caused itch and anxiety in the child. The "worms"
were sent to the laboratory. During examination the physician saw white
filiform helminths 0,5-1 cm long, with pointed ends, some helminths
had twisted ends. What is the most likely diagnosis?
A. Enterobiasis
B. Diphyllobothriasis
C. Teniasis
D. Ascaridiasis
E. Opisthorchiasis

40. Part of the DNA chain turned about 180 degree due to gamma
radiation. What type of mutation took place in the DNA chain?
A. Inversion
B. Deletion
C. Doubling
D. Translocation
E. Replication

41. 46 chromosomes were revealed on karyotype examination of the 5-year-old


girl. One of the 15th pair of chromosomes is longer than usual due to
connected chromosome from the 21 pair. What type of mutation does this
girl have?
A. Translocation
B. Deletion
C. Inversion
D. Insufficiency
E. Duplication

42. A pregnant woman had been having toxicosis with severe repeated vomiting
for 24 hours. In the end of the day there appeared tetanic convulsions and fluid
loss. What shift of acid-base state caused these changes?
A. Excretory alkalosis
B. Gaseous alkalosis
C. Gaseous acidosis
D. Metabolic acidosis
E. Excretory acidosis

43. Genetic structure of eukaryote is "exon-intron-exon". This structure-


functional organization of gene caused transcription peculiarities. What will
be pro-i-RNA according to the schema?
A. Exon-intron-exon
B. Exon-exon-intron
C. Exon-exon
D. Intron-exon
E. Exon-intron

44. Medical examination at the military registration and enlistment office


revealed that a 15-year-old boy was high, with eunuchoid body proportions,
gynecomastia, female pattern of pubic hair distribution. The boy had also fat
deposits on the thighs, no facial hair, high voice, subnormal intelligence
quotient. Which karyotype corresponds with this disease?
A. 47, XXY
B. 45, XO
C. 46, XX
D. 46, XY
E. 47, XXX

45. RNA that contains AIDS virus penetrated into a leukocyte and by
means of reverse transcriptase forced a cell to synthetize a viral
DNA. This process is based upon:
A. Reverse transcription
B. Operon repression
C. Reverse translation
D. Operon depression
E. Convariant replication

46. According to the model of double DNA helix that was suggested by Watson
and Creek, it was established that one of chains would not be lost during
replication and the second chain would be synthesized complementary to
the first one. What way of replication is it?
A. Semiconservative
B. Analogous
C. Identical
D. Dispersed
E. Conservative

47. A teenager was irradiated with high radiation dose that resulted in
serious damages of lymphoid system, lysis of many lymphocytes.
Restoration of normal hemogram is possible due to the functioning of
the following gland:
A. Thymus
B. Thyroid
C. Liver
D. Pancreas
E. Adrenal

48. It was revealed that T-lymphocytes were affected by HIV. Virus enzyme -
reverse transcriptase (RNA-dependent DNA polymerase) - catalyzes the
synthesis of:
A. DNA on the matrix of virus mRNA
B. Virus informational RNA on the matrix of DNA
C. DNA on virus ribosomal RNA
D. Viral DNA on DNA matrix
E. mRNA on the matrix of virus protein
49. A patient after pathological process has a thickened alveolar
membrane. The direct consequence of the process will be the
reduction of:
A. Diffuse lung capacity
B. Oxygen capacity of blood
C. Minute respiratory capacity
D. Alveolar lung ventilation
E. Reserve expiratiory capacity

50. Autopsy of a newborn boy revealed polydactylia, microcephalia,


cheiloschisis and uranoschisis as well as hypertrophy of parenchimatous
organs. These defects correspond with the description of Patau's syndrome.
What is the most probable cause of this pathology?
A. Trisomy of the 13th chromosome
B. Trisomy of the 18th chromosome
C. Trisomy of the 21st chromosome
D. Nondisjunction of sex chromosomes
E. Partial monosomy

51. An ovary specimen stained by hematoxylin-eosin presents a follicle, where


cells of follicular epithelium are placed in 1 -2 layers and have cubic form,
there is a bright-red membrane around the ovocyte. What follicle is it?
A. Primary
B. Primordial
C. Secondary
D. Mature
E. Atretic

52. A patient has symptoms of inflammation of urogenital tracts. Examination of


a vaginal smear revealed big monocellular, pear-shaped organisms with the
pointed spike at the posterior end of body, big nucleus and undulating
membrane. What protozoa were found in the smear?
A. Trichomonas vaginalis
B. Trichomonas hominis
C. Trichomonas buccalis
D. Trypanosoma gambiense
E. Lamblia intestinalis

53. During regular examination of schoolchildren it was revealed that a 10


year old girl had asymmetric oval eggs with a larva in the scrape from her
perianal folds. What diagnosis should be made?
A. Enterobiasis
B. Ascariasis
C. Amebiasis
D. Trichocephalosis
E. Ankylostomiasis

54. A woman delivered a dead child with multiple developmental defects.


What protozoan disease might have caused the intrauterine death?
A. Toxoplasmosis
B. Leishmaniasis
C. Malaria
D. Amebiasis
E. Lambliasis

55. Tuberculosis can be treated by means of combined chemotherapy that


includes substances with different mechanisms of action.What
antituberculous medication inhibits transcription of RNA into DNA in
mycobacteria?
A. Rifampicin
B. Isoniazid
C. Streptomycin
D. Ethionamide
E. Para-aminosalicylic acid

56. A patient experienced a sudden temperature rise up to 39 oС. After 6 hours


the temperature normalized. On the 2-nd day the attack recurred: in the
period of paroxysm the temperature reached 41 oС, apyrexial period began
after 8 hours. What type of temperature profile is it?
A. Intermitting
B. Recurrent
C. Hectic
D. Septic
E. Continued

57. Larvae were detected occasionally on the microscopic examination of


the sputum of the patient with pneumonia. Eosinophiles were detected
on the blood examination. What helminthiasis can be diagnosed?
A. Ascariasis
B. Enterobiosis
C. Trichocephaliasis
D. Paragonimiasis
E. Opistorchis

58. You are studying functioning of a bacteria operon. The operator gene has
been released from the repressor gene. Immediately after this the following
process will start in the cell:
A. Transcription
B. Translation
C. Replication
D. Processing
E. Repression

59. While studying maximally spiralized chromosomes of human karyotype


the process of cell division was stopped in the following phase:
A. Metaphase
B. Prophase
C. Interphase
D. Anaphase
E. Telophase

60. It is known that the gene responsible for development of blood groups
according to AB0 system has three allele variants. If a man has IV blood
group, it can be explained by the following variability form:
A. Combinative
B. Mutational
C. Phenotypic
D. Genocopy
E. Phenocopy

61. A patient complains of pain in the area of his liver. Duodenal intubation
revealed yellowish, oval, narrowed at the poles eggs with an operculum at
the end. Size of these eggs is the smallest among all helminth eggs. What is
the most probable diagnosis?
A. Opisthorchosis
B. Teniasis
C. Beef tapeworm infection
D. Echinococcosis
E. Diphyllobothriasis

62. A patient consulted an urologist about pain during urination. Analysis of his
urine taken in the daytime revealed eggs with a characteristic sharp point. It
is known from the anamnesis that the patient has recently returned from
Australia. What is the most likely diagnosis?
A. Urogenital schistosomiasis
B. Intestinal schistosomiasis
C. Japanese schistosomiasis
D. Opisthorchiasis
E. Dicroceliasis

63. A child complains of general weakness, loss of appetite, a troubled sleep,


itching in the perianal area. The provisional diagnosis is enterobiasis. In
order to specify this diagnosis it is necessary to perform:
A. Scraping from perianal folds
B. Roentgenoscopy
C. Biopsy of muscle tissue
D. Immune diagnostics
E. Duodenal contents analysis

64. The cell of the laboratory animal was overdosed with Roentgen rays. As a
result albuminous fragments formed in the cytoplasm. What cell organoid
will take part at their utilization?
A. Lysosomes
B. Golgi complex
C. Ribosome
D. Endoplasmic reticulum
E. Cells centre

65. In course of practical training students studied a stained blood smear of


a mouse with bacteria phagocyted by leukocytes. What cell organella
completes digestion of these bacteria?
A. Lisosomes
B. Mytochondrions
C. Granular endoplasmic reticulum
D. Golgi apparatus
E. Ribosomes

66. A woman with ІІІ (B), Rh- blood group born a child with ІІ (A) blood group.
The child is diagnosed with hemolytic disease of newborn as a result of
rhesus incompatibility. What blood group is the child's father likely to have?
A. ІІ (A), Rh+
B. I (0), Rh+
C. III (B), Rh+
D. I (0), Rh-
E. II (A), Rh-

67. Two days after consumption of smoked pork a patient got face and eye-lid
edemata, gastrointestinal disturbances, abrupt temperature rise, muscle
pain. Blood analysis showed full-blown eosinophilia. What helminth could
the patient be infected with?
A. Trichina
B. Pinworm
C. Ascarid
D. Whipworm
E. Hookworm

68. A lymph node punctate of a patient with suspected protozoal disease was
examined. Examination of the stained specimen (Romanovsky's stain)
revealed some crescent bodies with pointed end, blue cytoplasm and red
nucleus. What protozoan were revealed in the smears?
A. Toxoplasms
B. Malarial plasmodiums
C. Dermotropic leishmania
D. Viscerotropic leishmania
E. Trypanosomes

69. The guide of the scientific expedition in India was native who always was
with his dog. What invasive diseases can be transmitted by the dog if it is
the source of invasion?
A. Echinococcosis
B. Teniasis
C. Paragonimiasis
D. Dicroceliasis
E. Fascioliasis

70. A patient has acne on his face. Microspcopic examination of scrapings from
the affected areas revealed living porrect vermiform arthropoda 0,2-0,5
mm large with four pairs of short extremities in the front part of their
bodies. What is the laboratory diagnosis?
A. Demodicosis
B. Scabies
C. Myiasis
D. Pediculosis
E. Phthiriasis

71. A family of students who came from Africa got a child with anemia signs. The
child died soon. Examination revealed that the child's erythrocytes have
abnormal semilunar shape. Specify genotypes of the child's parents:
A. Аа х Аа
B. Аа х аа
C. АА х АА
D. аа х аа
E. Аа х АА

72. Examination of cell culture got from a patient with lysosomal pathology
revealed accumulation of great quantity of lipids in the lysosomes. What of the
following diseases is this disturbance typical for?
A. Tay-Sachs disease
B. Gout
C. Phenylketonuria
D. Wilson disease
E. Galactosemia

73. A 39-year-old woman has madescence in the region of mammilla, a small


ulcer with inflammatory hyperemia and cutaneous edema. Histologic
examination of tissue sampling from this area revealed in the malpighian
layer of thickened epidermis atypical cells with light and optically empty
cytoplasm, with no intracellular bridges. Such cells were also found in the
orifice of big mammal gland ducts. What is the most probable diagnosis?
A. Paget's disease
B. Intraductal cancer
C. Basal cell carcinoma
D. Epidermoid cancer
E. Melanocarcinoma

74. Golgi complex exports substances from a cell due to the fusion of the
membrane saccule with the cell membrane. The saccule contents flows
out. What process is it?
A. Exocytosis
B. Endocytosis
C. Active transport
D. Facilitated diffusion
E. All answers are false

75. A patient working at a pig farm complains about paroxysmal abdominal


pain, liquid feces with admixtures of mucus and blood, headache, weakness,
fever. Examination of large intestine revealed ulcers from 1 mm up to several
cm large, feces contained oval unicellular organisms with cilia. What disease
should be suspected?
A. Balantidiasis
B. Amebiasis
C. Toxoplasmosis
D. Lambliasis
E. Trichomoniasis

76. A boy found a spider with the following morphological characteristics: it


is 2 cm long, has roundish black abdomen with two rows of red spots on
its dorsal side; four pairs of jointed limbs are covered with small black
hairs. What arthropod is it?
A. Karakurt spider
B. Scorpion
C. Solpuga
D. Mite
E. Tarantula

77. Normal, actively dividing cells of human red bone marrow are analyzed.
What number of cells' chromosomes is typical for G1 period?
A. 46
B. 48
C. 47
D. 45
E. 23
78. A man has worked in an African country for 3 years. A month after his
return to Ukraine he consulted an ophthalmologist and complained about
eye ache, eyelid edema, lacrimation and temporary visual impairment.
Underneath the eye conjunctiva the doctor revealed helminths 30-50 mm
long with elongated filiform body. What diagnosis might be suspected?
A. Filariasis
B. Diphyllobothriasis
C. Ascaridiasis
D. Enterobiasis
E. Trichocephaliasis

79. Life cycle of a cell includes the process of DNA autoreduplication. As a result
of it monochromatid chromosomes turn into bichromatid ones. What
period of cell cycle does this phenomenon fall into?
A. S
B. Go
C. G1
D. G2
E. M

80.A 28-year-old female patient consulted a gynecologist about sterility.


Examination revealed underdeveloped ovaries and uterus, irregular
menstrual cycle. Analysis of the sex chromatin revealed 2 Barr’s bodies in
most somatic cells. What chromosome disease is most likely?
A. Triple X Syndrome
B. Edwards Syndrome
C. Patau’s Syndrome
D. Klinefelter’s Syndrome
E. Turner’s Syndrome

81. A cell at the stage of mitosis anaphase was stimulated by


colchicine that inhibits chromosome separation to the poles. What
type of mutation will be caused?
A. Polyploidy
B. Inversion
C. Deletion
D. Duplication
E. Translocation

82. A patient in a transplantation centre underwent heart transplantation. The


organ was taken from a donor who died in a road accident. Foreign heart
can be rejected as a result of development of transplantation immunity. It is
usually prevented by means of:
A. Immunosuppressors
B. Chemotherapy
C. Ultrasound
D. Enzymes
E. X-ray therapy

83. Following exposure to radiation a lot of mutant cells appeared in a patient.


Some time later most of them were detected and destroyed by the following
cells of the immune system:
A. T-lymphocytes-killers
B. Plasmoblasts
C. T-lymphocytes-supressors
D. B-lymphocyte
E. Stem cells

84. A gynaecologist was examining a patient and revealed symptoms of genital


tract inflammation. A smear from vagina contains pyriform protozoa with a
spine, flagella at their front; there is also an undulating membrane. What
disease can be suspected?
A. Urogenital trichomoniasis
B. Lambliasis
C. Intestinal trichomoniasis
D. Toxoplasmosis
E. Balantidiasis

85. A boy has I (I^0I^0) blood group and his sister has IV (I^AI^B) blood
group. What blood groups do their parents have?
A. II (I А I 0) and III (I ВI 0)
B. II (I А I А ) and III (I ВI 0 )
C. I (I 0 I 0 ) and IV (I A I B)
D. III (I ВI 0 ) and IV (I A I B)
E. I (I 0 I 0 ) and III (I ВI 0 )

86. For the purpose of myocardium infarction treatment a patient was


injected with embryonal stem cells derived from this very patient by
means of therapeutic cloning . What transplantation type is it?
A. Autotransplantation
B. Allotransplantation
C. Xenotransplantation
D. Isotransplantation
E. Heterotransplantation

87. An alcoholic woman has born a girl with mental and physical
developmental lag. Doctors diagnosed the girl with fetal alcohol
syndrome. What effect is the cause of the girl's state?
A. Teratogenic
B. Mutagenic
C. Malignization
D. Carcinogenic
E. Mechanic
88. Sex chromosomes of a woman didn't separate and move to the opposite
poles of a cell during gametogenesis (meiosis). The ovum was impregnated
with a normal spermatozoon. Which chromosomal disease can be found in
her child?
A. Turner's syndrome
B. Down's syndrome
C. Patau's syndrome
D. Edwards' syndrome
E. Cat cry syndrome

89. Hartnup disease is caused by point mutation of only one gene which results
in disturbance of tryptophane absorption in the bowels and its resorption in
the renal tubules. It is the reason for disorder of both digestive and urination
systems. What genetic phenomenon is observed in this case?
A. Pleiotropy
B. Complementary interaction
C. Polymery
D. Codominance
E. Semidominance

90. Cytogenetic examination of a patient with reproductive dysfunction revealed


normal karyotype 46 ХY in some cells, but most cells have karyotype of
Klinefelter's syndrome - 47 ХХY. Such cell heterogenity is called:
A. Mosaicism
B. Inversion
C. Transposition
D. Duplication
E. E Monomorphism

91. As a result of prophylactic medical examination a 7 year old boy was


diagnosed with Lesch-Nyhan syndrome (only boys fall ill with it). The
boy's parents are healthy but his grandfather by his mother's side
suffers from the same disease. What type of disease inheritance is it?
A. Recessive, sex-linked
B. Dominant, sex-linked
C. Autosomal recessive
D. Autosomal dominant
E. Semidominance

92. A doctor revealed tissues injury on patient's scalp with localized


suppurations and diagnosed his disease as myiasis. This infestation is
caused by larvae of the following insect:
A. Wohlfahrt fly
B. Kissing bug
C. Stable fly
D. Malarial mosquito
E. Mosquito
Krok 1 – 2015 Anatomy Base
1. A 10-year-old child complains of weakness, nausea, irritability. Helminthes
of while color and 5-10 mm long were found on the underwear. On
microscopy of the scrape from the perianal folds achromic ovums of
unsymmetrical form were revealed. Indicate what helminth is parasiting on
the child?
A. Enterobins vermicularis
B. Ancylostoma duodenalis
C. Trichuris
D. Trichina
E. Ascaris lumbricoides

2. A patient has been brought to the hospital with the complaints of headache,
pain in left hypochondrium. He has been ill for 1,5 weeks. The sudden illness
began with the increase of body temperature up to 39,9oC. In 3 hours the
temperature decreased and hydropoiesis began. The attacks repeat
rhythmically in 48 hours. The patient had visited one an African country.
The doctors have suspected malaria. What method of laboratory diagnostics
is necessary to use?
A. Blood examination
B. Immunological tests
C. Stool examination
D. Examination of vaginal and urethral discharge
E. Urine examination

3. Slime, blood and protozoa 30-200 microns of length have been revealed in a
man's feces. The body is covered with cilias and has correct oval form with a
little bit narrowed forward and wide round shaped back end. On the forward
end a mouth is visible. In cytoplasm there are two nucleuses and two short
vacuoles. For whom are the described attributes typical?
A. Balantidium
B. Lamblia
C. Dysenteric amoeba
D. Trichomonas
E. Intestinal amoeba

4. A patient in three weeks after acute myocardial infarction has pain in the
heart and joints and pneumonia. What is the main mechanism of
development of post-infarction Dressler’s syndrome?
A. Autoimmune inflammation
B. Ischemia of myocardium
C. Resorption of enzymes from necrotized area of myocardium
D. Secondary infection
E. Vessels ' thrombosis

5. During the fetal period of the development in the vascular system of the fetus
large arterial (Botallo's) duct is functioning which converts into lig.
arteriosum after birth. What anatomical formations does this duct connect
between each other?
A. Pulmonary trunk and aorta
B. Right and left auricle
C. Aorta and inferior vena cava
D. Pulmonary trunk and superior vena cava
E. Aorta and superior vena cava

6. Young man felt sharp pain in the back during active tightening on the
horizontal bar. Objectively: pain while moving of upper extremity, limited
pronation and adduction functions. Sprain of what muscle is presented?
A. M. latissimus dorsi
B. M. levator scapulae
C. M. romboideus major
D. M. Trapezius
E. M. subscapularis

7. Children often have heavy nasal breathing resulting from excessive


development of lymphoid tissue of pharyngeal mucous membrane. What
tonsils growth may cause this effect?
A. Tonsilla pharyngea
B. Tonsilla palatina
C. Tonsilla lingualis
D. Tonsilla tubaria
E. All above mentioned tonsils

8. In course of laparotomy a surgeon revealed gangrenous lesion of


descending colon. It was caused by thrombosis of the following artery:
A. Sinister colic
B. Median colic
C. Dexter colic
D. Ileocolic
E. Superior mesenteric artery

9. Three separate bones connected with cartilage in the area of pelvis cavity are
noticed on the X-ray of the pelvis. What are these bones?
A. Iliac, pubic, sciatic
B. Pubic, sciatic, femoral
C. Sciatic, femoral, sacral
D. Iliac, sacral, coccyx
E. Sacral, pubic, coccyx

10. A 60-year-old patient has reduced perception of high-frequency sounds.


What structures' disorder of auditory analizer caused these changes?
A. Main membrane of cochlea near the oval window
B. Main membrane of cochlea near helicotrema
C. Eustachian tube
D. Muscles of middle ear
E. Tympanic membrane

11. The alveolar ventilation of the patient is 5 L/min, the breath frequency is
10 per/min, and the tidal volume is 700 ml. What is the patient's dead
space ventilation?
A. 2,0 L/min
B. 0,7 L/min
C. 1,0 L/min
D. 4,3 L/min

12. During investigation of patient, it was found formation in the white


substance of cerebral hemispheres with location in the knee and frontal
part of posterior crus of internal capsule. Fibres of what conductive tract
of the brain will be disrupted?
A. Tr. Pyramidalis
B. Tr. Frontothalamicus
C. Тr. Thalamocorticalis
D. Tr. Frontopontinus
E. Tr. parietooccipitopontinus

13. Only one factor can influence the charge of amino acid radicals in the active
centre of enzyme. Name this factor:
A. pH medium
B. Pressure
C. Temperature
D. The presence of a competitive inhibitor
E. The surplus of a product

14. Succinate dehydrogenase catalyses the dehydrogenation of succinate.


Malonic acid HOOC- CH2-COOH is used to interrupt the action of this
enzyme. Choose the inhibition type:
A. Competitive
B. Allosteris
C. Non-competitive
D. Limited proteolysis
E. Dephosphorylation

15. The gluconeogenesis is activated in the liver after intensive physical


trainings .What substance is utilized in gluconeogenesis first of all
in this case:
A. Lactate
B. Pyruvate
C. Glucose
D. Glutamate
E. Alanine

16. A patient with a stab wound of the anterior stomach wall is in surgical care.
What formation of abdominal cavity did the stomach contents get into?
A. Antegastrial bursa
B. Omental bursa
C. Hepatic bursa
D. Left mesenteric sinus
E. Right mesenteric sinus

17. A patient had been taking glucocorticoids for a long time. When the
preparation was withdrawn he developed the symptoms of disease
aggravation, decreased blood pressure and weakness. What is the reason
of this condition?
A. Appearance of adrenal insufficiency
B. Hyperproduction of ACTH
C. Sensibilization
D. Habituation
E. Cumulation

18. A patient has tissue ischemia below the knee joint accompanied
with intermittent claudication. What artery occlusion should be
suspected?
A. Popliteal artery
B. Peroneal artery
C. Posterior tibial artery
D. Anterior tibial artery
E. Proximal part of femoral artery

19. A patient, who suffers from congenital erythropoietic porphyria, has skin
photosensitivity. The accumulation of what compound in the skin can cause
it?
A. Uroporphyrinogen 1
B. Protoporphyrin
C. Uroporphyrinogen 2
D. Coproporphyrinogen 3
E. Heme

20. A patient has a malignisation of thoracic part of esophagus. What


lymphatic nodes are regional for this organ?
A. Anulus lymphaticus cardiae
B. Nodi lymphatici paratrachealis
C. Nodi lymphatici prevertebralis
D. Nodi lymphatici pericardiales laterales
E. Nodi lymphatici mediastinales posteriores

21. The patient has come to the hospital from the smelting workshop in
the condition of hyperthermia. What is the direct cause of loss of
consciousness at the heat stroke?
A. Decreased brain blood supply
B. Arterial pressure drop
C. Increased water loss through sweating
D. Decrease of heart output
E. Dilatation of peripheral vessels

22. A patient's blood was analyzed and the decreased erythrocyte’s


sedimentation rate (ESR) was discovered. What disease from the listed below
is accompanied with decreased ESR?
A. Polycytemia
B. Hepatitis
C. Splenomegaly
D. Vitamin B deficiency
E. Myocardial infarction

23. A 55-year-old patien was hospitalized in result of the trauma of the medial
group of femoral muscles. What kind of movements is the patient unable to
do?
A. Adduction of femur
B. Abduction of femur
C. Flexion of femur
D. Extension of femur
E. Suppination of femur

24. A mother of a newborn complains of her baby's constant belching with


undigested milk. Which developmental anomaly is it an evidence of?

Fiatula=abnormal connection between es


A. Esophageal atresia
B. Labium leporium
C. Faux lupinum phagus and trachea.
Atresia=birth defect esophagus not proper
D. Anal atresia

devel.
E. Esophageal fistula

25. During the endoscopy the inflammation of a major papilla of the


duodenum and the disturbances of bile secretion were found. In which part
of duodenum were the problems found?
A. Descendent part
B. Ascendant part
C. Bulb
D. Upper horizontal part
E. Lower horizontal part
26. A 18-year-old patient came to the out-patient department with the
complaints of bleeding trauma in the vestibule of his nose. On examination:
the mechanical injure of the mucous layer of the vestibule without
continuation into nasal cavity proper. What is the boundary between the
vestibule and nasal cavity proper?
A. Nasal limen
B. Nasal roller
C. Nasal septa
D. Choanes
E. Nostrils

27. A 32-year-old patient has been diagnosed with bartholinitis (inflammation


of Bartholin's glands ). In what part of the female urogenital system are the
Bartholin's glands located?
A. The labia major
B. The labia minor
C. The clitoris
D. The vagina
E. The uterus

28. A 50 year-old patient had hemorrhage of the brain and was taken to the
hospital. The place of hemorrhage was revealed on the lateral hemispheres
surfaces during the medical examination. What artety was injured ?
A. The middle cerebral artery
B. The anterior cerebral artery
C. The posterior cerebral artery
D. The anterior communicating artery
E. The posterior communicating artery

29. A 30-year-old patient was hospitalized due to bleeding of the facial artery .
What place on the face has to be pressed to stop bleeding?
A. The mandible’s edge
B. The mental process
C. The mandible’s branch
D. The nose’s back
E. The molar bone

30. A 19 year-old patient was diagnosed with appendicitis and was


hospitalized. The surgical operation on ablating appendix vermiformis is
to be performed. What artery must be fixed to stop bleeding during the
surgical operation?
A. The ileocolic artery
B. The colica dextra
C. The colica media
D. The colica sinistra
E. The iliac
31. A 45-year-old man fell on the right knee and felt the acute pain in the
joint. On examination: severe edema on the anterior surface of the knee
joint. Crunching sounds are heard while moving the joint. Which bone is
destroyed?
A. Knee-cap
B. Neck of the thigh bone
C. Left epicondyle of the thigh
D. Right epicondyle of the thigh
E. Head of the thigh bone

32. During the operation on the hip joint of a 5-year- old child her ligament
was damaged which caused bleeding.What ligament was damaged?
A. The head of the thigh
B. Perpendicular of the acetabule
C. Iliofemoral
D. Pubofemoral
E. Ischiofemoral

33. A 6-year-old child fell on the cutting object and traumatized soft tissues
between tibia and fibula . What kind of bone connection was injured?
A. Membrane
B. Suture
C. Ligament
D. Fontanel
E. Gomphosis

34. Usually the intravenous injection is done into median cubital vein
because it is slightly movable due to fixation by the soft tissues. What
does it fix in the cubital fossa?
A. Aponeurosis of biceps muscle
B. Tendon of the triceps muscle
C. Brachial muscle
D. Brachioradial muscle
E. Anconeus muscle

35. Victim has elbow joint trauma with avulsion of medial epicondyle of
humerus. What nerve can be damaged in this trauma?
A. Ulnar
B. Radial
C. Musculocutaneous nerve
D. Cardiac cutaneous nerve
E. Medial cutaneous nerve of forearm

36. A 54-year-old man was admitted to the hospital with complaints of pain in
the right subcostal region, vomiting with blood. Objectively: enlarged liver,
varicose veins in the stomach and esophagus. Disfunction of what vessel is
likely to be?
A. Vena porta
B. Aorta abdominalis
C. Vena hepatica
D. Vena cava superior
E. Vena cava inferior

37. Examination of a 2-year-old child revealed physical developmental lag,


the child often has pneumonias. The child was diagnosed with nonclosure
of ductus arteriosus. Haemodynamics disorder was caused by the
intercommunication of the following vessels:
A. Aorta and pulmonary trunk
B. Pulmonary trunk and pulmonary veins
C. Superior cava and aorta
D. Superior cava and pulmonary trunk
E. Aorta and pulmonary veins

38. A 5-year-old child was admitted to the otorhinolaryngological department


with diagnosis - suppurative inflammation of the middle ear. Disease
started from the inflammation of the nasopharynx. Through the what
canal of the temporal bone did the infection get into the tympanic cavity?
A. Musculortubal canal
B. Tympanic Canaliculus tympanicus
C. Carotid canal
D. Canaliculus chordal tympani
E. Canaliculi caroticotympanici

39. Inflammation of the tympanic cavity (purulent otitis media) was


complicated by inflammation of mammillary process sockets. What wall of
tympanic cavity did the pus penetrate into the sockets through?
A. Posterior
B. Anterior
C. Medial
D. Lateral
E. Superior

40. A 19-year-old female suffers from tachycardia in rest condition, weight loss,
excessive sweating, exophtalmos and irritability. What hormone would you
expect to find elevated in her serum?
A. Thyroxine
B. Cortisol
C. Mineralocorticoids
D. ACTH
E. Insulin
41. A 60-year-old patient was diagnosed with hypothalamic lateral nuclei stroke.
What changes in patient’s behavior may be expected?
A. The rejection of food
B. Aggressive behaviour
C. Depression
D. Thirst
E. Unsatisfied hunger

42. The process of heart transplantation determined the viability of


myocardial cells. The determination of what myocardium parameter
is the most important?
A. Rest potential of cardiomyocytes
B. Heart temperature
C. Concentration of oxygen in heart vessels
D. Concentration of calcium-ions in myofibrils
E. Concentration of Ca-ions in heart vessels

43. It is necessary to take the cerebrospinal fluid from a patient with suspected
inflammation of brain tunics. Diagnostic puncture was performed between
the arches of the lumbar vertebras. During the puncture the needle went
through the following ligament:
A. Yellow (flaval)
B. Iliolumbar
C. Anterior longitudinal
D. Posterior longitudinal
E. Intertransverse

44. Nowadays about 50 minor bases have been found in the t-RNA structure
besides the main four nitrogenous bases. Choose the minor nitrogenous
base:
A. Dihydrouracil
B. Uracil
C. Cysteine
D. Adenine
E. Cytosine

45. Obturative jaundice developed in a 60-year-old patient because of


malignant tumour of the big papillary of the duodenal. Lumen of what
anatomical structure is squeezed with tumour?
A. Hepatopancreatic ampulla
B. Cystic duct
C. Common hepatic duct
D. Right hepatic duct
E. Left hepatic duct
46. The patient with thymoma (thymus gland tumour) has cyanosis, extention
of subcutaneous venous net and edema of the soft tissues of face, neck,
upper part of the trunk and upper extremities. What venous trunk is
pressed with tumour?
A. Superior vena cava
B. External jugular vein
C. Clavicular vein
D. Internal jugular vein
E. Frontal jugular vein

47. A patient operated on complicated appendicitis has the following


changes of blood count: erythrocytes - 4,0*101 2/l, Нb - 120 g/l, color
index - 0,9, leukocytes – 18*109 /l, basophils - 0, eosinophils - 0,
myelocytes - 0, juvenile - 0, stab neutrophils - 20, segmentonuclear
neutrophils - 53, lymphocytes - 21, monocytes - 5. How is such nuclear
shift of leukocytic formula called?
A. Degenerative left shift
B. Right shift
C. Regenerative left shift
D. Hyperregenerative
E. Regeneratively-degenerative

48. Patient complains of frequent and difficult urination. Imperfection of what


formation can cause it?
A. Prostate
B. Testicles
C. Bulb-uretic glands
D. Testicle adnexa
E. Sperm bubbles

49. Child inspired button. Where is it likely tobe?


A.In the right main bronchus
B.In the left main bronchus
C.In the trachea
D.In the larynx
E.In the gullet

50. Vegetative abnormalities in the sleep, heat regulation, all kinds of


metabolism, diabetes insipidus are developing in the patient due to grouth
of the tumour in the III ventricle of brain. Irritation of the nucleus of what
part of the brain can cause this symptoms?
A. Hypothalamus
B. Cerebral peduncles (cruces cerebri)
C. Mesencephalic tegmentum
D. Pons cerebelli
E. Medulla
51. A patient with infectious mononucleosis had been taking glucocorticoids for
two weeks. He was brought into remission, but he fell ill with acute attack of
chronic tonsillitis. What action of glucocorticoids caused this complication?
A. Immunosuppressive
B. Anti-inflammatory
C. Antishock
D. Antiallergic
E. Antitoxic

52. A 45-year-old man with domestic apper arm injuiry came to the trauma
unit. The objective data are: there are no extension, adduction or
pronation functions of the arm. What muscle damage caused this
condition?
A. Teres major
B. Subscapular
C. Teres minor
D. Subspinous
E. Supraspinous

53. A man with internal abdominal right side injury and suspicion of liver
rupture was admitted to the traumatological department. In what peritonial
structure will blood accumulate?
A. Excavatio rectovesicalis
B. Bursa omentalis
C. Recessus intersigmoideus
D. Fossa ischio-analis
E. Recessus duodenalis inferior

54. A histological spaceman presents parenchymal organ, which has cortex and
medulla. Cortex consists of epitheliocytes bars with blood capillaries
between them; the bars form three zones. Medulla consists of
chromaffinocytes and venous sinusoids. Which organ has these
morphological features?
A. Adrenal gland
B. Kidney
C. Lymph node
D. Thymus
E. Thyroid

55. After trauma a 44-year-old patient had a rupture of left palm muscle
tendons and of the surface of blood vessels. After operation and removal
of the most part of the necrotically changed muscle tissue the
bloodstream was normalized. What vessels have helped with restoration
of bloodstream?
A. Arcus palmaris profundus
B. Arcus palmaris superficialis
C. Aa. digitales palmares communes
D. Aa. metacarpeae palmares
E. Aa. Perforantes

56. A 45-year-old man applied to the trauma unit because of domestic


shoulder trauma. Objectively: flexibility, reduction and pronation
functions of the shoulder are absent. What muscle was injured?
A. Teres major muscle
B. Subscapular muscle
C. Teres minor muscle
D. Infraspinous muscle
E. Supraspinous muscle

57. Ovarian tumour was diagnozed in the woman. Surgery was indicated. What
ligament should be cut by the surgeon to disconnect the ovary and the
uterus?
A. The ovarial ligament
B. Broad ligament of uterus
C. Lateral umbilical ligament
D. Suspensory ligament of ovary
E. Round ligament of uterus

58. An old woman was hospitalized with acute pain, edema in the right hip
joint; the movements in the joint are limited. Which bone or part of it was
broken?
A. The neck of the thigh
B. The body of the thigh bone
C. Condyle of the thigh
D. Pubic bone
E. Ischial bone

59. A 53-year-old female patient was diagnosed with liver rupture resulting
from a blunt abdominal injury. The escaped blood will be assembled in the
following anatomic formation:
A. Rectouterine pouch
B. Vesicouterine pouch
C. Right mesenteric sinus
D. Omental bursa
E. Left mesenteric sinus

60. A patient complains about edemata of legs, skin cyanosis, small ulcers on
one side of the lateral condyle. Examination revealed a swelling, enlarged
veins, formation of nodes. The pathological process has started in the
following vein:
A. V. saphena parva
B. V. saphena magna
C. V. femoralis
D. V. profunda femoris
E. V. iliaca externa

61. A 70 year old female patient was diagnosed with fracture of left femoral neck
accompanied by disruption of ligament of head of femur. The branch of the
following artery is damaged:
A. Obturator
B. Femoral
C. External iliac
D. Inferior gluteal
E. Internal pudendal

62. A woman underwent an operation on account of extrauterine (tubal)


pregnancy. In course of the operation the surgeon should ligate the
branches of the following arteries:
A. Uterine and ovarian
B. Superior cystic and ovarian
C. Inferior cystic and ovarian
D. Uterine and superior cystic
E. Uterine and inferior cystic

63. A 6 month old baby ill with bronchitis was taken for an X-ray of chest.
Apart of changes associated with bronchi the X-ray film showed a shadow
of thymus gland. What might have caused such changes?
A. The above-mentioned condition is a normal variant for this age
B. It's the effect of bronchitis
C. It is caused by abnormal position
D. It is caused by thymus inflammation
E. It is caused by neoplastic process

64. A 7-year-old child can't abduct the shoulder, raise it to the horizontal level.
He can raise the hand to the face only with dorsal side with abduction of the
shoulder (with help of supraspinous muscle ) - "bugler" arm. Active function
of what muscle is absent?
A. Deltoid
B. Infraspinous
C. Pectoral major
D. Teres minor
E. Teres major

65. A 7-year-old girl has signs of anemia. Laboratory examination revealed


pyruvate kinase deficiency in erythrocytes. What process disturbance
plays the main role in anemia development?
A. Anaerobic glycolysis
B. Oxidative phosphorylation
C. Tissue respiration
D. Peroxide decomposition
E. Aminoacids desamination

66. A patient complained about being unable to adduct and abduct fingers in
the metacarpophalangeal articulations towards and away from the 3rd
finger. Which muscles' function is impaired?
A. Interosseous muscles
B. Lumbrical muscles
C. Breviflexors of fingers
D. Long flexors of fingers
E. Extensors

67. A 45-year-old patient was admitted to the surgical department with


complaints of abrupt sharp pain in the epigastric region. After examination it
was diagnosed: perforated ulcer of the posterior wall of the stomach. Where
did content of the stomach flow out while perforation?
A. To the omental bursa
B. To the liver bursa
C. To the proventriculus sack
D. To the left mesenteric sinus
E. To the right mesenteric sinus

68. A 25-year-old patient complained of the decreased vision. Accommodation


disorders, dilated pupil, not reacting on the light were revealed on
examination. Function of what muscles is disturbed?
A. Pupil narrowing muscle, ciliary
B. Pupil dilating muscle, ciliary
C. Inferir oblique muscle, ciliary
D. Lateral rectus muscle, pupil narrowing
E. Pupil narrowing and dilating muscle

69. Microspecimen of spinal cord contains a nucleus that should be analyzed.


Its neurons form motor endings in the skeletal muscles. What nucleus of
spinal cord is meant?
A. Proper nucleus of the anterior horn
B. Thoracic nucleus
C. Intermediate lateral nucleus
D. Proper nucleus of the posterior horn
E. Proper nucleus of gray substance

70. A man with cut wound of his right foot sole was admitted to the hospital
ward. The patient has limited elevation of the lateral foot edge. In course of
wound management the injury of a muscle tendon was revealed. What
muscle is injured?
A. Long peroneal
B. Anterior tibial
C. Long extensor muscle of toes
D. Triceps muscle of crus
E. Short peroneal

71. A 35 year old man with a trauma of his left hand was admitted to the
traumatology department. Objectively: cut wound of palmar surface of left
hand; middle phalanxes of II–V fingers don't bend. What muscles are
damaged?
A. Superficial finger flexor
B. Profound finger flexor
C. Lumbrical muscles
D. Palmar interosseous muscles
E. Dorsal interosseous muscles

72. A 38-year-old patient came to a traumatology centre and complained about


an injury of his right hand. Objectively: the patient has a cut wound in the
region of the thenar eminence on the right hand; distal phalanx of the I
finger cannot be flexed. What muscle was injured?
A. Long flexor muscle of thumb
B. Short flexor muscle of thumb
C. Short abductor muscle of thumb
D. Opposer muscle of thumb
E. Abductor muscle of thumb

73. A patient with neuritis of femoral nerve has disturbed flexion of thigh as well
as disturbed crus extension in the knee joint. What muscle's function is
disturbed?
A. Quadriceps muscle of thigh
B. Biceps muscle of thigh
C. Triceps muscle of thigh
D. Semitendinous muscle
E. Semimembranous muscle

74. In the specimen of one of the parts of respiratory system a tubular organ was
found. It has low epithelium, well developed muscular tunic, glands and
cartilage are absent. Name this organ:
A. Minor bronchs
B. Trachea
C. Larynx
D. Major bronchs
E. Median bronchs

75. Where should the cathetor for evacuation of the lymph from the
thoracic lymph duct be inserted?
A. To the left venous corner
B. To the right venous corner
C. To the superior vena cava
D. To the inferior vena cava
E. To the left inguinal vein

76. A patient has difficulties with hand movement. Examination revealed


inflammation of common synovial sheath of flexor muscles. It is known from
the patient's anamnesis that he got a stab wound of finger a week ago. Which
finger was most probably damaged?
A. Digitus minimus
B. Pollex
C. Digitus medius
D. Index
E. Digitus anularis

77. An injured man has bleeding from branches of carotid artery. For a
temporary arrest of bleeding it is necessary to press the carotid artery to the
tubercle of a cervical vertebra. Which vertebra is it?
A. VI
B. V
C. IV
D. III
E. II

78. As a result of an accident a patient has intense painfullness and edema of


the anterior crus surface; dorsal flexion of foot is hindered. Function of
which crus muscle is most likely to be disturbed?
A. M.tibialis anterior
B. M.flexor digitorum longus
C. M.flexor hallucis longus
D. M.peroneus longus
E. M.peroneus brevis

79. A 40-year-old patient complains of intensive heartbeats, sweating, nausea,


visual impairment, arm tremor, hypertension. From his anamnesis: 2
years ago he was diagnosed with pheochromocytoma. Hyperproduction of
what hormones causes the given pathology?
A. Catecholamines
B. Aldosterone
C. Glucocorticoids
D. ACTH
E. Thyroidal hormones

80.A patient who suffers from cancer of back of tongue has an intense
bleeding as a result of affection of dorsal lingual artery by the tumour.
What vessel should be ligated to stop bleeding?
A. Lingual artery
B. Dorsal lingual artery
C. Deep lingual artery
D. Facial artery
E. Ascending pharyngeal artery

81. A 58-year-old patient with acute cardiac insufficiency has decreased


volume of daily urine - oliguria. What is the mechanism of this
phenomenon?
A. Decreased glomerular filtration
B. Decreased number of functioning glomerules
C. Drop of oncotic blood pressure
D. Rise of hydrostatic blood pressure in capillars
E. Reduced permeamility of renal filter

82. An 18-year-old man was delivered to the hospital after a road accident.
Examination at the traumatological department revealed multiple injuries
of soft tissues of face in the region of the medial eye angle. The injuries
caused massive haemorrhage. What arterial anastomosis might have been
damaged in this region?
A. a.carotis externa et a. carotis interna
B. a.carotis externa et a. subclavia
C. a.carotis interna et a. subclavia
D. a.subclavia et a. ophthalmica
E. a.carotis interna et a. ophthalmica

83. After a 2 y.o. child has had flu, there appeared complaints about ear ache. A
doctor revealed hearing impairment and inflammation of the middle ear.
How did the infection penetrate into the middle ear?
A. Through the auditory tube
B. Through foramen jugularis
C. Through canalis caroticus
D. Through atrium mastoideum
E. Through canalis nasolacrimalis

84. A surgeon has to find the common hepatic duct during the operative
intervention on account of concrements in the gall ducts. The common
hepatic duct is located between the leaves of:
A. Hepatoduodenal ligament
B. Hepatogastric ligament
C. Hepatorenal ligament
D. Round ligament of liver
E. Venous ligament

85. Cerebral trauma caused increase of ammonia formation. What


aminoacid takes part in removal of ammonia from cerebral tissue?
A. Glutamic
B. Tyrosine
C. Valine
D. Tryptophan
E. Lisine

86. A 40-year-old woman was admitted to the infectious diseases department


with high body temperature. Objectively: marked meningeal symptoms. A
spinal cord punction was made. What anatomic formation was
puncturated?
A. Spantiumsubarach noidem
B. Spatium subdurale
C. Spatium epidurale
D. Cavum triheminale
E. Cisterna cerebellomedullaris posterior

87. After resection of the middle third of femoral artery obliterated by a thromb
the lower extremity is supplied with blood due to the surgical bypass. Name
an artery that plays the main role in reestablishment of blood flow:
A. Deep femoral artery
B. Superficial circumflex artery of hip bone
C. Descending genicular artery
D. Superficial epigastric artery
E. Deep external pudendal artery

88. A patient's knee joint doesn't extend, there is no knee-jerk reflex, skin
sensitivity of the anterior femoral surface is disturbed. What nerve structures
are damaged?
A. Femoral nerve
B. Superior gluteal nerve
C. Big fibular nerve
D. Obturator nerve
E. Inferior gluteal nerve

89. The electronic microphoto of kidney fragment has exposed afferent


glomerular arteriole, which has giant cells under its endothelium,
containing secretory granules. Name the type of these cells:
A. Juxtaglomerular
B. Mesangial
C. Smoothmuscular
D. Juxtavascular
E. Interstitial

90. A 50 y.o. patient was admitted to the hospital with complaints about pain
behind his breastbone, asphyxia during physical activity. Angiography
revealed pathological changes in the posterior interventricular branch of the
right coronary artery. What heart parts are affected?
A. Posterior wall of the right and left ventricles
B. Left atrium
C. Anterior wall of the right and left ventricles
D. Right atrium
E. Right atrioventricular valve

91. A patient was admitted to the surgical department with suspected


inflammation of Meckel's diverticulum. What part of bowels should be
examined in order to discover the diverticulum in course of an operation?
A. Ileum
B. Duodenum
C. Jejunum
D. Caecum
E. Colon ascendens

92. Examination of a newborn boy's genitals revealed a cleft of urethra that


opens on the inferior surface of his penis. What developmental anomaly is
meant?
A. Hypospadia
B. Hermaphroditism
C. Epispadia
D. Monorchism
E. Cryptorchism

93. A patient complains about impaired evacuatory function of stomach


(long-term retention of food in stomach). Examination revealed a tumour
of initial part of duodenum. Specify localization of the tumour:
A. Pars Superior
B. Pars Inferior
C. Pars descendens
D. Pars ascendens
E. Flexura duodeni inferior

94. A man after 1,5 litre blood loss has suddenly reduced diuresis. The
increased secretion of what hormone caused such diuresis alteration?
A. Vasopressin
B. Corticotropin
C. Natriuretic
D. Cortisol
E. Parathormone

95. A young man consulted a doctor about disturbed urination. Examination of


his external genitals revealed that urethra is split on top and urine runs out
of this opening. What anomaly of external genitals development is the case?
A. Epispadia
B. Phimosis
C. Hermaphroditism
D. Paraphimosis
E. Hypospadia

96. A 35 year old patient applied to a doctor with complaints about having
intense rhinitis and loss of sense of smell for a week. Objectively: nasal cavity
contains a lot of mucus that covers mucous membrane and blocks olfactory
receptors. In what part of nasal cavity are these receptors situated?
A. Superior nasal turbinate
B. Median nasal turbinate
C. Inferior nasal turbinate
D. Common nasal meatus
E. Vestibule of nose

97. Inflammatory process of modified subserous layer around cervix of the


uterus caused an intensive pain syndrome. In what region of genitals
does the pathological process take place?
A. Parametrium
B. Mesometrium
C. Myometrium
D. Endometrium
E. Perimetrium

98. In course of an operation surgeon removed a part of a lung that was


ventilated by a tertiary bronchus accompanied by branches of pulmonary
artery and other vessels. What part of a lung was removed?
A. Bronchopulmonary segment
B. Middle lobe
C. Inferior lobe
D. Superior lobe
E. Pulmonary lobule

99. While examining the oral cavity a stomatologist revealed inflammation of


papillae on the border of the median and posterior third of the back of
tongue. What papillae are inflamed?
A. Papillae vallatae
B. Papillae fungiformes
C. Papillae foliatae
D. Papillae filiformes
E. Papillae conicae

100. Examination of a patient revealed an abscess of pterygopalatine


fossa. Where can the infection spread to unless the disease is managed
in time?
A. To the orbit
B. To the interpterygoid space
C. To the frontal sinus
D. To the subgaleal temporal space
E. To the tympanic cavity

101. During complicated labour the symphysis pubis ruptured. What


organ can be damaged mostly?
A. Urinary blader
B. Rectum
C. Ovaria
D. Uterine tubes
E. Uterus

102. A patient with cholelithiasis fell ill with mechanic jaundice.


Examination revealed that the stone was in the common bile duct. What
bile-excreting ducts make up the obturated duct?
A. Ductus hepaticus communis et ductus cysticus
B. Ductus hepaticus dexter et sinister
C. Ductus hepaticus dexter et ductus cysticus
D. Ductus hepaticus sinister et ductus cysticus
E. Ductus hepaticus communis et ductus choledochus

103. Neurological examination of a 65 y.o. patient revealed a


haemorrhage within the superior temporal gyrus. In the blood supply
area of which artery is it?
A. Middle cerebral artery
B. Anterior cerebral artery
C. Posterior cerebral artery
D. Anterior communicating artery
E. Basilar artery

104. A 70 y.o. man has cut an abscess off in the area of mammiform process
during shaving. Two days later he was admitted to the hospital with
inflammation of arachnoid membranes. How did the infection penetrate into
the cavity of skull?
A. V.emissariaе mastoideaе
B. V.v.labyrinthi
C. V.v.tympanicae
D. V.facialis
E. V.v.auriculares

105. The cerebrospinal fluid is being examined for the purpose of


diffrential meningitis diagnostics. At what site is the lumbal
puncture safe?
A. L III-L IV
B. L II-L III
C. L I-L II
D. Th XII-L I
E. L V-S I

106. While preparing a patient to the operation the heart chambers'


pressure was measured. In one of them the pressure changed during one
heart cycle from 0 to 120 mm Hg. What chamber of heart was it?
A. Left ventricle
B. Right ventricle
C. Right atrium
D. Left atrium

107. Part of alveoles of a preterm infant didn't spread because of enhanced


elastic recoil of lungs. How can this recoil be reduced?
A. By surfactant introduction
B. By pure oxygen inhalation
C. By artificial pulmonary ventilation
D. By fluid suction from the respiratory tracts
E. By glucose introduction

108. Preventive examination of a patient revealed an enlarged lymph node


of metastatic origin on the medial wall of the left axillary crease. Specify the
most likely localization of the primary tumour:
A. Mammary gland
B. Submandibular salivary gland
C. Lung
D. Stomach
E. Thyroid gland

109. A man suffering from osteochondrosis got acute pain in the abdominal
muscles (lateral and anterior). During objective examination a physician
diagnosticated increased pain sensitivity of skin in the hypogastric region.
This pain might be caused by affection of the following nerve:
A. Iliohypogastric
B. Sciatic
C. Obturator
D. Femoral
E. Genitofemoral

110. A woman suffering from osteochondrosis felt acute pain in her


humeral articulation that became stronger when she abducted her
shoulder. These symptoms might be caused by damage of the following
nerve:
A. Axillary nerve
B. Subscapular nerve
C. Dorsal scapular nerve
D. Subclavicular nerve
E. Throracodorsal nerve

111. Pyeloureterography X-ray photo showed a renal pelvis with minor


calyces only (major calyces were absent). What form of urinary tracts of a
kidney was revealed?
A. Embryonal
B. Fetal
C. Mature
D. Ampullar

112. A patient has a deep cut wound on the posterior surface of his
shoulder in its middle third. What muscle might be injured?
A. Triceps muscle of arm
B. Biceps muscle of arm
C. Anconeus muscle
D. Brachial muscle
E. Coracobrachial muscle

113. A patient has pain, edema and reddening of his skin in the
anterosuperior area of his thigh and his foot's thumb. What lymph nodes of
his lower extremity responded to the inflammatory process?
A. Superficial inguinal
B. Deep inguinal
C. Internal longitudinal
D. Superficial longitudinal
E. General longitudinal

114. A patient got a craniocerebral trauma that resulted in right-side


convergent strabismus. Damage of which craniocerebral nerve caused
such consequences?
A. n.abducens
B. n.facialis
C. n.trigeminus
D. n.trochlearis
E. n.aculomotorius

115. In case of a penetrating wound of the anterior abdominal wall the


wound tract went above the lesser curvature of stomach. What peritoneum
formation is most likely to be injured?
A. Ligamentum hepatogastricum
B. Ligamentum gastrocolicum
C. Ligamentum hepatoduoduodenale
D. Ligamentum hepatorenale
E. Ligamentum triangulare sinistrum

116. After a trauma a patient lost ability of elbow extension. This might
have been caused by dysfunction of the following main muscle:
A. m. triceps brachii
B. m. subscapularis
C. m. teres major
D. m. infraspinatus
E. m. levator scapulae

117. A patient has been diagnosed with a compression fracture of a lumbar


vertebra. As a result he has a considerable increase in curvature of the lumbar
lordosis. Which ligament damage can induce such changes in the spine
curvature?
A. Anterior longitudinal ligament
B. Posterior longitudinal ligament
C. Yellow ligament
D. Iliolumbar ligament
E. Interspinous ligament

118. After a craniocerebral trauma a patient lost the ability to execute


learned purposeful movements (apraxia). The injury is most likely localized
in the following region of the cerebral cortex:
A. Gyrus supramarginalis
B. Gyrus angularis
C. Gyrus paracentralis
D. Gyrus lingualis
E. Gyrus parahippocampalis

119. A patient got an injury of spinal marrow in a road accident


that caused loss of tactile sensation, posture sense, vibration sense.
What conduction tracts are damaged?
A. Fascicle of Goll and cuneate fascicle
B. Anterior spinocerebellar tract
C. Rubrospinal tract
D. Reticulospinal tract
E. Tectospinal tract

120. Examination of a patient revealed hypertrophy and inflammation of


lymphoid tissue, edema of mucous membrane between palatine arches (acute
tonsillitis). What tonsil is normally situated in this area?
A. Tonsilla palatina
B. Tonsilla pharyngealis
C. Tonsilla tubaria
D. Tonsilla lingualis
121. While performing an operation in the area of axillary crease a
surgeon has to define an arterial vessel surrounded by fascicles of
brachial plexus. What artery is it?
A. A.axillaris
B. A.vertebralis
C. A.transversa colli
D. A.profunda brachii
E. A.subscapularis

122. Examination of a patient with impaired blood coagulation revealed


thrombosis of a branch of inferior mesenteric artery. What bowel segment is
damaged?
A. Colon sigmoideum
B. Ileum Ant=aponeurosis of abd. Ext. Oblique M.
C. Caecum Post.=Transverse fascia.
D. Colon transversum Superficial=loose inf. Edge of trans abd M.
E. Colon ascendens INF.=inginal lig.

123. A patient was admitted to the surgical department with inguinal


hernia. During the operation the surgeon performs plastic surgery on
posterior wall of inguinal canal. What structure forms this wall?
A. Transverse fascia
B. Aponeurosis of abdominal external oblique muscle
C. Inguinal ligament
D. Loose inferior edge of transverse abdominal muscle
E. Peritoneum

124. A man with an injury of the dorsal area of his neck was
admitted to the resuscitation department. What muscle occupies
this area?
A. M.trapezius
B. M.sternocleidomastoideus
C. M.latissimus dorsi
D. M.rhomboideus minor
E. M.scalenus anterior

125. A patient complains of dizziness and hearing loss. What nerve is


damaged?
A. Vestibulocochlear
B. Trigeminus
C. Sublingual
D. Vagus
E. Trochlear

126. While palpating mammary gland of a patient a doctor revealed an


induration in form of a node in the inferior medial quadrant. Metastases may
extend to the following lymph nodes:
A. Parasternal
B. Posterior mediastinal
C. Profound lateral cervical
D. Bronchopulmonary
E. Superior diaphragmal

127. A patient got a trauma that caused dysfunction of motor centres


regulating activity of head muscles. In what parts of cerebral cortex is the
respective centre normally localized?
A. Inferior part of precentral gyrus
B. Superior part of precentral gyrus
C. Supramarginal gyrus
D. Superior parietal lobule
E. Angular gyrus

128. Examination of a 6-month-old child revealed a delay in closure of


the occipital fontanelle. When should it normally close?
A. Until 3 months
B. Before the child is born
C. Until 6 months
D. Until the end of the first year of life
E. Until the end of the second year of life

129. A patient was diagnosed with paralysis of facial and masticatory


muscles. The haematoma is inside the genu of internal capsule. What
conduction tract is damaged?
A. Tr. cortico-nuclearis
B. Tr. cortico-spinalis
C. Tr. cortico-thalamicus
D. Tr. cortico-fronto-pontinus
E. Tr. cortico-temporo-parieto-occipito-pontinus

130. A patient has lost skin sensitivity in the region of the medial surface
of his shoulder. This is the result of dysfunction of the following nerve:
A. Medial brachial cutaneous nerve
B. Medial antebrachial cutaneous nerve
C. Radial nerve
D. Ulnar nerve
E. Axillary nerve

131. A foreign body (a button) closed space of the right superior lobar
bronchus. What segments of the right lung won't be supplied with air?
A. Apical, posterior, anterior
B. Superior and inferior lingular
C. Apical and posterior basal
D. Apical and median basal
E. Medial and lateral

132. A patient was diagnosed with bartholinitis (inflammation of greater


vulvovaginal glands). In which organ of urogenital system are these
glands localized?
A. Large lips of pudendum
B. Small lips of pudendum
C. Clitoris
D. Vagina
E. Uterus

133. While performing an inguinal canal operation on account of hernia


a surgeon damaged the canal's contents. What exactly was damaged?
A. Funiculus spermaticus
B. Urarchus
C. Lig. teres uteri
D. Lig. Inguinalе

134. Ultrasonic examination of a patient revealed aneurism in the area of


aortic arch that caused alteration of vocal function of larynx. What nerve
was constricted?
A. Recurrent laryngeal
B. Diaphragmatic
C. Superior laryngeal
D. Mandibular
E. Sublingual

135. Surgical approach to the thyroid gland from the transverse (collar)
approach involves opening of interaponeurotic suprasternal space. What
anatomic structure localized in this space is dangerous to be damaged?
A. Jugular venous arch
B. External jugular vein
C. Subclavicular vein
D. Inferior thyroid arthery
E. Superior thyroid arthery

136. In course of a small pelvis operation it became necessary to ligate an


ovarian artery. What formation may be accidentally ligated together with
it?
A. Ureter
B. Uterine tube
C. Round ligament of uterus
D. Internal iliac vein
E. Urethra
137. An injured person was delivered to the hospital with a penetrating
wound in the left lateral region of abdomen. What part of the large
intestine is most likely damaged?
A. Colon descendens
B. Colon ascendens
C. Colon transverses
D. Caecum
E. Rectum

138. After a road accident a driver was delivered to the hospital with
an injury of the medial epicondyle of humerus. What nerve might be
damaged in this case?
A. n. Ulnaris
B. n. radialis
C. n. axillaris
D. n. muscolocutaneus
E. n. medianus

139. A patient with a knife wound in the left lumbal part was delivered to
the emergency hospital. In course of operation a surgeon found that internal
organs were not damaged but the knife injured one of muscles of renal pelvis.
What muscle is it?
A. Greater psoas muscle
B. Iliac muscle
C. Erector muscle of spine
D. Abdominal internal oblique muscle
E. Abdominal external oblique muscle

140. A 25 year old patient was examined by a medical board. Examination


revealed pathology of chest. Transverse dimensions were to small and the
sternum was strongly protruding. What chest type is it?
A. Keeled chest
B. Funnel chest
C. Flat chest
D. Cylindrical chest
E. Barrel chest

141. A man with a stab wound in the area of quadrilateral foramen applied to
a doctor. Examination revealed that the patient was unable to draw his arm
aside from his body. What nerve is most probably damaged?
A. N.axillaris
B. N.medianus
C. N.radialis
D. N.ulnaris
E. N.subclavius
142. Brain tomography revealed a tumour in the region of red
nucleus. What part of brain isdamaged?
A. Midbrain
B. Medulla oblongata
C. Cerebellum
D. Interbrain
E. Pons cerebell
Krok 1 – 2015 Biochemistry Base
1. After consumption of rich food a patient has nausea and heartburn,
steatorrhea. This condition might be caused by:
A. Bile acid deficiency
B. Increased lipase secretion
C. Disturbed tripsin synthesis
D. Amylase deficiency
E. Disturbed phospholipase synthesis

2. Galactosemia is revealed in the child. Concentration of glucose in the


blood is not considerably changed. Deficiency of what enzyme caused
this illness?
A. Galactose-1-phosphate uridyltransferase
B. Amylo-1,6-glucosidase
C. Phosphoglucomutase
D. Galactokinase
E. Hexokinase

3. Fatty of phospholipids is disordered due to fat infiltration of the liver.


Indicate which of the presented substances can enhance the process of
methylation during phospholipids synthesis?
A. Methionine
B. Ascorbic acid
C. Glucose
D. Glycerin
E. Citrate

4. Characteristic sign of glycogenosis is muscle pain during physical work.


Blood examination reveals usually hypoglycemia. This pathology is caused
by congenital deficiency of the following enzyme:
A. Glycogen phosphorylase
B. Glucose 6-phosphate dehydrogenase
C. Alpha amylase
D. Gamma amylase
E. Lysosomal glycosidase

5. An infant has apparent diarrhea resulting from improper feeding. One of the
main diarrhea effects is plentiful excretion of sodium bicarbonate. What form
of acid-base balance disorder is the case?
A. Metabolic acidosis
B. Metabolic alkalosis
C. Respiratory acidosis
D. Respiratory alkalosis
E. No disorders of acid-base balance will be observed
6. Methotrexate (structural analogue of the folic acid which is competitive
inhibitor of the dihydrofolatreductase) is prescribed for treatment of the
malignant tumour.On which level does methotrexate inhibit synthesis of
the nucleic acids?
A. Mononucleotide synthesis
B. Replication
C. Transcription
D. Reparation
E. Processing

7. RNA-polymeraseB(II) is blocked due to amanitine poisoning


(poison of death-cup). It disturbes:
A. Synthesis of m-RNA
B. Synthesis of t-RNA
C. Reverse transcription
D. Primers synthesis
E. Maturation of m-RNA

8. Pain along large nervous stems and increased amount of pyruvate in the
blood were revealed in the patient. Insufficiency of what vitamin can cause
such change?
A. В1
B. В2
C. РР
D. Pantothenic acid
E. Biotin

9. Patient with encephalopathy was admitted to the neurological in-patient


department. Correlation of increasing of encephalopathy and substances
absorbed by the bloodstream from the intestines was revealed.
Whatsubstances that are created in the intestines can cause endotoxemia?
A. Indole
B. Butyrate
C. Acetacetate
D. Biotin
E. Ornithine

10. Examination of a patient suffering from cancer of urinary bladder revealed


high rate of serotonin and hydroxyanthranilic acid. It is caused by excess of
the following amino acid in the organism:
A. Tryptophan
B. Alanine
C. Histidine
D. Methionine
E. Tyrosine
11. A mother consulted a doctor about her 5-year-old child who develops
erythemas, vesicular rash and skin itch under the influence of sun.
Laboratory studies revealed decreased iron concentration in the blood
serum, increased uroporphyrinogen I excretion with the urine. What is the
most likely inherited pathology in this child?
A. Erythropoietic porphyria
B. Methemoglobinemia
C. Hepatic porphyria
D. Coproporphyria
E. Intermittent porphyria

12. A 3 year old child with fever was given aspirin. It resulted in intensified
erythrocyte haemolysis. Hemolytic anemia might have been caused by
congenital insufficiency of the following enzyme:
A. Glucose 6-phosphate dehydrogenase
B. Glucose 6-phosphatase
C. Glycogen phosphorylase
D. Glycerol phosphate dehydrogenase
E. γ-glutamiltransferase

13. Blood of a 12 year old boy presents low concentration of uric acid and
accumulation of xanthine and hypoxanthine. This child has genetic
defect of the following enzyme:
A. Xanthine oxidase
B. Arginase
C. Urease
D. Ornithine carbamoyltransferase
E. Glycerylkinase

14. Increased amount of free fat acids is observed in the blood of the patients
with diabetes mellitus. It can be caused by:
A. Increased activity of triglyceridelipase adipocytes
B. Storage of palmitatoil-CoA
C. Activation of the ketone bodies utilization
D. Activation of the synthesis of the apolipoproteins
E. Decreased activity of phosphatidylcholine-cholesterol-acyltransferase blood
plasma

15. A 46-year-old female patient has a continuous history of progressive


muscular (Duchenne's) dystrophy. Which blood enzyme changes will be of
diagnostic value in this case?
A. Creatine phosphokinase
B. Lactate dehydrogenase
C. Pyruvate dehydrogenase
D. Glutamate dehydrogenase
E. Adenylate cyclase
16. A patient is ill with diabetes mellitus that is accompanied by hyperglycemia
of over 7,2 millimole/l on an empty stomach. The level of what blood plasma
protein allows to estimate the glycemia rate retrospectively (4-8 weeks
before examination)?
A. Glycated hemoglobin
B. Albumin
C. Fibrinogen
D. C-reactive protein
E. Ceruloplasmin

17. In case of enterobiasis acrihine - the structural analogue of vitamin B2 - is


administered. The synthesis disorder of which enzymes does this medicine
cause in microorganisms?
A. FAD-dependent dehydrogenases
B. Cytochromeoxidases
C. Peptidases
D. NAD-dependet dehydrogenases
E. Aminotransferases

18. A 10-year-old girl often experiences acute respiratory infections with


multiple spotty haemorrages in the places of clothes friction.
Hypovitaminosis of what vitamin is present at the girl?
A. С
B. В6
C. В1
D. А
E. В2

19. Hydroxylation of endogenous substrates and xenobiotics requires a donor


of protons. Which of the following vitamins can play this role?
A. Vitamin C
B. Vitamin P
C. Vitamin B6
D. Vitamin E
E. Vitamin A

20. The formation of a secondary mediator is obligatory in membrane-


intracellular mechanism of hormone action. Point out the substance that is
unable to be a secondary mediator:
A. Glycerol
B. Diacylglycerol
C. Inositol-3,4,5-triphosphate
D. CAMP
E. Ca2 +
21. A 4 y.o. child with signs of durative proteinic starvation was admitted to the
hospital. The signs were as follows: growth inhibition, anemia, edemata,
mental deficiency. Choose a cause of edemata development:
A. Reduced synthesis of albumins
B. Reduced synthesis of globulins
C. Reduced synthesis of hemoglobin
D. Reduced synthesis of lipoproteins
E. Reduced synthesis of glycoproteins

22. Researchers isolated 5 isoenzymic forms of lactate dehydrogenase from


the human blood serum and studied their properties. What property
indicates that the isoenzymic forms were isolated from the same enzyme?
A. Catalyzation of the same reaction
B. The same molecular weight
C. The same physicochemical properties
D. Tissue localization
E. The same electrophoretic mobility

23. On some diseases it is observed aldosteronism with hypertension


and edema due to sodium retention in the organism. What organ of
the internal secretion is affected on aldosteronism?
A. Adrenal glands
B. Testicle
C. Ovaries
D. Pancreas
E. Hypophysis

24. An experiment proved that UV-radiated cells of patients with xeroderma


pigmentosum restore the native DNA structure slower than cells of healthy
individuals as a result of reparation enzyme defection. What enzyme helps
this process?
A. Endonuclease
B. RNA ligase
C. Primase
D. DNA polymerase III
E. DNA gyirase

25. A patient with suspicion on epidemic typhus was admitted to the hospital.
Some arachnids and insects have been found in his flat. Which of them
may be a carrier of the pathogen of epidemic typhus?
A. Lice
B. Spiders
C. Bed-bugs
D. Cockroaches
E. Houseflies
26. A businessman came to India from South America. On examination the
physician found that the patient was suffering from sleeping-sickness. What
was the way of invasion?
A. As a result of bug's bites
B. As a result of mosquito's bites
C. With contaminated fruits and vegetables
D. Through dirty hands
E. After contact with a sick dogs

27. Tere is observed inhibited fibrillation in the patients with bile ducts
obstruction, bleeding due to low level of absorbtion of some vitamin. What
vitamin is in deficit?
A. К
B. А
C. D
D. Е
E. Carotene

28. A 52 year-old patient with bronchial asthma was treated with


glucocorticoids. Fever reaction appeared as a result of postinjective abscess.
The patient had subfebrile temperature, which didn’t correspond to latitude
and severity of inflammatory process. Why did patient have low fever
reaction?
A. Inhibited endogen pyrogens production
B. Violation of heat loss through lungs
C. Inflammatory barrier formation in injection place
D. Violation of heat-producing mechanisms
E. Thermoregulation center inhibition

29. A 35-year-old man under the treatment for pulmonary tuberculosis has
acute-onset of right big toe pain, swelling, and low-grade fever. The gouty
arthritis was diagnosed and high serum uric acid level was found. Which of
the following antituberculosis drugs are known for causing high uric acid
levels?
A. Pyrazinamide
B. Cycloserine
C. Thiacetazone
D. Rifampicin
E. Aminosalicylic acid

30. During metabolic process, active forms of the oxygen including superoxide
anion radical are formed in the human body. With help of what enzyme is
this anion activated?
A. Superoxide dismutase
B. Catalase
C. Peroxidase
D. Glutathioneperoxidase
E. Glutathionereductase

31. A patient presents high activity of LDH1,2, aspartate aminotransferase,


creatine phosphokinase. In what organ (organs) is the development of a
pathological process the most probable?
A. In the heart muscle (initial stage of myocardium infarction)
B. In skeletal muscles (dystrophy, atrophy)
C. In kidneys and adrenals
D. In connective tissue
E. In liver and kidneys

32. Buffer capacity of blood was decreased in the worker due to exhausting
muscular work. Entry of what acid substance to the blood can this state be
explained?
A. Lactate
B. Pyruvate
C. 1,3-bisphosphoglycerate
D. α-ketoglutarate
E. 3-phosphoglycerate

33. While examining the child the doctor revealed symmetric cheeks
roughness, diarrhea, disfunction of the nervous system. Lack of what
food components caused it?
A. Nicotinic acid, tryptophane
B. Lysine, ascorbic acid
C. Threonine, panthothenic acid
D. Methionine, lipoic acid
E. Phenylalanine, pangamic acid

34. A13-year-old boy complains of general weakness, dizziness, tiredness. He


is mentally retarded. Increased level of valine, isoleucine, leucine is in the
blood and urine. Urine has specific smell. What is the diagnosis?
A. Maple syrup urine disease
B. Addison's disease
C. Tyrosinosis
D. Histidinemia
E. Graves' disease

35. Increased breaking of vessels, enamel and dentine destruction in scurvy


patients are caused by disorder of collagen maturing. What stage of
modification of procollagen is disordered in this avitaminosis?
A. Hydroxylation of proline
B. Formation of polypeptide chains
C. Glycosylation of hydroxylysine residues
D. Removal of C-ended peptide from procollagen
E. Detaching of N-ended peptide
36. A 62-year-old female patient has developed a cataract (lenticular opacity)
secondary to the diabetes mellitus. What type of protein modification is
observed in case of diabetic cataract?
A. Glycosylation
B. Phosphorylation
C. ADP-ribosylation
D. Methylation
E. Limited proteolysis

37. Aspirin has antiinflammatory effect due to inhibition of the cyclooxygenase


activity. Level of what biological active acids will decrease?
A. Prostaglandins
B. Leucotriens
C. Catecholamines
D. Biogenic amines
E. Iodinethyronyns

38. At the aboratory experiment the eukocyte culture was mixed with
staphylococci. Neutrophile leukocytes engulfed and digested bacterial cells.
This processes are termed:
A. Phagocytosis
B. Pinocytosis
C. Diffusion
D. Facilitated diffusion
E. Osmosis

39. Examination of a patient revealed typical presentations of collagenosis.


This pathology is characterized by increase of the following urine index:
A. Hydroxyproline
B. Arginine
C. Glucose
D. Mineral salts
E. Ammonium salts

40. Marked increase of activity of МВ-forms of CPK (creatinephosphokinase)


and LDH-1 were revealed on the examination of the patient's blood. What is
the most likely pathology?
A. Miocardial infarction
B. Hepatitis
C. Rheumatism
D. Pancreatitis
E. Cholecystitis
41. Untrained people often have muscle pain after sprints as a result of lactate
accumulation. This might be caused by intensification of the following
biochemical process:
A. Glycolysis
B. Gluconeogenesis
C. Pentose phosphate pathway
D. Lipogenesis
E. Glycogenesis

42. A 16-year-old boy was performed an appendectomy. He has been hospitalized


for right lower quadrant abdominal pain within 18 hours. The surgical
specimen is edematous and erythematous. Infiltration by what of the
following cells is the most typical for the process occuring here?
A. Neutrophils
B. Eosinophils
C. Basophils
D. Limphocytes
E. Monocytes

43. ATP synthesis is totaly blocked in a cell. How will the value of
membrane rest potential change?
A. It will disappear
B. It will be slightly increased
C. It will be considerably increased
D. First it will increase, then decrease
E. First it will decrease, then increase

44. The concentration of albumins in human blood sample is lower than


normal. This leads to edema of tissues. What blood function is damaged?
A. Maintaining the oncotic blood pressure
B. Maintaining the Ph level
C. Maintaining the body temperature
D. Maintaining the blood sedimentation system
E. All answers are correct

45. Examination of a patient suffering from frequent haemorrhages in the


inner organs and mucous membranes revealed proline and lysine being
included in collagen fibers. Impairment of their hydroxylation is caused
by lack of the following vitamin:
A. C
B. E
C. K
D. A
E. D

46. A 20 year old patient complains of general weakness, dizziness, quick


fatigability. Blood analysis results: Hb- 80 g/l. Microscopical examination
results: erythrocytes are of modified form. This condition might be caused
by:
A. Sickle-cell anemia
B. Hepatocellular jaundice
C. Acute intermittent porphyria
D. Obturative jaundice
E. Addison's disease

47. A 48 year old patient complained about intense pain, slight swelling and
reddening of skin over the joints, temperature rise up to 38oC. Blood
analysis revealed high concentration of urates. This condition might be
caused by disturbed metabolism of:
A. Purines
B. Collagen
C. Cholesterol
D. Pyrimidines
E. Carbohydrates

48. A patient has yellow skin colour, dark urine, dark-yellow feces. What
substance will have strengthened concentration in the blood serum?
A. Unconjugated bilirubin
B. Conjugated bilirubin
C. Mesobilirubin
D. Verdoglobin
E. Biliverdin

49. A patient has an increased pyruvate concentration in blood. A large


amount of it is excreted with the urine. What vitamin is lacking in this
patient?
A. B1
B. E
C. B3
D. B6
E. B2

50. Ammonia is a very toxic substance, especially for nervous system. What
substance takes the most active part in ammonia detoxication in brain
tissues?
A. Glutamic acid
B. Lysine
C. Proline
D. Histidine
E. Alanine

51. A patient has pellagra. Interrogation revealed that he had lived mostly on
maize for a long time and eaten little meat. This disease had been caused by
the deficit of the following substance in the maize:
A. Tryptophan
B. Tyrosine
C. Proline
D. Alanine
E. Histidine

52. Increased production of thyroidal hormones T3 and T4, weight loss,


tachycardia, psychic excitement and so on present on thyrotoxicosis. How do
thyroidal hormones effect energy metabolism in the mitochondrion of cells?
A. Disconnect oxidation and oxidated phosphorylation
B. Activates phosphorylation of substance
C. Stops phosphorylation of substance
D. Stops respiratory chain
E. Activates oxidated phosphorylation

53. A patient with high rate of obesity was advised to use carnitine as a food
additive in order to enhance "fat burning". What is the role of carnitine
in the process of fat oxidation?
A. Transport of FFA (free fatty acids) from cytosol to the
mitochondria
B. Transport of FFA from fat depots to the tissues
C. It takes part in one of reactions of FFA beta-oxidation
D. FFA activation
E. Activation of intracellular lipolysis

54. An experimantal animal that was kept on protein-free diet developed


fatty liver infiltration, in particular as a result of deficiency of
methylating agents. This is caused by disturbed generation of the
following metabolite:
A. Choline
B. DOPA
C. Cholesterol
D. Acetoacetate
E. Linoleic acid

55. A patient consulted a doctor about symmetric dermatitis of open skin


areas. It was found out that the patient lived mostly on cereals and ate
too little meat, milk and eggs. What vitamin deficiency is the most
evident?
A. Nicotinamide
B. Calciferol
C. Folic acid
D. Biotin
E. Tocopherol
56. A 46 year old woman suffering from chololithiasis developed jaundice.
Her urine became dark-yellow and feces became colourless. Blood serum
will have the highest concentration of the following substance:
A. Conjugated bilirubin
B. Unconjugated bilirubin
C. Biliverdin
D. Mesobilirubin
E. Urobilinogen

57. A 46 year old patient applied to a doctor complaining about joint pain
that becomes stronger the day before weather changes. Blood
examination revealed strengthened concentration of uric acid. The most
probable cause of the disease is the intensified disintegration of the
following substance:
A. Adenosine monophosphate
B. Cytidine monophosphate
C. Uridine triphosphate
D. Uridine monophosphate
E. Thymidine monophosphate

58. A 42-year man suffering from gout has increased level of urinary acid in the
blood. Allopurinol was prescribed to decrease the level of urinary acid.
Competitive inhibitor of what enzyme is allopurinol?
A. Xanthinoxidase
B. Adenosinedeaminase
C. Adeninephosphoribosiltransferase
D. Hypoxantinphosphoribosiltransferase
E. Guaninedeaminase

59. A 38 year old patient suffers from rheumatism in its active phase.
What laboratory characteristic of blood serum is of diagnostic
importance in case of this pathology?
A. C-reactive protein
B. Uric acid
C. Urea
D. Creatinine
E. Transferrin

60. Patient experienced increased susceptibility of the skin to the sunlight.


His urine after some time became dark-red. What is the most likely cause
of this?
A. Porphyria
B. Hemolytic jaundice
C. Albinism
D. Pellagra
E. Alkaptonuria

61. A patient with serious damage of muscular tissue was admitted to


the traumatological department. What biochemical urine index will
be increased in this case?
A. Creatinine
B. Common lipids
C. Glucose
D. Mineral salts
E. Uric acid

62. 12 hours after an accute attack of retrosternal pain a patient presented a


jump of aspartate aminotransferase activity in blood serum. What
pathology is this deviation typical for?
A. Myocardium infarction
B. Viral hepatitis
C. Collagenosis
D. Diabetes mellitus
E. Diabetes insipidus

63. Donor skin transplantation was performed to a patient with extensive


burns. On the 8-th day the graft became swollen and changed colour; on
the 11-th day graft rejection started. What cells take part in this process?
A. T-lymphocytes
B. Erythrocytes
C. Basophils
D. Eosinophils
E. B-lymphocytes

64. A 30 y.o. woman had been ill for a year when she felt pain in the area of joints
for the first time, they got swollen and skin above them became reddened.
Provisional diagnosis is rheumatoid arthritis. One of the most probable
causes of this disease is a structure alteration of a connective tissue protein:
A. Collagen
B. Mucin
C. Myosin
D. Ovoalbumin
E. Troponin

65. Autopsy of a 12-year-old girl revealed: multiple cutaneous hemmorhages


(mostly into the skin of buttocks, lower extremities), serous and mucous
memrane hemmorhages, cerebral hemmorhages. Adrenal glands show focal
necrosis and massive hemmorhages; kidneys show necrotic nephrosis,
suppurative arthritis, iridocyclitis, vasculitis. What is the most probable
diagnosis?
A. Meningococcemia
B. Epidemic typhus
C. Periarteritis nodosa
D. Systemic lupus erythematosus
E. Radiation sickness

66. Examination of a 27-year-old patient revealed pathological changes in liver


and brain. Blood plasma analysis revealed an abrupt decrease in the copper
concentration, urine analysis revealed an increased copper concentration.
The patient was diagnosed with Wilson’s degeneration. To confirm the
diagnosis it is necessary to study the activity of the following enzyme in
blood serum:
A. Ceruloplasmin
B. Carbonic anhydrase
C. Xanthine oxidase
D. Leucine aminopeptidase
E. Alcohol dehydrogenase

67. A patient complains about dyspnea provoked by the physical activity.


Clinical examination revealed anaemia and presence of the paraprotein
in the zone of gamma-globulins. To confirm the myeloma diagnosis it is
necessary to determine the following index in the patient’s urine:
A. Bence Jones protein
B. Bilirubin
C. Haemoglobin
D. Ceruloplasmin
E. Antitrypsin

68. As a result of exhausting muscular work a worker has largely reduced buffer
capacity of blood. What acidic substance that came to blood caused this
phenomenon?
A. Lactate
B. Pyruvate
C. 1,3-bisphosphoglycerate
D. 3-phosphoglycerate

69. A 2-year-old child with mental and physical retardation has been delivered
to a hospital. He presents with frequent vomiting after having meals.
There is phenylpyruvic acid in urine. Which metabolism abnormality is
the reason for this pathology?
A. Amino-acid metabolism
B. Lipidic metabolism
C. Carbohydrate metabolism
D. Water-salt metabolism
E. Phosphoric calcium metabolism
70. A patient was delivered to the hospital by an emergency team. Objectively:
grave condition, unconscious, adynamy. Cutaneous surfaces are dry, eyes are
sunken, face is cyanotic. There is tachycardia and smell of acetone from the
mouth. Analysis results: blood glucose - 20,1 micromole/l (standard is 3,3-5,5
micromole/l), urine glucose - 3,5% (standard is - 0). What is the most
probable diagnosis?
A. Hyperglycemic coma
B. Hypoglycemic coma
C. Acute heart failure
D. Acute alcoholic intoxication
E. Anaphylactic shock

71. Profuse foam appeared when dentist put hydrogen peroxide on the mucous
of the oral cavity. What enzyme caused such activity?
A. Catalase
B. Cholinesterase
C. Acetyltransferase
D. Glucose-6-phosphatdehydrogenase
E. Methemoglobinreductase

72. A 62 y.o. woman complains of frequent pains in the area of her chest and
backbone, rib fractures. A doctor assumed myelomatosis (plasmocytoma).
What of the following laboratory characteristics will be of the greatest
diagnostical importance?
A. Paraproteinemia
B. Hyperalbuminemia
C. Proteinuria
D. Hypoglobulinemia
E. Hypoproteinemia

73. A newborn child has convulsions that have been observed after
prescription of vitamin B6. This most probable cause of this effect is that
vitamin B6 is a componet of the following enzyme:
A. Glutamate decarboxylase
B. Pyruvate dehydrostase
C. Metoglubarate dehydromine
D. Aminolevulinate synthase
E. Glycogen phosphorylase

74. Nappies of a newborn have dark spots that witness of formation of


homogentisic acid. Metabolic imbalance of which substance is it
connected with?
A. Thyrosine
B. Galactose
C. Methionine
D. Cholesterine
E. Tryptophane

75. Pathological changes of the liver and brain were revealed in a 27-year-old
patient.The copper concentration is abruptly decreased in blood plasma and
increased in the urine. Wilson's disease was diagnosed. Activity of what
enzyme in the blood serum should be examined to prove diagnisis?
A. Ceruloplasmin
B. Carboanhydraze
C. Xanthioxidase
D. Leucinamineopeptidaze
E. Alcoholdehydrogenaze

76. A 50-year-old patient complains about general weakness, appetite


loss and cardiac arrhythmia. The patient presents with muscle
hypotonia, flaccid paralyses, weakened peristaltic activity of the
bowels. Such condition might be caused by:
A. Hypokaliemia
B. Hypoproteinemia
C. Hyperkaliemia
D. Hypophosphatemia
E. Hyponatremia

77. An 18-year-old patient has enlarged inguinal lymphnodes, they are painless,
thickened on palpation. In the area of genital mucous membrane there is a
small-sized ulcer with thickened edges and "laquer" bottom of greyish
colour. What is the most probable diagnosis?
A. Syphilis
B. Tuberculosis
C. Lepra
D. Trophic ulcer
E. Gonorrhea

78. Concentration of pyruvate is increased in the patient's blood, the most of


which is excreted with urine. What avitaminosis is observed in the
patient?
A. Avitaminosis В1
B. Avitaminosis E
C. Avitaminosis В3
D. Avitaminosis B6
E. Avitaminosis В2

79. Carnitine including drug was recomended to the sportsman for


improving results. What process is activated most of all with help of
carnitine?
A. Transport of fatty acids to the mitochondria
B. Synthesis of steroid hormones
C. Synthesis of ketone bodies
D. Synthesis of lipids
E. Tissue respiration

80.A patient with suspected diphtheria went through bacterioscopic


examination. Examination of throat swab revealed rod-shaped bacteria
with volutin granules. What etiotropic preparation should be chosen in this
case?
A. Antidiphtheric antitoxic serum
B. Bacteriophage
C. Diphtheria antitoxin
D. Eubiotic
E. Interferon

81. Patient with diabetes mellitus experienced loss of consciousness and


convulsions after injection of insulin. What is the result of biochemical
blood analysis for concentration of the sugar?
A. 1,5 mmol/L
B. 8,0 mmol/L
C. 10,0 mmol/L
D. 3,3 mmol/L
E. 5,5 mmol/L

82. A woman who has been keeping to a clean-rice diet for a long time was
diagnosed with polyneuritis (beriberi). What vitamin deficit results in
development of this disease?
A. Thiamine
B. Ascorbic acid
C. Pyridoxine
D. Folic acid
E. Riboflavin

83. Removal of gall bladder of a patient has disturbed processes of Ca


absorption through the intestinal wall. What vitamin will stimulate this
process?
A. D3
B. PP
C. C
D. B12
E. K

84. A 63-year-old woman developed signs of rheumatoid arthritis. Increase of


which indicated blood values level could be helpful in proving diagnosis?
A. Additive glycosaminoglycans
B. Lipoproteids
C. Acid phosphatase
D. General cholesterol
E. R-glycosidase

85. A 1,5-year-old child presents with both mental and physical lag,
decolorizing of skin and hair, decrease in catecholamine concentration in
blood. When a few drops of 5% solution of trichloroacetic iron had been
added to the child’s urine it turned olive green. Such alteration are typical
for the following pathology of the amino acid metabolism:
A. Phenylketonuria
B. Alkaptonuria
C. Tyrosinosis
D. Albinism
E. Xanthinuria

86. A patient complains of frequent diarrheas, especially after consumption of


fattening food, and of body weight loss. Laboratory examination revealed
steatorrhea; hypocholic feces. What can be the cause of this condition?
A. Obturation of biliary tracts
B. Mucous membrane inflammation of small intestine
C. Lack of pancreatic lipase
D. Lack of pancreatic phospholipase
E. Unbalanced diet

87. On the empty stomach in the patients blood glucose level was 5,65 mmol/L,
in an hour after usage of sugar it was 8,55 mmol/L, in a 2 hours - 4,95
mmol/L. Such indicators are typical for:
A. Healthy person
B. Patient with hidden diabetes mellitus
C. Patient with insulin-dependent diabetes mellitus
D. Patient with non-insulin dependent diabetes mellitus
E. Patient with tireotoxicosis

88. A child is languid, apathetic. Liver is enlarged and liver biopsy revealed a
significant excess of glycogene. Glucose concentration in the blood stream is
below normal. What is the cause of low glucose concentration?
A. Low (absent) activity of glycogene phosphorylase in liver
B. Low (absent) activity of hexokinase
C. High activity of glycogen synthetase
D. Low (absent) activity of glucose 6-phosphatase
E. Deficit of a gene that is responsible for synthesis of glucose 1 -phosphaturidine
transferase

89. A 65 year old man suffering from gout complains of kidney pain.
Ultrasound examination revealed renal calculi. The most probable cause
of calculi formation is the strengthened concentration of the following
substance:
A. Uric acid
B. Cholesterol
C. Bilirubin
D. Urea
E. Cystine

90. A 65-year-old suffering from the gout man complains of the pain in the
kidney's region. On ultrasonic examination the renal calculi were revealed. As
a result of what process were they formed?
A. Decay of purine nucleotides
B. Protein catabolism
C. Ornithine cycle
D. Heme decay
E. Restoration of cysteine

91. A 5-year-old child who often fells ill with respiratory diseases has
eczematous appearances after consumption of some food products,
tendency to prolonged course of inflammatory processes. What kind of
diathesis can be suspected in this case?
A. Exudative-catharral
B. Hemmorhagic
C. Arthritism
D. Lymphohypoplastic
E. Asthenic

92. The greater amount of nitrogen is excreted from the organism in form of
urea. Inhibition of urea synthesis and accumulation of ammonia in blood
and tissues are induced by the decreased activity of the following liver
enzyme:
A. Carbamoyl phosphate synthetase
B. Aspartate aminotransferase
C. Urease
D. Amylase
E. Pepsin

93. A 35 y.o. patient who often consumes alcohol was treated with diuretics.
There appeared serious muscle and heart weakness, vomiting, diarrhea,
AP- 100/60 mm Hg, depression. This condition is caused by intensified
excretion with urine of:
A. Potassium
B. Sodium
C. Chlorine
D. Calcium
E. Phosphates

94. After intake of rich food a patient feels nausea and sluggishness; with time
there appeared signs of steatorrhea. Blood cholesterine concentration is 9,2
micromole/l. This condition was caused by lack of:
A. Bile acids
B. Triglycerides
C. Fatty acids
D. Phospholipids
E. Chylomicrons

95. Examination of a man who hadn't been consuming fats but had been
getting enough carbohydrates and proteins for a long time revealed
dermatitis, poor wound healing, vision impairment. What is the probable
cause of metabolic disorder?
A. Lack of linoleic acid, vitamins A, D, E, K
B. Lack of palmitic acid
C. Lack of vitamins PP, H
D. Low caloric value of diet
E. Lack of oleic acid

96. A 44-year-old woman complains of common weakness, heart pain,


considerable increase of body weigt. Objectively: moon-like face, hirsutism,
AP- 165/100 mm Hg, height - 164 cm, weight - 103 kg; fat is mostly
accumulated in the region of neck, upper shoulder girdle, stomach. What is
the main pathogenetic mechanism of obesity?
A. Increased production of glucocorticoids
B. Decreased production of thyroidal hormones
C. Increased production of insulin
D. Decreased production of glucagon
E. Increased production of mineralocorticoids

97. An experimental animal has been given excessive amount of carbon-labeled


glucose for a week. What compound can the label be found in?
A. Palmitic acid
B. Methionine
C. Vitamin A
D. Choline
E. Arachidonic acid

98. After a serious viral infection a 3-year-old child has repeated vomiting, loss of
consciousness, convulsions. Examination revealed hyperammoniemia. What
may have caused changes of biochemical blood indices of this child?
A. Disorder of ammonia neutralization in ornithinic cycle
B. Activated processes of aminoacids decarboxylation
C. Disorder of biogenic amines neutralization
D. Increased purtefaction of proteins in intestines
E. Inhibited activity of transamination enzymes
99. Examination of a patient with frequent hemorrhages from internals and
mucous membranes revealed proline and lysine being a part of collagene
fibers. What vitamin absence caused disturbance of their hydroxylation?
A. Vitamin C
B. Vitamin K
C. Vitamin A
D. Thiamine
E. Vitamin E

100. A 36-year-old female patient has a history of collagen disease.


Urine analysis is likely to reveal an increased concentration of the
following metabolite:
A. Oxyproline
B. Indican
C. Creatinine
D. Urea
E. Urobilinogen

101. Albinos can't stand sun impact - they don't aquire sun-tan but
get sunburns. Disturbed metabolism of what aminoacid underlies this
phenomenon?
A. Phenilalanine
B. Methionine
C. Tryptophan
D. Glutamic acid
E. Histidine

102. A patient with continious bronchopneumonia was admitted to the


therapeutic department. Antibiotic therapy didn't give much effect. What
medication for improvement of immune state should be added to the
complex treatment of this patient?
A. Timaline
B. Analgin
C. Sulfocamphocaine
D. Benadryl
E. Paracetamol

103. A patient suffers from hepatic cirrhosis. Examination of which of


the following substances excreted by urine can characterize the state of
antitoxic function of liver?
A. Hippuric acid
B. Ammonium salts
C. Kreatinine
D. Uric acid
E. Aminoacids

104. Vitamin A together with specific cytoreceptors penetrates through


the nuclear membranes, induces transcription processes that stimulate
growth and differentiation of cells. This biological function is realized by
the following form of vitamin A:
A. Trans-retinoic acid
B. Trans-retinal
C. Cis-retinal
D. Retinol
E. Carotin

105. Products of some proteins hydrolysis and modification are the


biologically active substances called hormones. Lipotropin, corticotropin,
melanotropin and endorphins are synthesized in the hypophysis of the
following protein:
A. Proopiomelanocortin (POMC)
B. Neuroalbumin
C. Neurostromin
D. Neuroglobulin
E. Thyreoglobulin

106. In patients with the biliary tract obstruction the blood coagulation is
inhibited; the patients have frequent haemorrhages caused by the subnormal
assimilation of the following vitamin:
A. K
B. A
C. D
D. E
E. C

107. The study of the genealogy of a family with hypertrichosis (helix


excessive pilosis) has demonstrated that this symptom is manifested in all
generations only in men and is inherited by son from his father. What is the
type of hypertrichosis inheritance?
A. Y-linked chromosome
B. Autosome-recessive
C. Autosome-dominant
D. X-linked recessive chromosome
E. X-linked dominant chromosome

108. A newborn child suffers from milk curdling in stomach, this means
that soluble milk proteins (caseins) transform to insoluble proteins
(paracaseins) by means of calcium ions and a certain enzyme. What
enzyme takes part in this process?
A. Renin
B. Pepsin
C. Gastrin
D. Secretin
E. Lipase
109. The penetration of the irritable cell membrane for potassium ions has
been increased during an experiment. What changes of membrane electric
status can occur?
A. Hyperpolarization
B. Depolarization
C. Action potential
D. Local response
E. No changes

110. A sportsman was recommended to take a medication that


contains carnitine in order to improve his results. What process is
activated by carnitine the most?
A. Fatty acids transport to mitochondrions
B. Synthesis of steroid hormones
C. Synthesis of ketone bodies
D. Synyhesis of lipids
E. Tissue respiration

111. To prevent postoperative bleeding a 6 y.o. child was administered


vicasol that is a synthetic analogue of vitamin K. Name post-translational
changes of blood coagulation factors that will be activated by vicasol:
A. Carboxylation of glutamin acid
B. Phosphorylation of serine radicals
C. Partial proteolysis
D. Polymerization
E. Glycosylation

112. A 4 y.o. boy has had recently serious viral hepatitis. Now there are such
clinical presentations as vomiting, loss of consciousness, convulsions. Blood
analysis revealed hyperammoniemia. Disturbunce of which biochemical
process caused such pathological condition of the patient?
A. Disturbed neutralization of ammonia in liver
B. Disturbed neutralization of biogenic amines
C. Increased putrefaction of protiens in bowels
D. Activation of aminoacid decarboxylation
E. Inhibition of transamination enzymes

113. During examination of an 11-month-old infant a pediatrician


revealed osteoectasia of the lower extremities and delayed
mineralization of cranial bones. Such pathology is usually provoked by
the deficit of the following vitamin:
A. Cholecalciferol
B. Thiamin
C. Pantothenic acid
D. Bioflavonoids
E. Riboflavin
114. Examination of a patient suffering from chronic hepatitis revealed a
significant decrease in the synthesis and secretion of bile acids. What process
will be mainly disturbed in the patient’s bowels?
A. Fat emulsification
B. Protein digestion
C. Carbohydrate digestion
D. Glycerin absorption
E. Amino acid absorption

115. The energy inputs of a healthy man have been measured. In what
position was the patient if his energy inputs were less than the main
exchange?
A. Sleep
B. Rest
C. Easy work
D. Nervous exertion
E. Calmness

116. Glutamate decarboxylation results in formation of inhibitory


transmitter in CNS. Name it:
A. GABA
B. Glutathione
C. Histamine
D. Serotonin
E. Asparagine

117. In course of histidine catabolism a biogenic amin is formed that has


powerful vasodilatating effect. Name it:
A. Histamine
B. Serotonin
C. Dioxyphenylalanine
D. Noradrenalin
E. Dopamine

118. Utilization of arachidonic acid via cyclooxigenase pathway


results in formation of some bioactive substances. Name them:
A. Prostaglandins
B. Thyroxine
C. Biogenic amins
D. Somatomedins
E. Insulin-like growth factors
119. After a sprint an untrained person develops muscle hypoxia. This leads
to the accumulation of the following metabolite in muscles:
A. Lactate
B. Ketone bodies
C. Acetyl CoA
D. Glucose 6-phosphate
E. Oxaloacetate

120. Myocyte cytoplasm contains a big number of dissolved


metabolites of glucose oxidation. Name one of them that turns directly
into a lactate:
A. Pyruvate
B. Oxaloacetate
C. Glycerophosphate
D. Glucose 6-phosphate
E. Fructose 6-phosphate

121. Emotional stress causes activation of hormon-sensitive triglyceride


lipase in the adipocytes. What secondary mediator takes part in this
process?
A. Cyclic adenosine monophosphate
B. Cyclic guanosine monophosphate
C. Adenosine monophosphate
D. Diacylglycerol
E. Ions of Са2 +

122. A patient has been diagnosed with alkaptonuria. Choose an enzyme


whose deficiency can be the reason for this pathology:
A. Homogentisic acid oxidase
B. Phenylalanine hydroxylase
C. Glutamate dehydrogenase
D. Pyruvate dehydrogenase
E. Dioxyphenylalanine decarboxylase

123. A patient diagnosed with carcinoid of bowels was admitted to the


hospital. Analysis revealed high production of serotonin. It is known that
this substance is formed of tryptophane aminooacid. What biochemical
mechanism underlies this process?
A. Decarboxylation
B. Desamination
C. Microsomal oxydation
D. Transamination
E. Formation of paired compounds

124. A genetics specialist analyzed the genealogy of a family and found


that both males and females may have the illness, not across all the
generations, and that healthy parents may have ill children. What is the
type of illness inheritance?
A. Autosomal recessive
B. Autosomal dominant
C. X-linked dominant
D. X-linked recessive
E. Y-linked

125. Analysis of amniotic fluid that was obtained as a result of


amniocentesis (puncture of amniotic sac) revealed cells the nuclei of which
contain sex chromatin (Barr's body). What can it be evidence of?
A. Development of female fetus
B. Development of male fetus
C. Genetic disorders of fetus development
D. Trisomy
E. Polyploidy

126. Vitamin B1 deficiency results in disturbance of


oxidative decarboxylation of α-ketoglutaric acid. This will
disturb synthesis of the following coenzyme:
A. Thiamine pyrophosphate
B. Nicotinamide adenine dinucleotide (NAD)
C. Flavine adenine dinucleotide (FAD)
D. Lipoic acid
E. Coenzyme A

127. A child's blood presents high content of galactose, glucose


concentration is low. There are such presentations as cataract, mental
deficiency, adipose degeneration of liver. What disease is it?
A. Galactosemia
B. Diabetes mellitus
C. Lactosemia
D. Steroid diabetes
E. Fructosemia

128. According to clinical indications a patient was administered


pyridoxal phosphate. What processes is this medication intended to
correct?
A. Transamination and decarboxylation of aminoacids
B. Oxidative decarboxylation of ketonic acids
C. Desamination of purine nucleotide
D. Synthesis of purine and pyrimidine bases
E. Protein synthesis

129. A 45 y.o. woman suffers from Cushing's syndrome - steroid


diabetes. Biochemical examination revealed: hyperglycemia,
hypochloremia. Which of the under-mentioned processes is the first to
be activated?
A. Gluconeogenesis
B. Glycogenolysis
C. Glucose reabsorption
D. Glucose transport to the cell
E. Glycolysis

130. Autopsy of a 46-year-old man revealed multiple brown-and-green


layers and hemmorhages on the mucous membrane of rectum and sigmoid
colon; slime and some blood in colon lumen; histologically - fibrinous colitis.
In course of bacteriological analysis of colon contents S.Sonne were found.
What is the most probable diagnosis?
A. Dysentery
B. Cholera
C. Salmonellosis
D. Yersiniosis
E. Crohn's disease

131. A patient had been ill with bronchial asthma for many years and died
from asthmatic fit. Histologic lung examination revealed: lumen of
bronchioles and small bronches contain a lot of mucus with some
eosinophils, there is sclerosis of alveolar septums, dilatation of alveole lumen.
What mechanism of development of hypersensibility reaction took place?
A. Reagin reaction
B. Cytotoxic reaction
C. Immunocomplex reaction
D. Cytolysis determined by lymphocytes
E. Granulomatosis

132. Desulfiram is widely used in medical practice to prevent alcocholism. It


inhibits aldehyde dehydrogenase. Increased level of what metabolite causes
aversion to alcochol?
A. Acetaldehyde
B. Ethanol
C. Malonyl aldehyde
D. Propionic aldehyde
E. Methanol

133. A 1-year-old child with symptoms of muscle involvement was


admitted to the hospital. Examination revealed carnitine deficiency in his
muscles. What process disturbance is the biochemical basis of this
pathology?
A. Transporting of fatty acids to mitochodrions
B. Regulation of Ca2 + level in mitochondrions
C. Substrate phosphorylation
D. Lactic acid utilization
E. Actin and myosin synthesis

134. The patient with complaints of permanent thirst applied to the doctor.
Hyperglycemia, polyuria and increased concentration of 17-ketosteroids in
the urine were revealed. What disease is the most likely?
A. Steroid diabetes
B. Insulin-dependent diabetes mellitus
C. Myxoedema
D. Type I glycogenosis
E. Addison's disease

135. Index of pH of the blood changed and became 7,3 in the patient with
diabetus mellitus. Detecting of the components of what buffer system is used
while diagnosing disorder of the acid-base equilibrium?
A. Bicarbonate
B. Phosphate
C. Hemoglobin
D. Oxyhemoglobin
E. Protein

136. As a result of posttranslative modifications some proteins taking part


in blood coagulation, particularly prothrombin, become capable of calcium
binding. The following vitamin takes part in this process:
A. K
B. C
C. A
D. B1
E. B2

137. Objective examination of a patient revealed: slender figure, big skull,


highly developed frontal region of face, short extremities. What
constitutional type is it characteristic for?
A. Respiratory
B. Muscular
C. Digestive
D. Cerebral
E. Mixed

138. A 38-year-old patient died during intractable attack of bronchial


asthma. Histologic examination revealed mucus accumulation in bronchial
lumen, a lot of fat cells (labrocytes) in the wall of bronches, many of them are
in the state of degranulation, there are also a lot of eosinophils. What
pathogenesis of bronchial changes is it?
A. Atopy
B. Cytotoxic, cytolytic action of antibodies
C. Immunocomplex mechanism
D. Cellular cytolysis
E. Granulomatosis

139. Diabetes mellitus causes ketosis as a result of activated oxidation


of fatty acids. What disorders of acid-base equilibrium may be caused by
excessive accumulation of ketone bodies in blood?
A. Metabolic acidosis
B. Metabolic alcalosis
C. Any changes woun't happen
D. Respiratory acidosis
E. Respiratory alcalosis

140. A woman with 0 (I) bllod group has born a child with AB blood group.
This woman's husband has A blood group. What genetic interaction explains
this phenomenon?
A. Recessive epistasis
B. Codominance
C. Polymery
D. Incomplete dominance
E. Complementation

141. Depressions and emotional insanities result from the deficit of


noradrenalin, serotonin and other biogenic amines in the brain. Their
concentration in the synapses can be increased by means of the
antidepressants that inhibit the following enzyme:
A. Monoamine oxidase
B. Diamine oxidase
C. L-amino-acid oxidase
D. D-amino-acid oxidase
E. Phenylalanine-4-monooxygenase

142. A 3 year old child with symptoms of stomatitis, gingivitis and


dermatitis of open skin areas was delivered to a hospital. Examination
revealed inherited disturbance of neutral amino acid transporting in the
bowels. These symptoms were caused by the deficiency of the following
vitamin:
A. Niacin
B. Pantothenic acid
C. Vitamin A
D. Cobalamin
E. Biotin

143. During hypersensitivity test a patient got subcutaneous injection of an


antigen which caused reddening of skin, edema, pain as a result of histamine
action. This biogenic amine is generated as a result of transformation of the
following histidine amino acid:
A. Decarboxylation
B. Methylation
C. Phosphorylation
D. Isomerization
E. Deaminization

144. A patient with suspected diagnosis "progressing muscular dystrophy"


got his urine tested. What compound will confirm this diagnosis if found in
urine?
A. Kreatine
B. Collagen
C. Porphyrin
D. Myoglobin
E. Calmodulin

145. A patient complained about dizziness, memory impairment, periodical


convulsions. It was revealed that these changes were caused by a product of
decarboxylation of glutamic acid. Name this product:
A. GABA
B. Pyridoxal phosphate
C. TDP
D. ATP
E. THFA

146. A sportsman needs to improve his sporting results. He was


recommended to take a preparation that contains carnitine. What process is
activated the most by this compound?
A. Fatty acids transporting
B. Amino acids transporting
C. Calcium ions transporting
D. Glucose transporting
E. Vitamin K transporting

147. A doctor examined a child and revealed symptoms of rachitis.


Development of this desease was caused by deficiency of the following
compound:
A. 1,25 [ОН]-dichydroxycholecalciferol
B. Biotin
C. Tocopherol
D. Naphtaquinone
E. Retinol

148. Laboratory examination of a child revealed increased concentration


of leucine, valine, isoleucine and their ketoderivatives in blood and urine.
Urine smelt of maple syrup. This disease is characterized by the deficit of
the following enzyme:
A. Dehydrogenase of branched amino acids
B. Aminotransferase
C. Glucose-6-phosphatase
D. Phosphofructokinase
E. Phosphofructomutase

149. A 9-month-old infant is fed with artificial formulas with unbalanced


vitamin B6 concentration. The infant presents with pellagral dermatitis,
convulsions, anaemia. Convulsion development might be caused by the
disturbed formation of:
A. GABA
B. Histamine
C. Serotonin
D. DOPA
E. Dopamine

150. It was found out that some compounds, for instance fungi toxins and
some antibiotics can inhibit activity of RNA-polymerase. What process will
be disturbed in a cell in case of inhibition of this enzyme?
A. Transcription
B. Processing
C. Replication
D. Translation
E. Reparation

151. When blood circulation in the damaged tissue is restored, then lactate
accumulation comes to a stop and glucose consumption decelerates. These
metabolic changes are caused by activation of the following process:
A. Aerobic glycolysis
B. Anaerobic glycolysis
C. Lipolysis
D. Gluconeogenesis
E. Glycogen biosynthesis

152. During starvation muscle proteins break up into free amino acids.
These compounds will be the most probably involved into the following
process:
A. Gluconeogenesis in liver
B. Gluconeogenesis in muscles
C. Synthesis of higher fatty acids
D. Glycogenolysis
E. Decarboxylation

153. Surgical removal of a part of stomach resulted in disturbed absorption


of vitamin B12, it is excreted with feces. The patient was diagnosed with
anemia. What factor is necessary for absorption of this vitamin?
A. Gastromucoprotein
B. Gastrin
C. Hydrochloric acid
D. Pepsin
E. Folic acid

154. A newborn develops dyspepsia after the milk feeding. When the milk
is substituted by the glucose solution the dyspepsia symptoms disappear.
The newborn has the subnormal activity of the following enzyme:
A. Lactase
B. Invertase
C. Maltase
D. Amylase
E. Isomaltase

155. Patients who suffer from severe diabetes and don't receive insulin
have metabolic acidosis. This is caused by increased concentration of the
following metabolites:
A. Ketone bodies
B. Fatty acids
C. Unsaturated fatty acids
D. Triacylglycerols
E. Cholesterol

156. A 4 year old child with hereditary renal lesion has signs of rickets,
vitamin D concentration in blood is normal. What is the most probable
cause of rickets development?
A. Impaired synthesis of calcitriol
B. Increased excretion of calcium
C. Hyperfunction of parathyroid glands
D. Hypofunction of parathyroid glands
E. Lack of calcium in food

157. A 6 year old child was delivered to a hospital. Examination revealed


that the child couldn't fix his eyes, didn't keep his eyes on toys, eye ground
had the cherry-red spot sign. Laboratory analyses showed that brain, liver
and spleen had high rate of ganglioside glycometide. What congenital
disease is the child ill with?
A. Tay-Sachs disease
B. Wilson's syndrome
C. Turner's syndrome
D. Niemann-Pick disease
E. MacArdle disease
158. In clinical practice tuberculosis is treated with izoniazid preparation
- that is an antivitamin able to penetrate into the tuberculosis bacillus.
Tuberculostatic effect is induced by the interference with replication
processes and oxidation-reduction reactions due to the buildup of pseudo-
coenzyme:
A. NAD
B. FAD
C. FMN
D. TDP
E. CoQ

159. A newborn child was found to have reduced intensity of sucking,


frequent vomiting, hypotonia. Urine and blood exhibit increased
concentration of citrulline. What metabolic process is disturbed?
A. Ornithinic cycle
B. Tricarboxylic acid cycle
C. Glycolysis
D. Glyconeogenesis
E. Cori cycle

160. A male patient has been diagnosed with acute radiation disease.
Laboratory examination revealed a considerable reduction of platelet
serotonin level. The likely cause of platelet serotonin reduction is the
disturbed metabolism of the following substance:
A. 5-oxytryptofane
B. Tyrosine
C. Histidine
D. Phenylalanine
E. Serine

161. Dietary intake of a 30 year old nursing woman contains 1000 mg of


calcium, 1300 mg of phosphorus and 20 mg of iron per day. It is necessary
to change content of these mineral substances in the following way:
A. To increase phosphorus content
B. To increase calcium content
C. To reduce fluorine content
D. To increase iron content
E. To reduce iron content

162. Cardinal symptoms of primary hyperparathyroidism are osteoporosis


and renal lesion along with development of urolithiasis. What substance
makes up the basis of these calculi in this disease?
A. Calcium phosphate
B. Uric acid
C. Cystine
D. Bilirubin
E. Cholesterol

163. Study of conversion of a food colouring agent revealed that


neutralization of this xenobiotic takes place only in one phase - microsomal
oxydation. Name a component of this phase:
A. Cytochrome Р-450
B. Cytochrome B
C. Cytochrome C
D. Cytochrome A
E. Cytochrome oxidase

164. A patient had hemorrhagic stroke. Blood examination revealed


strengthened kinin concentration.The patient was prescribed contrical. It was
administered in order to inhibit the following proteinase:
A. Kallikrein
B. Pepsin
C. Trypsin
D. Chemotrypsin
E. Collagenase

165. A 49-year-old driver complains about unbearable constricting pain


behind the breastbone irradiating to the neck. The pain arose 2 hours ago.
Objectively: the patient’s condition is grave, he is pale, heart tones are
decreased. Laboratory studies revealed high activity of creatine kinase and
LDH1. What disease are these symptoms typical for?
A. Acute myocardial infarction
B. Acute pancreatitis
C. Stenocardia
D. Cholelithiasis
E. Diabetes mellitus

166. Plasmic factors of blood coagulation are exposed to post-translational


modification with the participation of vitamin K. It is necessary as a
cofactor in the enzyme system of γ-carboxylation of protein factors of blood
coagulation due to the increased affinity of their molecules with calcium
ions. What amino acid is carboxylated in these proteins?
A. Glutamic
B. Valine
C. Serine
D. Phenylalanine
E. Arginine

167. Pharmacological effects of antidepressants are connected with


inhibition of an enzyme catalyzing biogenic amines noradrenaline and
serotonine in the mitochondrions of cerebral neurons. What enzyme
participates in this process?
A. Monoamine oxidase
B. Transaminase
C. Decarboxylase
D. Peptidase
E. Lyase

168. An oncological patient was prescribed methotrexate. With the lapse of


time target cells of the tumour lost susceptibility to this drug. There is change
of gene expression of the folowing enzyme:
A. Dehydrofolate reductase
B. Thiaminase
C. Deaminase
D. Folate oxidase
E. Folate decarboxylase
Krok 1 – 2015 Histology Base
1. A patient visited a dentist with complaints of redness and edema of his
mouth mucous membrane in a month after dental prosthesis. The
patient was diagnosed with allergic stomatitis. What type of allergic
reaction by Gell and Cumbs underlies this disease?
A. Delayed type hypersensitivity
B. Cytotoxic
C. Immunocomplex
D. Anaphylactic
E. Stimulating

2. An electronic microphotograph shows a macrophagic cell with erythrocytes at


different stages of differentiation located along its processes. This is the cell
of the following organ:
A. Red bone marrow
B. Thymus
C. Spleen
D. Tonsil
E. Lymph node

3. Decreased blood supply to the organs causes hypoxia that activates


fibroblasts function. Volume of what elements is increased in this case?
A. Intercellular substance
B. Vessels of microcircular stream
C. Nerve elements
D. Parenchymatous elements of the organ
E. Lymphatic vessels

4. A patient has undergone an amputation of lower extremity. Some time


later painful nodules appeared in a stump. Amputatious neuromas were
found out at the microscopic examination. To what pathological processes
do those formations relate?
A. Regeneration
B. Dystrophy
C. Inflammation
D. Hyperemia
E. Metaplasia

5. A 22-year-old patient was admitted to the hospital with complaints of heavy


nasal breathing. During the examination of her nasal cavity the doctors
found thickened mucous membrane, a lot of mucus and nodular infiltrates
without erosions in the nose. The nasal rhinoscleroma was diagnosed. The
biopsy was taken. What typical morphological changes may be found?
A. Granulomas with Mikulicz’s cells
B. Granulomas with Virchow’s cells
C. Granulomas with Langhan’s cells
D. Granulomas with foreign body cells
E. Interstitial inflammation

6. Lung of premature infant is presented on electronic photomicrography of


biopsy material. Collapse of the alveolar wall caused by the deficiency of
surfactant was revealed. Disfunction of what cells of the alveolar wall caused
it?
A. Alveocytes type II
B. Alveocytes type I
C. Alveolar macrophages
D. Secretory cells
E. Fibroblasts

7. During histological examination of the stomach it was found out that glands
contain very small amount of pariental cells or they are totally absent.
Mucose membrane of what part of the stomach was studied?
A. Pyloric part
B. Fundus of stomach
C. Cardiak part
D. Body of stomach

8. Live vaccine is injected into the human body. Increasing activity of what
cells of connective tissue can be expected?
A. Plasmocytes and lymphocytes
B. Macrophages and fibroblasts
C. Pigmentocytes and pericytes
D. Adipocytes and adventitious cells
E. Fibroblasts and labrocytes

9. In the blood of a 26-year-old man it was revealed 18% of erythrocytes of


the spherical, ball-shaped, flat and thorn-like shape. Other eritrocytes
were in the form of the concavo-concave disks. How is such phenomenon
called?
A. Physiological poikilocytosis
B. Pathological poikilocytosis
C. Physiological anisocytosis
D. Pathological anisocytosis
E. Erytrocytosis

10. When the pH level of the stomach lumen decreases to less than 3, the antrum
of the stomach releases peptide that acts in paracrine fashion to inhibit
gastrin release. This peptide is:
A. GIF
B. Acetylcholine
C. Gastrin-releasing peptide (GRP)
D. Somatostatin
E. Vasoactive intestinal peptide (VIP)

11. A 50-year-old male farm worker has been brought to the emergency room.
He was found confused in the orchard and since then has remained
unconscious. His heart rate is 45 and his blood pressure is 80/40 mm Hg.
He is sweating and salivating profusely. Which of the following should be
prescribed?
A. Atropine
B. Norepinephrine
C. Proserine
D. Physostigmine
E. Pentamine

12. A 13-year-old girl with history of asthma complained of cough, dyspnea and
wheezing. Her symptoms became so severe that her parents brought her to
the emergency room. Physical examination revealed diaphoresis, dyspnea,
tachycardia and tachypnea. Her respiratory rate was 42/min, pulse rate was
110 beats per minute, and blood pressure was 130/70 mm Hg. Choose from
the following list the most appropriate drug to reverse the
bronchoconstriction rapidly:
A. Salbutamol
B. Cromolyn
C. Beclomethasone
D. Methylprednidsolone
E. Ipratropium

13. A doctor administered Allopurinol to a 26-year-old young man with the


symptoms of gout. What pharmacological action of Allopurinol ensures
therapeutical effect?
A. By inhibiting uric acid synthesis
B. By increasing uric acid excretion
C. By inhibiting leucocyte migration into the joint
D. By general anti-inflammatory effect
E. By general analgetic effect

14. The pulmonalis embolism has suddenly developed in a 40-year-old


patient with opened fracture of the hip. Choose the possible kind of
embolism.
A. Fat
B. Thrombus-embolus
C. Air
D. Tissue
E. Foreign body
15. Hypertrychosis of auricles is caused by a gene that is localized in Y -
chromosome. Father has this feature. What is the probability to give birth to
a boy with such anomaly?
A. 100%
B. 0%
C. 25%
D. 35%
E. 75%

16. On autopsy a 35-year-old man the focus of carnification 5 cm in diametre


enclosed in a thin capsule was revealed in the second segment of the right
lung . The focus consists of a tough dry friable tissue with a dim surface. For
what disease are these morphological changes typical?
A. Tuberculoma
B. Lung cancer
C. Chondroma
D. Tumorous form of silicosis
E. Postinflammatory pneumosclerosis

17. In the microspecimen of red bone marrow there were revealed multiple
capillares through the walls of which mature blood cells penetrated. What
type of capillares is it?
A. Sinusoidal
B. Fenestrational
C. Somatical
D. Visceral
E. Lymphatic

18. M-r S presents all signs of the hepatic coma: loss of consciousness,
absence of reflexes, cramps, convulsion, disorder of heart activity,
recurrent (periodical) respiration. What are cerebrotoxical substances
which accumulate in blood under hepar insufficiency?
A. Ammonia
B. IL-1
C. Autoantibody
D. Necrosogenic substances
E. Ketonic body

19. Autopsy of a man who died from chronic cardiacvascular collapse


revealed "tiger heart". Sidewards of endocardium a yellowish-white
banding can be seen; myocardium is dull, dark-yellow. What process
caused this pathology?
A. Fatty parenchymatous degeneration
B. Carbohydrate degeneration
C. Hyaline degeneration
D. Fatty vascular -stromal degeneration
E. Amyloidosis

20. The low specific gravity of the secondary urine (1002) was found out in the
sick person. Wat is the most distant part of nephron where concentration of
secondary urine takes place?
A. In the collecting duck
B. In the nephron’s glomerulus
C. In proximal tubule of nephron
D. In ascending part of loop of Henle
E. In distal tubule of nephron

21. Blood sampling for bulk analysis is recommended to be performed on an


empty stomack and in the morning. What changes in blood composition can
occur if to perform blood sampling after food intake?
A. Increased contents of leukocytes
B. Increased contents of erythrocytes
C. Increased plasma proteins
D. Reduced contents of thrombocytes
E. Reduced contents of erythrocytes

22. Most participants of Magellan expedition to America died from


avitominosis. This disease declared itself by general weakness,
subcutaneous hemmorhages, falling of teeth, gingival hemmorhages.
What is the name of this avitiminosis?
A. Scurvy
B. Pellagra
C. Rachitis
D. Polyneuritis (beriberi)
E. Biermer's anemia

23. A histological specimen presents a receptor zone of a sensoepithelial sense


organ. Cells of this zone are placed upon the basal membrane and include
the following types: external and internal receptor cells, external and
internal phalangeal cell, stem cells, external limiting cells and external
supporting cell. The described receptor zone belongs to the following sense
organ:
A. Acoustic organ
B. Visual organ
C. Gustatory organ
D. Equilibrium organ
E. Olfactory organ

24. A patient died 3 days after the operation because of perforated colon with
manifestations of diffuse purulent peritonitis. The autopsy revealed: colon
mucos membrane was thickened and covered with a fibrin film, isolated
ulcers penetrated at different depth. The histology result: mucous
membrane necrosis, leukocytes infiltration with hemorrhages focuses.
What disease complication caused the patient's death?
A. Dysentery
B. Typhoid
C. Nonspecific ulcerative colitis
D. Crohn's disease
E. Amebiasis

25. A patient was admitted to the hospital with an asphyxia attack provoked by a
spasm of smooth muscles of the respiratory tracts. This attack was mainly
caused by alterations in the following parts of the airways:
A. Small bronchi
B. Median bronchi
C. Large bronchi
D. Terminal bronchioles
E. Respiratory part

26. The increased intraocular tension is observed in the patient with


glaucoma.Secretion of aqueous humor by the ciliar body is normal. Injury of
what structure of the eyeball wall caused the disorder of flow-out from the
anterior chamber?
A. Venous sinus
B. B Ciliar body
C. Choroid
D. Ciliary muscle
E. Back epithelium of cornea

27. Examination of a 43 y.o. patient revealed that his stomach has difficulties
with digestion of protein food. Gastric juice analysis revealed low acidity.
Function of which gastric cells is disturbed in this case?
A. Parietal exocrinocytes
B. Main exocrinocytes
C. Mucous cells (mucocytes)
D. Endocrinous cells
E. Cervical mucocytes

28. A patient has been given high doses of hydrocortisone for a long time. This
caused atrophy of one of the adrenal cortex zones. Which zone is it?
A. Fascial
B. Glomerular
C. Reticular
D. Glomerular and reticular

29. Low level of albumins and fibrinogen was detected in the patient's blood.
Decreased activity of what organelle of the liver hepatocytes can cause it?
A. Granular endoplasmatic net
B. Agranular endoplasmatic net
C. Mitochondrions
D. Golgi complex
E. Lysosomes

30. In a histological specimen parenchyma of an organ is represented by


lymphoid tissue that forms lymph nodes; the latter are arranged in a diffuse
manner and enclose a central artery. What anatomic formation has such
morphological structure?
A. Spleen
B. Tonsil
C. Lymph node
D. Thymus
E. Red bone marrow

31. A histological specimen of a kidney shows a part of the distal tubule going
between the afferent and efferent arteriole. The cells building the tubule
wall have dense nuclei; basal membrane is absent. Such structural
formation is called:
A. Macula densa
B. Juxtaglomerular cells
C. Mesangial cells
D. Juxtavascular cells

32. Patient with injured muscles of the lower extremities was admitted to the
traumatological department. Due to what cells is reparative regeneration of
the muscle fibers and restoration of the muscle function possible?
A. Satellite-cells
B. Myoblasts
C. Myofibroblasts
D. Fibroblasts
E. Myoepithelial cells

33. In course of an experiment a big number of stem cells of red bone marrow
was in some way destructed. Regeneration of which cell populations in the
loose connective tissue will be inhibited?
A. Of macrophags
B. Of fibroblasts
C. Of pigment cells
D. Of lipocytes
E. Of pericytes

34. Histological examination of a 40 y.o. man's thymus revealed decreased share


of parenchymatous gland elements, increased share of adipose and loose
connective tissue, its enrichment with thymus bodies. The organ's mass was
unchanged. What phenomenon is it?
A. Age involution
B. Accidental involution
C. Hypotrophy
D. Dystrophy
E. Atrophy

35. A histological specimen shows a blood vessel. Its inner coat is composed
by endothelium, subendothelium and internal elastic membrane. The
middle coat is enriched with smooth myocytes. Such morphological
characteristics are typical for the following vessel:
A. Muscular-type artery
B. Elastic-type artery
C. Capillary
D. Non-muscular vein
E. Muscular-type vein

36. In course of indirect histogenesis of tubular bone tissue a plate is formed


between epiphyseal and diaphyseal ossification centres that provides
further lengthwise growth of bones. What structure is it?
A. Metaphyseal plate
B. Osseous cuff
C. Osseous plate
D. Osteon
E. Layer of interior general plates

37. A 2-year-old child has got intestinal dysbacteriosis, which results in


hemorrhagic syndrome. What is the most likely cause of hemorrhage of the
child?
A. Vitamin K insufficiency
B. Activation of tissue thromboplastin
C. PP hypovitaminosis
D. Fibrinogen deficiency
E. Hypocalcemia

38. Histological specimen presents a vessel the wall of which consists of


endothelium, basal membrane and loose connective tissue. What type of
vessel is it?
A. Vein of non-muscular type
B. Artery
C. Vein of muscular type
D. Hemocapillary
E. Lymphocapillary
39. A patient died from acute cardiac insufficiency. The histological examination
of his heart revealed the necrotized section in myocardium of the left
ventricle, which was separated from undamaged tissue by the zone of
hyperimic vessels, small hemorrhages and leukocytic infiltration. What is the
most likely diagnosis?
A. Myocardial infarction
B. Myocardial ischemic dystrophy
C. Focal exudate myocarditis
D. Diffuse exudate myocarditis
E. E Productive myocarditis

40. Vitamin A deficit results in the impairment of twilight vision. Name the
cells that have the above-mentioned photoreceptor function:
A. Rod receptor cell
B. Horizontal neurocytes
C. Cone receptor cells
D. Bipolar neurons
E. Ganglion neurocytes

41. Kidneys of a man under examination show increased resorbtion of


calcium ions and decreased resorbtion of phosphate ions. What
hormone causes this phenomenon?
A. Parathormone
B. Thyrocalcitonin
C. Hormonal form D3
D. Aldosterone
E. Vasopressin

42. During pubescence the cells of male sexual glands begin to produce male sex
hormon testosterone that calls forth secondary sexual characters. What cells
of male sexual glands produce this hormone?
A. Leidig cells
B. Sustentocytes
C. Sertoli's cells
D. Supporting cells
E. Spermatozoa

43. The specimens present sections of haemopoetic and immunogenetic organs.


Organ has lymph tissue forming different structures (lymph nodes,lobules,
bars). In what organ does antigen-independent proliferation and
differantiation take place?
A. Thymus
B. Lymphatic nodes
C. Spleen
D. Hemolymph nodes
E. Tonsil
44. A patient with thrombophlebitis of lower extremities had got chest pains,
blood spitting, growing respiratory failure that caused his death. Autopsy
revealed multiple pulmonary infarctions. What is the most probable
reason of their development?
A. Pulmonary artery embolism
B. Pulmonary artery thrombosis
C. Bronchial artery thrombosis
D. Bronchial artery embolism
E. Pulmonary venous thrombosis

45. A patient complains of dryness of head skin, itching, fragility and loss of hair.
After examination he was diagnosed with seborrhea. Disturbed activity of
which cells caused this condition?
A. Cells of sebaceous glands
B. Cells of sudoriferous glands
C. Epithelial cells
D. Adipocytes
E. Melanocytes

46. A sensitive neural ganglion consists of roundish neurocytes with one


extension that divides into axon and dendrite at some distance from the
perikaryon. What are these cells called?
A. Pseudounipolar
B. Unipolar
C. Bipolar
D. Multipolar
E. Apolar

47. An embryo displays disturbed process of dorsal mesoderm


segmentation and somite formation. What part of skin will have
developmental abnormalities?
A. Derma
B. Hair
C. Sebaceous glands
D. Epidermis
E. Sudoriferous glands

48. An electron microphotography of a fragment of proper gastric gland shows a


big irregular round-shaped cell. There are a lot of intracellular tubules and
mitochondria in the cytoplasm. Specify this cell:
A. Parietal cell
B. Principal cell
C. Undifferentiated cell
D. Mucous cell
E. Endocrine cell
49. A scheme presents an exocrinous gland that has unbranched excretory duct
with a terminal part in form of a saccule openining into the duct. How is this
gland called according to the morphological classification of exocrinous
glands?
A. Simple unbranched alveolar
B. Compound branched alveolar
C. Simple branched tubular
D. Compound unbranched alveolar
E. Compound unbranched alveolar tubular

50. Roentgenological examination of skull base bones revealed enlargement of


sellar cavity, thinning of anterior clinoid processes, destruction of different
parts, destruction of different parts of sella turcica. Such bone destruction
might be caused by a tumour of the following wndocrinous gland:
A. Hypophysis
B. Epiphysis
C. Thymus gland
D. Adrenal glands
E. Thyroid gland

51. Electronic microphotography of pulmonary alveole's wall presents a big cell.


Its cytoplasm has a lot of mitochondria, developed Golgi apparatus,
osmiophil lamellated corpuscles. What is the main function of this cell?
A. It produces surfactant
B. It is a component of blood-air barrier
C. It warms the air
D. It purifies the air
E. It absorbs microorganisms

52. A pathological process in bronchi resulted in epithelium


desquamation. What cells will regenerate bronchial epithelium?
A. Basal
B. Intercalary
C. Ciliate
D. Endocrinal
E. Goblet

53. A viral infection has damaged cells that form walls of bile capillaries. This
stimulated conditions for inflow of bile into the blood of sinusoidal
capillaries. What cells are damaged?
A. Hepatocytes
B. Kupffer's cells
C. Ito cells
D. Pit-cells
E. Endotheliocytes

54. A histological specimen of spleen shows a vessel with a wall consisting of


endothelium and subendothelial layer, median membrane is absent, exterior
membrane inosculates with the layers of spleen connective tissue. What
vessel is it?
A. Vein of non-muscular type
B. Vein of muscular type
C. Artery of muscular type
D. Arteriole
E. Capillary

55. In course of a conditional experiment the development of mesenchyma cells


was completely inhibited. Development of the following muscular tissue will
be disturbed:
A. Smooth muscular tissue
B. Neural muscular tissue
C. Epidermal muscular tissue
D. Cardiac muscular tissue
E. Skeletal muscular tissue

56. A patient ill with chronic gastritis went for endogastric pH-metry that
allowed to reveal decreased acidity of gastric juice. It is indicative of
diminished function of the following cells:
A. Parietal exocrinocytes
B. B Chief exocrinocytes
C. Endocrinocytes
D. Cervical cells
E. Accessory cells

57. Ultramicroscopical examination of "dark" hepatocyte population in the cell


cytoplasm detected a developed granular endoplasmic reticulum. What
function has this organella in these cells?
A. Synthesis of blood plasma proteins
B. Carbohydrate synthesis
C. Deintoxicative function
D. Bile production
E. Calcium ion depositing

58. An endocrinal gland with parenchyma consisting of epithelium and neural


tissue is under morphological examination. Epithelial trabecules have two
types of cells: chromophilic and chromophobic. Identify this organ:
A. Hypophysis
B. Adrenal glands
C. Hypothalamus
D. Thyroid gland
E. Parathyroid gland
59. A histological specimen presents an artery. One of the membranes of its
wall has flat cells lying on the basal membrane. What type of cells is it?
A. Endothelium
B. Mesothelium
C. Smooth myocytes
D. Fibroblasts
E. Macrophages

60. One of sections of central nervous system has layerwise arrangement of


neurocytes. Among them there are cells of the following forms: stellate,
fusiform, horizontal, pyramidal. What section of central nervous system is
this structure typical for?
A. Cortex of cerebrum
B. Spinal cord
C. Cerebellum
D. Medulla oblongata
E. Hypothalamus

61. Study of fingerprints (dactylography) is used by criminalists for personal


identification as well as for diagnostics of genetic abnormalities, particularly
Dawn's disease. What layer of skin determines individuality of fingerprints?
A. Dermopapillary
B. Horny
C. Reticular
D. Clear (stratum lucidum epidermidis)
E. Basal

62. A microspecimen of the submandibular salivary gland shows some


basket-shaped cells concentrated around the acines and excretory ducts.
These cells surround bases of the serous cells and are called
myoepitheliocytes. These cells relate to the following tissue:
A. Muscular tissue
B. B Epithelial tissue
C. Neural tissue
D. Special connective tissue
E. Loose fibrous connective tissue

63. An infectious disease caused contractive activity of muscles that contract


and dilate eye pupil (paralytic state). What functional eye system was
damaged?
A. Accomodative
B. Dioptric
C. Ancillary
D. Photosensory
E. Lacrimal apparatus
Krok 1 – 2015 Physiology Base
1. Patient with hypersecretion of the gastric juices was recomended to
exclude from the diet concentrated bouillons and vegetable decoctions
because of their stimulation of gastric secretion. What is dominating
mechanism of stimulation of secretion in this case?
A. Stimulation of gastrin production by G-cells
B. Irritation of taste receptors
C. Irritation of mechanoreceptors of the oral cavity
D. Irritation of mechanoreceptors of the stomach
E. Stimulation of excretion of secretin in the duodenum

2. Person felt thirsty after staying in heat for a long time. Signals of what
receptors caused it first of all?
A. Osmoreceptors of hypothalamus
B. Sodium receptors of hypothalamus
C. Osmoreceptorsof the liver
D. Glucoreceptors of hypothalamus
E. Baroreceptors of aortic arch

3. An individual is characterized by rounded face, broad forehead, a


mongolian type of eyelid fold, flattened nasal bridge, permanently open
mouth, projecting lower lip, protruding tongue, short neck, flat hands, and
stubby fingers. What diagnosis can be put to the patient?
A. Down's syndrome
B. Klinefelter's syndrome
C. Alkaptonuria
D. Supermales
E. Turner's syndrome

4. Purulent endometritis developed in a woman after delivery. Treating with


antibiotics inhibitors of murein synthesis was ineffective. Wide spectrum
bactericidal antibiotic was administered to her. In 6 hours temperature
rapidly increased up to $40^0C$ with shiver. Muscle pains have appeared.
BP dropped down to 70/40 mmHg. Oligura has developed. What is the
main reason for the development of this condition?
A. Endotoxic shock
B. Toxic effect of preparation
C. Internal bleeding
D. Anaphylactic shock
E. Bacteremia

5. Secretion of which gastrointestinal hormones is primerily decreased in


patient with removed duodenum?
A. Cholecystokinin and secretin
B. Gastrin
C. Histamine
D. Gastrin and histamine
E. Neurotensin

6. A 2-year-old child experienced convulsions because of lowering calcium ions


concentration in the blood plasma. Function of what structure is decreased?
A. Parathyroid glands
B. Hypophysis
C. Adrenal cortex
D. Pineal gland
E. Thymus

7. While emotional excitement the heart rate in a 30-year -old person run up
to 112 Bpm. What part of the conducting system of the heart caused it?
A. Synoatrial node
B. Purkinje's fibers
C. His bundle branches
D. Intraventricular node
E. His bundle

8. Usage of oral contraceptives with sex hormones inhibits secretion of


the hypophysiae hormones. Secretion of which of the indicated
hormones is inhibited while using oral contraceptives with sex
hormones?
A. Follicle-stimulating
B. Vasopressin
C. Thyrotropic
D. Somatotropic
E. Oxytocin

9. During the breakout of acute respiratory infection in order to diagnose


influenza the express-diagnosis, based on revealing of specific viral
antigen in the examined material (nasopharyngial lavage), is carried
out. Which reaction is used for this?
A. Immunofluorescence
B. Complement binding
C. Agglutination
D. Precipitation
E. Opsonization

10. A 38-year-old woman was admitted to the admission-diagnostic


department with uterine bleeding. What are the most likely changes of
blood?
A. Reduction of haematocrite rate
B. Increase of haematocrite rate
C. Leukopenia
D. Leucocytosis
E. Polycythemia
11. Due to action of electric current on the exitable cell there appeared
depolarization of it's membrane. Movement of what ions through the
membrane caused depolarisation?
A. Na +
B. НСО3-
C. Са2 +
D. Сl-
E. К+

12. The high level of Lactate Dehydrogenase (LDH) isozymes concentration


showed the increase of LDH-1 and LDH-2 in a patient’s blood plasma. Point
out the most probable diagnosis:
A. Myocardial infarction
B. Skeletal muscle dystrophy
C. Diabetes mellitus
D. Viral hepatitis
E. Acute pancreatitis

13. There is only one hormone among the neurohormones which refers to the
derivatives of amino acids according to classification. Point it out:
A. Melatonin
B. Thyroliberin
C. Vasopressin
D. Oxytocin
E. Somatotropin

14. The sterile Petri dishes and pipettes are necessary to prepare for
microbiological tests in bacteriological laboratory. What way of
sterilization should be applied in this case?
A. Dry-heat sterilization
B. Tyndallization
C. Pasteurization
D. Steam sterilization in autoclave
E. Boiling

15. A patient after hypertension stroke does not have voluntary movements in
his right arm and leg with the increased muscle tone in these extremites.
What type of disfunction of nervous system is it?
A. Central paralysis
B. Peripheral paralysis
C. Peripheral paresis
D. Reflex paresis
E. Central paresis

16. Blood analysis of a patient showed signs of HIV infection (human


immunodeficiency virus). Which cells does HIV-virus primarily affect?
A. Cells that contain receptor T4 (T -helpers)
B. Cells that contain receptor IgM (B-lymphocytes)
C. Specialized nervous cells (neurons)
D. Mast cells
E. Proliferating cells (stem hematoplastic cells)

17. The preventive radioprotector was given to a worker of a nuclear


power station. What mechanism from the below mentioned is
considered to be the main mechanism of radioprotection?
A. Inhibition of free radicals formation
B. Prevention of tissue’s hypoxia
C. Activation of oxidation reactions
D. Increasing of tissue blood supply
E. Increasing of respiration

18. X-ray examination discovered lungs emphysema in the patient. What is the
reason of short breath development in this case?
A. Decreased lungs elasticity
B. Increased lungs elasticity
C. Inhibition of respiratory center
D. Excitation of respiratory center
E. Decreasing of alveoli receptors sensitivity

19. On experiment on the dog the peripheral part of nervus vagus of the neck
was irritated. What changes of the heart function would be observed?
A. Decreased contraction rate
B. Increased contraction force
C. Increased atrioventricular conduction
D. Increased contraction force and rate
E. Increased myocardial excitability

20. Power inputs of a boy increased from 500 to 2000 kJ pro hour. What can
be the cause of it?
A. Physical exercise
B. Raise of outer temperatute
C. Mental activity
D. Food intake
E. Transition from sleep to wakefulness

21. A student is thoroughly summarising a lecture. When his groupmates begin


talking the quality of the summarising worsens greatly. What type of
inhibition in the cerebral cortex is the cause of it?
A. External
B. Protective
C. Dying
D. Differential
E. Delayed
22. In the experiment on the animal the part of the cerebral cortex hemispheres
was removed. It caused elimination of previously formed conditioned reflex
to the light irritation. What part ot the cortex was removed?
A. Occipital cortex
B. Precentral convolution
C. Postcentral convolution
D. Limbic cortex
E. Temporal lobe

23. Inhibition of alpha-motoneuron of the extensor muscles was noticed after


stimulation of alpha-motoneuron of the flexor muscles during the
experiment on the spinal column. What type of inhibition can this process
cause?
A. Reciprocal
B. Presynaptic
C. Depolarizational
D. Recurrent
E. Lateral

24. Respiratory coefficient was studied in the patient who strictly kept certain
diet for 10 days. It was determined that it is 1. What diet does the patient
follow?
A. With domination of carbohydrates
B. With domination of proteins and fat
C. With domination of fat and carbohydrates
D. Mixed
E. With domination of proteins and carbohydrates

25. On blood grouping on the system ABO, standart serum of the I and II
groups caused erythrocytes agglutination of the examined blood and
serum group of the III didn't. What agglutinogens are in this
erythrocytes?
A. В
B. А
C. А and В
D. С
E. D and C

26. Punctata hemorrhage was found out in the patient after application of a
tourniquet. With disfunction of what blood cells is it connected?
A. Platelets
B. Eosinophiles
C. Monocytes
D. Lymphocytes
E. Neutrophiles
27. Students who are taking examinations often have dry mouth. The
mechanism that causes this state is the realization of the following
reflexes:
A. Conditioned sympathetic
B. Unconditioned parasympathetic
C. Conditioned parasympathetic
D. Unconditioned sympathetic
E. Unconditioned peripheral

28. Middle part of cochlear of internal ear was destroyed in animal while
experiment. It will cause abnormalities of the sound perception of the
following frequencies:
A. Middle
B. Low
C. High
D. High and low
E. No abnormalities

29. The person has decreased diuresis, hypernatremia, hypokalemia.


Hypersecretion of what hormon can cause such changes?
A. Aldosterone
B. Vasopressin
C. Auricular sodiumuretic factor
D. Adrenalin
E. Parathormone

30. The temperature of the ambient environment is 38oC and relative air
humidity is 50\%. What ways of heat emission provide maintaining a
constant temperature of the human body?
A. Evaporation
B. Radiation
C. Heat conduction
D. Convection
E. Convection and conduction

31. The minute blood volume in a patient with transplanted heart has
increased as a result of physical activity. What regulative mechanism is
responsible for these changes?
A. Catecholamines
B. Sympathetic unconditioned reflexes
C. Parasympathetic unconditioned reflexes
D. Sympathetic conditioned reflexes
E. Parasympathetic conditioned reflexes

32. Isolated muscle of a frog is rhythmically irritated with electric impulses.


Every next impulse is in a period of relaxation from the previus contraction.
What contraction of the muscle appears?
A. Waved tetanus
B. Single
C. Asynchronous
D. Continuous(smooth) tetanus
E. Tonic

33. A man has normal sensitivity of his finger skin, however he doesn’t sense
his wedding ring around the finger. What process induced by wearing of
the ring has caused this phenomenon?
A. Receptor adaptation
B. Development of the fibrous tissue
C. Abnormality of the epidermis structure
D. Impaired circulation
E. Abnormality of the receptor structure

34. An aged man had raise of arterial pressure under a stress. It was caused
by activation of:
A. Sympathoadrenal system
B. Parasympathetic nucleus of vagus
C. Functions of thyroid gland
D. Functions of adrenal cortex
E. Hypophysis function

35. ECG study showed that the T -waves were positive in the standard
extremity leads, their amplitude and duration were normal. The right
conclusion would be that the following process runs normally in the
heart ventricles:
A. Repolarization
B. Depolarization
C. Excitement
D. Contraction
E. Relaxation

36. Blood minute volume of a 30 year old woman at rest is 5 l/m. What blood
volume is pumped through the pulmonary vessels per minute?
A. 5 l
B. 3,75 l
C. 2,5 l
D. 2,0 l
E. 1,5 l

37. As a result of long-term starvation the glomerular filtration of a man was


accelerated by 20\%. The most probable cause of filtration changes under
such conditions is:
A. Fall of oncotic pressure of blood plasma
B. Rise of systemic arterial pressure
C. Increased permeability of renal filter
D. Growth of filtration coefficient
E. Increase of renal plasma flow

38. In course of an experiment a skeletal muscle is being stimulated by a series


of electric impulses. What type of muscle contraction will arise, if every
subsequent impulse comes in the period of shortening of the previous
single muscle contraction?
A. Holotetanus
B. Partial tetanus
C. Asynchronous tetanus
D. A series of single contractions
E. Muscle contracture

39. While shifting the gaze to the closely situated object the refracting power
of eye's optical mediums will increase by 10 diopters. It results from
changing of such eye structure:
A. Lens
B. Cornea
C. Vitreous body
D. Liquid of the anterior chamber of eye
E. Muscle that dilatates pupil

40. Spasm of smooth muscle of bronchi developed in the patient. Usage of


activators of what membrane cytoreceptors is fisiologically valid to
decrease attack?
A. beta-adrenoreceptors
B. alpha-аdrenoreceptors
C. alpha- та beta-аdrenoreceptors
D. Н-cholinoreceptors
E. М-cholinoreceptors

41. Intrapleural pressure is being measured in a person. In what phase has a


person hold his breath if the pressure is - 25 cm H2 O?
A. Speed up inspiration
B. Calm expiration
C. Calm inspiration
D. Speed up expiration

42. On examination of the person it was revealed that minute volume of heart is
3500mL, systolic volume is 50 mL. What is the frequency of cardiac
contraction?
A. 70 bpm
B. 60 bpm
C. 50 bpm
D. 80 bpm
E. 90 bpm
43. Due to activation of ion channels of external membrane of excitable cell it's
rest potential has significantly increased. What channels were activated?
A. Potassium Channels
B. Natrium Channels
C. Fast Calcium Channels
D. Slow calcium Channels
E. Natrium and Calcium Channels

44. The ventral roots of 5 frontal segment of spinal cord were cut during
experiment in the animal. What changes will take place in the
innervation region?
A. Loss of movements
B. Loss of touch sensitivity
C. Loss of temperature sensitivity
D. Loss of proprioceptive sensitivity
E. Hypersensitivity

45. Glomerular filtration rate (GFR) increased for 20% due to prolonged
starvation of the person. The most likely cause of filtration changes under
this condition is:
A. Decrease of oncotic pressure of blood plasma
B. Increase of systemic blood pressure
C. Increase of penetration of the renal filter
D. Increase of filtration coefficient
E. Increase of renal plasma stream

46. A patient has a transverse disruption of spinal cord below the IV


thoracic segment. What changes of respiration will it cause?
A. Respiration will stay unchanged
B. Respiration will stop
C. Respiration will become less frequent
D. Respiration will become deeper
E. Respiration will become more frequent

47. Due to cranial trauma, the patient developed the symptoms: intention
tremor, dysmetry, adiadochokinesis, dysarthria. What structure of the
brain is injured?
A. Cerebellum
B. Striatum
C. Motor cortex
D. Pale sphere
E. Black substance

48. A lightly dressed man is standing in a room, air temperature is +14 oC,
windows and doors are closed. In what way does he emit heat the
most actively?
A. Heat radiation
B. Heat conduction
C. Convection
D. Evaporation
E. Perspiration

49. ECG of a patient with hyperfunction of thyroid gland showed heart hurry.
It is indicated by depression of the following ECG element:
A. R-R interval
B. P-Q segment
C. P-Q interval
D. P-T interval
E. QRS complex

50. A peripheral segment of vagus nerve on a dog's neck was being stimulated in
course of an experiment. The following changes of cardiac activity could be
meanwhile observed:
A. Heart rate fall
B. Heart Hurry
C. Enhancement of atrioventricular conduction
D. Heart rate and heart force amplification
E. Increased excitability of myocardium

51. ECG of a patient shows prolongation of T -wave. This is caused by


deceleration in ventricles of:
A. Repolarization
B. Depolarization and repolarization
C. Depolarization
D. Contraction
E. Relaxation

52. In a healthy adult speed of the excitement conduction through the


atrioventricular node is 0,02-0,05 m/sec. Atrioventricular delay enables:
A. Sequence of atrial and ventricular contractions
B. Simultaneity of both atria contractions
C. Simultaneity of both ventricles contractions
D. Sufficient force of atrial contractions
E. Sufficient force of ventricular contractions

53. A 2 y.o. child has convulsions as a result of lowered concentration of


calcium ions in blood plasma. It is caused by reduced function of:
A. Parathyroid glands
B. Hypophysis
C. Adrenal cortex
D. Pineal gland
E. Thymus
54. What heat transfer mechanism is the most effective while the man
being at 80% of air moisture and the temperature +35 0 С?
A. Evaporation
B. Radiation
C. Heat conduction
D. Convection

55. During preparation of a patient to a heart surgery it was necessary to


measure pressure in heart chambers. In one of them pressure varied from 0
mm Hg up to 120 mm Hg within one cardiac cycle. What heart chamber is
it?
A. Left ventricle
B. Right ventricle
C. Right atrium
D. Left atrium

56. Heart rate of a man permanently equals 40 beats pro minute. What is
the pacemaker?
A. Atriventricular node
B. Sinoatrial node
C. His' bundle
D. His' bundle branches
E. Purkinje's fibers

57. Stimulation of an excitable cell by the electric current has led to the
depolarization of its membrane. The depolarization has been caused mainly
by the following ions penetrating into the cell through its membrane:
A. Na +
B. HCO3-
C. Ca2 +
D. Cl-
E. K+

58. Parents of a 10 y.o. boy consulted a doctor about extension of hair-covering,


growth of beard and moustache, low voice. Intensified secretion of which
hormone must be assumed?
A. Of testosterone
B. Of somatotropin
C. Of oestrogen
D. Of progesterone
E. Of cortisol

59. Lung ventilation in a person is increased as a result of physical activity.


Which of the following indices of the external respiration is much higher than
in a state of rest?
A. Respiratory volume
B. Vital capacity of lungs
C. Inspiratory reserve volume
D. Expiratory reserve volume
E. Total lung capacity

60. A man took a quiet expiration. Name an air volume that is meanwhile
contained in his lungs:
A. Functional residual capacity
B. Residual volume
C. Expiratory reserve volume
D. Respiratory volume
E. Vital lung capacity

61. Examination of an isolated cardiomyocyte revealed that it didn't generate


excitation impulses automatically. This cardiomyocyte was obtained from:
A. Ventricles
B. Sinoatrial node
C. Atrioventricular node
D. His' bundle
E. Purkinje's fibers

62. Examination of a man established that cardiac output equaled 3500 ml,
systolic output - 50 ml. What is the man's heart rate pro minute?
A. 70
B. 60
C. 50
D. 80
E. 90

63. As a result of continuous starvation the glomerular filtration rate has


increased by 20%. The most probable cause of the glomerular filtration
alteration under the mentioned conditions is:
A. Decrease in the oncotic pressure of blood plasma
B. Increase in the systemic arterial pressure
C. Increase in the permeability of the renal filter
D. Increase of the filtartion quotient
E. Increase of the renal blood flow

64. A man who went for a ride on a roundabout had amplification of heart
rate, sweating and nausea. What receptors stimulation is it primarily
connected with?
A. Vestibular
B. Proprioceptors
C. Tactors
D. Auditory
E. Visual
65. A man's intrapleural pressure is being measured. In what phase did the
man hold his breath, if his pressure is 7,5 cm Hg?
A. Quiet inspiration
B. Quiet expiration
C. Forced inspiration
D. Forced expiration

66. Atria of an experimental animal were superdistended by blood that


resulted in decreased reabsorption of Na + and water in renal tubules. This
can be explained by the influence of the following factor upon kidneys:
A. Natriuretic hormone
B. Aldosterone
C. Renin
D. Angiotensin
E. Vasopressin

67. A middle-aged man went to a foreign country because he had been


offered a job there. However he had been unemployed for quite a long
time. What endocrine glands were exhausted most of all in this man?
A. Adrenal glands
B. Parathyroid glands
C. Seminal glands
D. Substernal gland
E. Thyroid gland

68. A 60-year-old man after cerebral hemorrhage felt asleep for a long time.
Damage of what structure caused this state?
A. Reticular formation
B. Hippocampus
C. Nuclears of the cerebral nerves
D. Cortex of the large hemispheres
E. Black substances

69. A human body cools in water much faster that in the air. What way of heat
emission in water is much more efficient?
A. Heat conduction
B. Convection
C. Heat radiation
D. Sweat evaporation

70. As a result of spinal-cord trauma a 33 y.o. man has a disturbed pain


and temperature sensitivity that is caused by damage of the following
tract:
A. Spinothalamic
B. Medial spinocortical
C. Posterior spinocerebellar
D. Lateral spinocortical
E. Anterior spinocerebellar

71. After a surgery a 36-year-old woman was given an intravenous injection


of concentrated albumin solution. This has induced intensified water
movement in the following direction:
A. From the intercellular fluid to the capillaries
B. From the intercellular fluid to the cells
C. From the cells to the intercellular fluid
D. From the capillaries to the intercellular fluid
E. No changes of water movement will be observed

72. A clinic observes a 49 year old patient with significant prolongation of


coagulation time, gastrointestinal haemorrhages, subcutaneous
hematomas. These symptoms might be explained by the deficiency of the
following vitamin:
A. K
B. B1
C. B6
D. H
E. E

73. Examination of a patient revealed hyperkaliemia and hyponatremia. Low


secretion of which hormone may cause such changes?
A. Aldosteron
B. Vasopressin
C. Cortisol
D. Parathormone
E. Natriuretic

74. Examination of a 43 y.o. anephric patient revealed anemia symptoms. What


is the cause of these symptoms?
A. Reduced synthesis of erythropoietins
B. Enhanced destruction of erythrocytes
C. Iron deficit
D. Vitamin B12 deficit
E. Folic acid deficit

75. A man is being measured power inputs on an empty stomach, in the lying
position, under conditions of physical and psychic rest at a comfortable
temperature. Power inputs will reach the maximum at:
A. 5-6 p.m.
B. 7-8 a.m.
C. 10-12 a.m.
D. 2-3 p.m.
E. 3-4 a.m.
76. When measuring power inputs of a man by the method of indirect
calorimetry the following results were obtained: 1000 ml oxygen
consumption and 800 ml carbon dioxide liberation per minute. The man
under examination has the following respiratory coefficient:
A. 0,8
B. 1,25
C. 0,9
D. 0,84
E. 1,0

77. While determining power inputs of a patient’s organism it was established


that the respiratory coefficient equaled 1,0. This means that in the cells of
the patient the following substances are mainly oxidized:
A. Carbohydrates
B. Proteins
C. Fats
D. Proteins and carbohydrates
E. Carbohydrates and fats

78. A patient has a disturbed absorbtion of fat hydrolysates. It might have been
caused by a deficit in the small intestine cavity:
A. Of bile acids
B. Of bile pigments
C. Of lipolytic enzymes
D. Of sodium ions
E. Of liposoluble vitamins

79. Inhabitants of territories with cold climate have high content of an


adaptive thermoregulatory hormone. What hormone is meant?
A. Thyroxin
B. Insulin
C. Glucagon
D. Somatotropin
E. Cortisol

80.A concentrated solution of sodium chloride was intravenously injected


to an animal. This caused decreased reabsorption of sodium ions in the
renal tubules. It is the result of the following changes of hormonal
secretion:
A. Aldosterone reduction
B. Aldosterone increase
C. Vasopressin reduction
D. Vasopressin increase
E. Reduction of atrial natriuretic factor

81. Accelerated frequency of the heart rate and increased blood pressure were
marked in the sportsman on the start before the competitions. Influence
of what parts of the CNS can explain these changes?
A. Cortex of the large hemispheres
B. Medulla
C. Mesencephalon
D. Diencephalon
E. Hypothalamus

82. People adapted to high external temperatures have such pecularity: profuse
sweating isn't accompanied by loss of large volumes of sodium chloride.
This is caused by the effect of the following hormone upon the perspiratory
glands:
A. Aldosterone
B. Vasopressin
C. Cortisol
D. Tgyroxin
E. Natriuretic

83. During an experiment the dorsal roots of the spinal cord of an animal have
been cut. What changes will be observed in the innervation zone?
A. Sensitivity loss
B. Loss of motor functions
C. Decrease in muscle tone
D. Increase in muscle tone
E. Sensitivity loss and loss of motor functions

84. As a result of destruction of certain brainstem structures an animal has


lost its orientative reflexes in response to strong photic stimuli. What
structures were destroyed?
A. Anterior tubercles of quadrigeminal plate
B. Posterior tubercles of quadrigeminal plate
C. Red nuclei
D. Vestibular nuclei
E. Black substance

85. As a result of damage to certain structures of brainstem an animal lost


orientation reflexes. What structures were damaged?
A. Quadritubercular bodies
B. Medial nuclei of reticular formation
C. Red nuclei
D. Vestibular nuclei
E. Black substance

86. Osmotic pressure of a man's blood plasma is 350 mosmole/l (standard


pressure is 300 mosmole/l). First of all it will result in high secretion of
the following hormone:
A. Vasopressin
B. Aldosteron
C. Cortisol
D. Adrenocorticotropin
E. Natriuretic

87. A hypertensive glucose solution was introduced to a patient. It will intensify


water movement:
A. From the cells to the intercellular liquid
B. From the intercellular liquid to the capillaries
C. From the intercellular liquid to the cells
D. From the capillaries to the intercellular liquid
E. There will be no changes of water movement

88. To prevent long-term effects of 4-day malaria a 42-year-old patient was


prescribed primaquine. On the 3-rd day from the begin of treatment there
appeared stomach and heart pains, dyspepsia, general cyanosis,
hemoglobinuria. What caused side effects of the preparation?
A. Genetic insufficiency of glucose 6-phosphate dehydrogenase
B. Cumulation of the preparation
C. Decreased activity of microsomal liver enzymes
D. Delayed urinary excretion of the preparation
E. Drug potentiation by other preparations

89. According to audiometry data a patient has a disturbed perception of


medium-frequency sounds. It might have been caused by a damage of:
A. Middle part of helix
B. Cochlear nuclie
C. Spiral ganglion
D. Quadritubercular structure
E. Lateral geniculate bodies

90. A 17-year-old boy fell seriously ill, body temperature rose up to 38,5 0 C,
there is cough, rhinitis, lacrimation, nasal discharges. What kind of
inflammation is it?
A. Catarrhal inflammation
B. Serous inflammation
C. Fibrinous inflammation
D. Suppurative inflammation
E. Hemorrhagic inflammation

91. A patient with disturbed cerebral circulation has problems with deglutition.
What part of brain was damaged?
A. Brainstem
B. Cervical part of spinal cord
C. Forebrain
D. Interbrain
E. Midbrain
92. A patient who has been treated with diazepam on account of neurosis
complains of toothache. Doctor administered him an analgetic, but its dose
was lower than average therapeutic dose. What phenomenon did the doctor
take into account while prescribing the patient an underdose?
A. Potentiation
B. Summation
C. Cumulation
D. Drug dependence
E. Tolerance

93. Long-term starvation cure of a patient resulted in diminished ratio of


albumines and globulines in plasma. What of the following will be
result of these changes?
A. Increase of ESR
B. Decrease of ESR
C. Increase of hematocrit
D. Decrease of hematocrit
E. Hypercoagulation

94. A patient has a decreased vasopressin synthesis that causes polyuria and as
a result of it evident organism dehydratation. What is the mechanism of
polyuria development?
A. Reduced tubular reabsorption of water
B. Reduced tubular reabsorption of Na ions
C. Reduced tubular reabsorption of protein
D. Reduced glucose reabsorption
E. Acceleration of glomerular filtration

95. To anaesthetize the manipulation related to burn surface treatment, a


patient was intravenously injected a medication for short-acting narcosis. 1
minute later the patient being under anaesthesia had increased blood
pressure, tachycardia, increased tone of sceletal muscles; reflexes remained.
After awakening the patient had desorientation and visual hallucinations.
What medication was the patient injected?
A. Ketamine
B. Sombrevin
C. Diethyl ether
D. Thiopental sodium
E. Nitrous oxide

96. A 35-year-old man consulted a dentist about reduced density of dental tissue,
high fragility of teeth during eating solid food. This patient suffers the most
probably from the deficiency of the following mineral element:
A. Calcium
B. Potassium
C. Sodium
D. Magnesium
E. Iron

97. A patient is 44 years old. Laboratory examination of his blood revealed that
content of proteins in plasma was 40 g/l. What influence will be exerted on
the transcapillary water exchange?
A. Filtration will be increased, reabsorption – decreased
B. Both filtration and reabsorption will be increased
C. Both filtration and reabsorption will be decreased
D. Filtration will be decreased, reabsorption – increased
E. Exchange will stay unchanged

98. After destruction of CNS structures an animal lost orientative reflexes.


What structure was destroyed?
A. Quadrigeminal plate
B. Red nucleus
C. Lateral vestibular nuclei
D. Black substance
E. Medial reticular nuclei

99. An isolated cell of human heart automatically generates excitation impulses


with frequency 60 times pro minute. What structure does this cell belong to?
A. Sinoatrial node
B. Atrium
C. Ventricle
D. Atrioventricular node
E. His' bundle

100. Examination of a patient revealed a strong, balanced, inert type of


higher nervous activity according to Pavlov. What temperament type does the
patient have (according to Hippocrates classification)?
A. Phlegmatic
B. Sanguine
C. Choleric
D. Melancholic

101. Examination of a patient revealed overgrowth of facial bones and


soft tissues, tongue enlargement, wide interdental spaces in the
enlarged dental arch. What changes of the hormonal secretion are the
most likely?
A. Hypersecretion of the somatotropic hormone
B. Hyposecretion of the somatotropic hormone
C. Hypersecretion of insulin
D. Hyposecretion of thyroxin
E. Hyposecretion of insulin

102. A patient has a haemorrhage into the posterior central gyrus. What
type of sensitivity on the opposite side will be disturbed?
A. Skin and proprioceptive
B. Visual
C. Auditory
D. Olfactory
E. Auditory and visual

103. Child asked you to puff up the balloon as much as possible for a
one exhalation. What air volume will you use?
A. Vital volume of the lungs
B. Inspiration volume
C. Functional residual volume
D. Total volume of the lungs
E. Backup volume of the inspiration

104. A 32-year-old patient consulted a doctor about the absence of lactation


after parturition. Such disorder might be explained by the deficit of the
following hormone:
A. Prolactin
B. Somatotropin
C. Vasopressin
D. Thyrocalcitonin
E. Glucagon

105. During influenza epidemic 40% of pupils who didn't go in for


sports were affected by the disease, and among the pupils who regularly
did physical exercises this index was only 20\%. What adaptative
mechanisms determined such a low sickness rate of pupils participating
in the sports?
A. Cross adaptation
B. Specific adaptation
C. Physiological adaptation
D. Biochemical adaptation
E. Genetic adaptation

106. A 30-year-old woman was diagnosed with insufficiency of exocrinous


function of pancreas. Hydrolisis of what nutrients will be disturbed?
A. Proteins, fats, carbohydrates
B. Proteins, fats
C. Proteins, carbohydrates
D. Fats, carbohydrates
E. Proteins

107. Short-term physical activity resulted in reflex amplification of heart


rate and raise of systemic arterial pressure. What receptors activation was the
main cause of pressor reflex realization?
A. Proprioreceptors of active muscles
B. Vascular chemoreceptors
C. Vascular volume receptors
D. Vascular baroceptors
E. Hypothalamus thermoreceptors

108. In course of an experiment a skeletal muscle is being stimulated by a


series of electric impulses. What type of muscle contraction will arise, if
every subsequent impulse comes in the period of relaxation of single
muscle contraction?
A. Partial tetanus
B. Holotetanus
C. A series of single contractions
D. Muscle contructure
E. Asynchronous tetanus

109. Packed cell volume of a man was 40% before the trauma. What
packed cell volume will be observed 24 hours after blood loss of 750 ml?
A. 30%
B. 40%
C. 55%
D. 45%
E. 50%

110. A patient staggers and walks astraddle. He has hypomyotonia of


arm and leg muscles, staccato speech. In what brain section is this
affection localized?
A. Cerebellum
B. Putamen
C. Caudate nucleus
D. Motor cortex
E. Red nucleus

111. A pregnant woman had her blood group identified. Reaction of


erythrocyte agglutination with standard serums of 0α\β(I), Bα(III) groups
didn't proceed with standard serum of Aβ(II) group. The blood group under
examination is:
A. Аβ(II)
B. 0α\β(I)
C. Вα(III)
D. АВ (IV)

112. Blood group of a 30 year old man was specified before an operation.
His blood is Rh-positive. Reaction of erythrocyte agglutination was absent
with standard sera of 0α\β(I), Аβ(II) , Bα(III) groups. The blood under
examination is of the following group:
A. 0α\β(I)
B. Аβ(II)
C. Вα(III)
D. АВ (IV)

113. During an experiment the myotatic reflex has been studied in frogs.
After extension in a skeletal muscle its reflectory contraction was absent. The
reason for it might be a dysfunction of the following receptors:
A. Muscle spindles
B. Nociceptors
C. Articular
D. Golgi tendon organs
E. Tactile

114. Vagus branches that innervate heart are being stimulated in course of
an experiment. As a result of it the excitement conduction from atria to the
ventricles was brought to a stop. It is caused by electrophysical changes in
the following structures:
A. Atrioventricular node
B. His' bundle
C. Sinoarial node
D. Ventricles
E. Atria

115. If a man has an attack of bronchiospasm it is necessary to reduce


the effect of vagus on smooth muscles of bronchi. What membrane
cytoreceptors should be blocked for this purpose?
A. M-cholinoreceptors
B. N-cholinoreceptors
C. α-adrenoreceptors
D. β-adrenoreceptors
E. α- and β-adrenoreceptors

116. A man weighs 80 kg, after long physical activity his circulating blood
volume is reduced down to 5,4 l, hematocrit makes up 50%, whole blood
protein is 80 g/l. These blood characteristics are determined first of all by:
A. Water loss with sweat
B. Increased number of erythrocytes
C. Increased protein concentration in plasm
D. Increased circulating blood volume
E. Increased diuresis

117. A 16 year old boy after an illness has diminished function of protein
synthesis in liver as a result of vitamin K deficiency. It will cause
disturbance of:
A. Blood coagulation
B. Erythrocyte sedimentation rate
C. Anticoagulant generation
D. Erythropoietin secretion
E. Osmotic blood pressure

118. In response to a change in body position from horizontal to vertical


blood circulation system develops reflectory pressor reaction. Which of the
following is its compulsory component?
A. Systemic constriction of the venous vessels
B. Systemic dilatation of the arterial resistive vessels
C. Decrease in the circulating blood volume
D. Increase in the heart rate
E. Weakening of the pumbing ability of heart

119. Examination of a pregnant woman revealed twice as much


concentration of fibrinogen in blood plasm. What ESR can this woman
have?
A. 40-50 mm/h
B. 10-15 mm/h
C. 2-12 mm/h
D. 5-10 mm/h
E. 0-5 mm/h

120. Introduction of a big dose of histamine to an experimental animal


caused abrupt drop of arterial pressure as a result of:
A. Dilatation of resistance vessels
B. Constriction of resistance vessels
C. Increase of heart rate
D. Decrease of heart rate
E. Decrease of heart rate and force

121. Systemic arterial pressure of an adult dropped from 120/70 to


90/50 mm Hg that led to reflectory vasoconstriction. The
vasoconstriction will be maximal in the following organ:
A. Bowels
B. Heart
C. Brain
D. Kidneys
E. Adrenals

122. Vagus branches that innervate heart are being stimulated during an
experiment. This caused reduction of heart rate due to the intensification of
the following process (through the cell membrane of cardiac pacemaker):
A. Potassium ion yield
B. Potassium ion entry
C. Calcium ion entry
D. Calcium ion yield
E. Calcium and potassium ion yield
123. Rest potential of a cell equals -80 mV. At what stage of
action potential did the membrane potential equal +30 mV?
A. Reverse polarization
B. After hyperpolarization
C. After depolarization
D. Depolarization

124. A 35 year old man got an injury that caused complete disruption of
spinal cord at the level of the first cervical segment. What respiration
changes will be observed?
A. It will come to a standstill
B. No changes will be observed
C. Diaphragmal respiration will be maintained, thoracic respiration will disappear
D. Thoracic respiration will be maintained, diaphragmal respiration will disappear
E. It will become infrequent and deep

125. A doctor asked a patient to breath out fully after taking a normal
breath. What muscles contract during such exhalation?
A. Abdominal muscles
B. External intercostal muscles
C. Diaphragm
D. Trapezius muscles
E. Pectoral muscles

126. A man was intoxicated with mushrooms. They contain muscarine


that stimulates muscarinic cholinoreceptors. What symptoms signalize
intoxication with inedible mushrooms?
A. Myotic pupils
B. Mydriatic pupils
C. Bronchi dilatation
D. Increased heart rate
E. Rise of arterial pressure

127. A man presents with increased heart rate, mydriatic pupils, dry
mouth. This condition results from the activation of the following system of
function regulation:
A. Sympathetic
B. Parasympathetic
C. Metasympathetic
D. Vago-insular
E. Hypothalamo-pituitary-adrenal

128. In course of an experiment a peripheral section of vagus of an


expiremental animal is being stimulated. What changes will be observed?
A. Heart rate fall
B. Heart hurry
C. Pupil dilation
D. Increase of respiration rate
E. Bronchi dilation

129. Voluntary breath-holding caused increase of respiration depth and


frequency. The main factor stimulating these changes of external respiration
is:
A. Increased tension of CO2 in blood
B. Increased tension of O 2 in blood
C. Decreased tension of O 2 in blood
D. Decreased tension of CO 2 in blood
E. Decreased concentration of H+ in blood

130. A patient has delayed conduction of excitement through the


atrioventricular node. What changes of ECG will be observed?
A. Prolongation of P-Q interval
B. Prolongation of Q-S interval
C. Negative T wave
D. S-T-segment displacement
E. Prolongation of Q-T interval

131. Surface with an intact toad on it was inclined to the right. Tone of
extensor muscles became reflectory higher due to the activation of the
following receptors:
A. Vestibuloreceptors of utricle and saccule
B. Vestibuloreceptors of semicircular ducts
C. Mechanoreceptors of foot skin
D. Photoreceptors of retina
E. Proprioreceptors

132. In course of an experiment a toad's right labyrinth was destroyed. It


will cause amyotonia of the following muscles:
A. Right extensors
B. Left flexors
C. Left extensors
D. Right flexors
E. Right and left extensors

133. An animal has an increased tonus of extensor muscles. This the


result of intensified information transmission to the motoneurons of
the spinal cord through the following descending pathways:
A. Vestibulospinal
B. Medial corticospinal
C. Reticulospinal
D. Rubrospinal
E. Lateral corticospinal
134. Workers of a hothouse farm work under conditions of
unfavourable microclimate: air temperature is +37 oC, relative humidity
is 90%, air speed is 0,2 m/s. The way of heat emission under these
conditions will be:
A. Evaporation
B. Heat conduction
C. Convection
D. Radiation
E. All the ways

135. Lungs of a preterm infant have areas of atelectasis (pulmonary


collapse). The main cause is:
A. Surfactant deficiency
B. Increased viscous resistance
C. Underdeveloped inspiration muscles
D. Diminished force of surface tension of lungs
E. Surfactant excess

136. Vagi of an experimental animal were cut on both sides. What


respiration changes will be observed?
A. It will become deep and infrequent
B. It will become shallow and frequent
C. It will become deep and frequent
D. It will become shallow and infrequent
E. No changes will be observed

137. A cardiac electric stimulator was implanted to a 75 year old man with
heart rate of 40 bpm. Thereafter the heart rate rose up to 70 bpm. The
electric stimulator has undertaken the function of the following heart part:
A. Sinoatrial node
B. Atrioventricular node
C. His' bundle branches
D. His' bundle fibers
E. Purkinje's fibers

138. A patient came to the hospital complaining about quick fatigability


and apparent muscle weakness. Examination revealed an autoimmune
disease that causes disorder of functional receptor condition in
neuromuscular synapses. What transmitter will be blocked?
A. Acetylcholine
B. Noradrenalin
C. Dopamine
D. Serotonin
E. Glycine

139. Which muscle contraction will be observed in the upper extremity


during holding (not moving) a load in a certain position?
A. Isometric
B. Isotonic
C. Auxotonic
D. Concentric
E. Excentric

140. Examination of a 35 year old patient revealed high acidity of gastric


juice. What receptors should be blocked in order to reduce it?
A. Histamine
B. α1-adrenoreceptors
C. α2-adrenoreceptors
D. β1-adrenoreceptors
E. β2-adrenoreceptors

141. A young woman who entered a production department where it


strongly smelt of paints and varnishes had a bronchospasm. This reflex was
caused by irritation of the following receptors:
A. Irritant
B. Juxtaglomerular
C. Pleura receptors
D. Central chemoreceptors
E. Peripheral chemoreceptors

142. A 60-year-old patient presents with weakened peristaltic activity of


the bowels. Which of the following foodstuffs would stimulate peristalsis
most of all?
A. Brown bread
B. White bread
C. Meat
D. Lard
E. Tea

143. An isolated muscle fiber is under examination. It was established


that the threshold of stimulation force became significantly lower.
What is the cause of this phenomenon?
A. Activation of sodium channels of membrane
B. Activation of potassium channels of membrane
C. Inactivation of sodium channels of membrane
D. Inactivation of potassium channels of membrane
E. Block of energy production in the cell

144. It was established that agglutination of the recipient’s blood


erythrocytes had been caused by the standard sera from the I and II groups.
Serum from the III group as well as anti-Rh serum hadn‘t provoke any
agglutination. Which blood group and rhesus is allowed to be transfused this
recipient?
A. B, α (III) Rh-
B. A, β (II) Rh-
C. 0, α,β(I) Rh+
D. AB (IV), Rh+
E. AB (IV), Rh-

145. A patient consumed a lot of reach in proteins food that caused


increase of rate of proteolytic enzymes of pancreatic juice. It is also
accompanied by increase of rate of the following enzyme:
A. Tripsin
B. Pepsin
C. Enterokinase
D. Gastricsin
E. Renin

146. In course of an experiment thalamocortical tracts of an animal were


cut. What type of sensory perception remained intact?
A. Olfactory
B. Auditory
C. Exteroreceptive
D. Visual
E. Nocice
Krok 1 – 2015 Microbiology Base
1. Quite often the cause of secondary immunodeficiency is an infection
involvement, when the causative agents propagate directly in the cells of
immune system and destroy it. The following diseases are characterized
by:
A. Infectious mononucleosis, AIDS
B. Tuberculosis, mycobacteriosis
C. Poliomyelitis, type A hepatitis
D. Dysentery, cholera
E. Q-febris, epidemic typhus

2. It is planned to use the territory of an old cattle burial ground (which is not
used for more than 50 years) for building houses. But ground analysis
revealed presence of the pathogen of the very dangerous illness. Which of the
indicated microorgonisms is likely to remain in the ground for such a long
time?
A. Bacillus anthracis
B. Francisella tularensis
C. Brucella abortus
D. Yersinia pestis
E. Mycobacterium bovis

3. From the nasopharynx of a 5-year-old child it was excreted amicroorganism


which is identical to Corynebacterium diphtheriae dose according to
morphological and biochemical signs.Microorganism does not produce
exotoxin. As a result of what process can this microorganism become
toxigenic?
A. Phage conversion
B. Cultivation in the telluric environment
C. Passing through the organism of the sensative animals
D. Growing with antitoxic serum
E. Chromosome mutation

4. While studying a microslide obtained from the punctuate of a regional


lymph node and stained by Romanovsky -Giemsa method a physician
revealed some light-pink thin microorganisms with 12-14 regular spiral
coils and pointed ends, up to 10-13 micrometer long. This might be the
causative agent of the following disease:
A. Syphilis
B. Trypanosomiasis
C. Leptospirosis
D. Relapsing fever
E. Leishmaniasis
5. Sanitary bacteriological research on water by the membrane filter method
revealed two red colonies on a membrane filter (Endo agar) through which
500 ml of analyzed water were passed. Calculate the coli index and coli
titer of the analyzed water:
A. 4 and 250
B. 2 and 500
C. 250 and 4
D. 500 and 2
E. 250 and 2

6. While examining a patient an otolaryngologist noticed hyperaemia and


significantly edematous tonsils with a grayish film upon them.
Microscopical examination of this film revealed some gram-positive bacilli
placed at an angle with each other. What disease might be suspected?
A. Diphtheria
B. Angina
C. Scarlet fever
D. Meningococcal nasopharyngitis
E. Epidemic parotitis

7. Patient with vomiting, dizziness, sensation of dubble vision, difficult


swallowing was admitted to the hospital. Doctor suspects botulism. What
diagnostic methods should be used for diagnosis approving?
A. Biological test, bacteriological
B. Allergic test, serological
C. Bacteriological, mycological
D. Protozoological, microscopical

8. A man who was bitten by the unknown dog applied to the surgeon. Wide
ragged woundes were localised on the face. What curative-prophylactic aid
should be given to prevent rabies?
A. Start immunisation with rabies vaccine
B. Prescribe combine antibiotic therapy
C. Immediate injection of DPT (Diphtheria, Pertusis, Tetanus) vaccine
D. Hospitalize the patient and keep under the doctor's supervision
E. Immediately inject normal gamma globulin

9. In a patient with clinical signs of immunodeficiency the number and


functional activity of T and B lymphocytes are not changed. Defect with
dysfunction of antigen-presentation to the immunocompetent cells was found
during investigation on the molecule level. Defect of what cells is the most
probable?
A. Macrophages, monocytes
B. Т-lymphocytes, В-lymphocytes
C. NK-cells
D. Fibroblasts, Т- lymphocytes, В-lymphocytes
E. 0-lymphocytes

10. A patient with complaints of 3-day-long fever, general weakness, loss of


appetite came to visit the infectionist. The doctor suspected enteric fever.
Which method of laboratory diagnosis is the best to confirm the diagnosis?
A. Detachment of blood culture
B. Detachment of myeloculture
C. Detachment of feces culture
D. Detachment of urine culture
E. Detachment of pure culture

11. A consumptive patient has an open pulmonary form of disease.


Choose what sputum staining should be selected for finding out the
tubercle (Koch's) bacillus?
A. Method of Ziel-Neelsen
B. Method of Romanowsky-Giemsa
C. Method of Gram
D. Method of Neisser
E. Method of Burry-Gins

12. During surgical operation a blood transfusion was made. The blood must
be checked to find antigens of some disease. What disease is expected to be
found?
A. Virus of hepatitis B
B. Virus of hepatitis A
C. Adenovirus
D. Enterovirus
E. Virus of hepatitis E

13. A 42-year-old man who has been injured in a car accident is brought
into the emergency room. His blood alcohol level on admission is 250
mg/dL. Hospital records show a prior hospitalization for alcohol related
seizures. His wife confirms that he has been drinking heavily for 3
weeks. What treatment should be provided to the patient if he goes into
withdrawal?
A. Diazepam
B. Phenobarbital
C. Pentobarbital
D. Phenytoin
E. None

14. A patient who came to the doctor because of his infertility was administered
to make tests for toxoplasmosis and chronic gonorrhoea. Which reaction
should be performed to reveal latent toxoplasmosis and chronic gonorrhoea
of the patient?
A. (R)CFT- Reiter's complement fixation test
B. IFA - Immunofluorescence assay
C. Immunoblot analysis
D. RDHA - Reverse direct hemagglutination assay
E. RIHA - Reverse indirect hemagglutination assay

15. On bacteriological study of rinsing water of the patient with food poisoning,
the pure bacterial culture was inoculated with the following properties: gram-
negative motile bacillus in the Endo environment grows like achromic colony.
Representative of what genus has caused the illness?
A. Salmonella
B. Shigella
C. Yersinia
D. Escherichia
E. E Citrobacter

16. The person was selling "homemade pork" sausages on the market. State
sanitary inspector suspected falcification of the sausages.With help of what
serological immune reaction can food substance be identified?
A. Precipitation test
B. Indirect hemagglutination test
C. Agglutination test
D. Immunofluorescence test
E. Complement- fixation test

17. While registering the child to the school Mantu's test was made to define
whether revaccination was needed test result is negative. What does this
result of the test mean?
A. Absence of cell immunity to the tuberculosis
B. Presence of cell immunity to the tuberculosis
C. Absence of antibodies for tubercle bacillus
D. Absence of antitoxic immunity to the tuberculosis
E. Presence of antibodies for tubercle bacillus

18. The donor who for a long time didn't donate the blood was investigated
with IFA method. Anti-HBs antibodies were revealed. What does positive
result of IFA in this case mean?
A. Previous hepatitis B
B. Acute hepatitis B
C. Acute hepatitis C
D. Chronic hepatitis В
E. Chronic hepatitis С

19. Bacteriological examination of purulent discharges from the urethra


revealed gram-negative bacteria looking like coffee beans. They were
localized in the leukocytes and could decompose glucose and maltose to
acid. These are the causative agents of the following disease:
A. Gonorrhoea
B. Syphilis
C. Veneral lymphogranulomatosis
D. Soft chancre
E. Melioidosis

20. Scraps of the mycelium of a fungus, spores, air bubbles and fat drops were
discovered on microscopy of the patient's hair excluded from the infected
areas. For what fungus disease is this microscopic picture characteristic?
A. Favus
B. Microspory
C. Trichophytosis
D. Epidermophytosis
E. Sporotrichosis

21. In order to speed up healing of a wound of oral mucosa a patient was


prescribed a drug that is a thermostable protein occuring in tears, saliva,
mother's milk as well as in a new-laid hen's egg. It is known that this protein
is a factor of natural resistance of an organism. What is it called?
A. Lysozyme
B. Complement
C. Interferon
D. Interleukin
E. Imanine

22. Study of bacteriological sputum specimens stained by the Ziel-Neelsen


method revealed some bright-red acid-resistant bacilli that were found in
groups or singularly. When inoculated onto the nutrient media, the signs of
their growth show up on the 10-15 day. These bacteria relate to the following
family:
A. Micobacterium tuberculosis
B. Yersinia pseudotuberculosis
C. Histoplasma dubrosii
D. Klebsiella rhinoscleromatis
E. Coxiella burnettii

23. A man was admitted to the hospital on the 5th day of disease that
manifested itself by jaundice, muscle aching, chill, nose bleedings. In course
of laboratory diagnostics a bacteriologist performed dark-field microscopy
of the patient's blood drop. Name a causative agent of this disease:
A. Leptospira interrogans
B. Borrelia dutlonii
C. Calymmatobacterium granulomatis
D. Bartonella bacilloformis
E. Rickettsia mooseri

24. Gramnegative bin-shaped diplococcus inside and outside of leucocytes were


detected on bacteriological examination of the purulent exudates from the
cervix of the uterus. Name the causative agent of purulent inflammation of
the cervix of the uterus.
A. Neisseria gonorroeae
B. Chlamidia trachomatis
C. Haemophilus vaginalis
D. Trichomonas vaginalis
E. Calymmatobacterium granulomatis

25. Patient with diarrhoea was admitted to the infection unit. Gramnegative
curved rod-like bacteria were founded on bacterioscopic examination of
faecal masses. What is the most likely disease in this patient?
A. Cholera
B. Typhoid fever
C. Salmonellosis gastroenteritis
D. Diphtheria
E. Intestinal form of plague

26. In a 2-year-old child with catarrhal presentations and skin rash a


pediatrician suspected scarlet fever. The child was given intracutaneously
a small dose of serum antibody to the streptococcal erythrogenic toxin; on
the site of injection the rash disappeared. What do the reaction results
mean?
A. The clinical diagnosis was confirmed
B. The child has hypersensitivity to the erythrogenic toxin
C. The disease wasn't caused by haemolytic streptococcus
D. The whole serum dose may be injected intravenously
E. The child has very weak immune system

27. From the defecation of a 6-year-old ill child, who has artificial feeding, the
intestinal bacillus with antigen structure 0-111 is excreted. What is the
diagnosis?
A. Coli-enteritis
B. Gastroenteritis
C. Cholera-like diseasis
D. Food poisoning
E. Disentery-like diseasis

28. For serological diagnostics of the whooping cough it was made large-scale
reaction with parapertussis and pertussis diagnosticums. At the bottom of
the test-tubes with diagnosticum of Bordetella parapertussis grain-like
sediment formed. What antibodies have this reaction revealed?
A. Agglutinins
B. Precipitins
C. Opsonins
D. Bacteriolysins
E. Antitoxins

29. A man died from an acute infectious disease accompanied by fever, jaundice,
haemorrhagic rash on the skin and mucous membranes as well as by acute
renal insufficiency. Histological examination of renal tissue (stained by
Romanovsky-Giemsa method) revealed some convoluted bacteria looking
like C und S letters. What bacteria were revealed?
A. Leptospira
B. Treponema
C. Spirilla
D. Borrelia
E. Campilobacteria

30. A 16 y.o. boy from a countryside entered an educational establishment.


Scheduled Manteux test revealed that the boy had negative reaction. What
are the most reasonable actions in this case?
A. To perform BCG vaccination
B. To repeat the reaction in a month
C. To perform serodiagnostics of tuberculosis
D. To isolate the boy temporarily from his mates
E. To perform rapid Price diagnostics

31. Examination of a patient with pustular skin lesions allowed to isolate a


causative agent that forms in the blood agar roundish yellow middle-sized
colonies surrounded by haemolysis zone. Smears from the colonies contain
irregular-shaped clusters of gram-positive cocci. The culture is oxidase- and
catalase-positive, ferments mannitol and synthesizes plasmocoagulase. What
causative agent was isolated?
A. Staphylococcus aureus
B. Streptococcus agalactiae
C. Streptococcus pyogenes
D. Staphylococcus epidermidis
E. Staphylococcus saprophyticus

32. Microscopic examination of a Gram-stained scrape from patient's tongue


revealed oval, round, elongated chains of dark-violet gemmating cells.
What disease can be caused by this causative agent?
A. Candidosis
B. Actinomycosis
C. Streptococcic infection
D. Staphylococcic infection
E. Diphtheria
33. From pharynx of a child with suspected diphtheria a pure culture of
microorganisms was isolated. Their morphological, tinctorial, cultural and
biochemical properties appeared to be typical for diphtheria causative
agents. What study should be conducted in order to drow a conclusion that
this is a pathogenic diphtheria bacillus?
A. Estimation of toxigenic properties
B. Estimation of proteolytic properties
C. Estimation of urease activity
D. Estimation of cystinous activity
E. Estimation of ability to decompose starch

34. Examination of a child revealed some whitish spots looking like coagulated
milk on the mucous membrane of his cheeks and tongue. Analysis of
smears revealed gram-positive oval yeast-like cells. What causative agents
are they?
A. Candida
B. Staphylococci
C. Diphtheria bacillus
D. Actinomycetes
E. Fusobacteria

35. A duodenal content smear of a patient with indigestion contains protosoa 10-
18 mcm large. They have piriform bodies, 4 pairs of filaments, two
symmetrically located nuclei in the broadened part of body. What kind of the
lowest organisms is it?
A. Lamblia
B. Dysentery ameba
C. Trichomonas
D. Intestinal ameba
E. Balantidium

36. Blood of a patient with presumable sepsis was inoculated into sugar broth.
There appeared bottom sediment. Repeated inoculation into blood agar
caused growth of small transparent round colonies surrounded by hemolysis
zone. Examination of a smear from the sediment revealed gram-positive cocci
in form of long chains. What microorganisms are present in blood of this
patient?
A. Streptococci
B. Micrococci
C. Staphylococci
D. Tetracocci
E. Sarcina

37. On bacteriological examination of the defecation of a 4-months-old baby with


the symptoms of acute bowel infection there were revealed red colonies
spread in the large quantity in the Endo environment. What microorganism
can it be?
A. Escherichia
B. Salmonella
C. Staphylococcus
D. Streptococcus
E. Shigell

38. Bacteriological examination of a patient with food poisoning required


inoculation of a pure culture of bacteria with the following properties: gram-
negative movable bacillus that grows in the Endo's medium in form of
colourless colonies. A representative of which species caused this disease?
A. Salmonella
B. Shigella
C. Iersinia
D. Esherichia
E. Citrobacter

39. Examination of a young man in the AIDS centre produced a positive result of
immune-enzyme assay with HIV antigens. Patient's complaints about state of
his health were absent. What can the positive result of immune-enzyme assay
be evidence of?
A. HIV infection
B. Being ill with AIDS
C. Being infected with HBV
D. Having had AIDS recently
E. HBV persistence

40. Microscopy of stained (Ziehl-Neelsen staining) smears taken from the


sputum of a patient with chronic pulmonary disease revealed red bacilli.
What property of tuberculous bacillus was shown up?
A. Acid resistance
B. Alkali resistance
C. Alcohol resistance
D. Capsule formation
E. Sporification

41. Reaction of passive hemagglutination conducted with erythrocytic typhoid


Vi-diagnosticum helped to reveal some antibodies in the dilution of the
patient's serum at a ratio of 1:80 that exceeds the diagnostic titer. Such
result witnesses of:
A. Being a potential carrier of typhoid bacilli
B. Being ill with acute typhoid fever
C. Typhoid fever recurrence
D. Incubation period of typhoid fever
E. Reconvalescence of a patient ill with typhoid fever
42. In order to determine toxigenicity of diphtheria bacilli a strip of filter paper
impregnated with antitoxic diphtherial serum was put on the dense nutrient
medium. There were also inoculated a microbal culture under examination
and a strain that is known to be toxigenic. If the microbal culture under
examination produces exotoxin, this wil result in formation of:
A. Precipitin lines
B. Haemolysis zones
C. Zones of diffuse opacification
D. Zones of lecithovitellinous activity
E. Precipitin ring

43. A 50-year-old patient with typhoid fever was treated with Levomycetin,
the next day his condition became worse, temperature rised to 39,6 0 С.
What caused worthening?
A. The effect of endotoxin agent
B. Allergic reaction
C. Irresponsiveness of an agent to the levomycetin
D. Secondary infection addition
E. Reinfection

44. In order to estimate toxigenity of diphtheria agents obtained from patients


the cultures were inoculated on Petri dish with nutrient agar on either side of
a filter paper strip that was put into the centre and moistened with
antidiphtheric antitoxic serum. After incubation of inoculations in agar the
strip-like areas of medium turbidity were found between separate cultures
and the strip of filter paper. What immunological reaction was conducted?
A. Precipitation gel reaction
B. Coomb's test
C. Agglutination reaction
D. Rings precipitation reaction
E. Opsonization reaction

45. A patient with clinical signs of encephalitis was delivered to the infectious
diseases hospital. Anamnesis registers a tick bite. Hemagglutination-
inhibition reaction helped to reveal antibodies to the causative agent of
tick-borne encephalitis in the dilution 1:20 which is not diagnostic. What
actions should the doctor take after he had got such result?
A. To repeat the examination with serum taken 10 days later
B. To examine the same serum
C. To apply more sensitive reaction
D. To repeat examination with another diagnosticum
E. To deny diagnosis of tick-borne encephalitis

46. The first grade pupils were examined in order to sort out children
for tuberculosis revaccination. What test was applied for this
purpose?
A. Mantoux test
B. Schick test
C. Supracutaneous tularin test
D. Burnet test
E. Anthraxine test

47. Clinical diagnosis of a female patient was gonorrhoea. What examination


method can be applied for confirmation of this diagnosis?
A. Microscopy of pathological material
B. Infection of laboratory animals
C. Test with bacteriophage
D. Hemagglutination reaction
E. Immobilization reaction

48. A patient suffering from periodical attacks caused by inhalation of


different flavoring substances was diagnosed with atopic bronchial
asthma. IgE level was increased. This is typical for the following type of
reactions:
A. Anaphylactic reactions
B. Cytotoxic reactions
C. Immunocomplex reactions
D. delayed-type hypersensitivity
E. Autoimmune reactions

49. Bacteriological laboratory examines canned meat whether it contains


botulinum toxin. For this purpose an extract of test specimen and antitoxic
antibotulinic serum of A, B, E types were introducted to a group of mice
under examination; a control group of mice got the extract without
antibotulinic serum. What serological reaction was applied?
A. Neutralization
B. Precipitation
C. Complement binding
D. Opsono-phagocytic
E. Double immune diffusion

50. For the purpose of retrtospective diagnostics of recent bacterial dysentery it


was decided to perform serological examination of blood serum in order to
determine antibody titer towards Shiga bacilli. What of the following
reactions should be applied?
A. Passive hemagglutination
B. Bordet-Gengou test
C. Precipitation
D. Hemolysis
E. Bacteriolysis
51. During the repeated Widal's agglutination test it was noticed that the ratio
of antibody titers and O-antigens S.typhi in the patient's serum had
increased from 1:100 to 1:400. How would you interpret these results?
A. The patient has typhoid fever
B. The patient is an acute carrier of typhoid microbes
C. The patient is a chronic carrier of typhoid microbs
D. The patient previously had typhoid fever
E. The patient was previously vaccinated against typhoid fever

52. A patient recovered from Sonne dysentery and was once more infected
with the same causative agent. What is such infection form called?
A. Reinfection
B. Recidivation
C. Superinfection
D. Persisting infection
E. Chronic infection

53. A 10-year-old child had the mantoux tuberculin test administered. 48 hours
later a papule up to 8 mm in diameter appeared on the site of the injection.
What type of hypersensitivity reaction developed after the tuberculin
injection?
A. Type IV hypersensitivity reaction
B. Arthus phenomenon
C. Seroreaction
D. Atopic reaction
E. Type II hypersensitivity reaction

54. A patient with clinical presentations of immunodeficiency went through


immunological examinations. They revealed significant loss of cells that
form rosettes with erythrocytes of a ram. What conclusion can be made
according to the analysis data?
A. Decrease of T-lymphocytes rate
B. Decrease of B-lymphocytes rate
C. Decrease of natural killer cell rate
D. Decrease of complement system rate
E. Insufficiency of effector cells of humoral immunity

55. As a result of durative antibiotic therapy a 37-year old patient developed


intestinal dysbacteriosis. What type of drugs should be used in order to
normalize intestinal microflora?
A. Eubiotics
B. Sulfanilamides
C. Bacteriophages
D. Autovaccines
E. Vitamins
56. Among junior children of an orphanage an outbreak of intestinal
infection with signs of colienteritis was registered. In order to identify
isolated causative agent it is necessary to:
A. Study antigenic properties of the causative agent
B. To determine sensitivity to antibiotics
C. To study sensitivity to bacteriophages
D. To study biochemical properties of the causative agent
E. To study virulence of the causative agent

57. Urine examination of a patient with acute cystitis revealed leukocytes and a
lot of gram-negative bacilli. Inoculation resulted in growth of colonies of
mucous nature that formed green soluble pigment. What microorganism is
the most probable cause of the disease?
A. Pseudomonas aeruginosa
B. Escherihia coli
C. Klebsiella pneumoniae
D. Proteus mirabilis
E. Salmonella enteritidis

58. A laboratory received a material from a patient's wound. Ppreliminary


diagnosis is gaseous gangrene. What microbiological method should be
applied to determine species of causative agent?
A. Bacteriological
B. Allergic
C. Bacterioscopic
D. Serological
E. RIA

59. A virological laboratory obtained pathological material (mucous


discharges from nasal meatuses) taken from a patient with provisional
diagnosis "influenza". What quick test will allow to reveal specific viral
antigen in the material under examination?
A. Direct and indirect immunofluorescence test
B. Direct and indirect fluorescence immunoassay
C. Hemagglutination inhibition assay
D. Radioimmunoassay

60. In the surgical department of a hospital there was an outbreak of hospital


infection that showed itself in often postoperative wound abscesses.
Bacteriological examination of pus revealed aurococcus. What
examination shall be conducted to find out the source of this causative
agent among the department personnel?
A. Phagotyping
B. Microscopical examination
C. Serological identification
D. Estimation of antibiotic susceptibility
E. Biochemical identification

61. A 7 year old child often suffers from streprococcic angina. Doctor suspected
development of rheumatism and administered serological examination.
The provisional diagnosis will be most probably confirmed by presence of
antibodies to the following streptococcic antigen:
A. O-streptolysin
B. C-carbohydrate
C. M-protein
D. Erythrogenic toxin
E. Capsular polysaccharide

62. A culture of monkey cells (Vero) and a group of mouse sucklings were
infected with an inoculum taken from a child with provisional diagnosis
"enterovirus infection". There was no cytopathic effect on the cell culture but
mouse sucklings died. What enteric viruses might have caused disease of this
child?
A. Coxsackie A
B. Coxsackie B
C. ECHO virus
D. Polioviruses
E. Unclassified enteric viruses 68-71

63. A patient has been suffering from elevated temperature and attacks of
typical cough for 10 days. Doctor administered inoculation of mucus from
the patient's nasopharynx on the agar. What microorganism is presumed?
A. Pertussis bacillus
B. Pfeiffer's bacillus
C. Listeria
D. Klebsiella
E. Staphylococcus

64. A patient of surgical department complains about pain in the small of her
back and in the lower part of her belly; painful and frequent urination.
Bacteriological examination of urine revealed gram-negative oxidase-
positive rod-like bacteria forming greenish mucoid colonies with specific
smell. What causative agent can it be?
A. Pseudomonas aeruginosa
B. Proteus mirabilis
C. E.coli
D. Str.pyogenes
E. Mycoplasma pneumonie

65. A female patient underwent liver transplantation. 1,5 month after it her
condition became worse because of reaction of transplant rejection. What
factor of immune system plays the leading part in this reaction?
A. T-killers
B. Interleukin-1
C. Natural killers
D. B-lymphocytes
E. T-helpers

66. Microscopical examination of a microbal culture revealed fusiform


spore-forming microorganisms that get violet-blue Gram's stain. What
microorganisms were revealed?
A. Clostridia
B. Streptococci
C. Spirochaete
D. Actinomycete
E. Diplococci

67. A specimen stained by Ozheshko method contains rod-like


microorganisms stained blue with round terminal components stained
red. What are these components called?
A. Spores
B. Cilia
C. Flagella
D. Capsules
E. Mesosomas

68. During the regular sanitary-epidemiological inspection of a pharmacy, the


bacteriological analysis of air was performed. The air was found to have
bacilli, yeast fungi, hemolytic streptococci, micrococci. Which of the detected
microorganisms indicate the direct epidemic danger?
A. Haemolytic streptococci
B. Micrococci
C. Bacilli
D. Yeast fungi

69. A bacteriological laboratory received sputum sample of a patient suffering


from tuberculosis. Bacterioscopic examination of smears and detection of
tuberculosis bacillus can be realized by one of enrichment methods that
involves processing of sputum only with solution of caustic soda. What is
this method called?
A. Homogenization
B. Inactivation
C. Flotation
D. Filtration
E. Neutralization
70. A pregnant woman was registered in an antenatal clinic and underwent
complex examination for a number of infections. Blood serum contained IgM
to the rubella virus. What is this result indicative of?
A. Of primary infection
B. Of a chronic process
C. The woman is healthy
D. Of exacerbation of a chronic disease
E. Of recurring infection with rubella virus

71. A 65-year-old man has purulent abscess on his neck. Analyses


revealed a culture of gram-positive cocci with plasmocoagulase
activity. This culture relates most likely to:
A. Staphylococcus aureus
B. Streptococcus pyogenes
C. Staphylococcus epidermidis
D. Staphylococcus saprophyticus

72. Material taken from a patient with provisional diagnosis "influenza" was
referred to a laboratory. For virological examination the hemadsorption
reaction was applied. This reaction can be applied for detection of the
following viruses:
A. Viruses containing hemagglutinins
B. All the simple viruses
C. All the complex viruses
D. DNA-genomic viruses
E. Any viruses

73. Vomiting matters of a patient suspected of having cholera were delivered to


the bacteriological laboratory. The material was used for preparing a
"hanging drop" specimen. What type of microscopy will be applied for
identification of the causative agent by its mobility?
A. Phase-contrast microscopy
B. Electron microscopy
C. Immune and electron microscopy
D. Fluorescence microscopy
E. Immersion microscopy

74. Inoculum from pharynx of a patient ill with angina was inoculated into
blood-tellurite agar. It resulted in growth of grey, radially striated (in form
of rosettes) colonies 4-5 mm in diameter. Gram-positive bacilli with
clublike thickenings on their ends placed in form of spread wide apart
fingers are visible by microscope. What microorganisms are these?
A. Diphtheria corynebacteria
B. Botulism clostridia
C. Diphtheroids
D. Streptococci
E. Streptobacilli

75. During examination of a patient a dentist revealed a lot of "white spots" -


zones of enamel demineralization. What microorganisms take part in the
development of this process?
A. Streptococcus mutans
B. Streptococcus salivarius
C. Streptococcus pyogenes
D. Veilonella parvula
E. Staphylococcus epidermidis

76. Planned mass vaccination of all newborn 5-7 day old children against
tuberulosis plays an important role in tuberculosis prevention. In this
case the following vaccine is applied:
A. BCG
B. Diphteria and tetanus toxoids and pertussis vaccine
C. Diphtheria and tetanus anatoxin vaccine
D. Adsorbed diphtheria vaccine

77. A 4-year-old child presents with general weakness, sore throat and
deglutitive problem. After his examination a doctor suspected diphtheria
and sent the material to the bacteriological laboratory. In order to determine
the diphtheria causative agent the material should be inoculated into the
following differential diagnostic medium:
A. Blood tellurite agar
B. Endo's agar
C. Ploskyrev's agar
D. Sabouraud's agar
E. Levenshtein-Yessen agar
Krok 1 – 2015 Path Anatomy Base
1. Multiple oval ulcers along the intestine were revealed on autopsy of the
person, who died from diffuse of peritonitis in the distant part of the small
intestine. Bottom parts of the ulcers are clear, smooth, formed with muscular
or serous covering, edges of ulcers are flat, rounded. There are perforations
up to 0,5 cm in diameter in two ulcers. What diseasis can be diagnosed?
A. Typhoid fever
B. Dysentery
C. Cholera
D. Tuberculosis
E. Typhus

2. Patient suffering from trombophlebitis of the deep veins suddenly died.


Autopsy has shown freely lying red friable masses with dim crimped surface
in the trunk and bifurcation of the pulmonary artery. What pathologic
process was revealed by morbid anatomist?
A. Tromboembolism
B. Thrombosis
C. Tissue embolism
D. Embolism with foreign body
E. Fat embolism

3. Examination of a patient revealed a dense, movable skin tumour that is


standing out distinctly from the surrounding tissues. Its section is found to
be white and composed of fibrous tissue. Microscopic examination revealed
interlacing collagen fibers and few cells. What tumour is it?
A. Fibroma
B. Myoma
C. Histiocytoma
D. Dermatofibroma
E. Desmoid

4. A 50-year-old man has felt vague abdominal discomfort within past 4


months. Physical examination revealed no lymphadenopathy, and no
abdominal masses or organomegaly at palpation. Bowel sounds are heard.
An abdominal CT scan shows a 20 cm retroperitoneal soft tissue mass
obscuring the left psoas muscle. A stool specimen tested for occult blood is
negative. Which of the following neoplasms is this man most likely to have?
A. Lipoma
B. Melanoma
C. Hamartoma
D. Adenocarcinoma
E. Lymphoma

5. A 40-year-old woman has had a feeling of abdominal discomfort for the past
8 months. On pelvic examination, there is the right adnexal mass.
Abdominal CT scan demonstrates a 7 cm cystic mass involving the right
ovary with small areas of calcification. The uterus is normal in size. The right
fallopian tube and ovary have been removed surgically. Grossly, the mass on
sectioning is filled with abundant hair and sebum. Microscopically, the mass
has glandular spaces lined by columnar epithelium, squamous epithelium
with hair follicles, cartilage, and dense connective tissue. What type of
tumour is it?
A. Teratoma
B. Squamous cell carcinoma of ovary
C. Melanoma
D. Sarcoma of ovary
E. Metastase of cervical carcinoma

6. A man died 8 days after the beginning of the disease. He was diagnosed
with dysentery. At the autopsy it was found out a thickened wall of the
sigma and rectum, fibrinous membrane on the surface of mucous
membrane. Histologically: there is a deep necrosis of mucous membrane
with infiltration of necrotic masses with fibrin. What kind of colitis does
correspond to the changes?
A. Diphtheritic
B. Catarrhal
C. Ulcerative
D. Chronic
E. Gangrenous

7. A woman suffering from dysfunctional metrorrhagia was made a diagnostic


abortion. Histologically in the scrape there were a lot of small stamped
glandulars covered with multirowed epithelium. The lumens of some
glandulars were cystically extended. Choose the variant of general
pathologic process in the endometrium.
A. Glandular-cystic hyperplasia of endometrium
B. Atrophy of endometrium
C. Metaplasia of endometrium
D. Neoplasm of endometrium
E. Hypertrophic growth

8. A 46 year-old man complains of difficult nose breathing. Mikulich cells,


storage of epithelioid cells, plasmocytes, lymphocytes, hyaline balls are
discovered in the biopsy material of the nose thickening. What is the most
likely diagnosis?
A. Scleroma
B. Virus rhinitis
C. Allergic rhinitis
D. Rhinovirus infection
E. Meningococcal nasopharyngitis

9. Extensive thromboembolic infarction of the left cerebral hemispheres, large


septic spleen, immunocomplex glomerulonephritis, ulcers on the edges of the
aortic valves, covered with polypous thrombus with colonies of
staphylococcus were revealed on autopsy of the young man who died in coma.
What disease caused cerebral thromboemboly?
A. Septic bacterial endocarditis
B. Septicemia
C. Acute rheumatic valvulitis
D. Septicopyemia
E. Rheumatic thromboendocarditis

10. A patient ill with diabetes mellitus felt acute pain in his right foot.
Objectively: foot thumb is black, foot tissues are edematous, there are
foci of epidermis desquamation, stinking discharges. What
clinicopathological form of necrosis is it?
A. Moist gangrene
B. Bedsore
C. Sequestrum
D. Dry gangrene
E. Infarction

11. A denaturation of proteins can be found in some substances. Specify the


substance that is used for the incomplete denaturation of hemoglobin:
A. Urea
B. Toluene
C. Sulfuric acid
D. Nitric acid
E. Sodium hydroxide

12. During surgery in a 17-year-old patient it was revealed the tumour of


4,5х5,0х3,5 sm in size on the lower surface of the liver with subserose
localization, of dark-red color. On the section tumour has cavities with
marked amount of blood. What is preliminary diagnosis?
A. Cavernous hemangioma
B. Capillar hemangioma
C. Hemangiopericytoma
D. Hemangioendothelioma
E. Lymphangioma

13. A sick man with high temperature and a lot of tiny wounds on the body has
been admitted to the hospital. Lice have been found in the folds of his
clothing. What disease can be suspected in the patient?
A. Epidemic typhus
B. Tularemia
C. Scabies
D. Malaria
E. Plague

14. On autopsy it is revealed that kidneys are enlarged, surface is large-granular


because of multiple cavities with smooth wall, which are filled with clear
fluid. What kidney disease did the patient have?
A. Polycystic kidney
B. Necrotic nephrosis
C. Pyelonephritis
D. Glomerulonephritis
E. Infarction

15. On autopsy it is revealed enlarged dense right lung, fibrin layers on the
pleura. Lung tissue is light green color on incision with muddy liqued
exudates. What lung disease are these symptoms typical for?
A. Lung-fever
B. Bronchopneumonia,
C. Interstitial pneumonia
D. Pulmonary gangreneі
E. Fibrosing alveolitis

16. On autopsy it is revealed: soft arachnoid membrane of the upper parts


of cerebral hemisphere is plethoric, it is of yellowish-green color,
soaked with purulent and fibrose exudate, it lookes like cap. For what
disease is it characteristical picture
A. Meningococcal meningitis
B. Tuberculous meningitis
C. Influenza meningitis
D. Meningitis at anthrax
E. Meningitis at typhus

17. On autopsy of the 58-year-old man it is revealed: mitral valve is deformed,


thickened, not totally closed. Microscopically: centers of collagen fibers are
eosinophilic, have positive fibrin reaction. The most likely it is:
A. Fibrinoid swelling
B. Fibrinoid inflammation
C. Mucoid swelling
D. Hyalinosis
E. Amyloidosis

18. Chronic inflammation and transformation of the one-layer ciliated


epithelium into multiple-layers flat epithelium was revealed in the
thickened mucous membrane of the bronchus bioptate of the patient
with smoke abuse. Which of the processes is the most likely?
A. Metaplasia
B. Hyperplasia of the epithelium
C. Squamous cancer
D. Leucoplacia
E. Epithelium hypertrophy

19. For a long time a 49-year-old woman was suffering from glomerulonephritis
which caused death.On autopsy it was revealed that kidneys size was 7х3х2.5
sm, weight is 65,0 g, they are dence and small-grained. Microscopically:
fibrinogenous inflammation of serous and mucous capsules, dystrophic
changes of parenchymatous organs, brain edema. What complication can
cause such changes of serous capsules and inner organs?
A. Uraemia
B. Anemia
C. Sepsis
D. DIC-syndrome
E. Thrombopenia

20. A patient died under conditions of cardiov ascular insufficiency. Autopsy


results: postinfarction cardiosclerosis, myocardium hypertrophy and
dilatation of its cavities, especially of its right ventricle. Liver is enlarged,
its surface is smooth, incision revealed that it was plethoric, with dark-red
specks against the background of brownish tissue. Histologically: plethora
of central parts of lobules; peritheral parts around portal tracts contain
hepatocytes in a state of adipose degeneration. How are these liver changes
called?
A. Nutmeg liver
B. Pseudonutmeg liver
C. Amyloidosis
D. Liver cirrhosis
E. Liver steatosis

21. A 59-year-old man has signs of the parenchymatous jaundice and portal
hypertension. On histological examination of the puncture of the liver
bioptate, it was revealed: beam-lobule structure is affected, part of
hepatocytes has signs of fat dystrophy, port-portal connective tissue
septa with formation of pseudo-lobules,with periportal lympho-
macrophage infiltrations. What is the most probable diagnosis?
A. Liver cirrhosis
B. Alcohol hepatitis
C. Chronic hepatosis
D. Viral hepatitis
E. Toxic dystrophy

22. On microscopic examination of the enlarged neck gland of a 14-year-old girl


it was revealed destruction of the tissue structure of the node, absence of the
lymph follicles, sclerotic and necrosis parts, cell constitution of the node is
polymorphous, lymphocites, eosinophiles, atypical cells of the large size with
multiple-lobule nuclei (Beresovsky-Shternberg cells) and onenucleus cells of
the large size are present. What is the most likely diagnosis?
A. Lymphogranulomatous
B. Acute lympholecosis
C. Chronic lympholecosis
D. Berkitt’s lymphoma
E. Fungous mycosis
23. A female patient suffering from bronchial asthma had got a viral infection
that provoked status asthmaticus with fatal outcome. Histological
examination of lungs revealed spasm and edema of bronchioles, apparent
infiltration of their walls with lymphocytes, eosinophils and other
leukocytes; labrocyte degranulation. What mechanism of hypersensitivity
underlies the described alterations?
A. Reagin reaction
B. Inflammatory
C. Autoimmune
D. Immune complex
E. Immune cytolysis

24. On autopsy of the 40-year-old woman suffering from rheumatic arthritis,


the enlarged solid spleen was revealed. On section its tissue is of the
mahogany color with enlarged follicles, which look like semi-transparent
grayish-whitish grains. What pathological process is the most likely?
A. Sago spleen
B. Glaze spleen
C. Waxy spleen
D. Hyaline spleen
E. Porphyric spleen

25. Local lymphonodules enlarged near the infected wound. Increased amount
of macrophages, lymphocytes, lymphatic follicles in the cortical layer and
large amount of plasma cells were revealed on histological examination.
What process in the lymphatic nodules represent these histological
changes?
A. Antigen stimulation
B. Acquired insufficiency of the lymphoid tissue
C. Innate insufficiency of the lymphoid tissue
D. Tumour transformation
E. Hypersensibility reaction

26. On autopsy of the man with alcohol abuse for a long time it was revealed:
dense, small-knobby, small size liver. Microscopically: small pseudo-
lobules, divided with thin layers of connective tissue with
lymphomacrophagial infiltrates; hepatocytes in the state of globular fatty
dystrophy. What is the most likely diagnosis?
A. Alcohol cirrhosis
B. Chronic active alcohol hepatitis
C. Chronic persistent alcohol hepatitis
D. Toxic liver dystrophy
E. Fatty hepatosis

27. An 8-year-old child was admitted to the infectious department with fever (up
to 38oC) and punctuate bright-red skin rash. The child was diagnosed as
having scarlet fever. Objectively: mucous membrane of pharynx is apparently
hyperaemic and edematic, the tonsils are enlarged and have dull yellowish-
grey foci with some black areas. What inflammation is the reason for the
pharynx alterations?
A. Purulent necrotic
B. Fibrinous
C. Haemorrhagic
D. Serous
E. Catarrhal

28. A 30-year-old patient with bacteriologically proved dysentery


developed the signs of paraproctitis. What is the stage of local changes
in this patient?
A. Ulceration stage
B. Fibrinous colitis
C. Follicular colitis
D. Catarrhal colitis
E. Healing of the ulcers stage

29. A patient who has been abusing tobacco smoking for a long time has got
cough accompanied by excretion of viscous mucus; weakness after minor
physical stress, pale skin. The patient has also lost 12,0 kg of body weight.
Endoscopic examination of biosy material his illness was diagnosed as
squamous cell carcinoma. Name a pathological process that preceded
formation of the tumour:
A. Metaplasia
B. Hypoplasia
C. Hyperplasia
D. Necrosis
E. Sclerosis

30. Diagnostic scraping was performed to the woman with dysfunctional


uterine bleeding. Multiple convoluted glands, ganglially dilated cavities
of some glands were revealed histologically in the scrape. Name the type
of general pathological process.
A. Glandulo-gangliac hyperplasia
B. Atrophy
C. Metaplasia
D. Displasia
E. Hypertrophic excrescence

31. Tuberculine was injected intracutaneously to the child for tuberculin test.
Marked hyperemia, tissue infiltration developed on the place of injection in
24 hours. What mechanism caused these modifications?
A. Cells cytotoxity
B. Reagin type cytotoxity
C. Antibody cytotoxity
D. Granuloma formation
E. Immunocomplex cytotoxity
32. The intraoperational biopsy of mammal gland has revealed the signs of
atypical tissue with disorder of parenchyma stroma proportion with
domination of the last, gland structures of the different size and shape, lined
with single-layer proliferative epithelium. What is the most appropriate
diagnosis?
A. Fibroadenoma
B. Papilloma
C. Noninfiltrative cancer
D. Infiltrative cancer
E. Mastitis

33. Arterial hypertension, hyperglycemia, glucosuria were observed clinically for


a long time in the patient with upper type of obesity. Death was due to the
cerebral haemorrhage. Basophilic hypophysis adenoma, hyperplasia of
adrenal gland cortex were revealed on pathomorphological examination.
What is the likely diagnosis?
A. Cushing disease
B. Diabetes mellitus
C. Acromegaly
D. Hypophysis nanism
E. Adiposogenitalis dystrophy

34. On autopsy it was revealed: large (1 -2 cm) brownish-red, easy crumbling


formations covering ulcerative defects on the external surface of the aortic
valve. What is the most likely diagnosis?
A. Polypus-ulcerative endocarditis
B. Recurrent warty endocarditis
C. Acute warty endocarditis
D. Fibroplastic endocarditis
E. Diffusive endocarditis

35. Purulent endometritis with fatal outcome was progressing in the woman
after abortion performed not at the hospital. On autopsy multiple lung
abscesses, subcapsule ulcers in the kidneys, spleen hyperplasia were
revealed. What form of sepsis developed in the patient?
A. Septopyemia
B. Septicemia
C. Chroniosepsis
D. Lung sepsis
E. Urosepsis

36. Autopsy of a 73-year-old man who had been suffering from the coronary
heart disease along with cardiac insufficiency for a long time revealed:
nutmeg liver, brown induration of lungs, cyanotic induration of kidneys and
spleen. What kind of circulation disorder was the cause of such effects?
A. General chronic venous congestion
B. Arterial hyperaemia
C. General acute venous congestion
D. Acute anaemia
E. Chronic anaemia

37. A 22 year old patient from the West Ukraine complains of laboured
nasal breathing. Morphological examination of biopsy material of nasal
mucous membrane revealed lymphoid, epithelioid, plasma cells as well
as Mikulicz's cells. What is the most probable diagnosis?
A. Rhinoscleroma
B. Glanders
C. Tuberculosis
D. Leprosy
E. Syphilis

38. Autopsy of a man who had been working as a miner for many years and
died from cardiopulmonary decompensation revealed that his lungs were
airless, sclerosed, their apexex had emphysematous changes, the lung
surface was greyish-black, the incised lung tissue was coal-black. What
disease caused death?
A. Anthracosis
B. Silicosis
C. Talcosis
D. Asbestosis
E. Aluminosis

39. Examination of coronary arteries revealed atherosclerotic calcific plaques


that close vessel lumen by 1/3. The muscle has multiple whitish layers of
connective tissue. What process was revealed in myocardium?
A. Diffuse cardiosclerosis
B. Tiger heart
C. Postinfarction cardiosclerosis
D. Myocarditis
E. Myocardium infarction

40. A 63 year old male patient who had been suffering from chronic diffuse
obstructive disease, pulmonary emphysema, for 15 years died from cardiac
insufficiency. Autopsy revealed nutmeg liver cirrhosis, cyanotic induration
of kidneys and spleen, ascites, edemata of lower limbs. These changes of
internal organs are typical for the following disease:
A. Chronic right-ventricular insufficiency
B. Acute right-ventricular insufficiency
C. Chronic left-ventricular insufficiency
D. Acute left-ventricular insufficiency
E. General cardiac insufficiency

41. Microscopical examination of an enlarged cervical lymph node revealed


blurring of its structure, absence of lymphoid follicles; all the microscopic
fields showed cells with roundish nuclei and thin limbus of basophil
cytoplasm. It is known from the clinical data that other groups of lymph
nodes are also enlarged as well as spleen and liver. What disease might be
suspected?
A. Lymphoid leukosis
B. Lymphogranulomatosis
C. Lymphosarcoma
D. Myeloid leukosis
E. Multiple myeloma

42. A worker of a cattle farm fell acutely ill and then died from the progressing
intoxication. Autopsy revealed enlarged, hyposthenic spleen of dark-cherry
colour when dissected; excessive pulp scraping. At the base and fornix of
brain pia maters are edematous, soaked with blood, dark-red ("scarlet
hat"). Microscopic examination revealed serous haemorrhagic
inflammation of brain tissues and tunics along with destruction of small
vessel walls. What is the most likely diagnosis?
A. Anthrax
B. Tularemia
C. Brucellosis
D. Plaque
E. Cholera

43. Histological examination of a skin tissue sampling revealed granulomas


consisting of macrophagal nodules with lymphocytes and plasmatic cells.
There are also some big macrophages with fatty vacuoles containing
causative agents of a disease packed up in form of spheres (Virchow's cells).
Granulation tissue is well vascularized. What disease is this granuloma
typical for?
A. Lepra
B. Tuberculosis
C. Syphilis
D. Rhinoscleroma
E. Glanders

44. A 40 year old man noticed a reddening and an edema of skin in the area of
his neck that later developed into a small abscess. The incised focus is dense,
yellowish-green. The pus contains white granules. Histological examination
revealed drusen of a fungus, plasmatic and xanthome cells, macrophages.
What type of mycosis is the most probable?
A. Actinomycosis
B. Aspergillosis
C. Candidosis
D. Sporotrichosis
E. Coccidioidomycosis

45. A physician examined a patient and found inguinal hernia. Through what
anatomic formation does it penetrate into the skin?
A. Hiatus saphenus
B. Anulus femoralis
C. Canalis adductorius
D. Lacuna musculorum
E. Anulus inguinalis superficialis

46. Autopsy of a man who died from burn disease revealed brain edema, liver
enlargement as well as enlargement of kidneys with wide light-grey cortical
layer and plethoric medullary area. Microscopic examination revealed
necrosis of tubules of main segments along with destruction of basal
membranes, intersticium edema with leukocytic infiltration and
haemorrhages. What is the most probable postmortem diagnosis?
A. Necrotic nephrosis
B. Tubulointerstitial nephritis
C. Pyelonephritis
D. Gouty kidney
E. Myeloma kidney

47. A 30 year old man had been suffering from acute respiratory disease and
died from cardiopulmonary decompensation. Autopsy revealed fibrinous-
haemorrhagic inflammation in the mucous membrane of larynx and
trachea, destructive panbronchitis, enlarged lungs that look black due to the
multiple abcesses, haemorrhages, necrosis. What is the most probable
postmortem diagnosis?
A. Influenza
B. Parainfluenza
C. Respiratory syncytial infection
D. Measles
E. Adenoviral infection

48. A man with a wound of his limb that had been suppurating for a long
time died from intioxication. Autopsy revealed extreme emaciation,
dehydration, brown atrophy of liver, myocardium, spleen and cross-
striated muscles as well as renal amyloidosis. What diagnosis
corresponds with the described picture?
A. Chroniosepsis
B. Septicopyemia
C. Septicemia
D. Chernogubov's syndrome
E. Brucellosis

49. 6 months after delivery a woman had uterine bleeding. Gynecological


examination revealed in the uterine cavity a dark-red tissue with multiple
cavities that resembled of "sponge". Microscopic examination of the tumour
revealed some atypic light epithelial Langhans cells and giant cells of
cyncytiotrophoblast in blood lacunas. What tumour is it?
A. Chorioepithelioma
B. Squamous cell nonkeratinous carcinoma
C. Adenocarcinoma
D. Fibromyoma
E. Vesicular mole

50. A patient with android-type obesity had been suffering from arterial
hypertension, hyperglycemia, glycosuria for a long time and died from
the cerebral haemorrhage. Pathologic examination revealed pituitary
basophil adenoma, adrenal cortex hyperplasia. What is the most likely
diagnosis?
A. Itsenko-Cushing's syndrome
B. Diabetes mellitus
C. Acromegalia
D. Pituitary nanism
E. Adiposogenital dystrophy

51. Mucous membrane of the right palatine tonsil has a painless ulcer with
smooth lacquer fundus and regular cartilagenous edges. Microscopically:
inflammatory infiltration that consists of lymphocytes, plasmocytes, a
small number of neutrophils and epithelioid cells; endovasculitis and
perivasculitis. What disease is it?
A. Syphilis
B. Actinomycosis
C. Tuberculosis
D. Pharyngeal diphtheria
E. Ulcerous necrotic Vincent's angina

52. Autopsy of a man with a malignant stomach tumour who had died from
cancer intoxication revealed in the posteroinferior lung fields some dense,
grayish-red irregular foci protruding above the section surface. Microscopic
examination revealed exudate containing a large amount of neutrophils in
the lumen and walls of small bronchi and alveoles. Such pulmonary
alterations indicate the following disease:
A. Acute purulent bronchopneumonia
B. Acute bronchitis
C. Croupous pneumonia
D. Intermittent pneumonia
E. Acute serous bronchopneumonia

53. Microscopical examination of a removed appendix revealed an edema,


diffuse neutrophilic infiltration of appendix wall along with necrosis and
defect of mucous membrane with affection of its muscle plate. What
appendicitis form was developed?
A. Ulcerophlegmonous
B. Phlegmonous
C. Gangrenous
D. Superficial
E. Apostematous
54. A 39 y.o. woman went through an operation in course of which surgeons
removed her uterine tube that was enlarged and a part of an ovary with a big
cyst. Histological examination of a tube wall revealed decidual cells, chorion
villi. What was the most probable diagnosis made after examination of the
uterine tube?
A. Tubal pregnancy
B. Placental polyp
C. Choriocarcinoma
D. Papyraceous fetus
E. Lithopedion

55. Autopsy of a 1,5-year-old child revealed haemorrhagic skin rash, moderate


hyperaemia and edema of nasopharyngeal mucous membrane, small
haemorrhages in the mucous membranes and internal organs; dramatic
dystrophic alterations in liver and myocardium; acute necrotic nephrosis;
massive haemorrhages in the adrenal glands. What disease are these
alterations the most typical for?
A. Meningococcal infection
B. Scarlet fever
C. Diphtheria
D. Measles
E. Epidemic typhus

56. 48 hours after performing tuberculin test (Mantoux test) to a child a 10 mm


papule appeared on the spot of tuberculin introduction. What
hypersensitivity mechanism underlies these changes?
A. Cellular cytotoxicity
B. Anaphylaxis
C. Antibody-dependent cytotoxicity
D. Immune complex cytotoxicity
E. Granulomatosis

57. Colonoscopy of a patient ill with dysentery revealed that mucous


membrane of his large intestine is hyperemic, edematic, its surface was
covered with grey-and-green coats. Name the morphological form of
dysenteric collitis:
A. Fibrinous
B. Catarrhal
C. Ulcerous
D. Purulent
E. Necrotic

58. A patient has been syffering from diarrhea for 5 day. On the fith day
colonoscopy revealed that membrane of rectum was inflamed, there were
greyish-green films closely adhering to the subjacent tissue. What is the most
probable diagnosis?
A. Dysentery
B. Typhoid fever
C. Nonspecific ulcerous colitis
D. Salmonellosis
E. Crohn's disease

59. Autopsy of a 48 y.o. man revealed a round formation 5 cm in diameter with


clear-cut outlines in the region of the 1st segment of his right lung. This
formation was encircled with a thin layer of connective tissue full of white
brittle masses. Make a diagnosis of the secondary tuberculosis form:
A. Tuberculoma
B. Caseous pneumonia
C. Acute cavernous tuberculosis
D. Acute focal tuberculosis
E. Fibrous cavernous tuberculosis

60. A man had worked in a coal mine for over 20 years. After his death autopsy
revealed that his lungs were dense, grayish-black and had large areas of
neogenic connective tissue containing a lot of microphages with black
pigment in the cytoplasm. What is the most likely diagnosis?
A. Anthracosis
B. Anthracosilicosis
C. Silicoanthracosis
D. Talcosis
E. Siderosis

61. Skin of a man who died from cardiac insufficiency has an eruption in form
of spots and specks. There are also bedsores in the area of sacrum and
spinous vertebral processes. Microscopical examination of CNS, skin,
adrenal glands revealed in the vessels of microcirculatory bed and in small
arteries destructive-proliferative endothrombovasculitis with Popov's
granulomas; interstitial myocarditis. What diagnosis corresponds with the
described picture?
A. Spotted fever
B. Q fever
C. Enteric fever
D. Nodular periarteritis
E. HIV

62. Autopsy of a 17 year old girl who died from pulmonary failure revealed a
small area of caseous necrosis in the inferior lobe of the right lung, and
occurences of caseous necrosis in the bronchopulmonary, bronchial and
bifurcational lymph nodes. What is the most probable postmortem
diagnosis?
A. Primary tuberculosis
B. Hematogenous progression of primary tuberculosis
C. Hematogenous tuberculosis with predominant lung affection
D. Tuberculoma
E. Caseous pneumonia under secondary tuberculosis

63. Autopsy of a man who died from the sepsis in his femoral bone revealed
phlegmonous inflammation that affected the marrow, haversian canals
and periosteum. Under the periosteum there are multiple abscesses,
adjoining soft tissues of thigh also have signs of phlegmonous
inflammation. What pathological process was described?
A. Acute hematogenous osteomyelitis
B. Osteoporosis
C. Chronic hematogenous osteomielitis
D. Osteopetrosis

64. An experimental animal was first sensibilized whereupon an antigen dose


was introduced subcutaneously. This injection resulted in the development
of a fibrinous inflammation with alteration of vessel walls, basal substance
and fibrous structures of connective tissue in form of mucoid and fibrinoid
swelling and necrosis. What immunological reaction took place?
A. Immediate hypersensitivity
B. Delayed-type hypersensitivity
C. Reaction of transplantation immunity
D. Normergic reaction
E. Granulomatosis

65. Examination of a 55 year old woman revealed under the skin of


submandibular area a movable slowly growing pasty formation with distinct
borders 1,0x0,7 cm large. Histological examination revealed lipocytes that
form segments of diffrent forms and sizes separated from each other by thin
layers of connective tissue with vessels. What is the most probable diagnosis?
A. Lipoma
B. Fibroma
C. Angioma
D. Liposarcoma
E. Fibrosarcoma

66. A 4 year old child complained of pain during deglutition, indisposition.


Objectively: palatine arches and tonsils are moderately edematic and
hyperemic, there are greyish-white films up to 1 mm thick closely adhering to
the subjacent tissues. What pathological process are these changes typical
for?
A. Inflammation
B. Dystrophy
C. Necrosis
D. Metaplasia
E. Organization

67. A 9 m.o. child has delayed dentition, it is also out of order. Upper jaw
configuration is horizontal ("high" palate); microscopically - irregular
mineralization of tooth enamel, wrinkled enamel prisms, some of them are
vacuolized. Predentin zone is extended; there are solitary denticles. What
disease is it?
A. Early rickets
B. Late rickets
C. Osteomalacia
D. Gout
E. Hypervitaminosis D

68. Microscopical renal examination of a 36 y.o. woman who died from renal
insufficiency revealed in the glomerules proliferation of capsule
nephrothelium as well as of podocytes and phagocytes accompanied by
formation of "crescents", capillary loop necrosis, fibrinous thrombs in their
lumens; sclerosis and hyalinosis of glomerules, atrophy of tubules and
fibrosis of renal stroma. What is the most probable diagnosis?
A. Subacute glomerulonephritis
B. Acute glomerulonephritis
C. Chronic glomerulonephritis
D. Focal segmentary sclerosis
E. Membranous nephropathy

69. A forensic medical expert examines the body of a 58 y.o. man who had been
consuming large amounts of alcochol for a long time and died at home.
Microscopicaly: the right lung is dense and enlarged, its incision revealed that
the tissue is greyish and homogenous, pleura is covered with greyish layers.
Microscopically - alveolar cavities contain fibrin, hemolyzed erythrocytes.
Make a diagnosis:
A. Croupous pneumonia
B. Focal pneumonia
C. Interstitial pneumonia
D. Primary pulmonary tuberculosis
E. Caseous pneumonia

70. Autopsy of a 50-year-old man revealed the following changes: his right lung
was moderately compact in all parts, the dissected tissue was found to be
airless, fine-grained, dryish. Visceral pleura had greyish-brown layers of
fibrin. What is the most likely diagnosis?
A. Croupous pneumonia
B. Tuberculosis
C. Bronchopneumonia
D. Interstitial pneumonia
E. Pneumofibrosis

71. Autopsy of a 56 y.o. man revealed in the right temporal part of brain a big
focus of softened grey matter that was semi-liquid and light grey. Arteries
of cerebral tela contain multiple whitish-yellow thickenings of intima that
abruptly narrow the lumen. What is your diagnosis?
A. Ischemic stroke
B. Brain abscess
C. Hemorrhage
D. Hemorrhagic infarction
E. Brain edema

72. A 22 y.o. woman has enlarged lymph nodes. Histologically: a lymph


node contains lymphocytes, histiocytes, reticular cells, small and big
Hodgkin's cells, multinucleated Sternberg cells, isolated foci of
caseous necrosis. What disease are these changes typical for?
A. Lymphogranulomatosis
B. Lymphosarcoma
C. Chronic leukosis
D. Acute leukosis
E. Lung cancer metastasis

73. Analysis of a punction biopsy material of liver revealed hepatocyte dystrophy


with necroses as well as sclerosis with disorder of beam and lobulous
structure, with formation of pseudolobules and regenerative nodes. What is
the most probable diagnosis:
A. Liver cirrhosis
B. Chronic hepatosis
C. Chronic hepatitis
D. Progressive massive liver necrosis
E. Acute hepatitis

74. Autopsy of a man, who had been suffering from the multiple
bronchiectasis for 5 years and died from chronic renal insufficiency,
revealed that kidneys were dense and enlarged, with thickened cortical
layer of white colour with greasy lustre. What renal disease might be
suspected?
A. Secondary amyloidosis
B. Glomerulonephritis
C. Chronic pyelonephritis
D. Necrotic nephrosis

75. Autopsy of a 49-year-old woman who died from chronic renal insufficiency,
revealed: kidneys were dense, reduced, multicoloured, with haemorrhagic
areas. Microscopic examination revealed some hematoxylin bodies in the
nuclei of the renal tubule epithelium; "wire-loop" thickening of the
glomerular capillary basement membrane; here and there in the capillaries
some hyaline thrombi and foci of fibrinoid necrosis were present. What is
the most likely diagnosis?
A. Systemic lupus erythematosus
B. Rheumatism
C. Arteriosclerotic pneumosclerosis
D. Amyloidosis
E. Atherosclerotic nephrosclerosis
76. Unpainfull formation without marked borders appeared in the soft tissues of
the thigh in the young man. On the tissue bioptate the formation lookes like
a meat of a fish, consisting of the immature fibroblast-like cells with
multiple mitosis, which grow through the muscles. What is the most likely
diagnosis?
A. Fibrosarcoma
B. Myosarcoma
C. Fibroma
D. Cancer
E. Myoma

77. A 45 y.o. patient consulted a doctor about plaque-shaped formation on his


neck. Histological examination of biopsy skin material revealed tumourous
cells of round and oval form with thin ring of basophilic cytoplasma that
resemble of cells of basal epidermal layer. What tumour is it?
A. Basalioma
B. Epidermal cancer
C. Hydradenoma
D. Trichoepithelioma
E. Syringoadenoma

78. A 63 y.o. man fell ill with acute tracheitis and bronchitis accompanied by
bronchial pneumonia. On the 10th day the patient died from
cardiopulmonary insufficiency. Autopsy revealed fibrinous hemorrhagic
laryngotracheobronchitis; lungs were enlarged, their incision revealed the
"coal-miner's" effect caused by interlacing of sections of bronchial
pneumonia, hemorrhages into the pulmonary parenchyma, acute abscesses
and atelectases. Internal organs have discirculatory and dystrophic
changes. What is the most probable diagnosis?
A. Influenza, severe form
B. Moderately severe influenza
C. Parainfluenza
D. Respiratory syncytial infection
E. Adenoviral infection

79. Autopsy of a man who died from influenza revealed that his heart was
slightly enlarged, pastous, myocardium was dull and had specks.
Microscopical examination of myocardium revealed signs of
parenchymatous adipose and hydropic dystrophy; stroma was edematic
with poor macrophagal and lymphocytic infiltration, vessels were
plethoric; perivascular analysis revealed petechial hemorrhages. What
type of myocarditis was developed in this case?
A. Serous diffuse
B. Interstitial proliferative
C. Serous focal
D. Purulent
E. Granulomatous
80.A boy is 7 y.o. Objectively: against the background of hyperemic skin
there is knobby bright-pink rash on his forehead, neck, at the bottom of
abdomen, in the popliteal spaces; nasolabial triangle is pale.
Examination of oropharyngeal surface revealed localized bright-red
hyperemia; tonsils are swollen, soft, lacunas contain pus, tongue is
crimson. Cervical lymph nodes are enlarged, dense and painful. What is
the most probable diagnosis?
A. Scarlet fever
B. Rubella
C. Whooping cough
D. Diphtheria
E. Infectious mononucleosis

81. Gynecological examination of the uterine cervix in a 30-year-old woman


revealed some bright-red lustrous spots that easily bleed when touched.
Biopsy showed that a part of the uterine cervix was covered with cylindrical
epithelium with papillary outgrowths; in the depth of tissue the growth of
glands was present. What pathology of the uterine cervix was revealed?
A. Pseudoerosion
B. True erosion
C. Endocervicitis
D. Glandular hyperplasia
E. Leukoplakia

82. A stillborn child was found to have thickened skin resembling of the tortoise
shell, underdeveloped auricles. Histological examination of skin revealed
hyperkeratosis, atrophy of the granular epidermis layer; inflammatory
changes were not present. What is the most likely diagnosis?
A. Ichthyosis
B. Leukoplakia
C. Xerodermia
D. Erythroplakia
E. Dermatomyositis

83. Acute renal impairment caused death of a bleeding patient. Autopsy


revealed enlarged kidneys with a broad pale pink cortical layer expressively
demarcated from dark red renal pyramids. Macroscopic examination
revealed lack of epithelial nuclei of convoluted tubules, tubulorrhexis,
phlebostasis. The cell nuclei of choroid glomus and straight tubules were
present. What pathology is it?
A. Necronephrosis
B. Infarction
C. Glomerulonephritis
D. Pyelonephritis
E. Nephrosis
84. A pathology-histology laboratory received a vermiform appendix up to 2,0 cm
thick. Its serous membrane was pale, thick and covered with yellowish-green
films. The wall was flaccid, of grayish-red colour. The appendix lumen was
dilated and filled with yellowish-green substance. Histological examination
revealed that the appendix wall was infiltrated with neutrophils. Specify the
appendix disease:
A. Acute phlegmonous appendicitis
B. Acute gangrenous appendicitis
C. Acute superficial appendicitis
D. Acute simple appendicitis
E. Chronic appendicitis

85. A 46 year old patient who had been suffering from tuberculosis for 6 years
died from massive pulmonary haemorrhage. Autopsy revealed different-sixed
foci of sclerosis and caseous necrosis in lungs, in the upper part of the right
lung there was a cavity 5 cm in diameter with dense grey walls, the cavity
contained liquid blood and blood clots. What type of tuberculosis is it?
A. Fibrocavernous
B. Acute cavernous
C. Infiltrative
D. Fibrous focal
E. Acute focal

86. A patient died from cardiopulmonary decompensation. Histological


examination revealed diffused pulmonary lesion together with interstitial
edema, infiltration of tissue by limphocytes, macrophages, plasmocytes;
pulmonary fibrosis, panacinar emphysema. What disease corresponds with
the described picture?
A. Fibrosing alveolitis
B. Chronic bronchitis
C. Bronchopneumonia
D. Pulmonary atelectasis
E. Bronchial asthma

87. A 50 year old patient has been taking treatment thrice for the last 6 months
because of fractures caused by domestic accidents. Microscopical
examination of bony tissue revealed foci of lacunar resolution, giant-cell
granulomas in the tumour-like formations, cysts. Bony tissue was
substituted by fibrous connective tissue. Examination revealed also
adenoma of parathyroid gland and hypercalcemia. What is the most
probable diagnosis?
A. Parathyroid osteodystrophy
B. Myelomatosis
C. Osteomyelitis
D. Osteopetrosis
E. Paget's disease

88. 2 days after labour a woman developed shock along with DIC syndrome
that caused her death. Autopsy revealed purulent endomyometritis,
regional purulent lymphangitis, lymphadenitis and purulent
thrombophlebitis. There were also dystrophic alterations and interstitial
inflammation of parenchymal organs. What is the most likely diagnosis?
A. Septicemia
B. Syphilis
C. Tuberculosis of genital organs
D. Chorioadenoma destruens
E. Hydatid mole

89. 2 hours after a skeletal extension was performed to a 27 year old patient with
multiple traumas (closed injury of chest, closed fracture of right thigh) his
condition abruptly became worse and the patient died from acute
cardiopulmonary decompensation. Histological examination of pulmonary
and cerebral vessels stained with Sudan III revealed orange drops occluding
the vessel lumen. What complication of polytrauma was developed?
A. Fat embolism
B. Gaseous embolism
C. Microbal embolism
D. Thromboembolism
E. Air embolism

90. A 50 year old patient underwent resection of tumour of large intestine wall.
Microscopically it presents itself as fascicles of divergent collagen fibers of
different thickness and form and some monomorphous fusiform cells that
are irregularly distributed among the fibers. Cellular atypia is not evident.
What tumour is it?
A. Hard fibroma
B. Fibromyoma
C. Soft fibroma
D. Desmoma
E. Fibrosarcoma

91. Autopsy of a 5 year old child revealed in the area of vermis of cerebellum a
soft greyish-pink node 2 cm in diameter with areas of haemorrhage.
Histologically this tumour consisted of atypical monomorphous small
roundish cells with big polymorphous nuclei. What tumour is it?
A. Medulloblastoma
B. Meningioma
C. Glioblastoma
D. Astrocytoma
E. Oligodendroglioma

92. In course of severe respiratory viral infection there appeared clinical


signs of progressing cardiac insufficiency that caused death of a patient
in the 2nd week of disease. Autopsy revealed that the heart was sluggish,
with significant cavity dilatation. Histological examination of
myocardium revealed plephora of microvessels and diffuse infiltration of
stroma by lymphocytes and histiocytes. What disease corresponds with
the described picture?
A. Myocarditis
B. Stenocardia
C. Acute coronary insufficiency
D. Myocardium infarction
E. Cardiomyopathy

93. A patient with high-titer antinuclear antibodies died from progressing


renal impairment. Autopsy revealed mesangioproliferative
glomerulonephritis and abacterial polypous endocarditis. There was
periarterial bulbar sclerosis in spleen and productive proliferative
vasculitis in skin. What is the most likely diagnosis?
A. Systemic lupus erythematosus
B. Nephrotic syndrome
C. Rheumatism
D. Dermatomyositis
E. Periarteritis nodosa

94. A 38 year old patient with full-blown jaundice, small cutaneous


hemorrhages, general weakness and loss of appetite underwent puncture
biopsy of liver. Histological examination revealed disseminated dystrophy,
hepatocyte necrosis, Councilman's bodies. Lobule periphery has signs of
significant infiltration by lymphocytes, there are also individual
multinuclear hepatocytes. What is the most probable diagnosis?
A. Acute viral hepatitis
B. Acute alcoholic hepatitis
C. Miliary hepatic cirrhosis
D. Toxic degeneration of liver
E. Chronic hepatitis

95. A 20 year old patient died from intoxication 8 days after artificial illegal
abortion performed in her 14-15th week of pregnancy. Autopsy of the corpse
revealed yellowish colour of eye sclera and of skin, necrotic suppurative
endometritis, multiple pulmonary abscesses, spleen hyperplasia with a big
number of neutrophils in its sinuses. What complication after abortion was
developed?
A. Septicopyemia
B. Septicemia
C. Hemorrhagic shock
D. Chroniosepsis
E. Viral hepatitis type A

96. A section of the left lung was found to have an area of dense red tissue. The
area was cone-shaped, stood out distinctly from the healthy tissue, with its
base directed to the pleura. The dissected tissue was granular, dark-red.
What is the most likely diagnosis?
A. Haemorrhagic infarction
B. Lung abscess
C. Lung gangrene
D. Primary tuberculous affection
E. Croupous pneumonia

97. A patient has a cluster of matted together dense lymph nodes on his neck.
Histological examination of a removed lymph node revealed proliferation
of reticular cells, presense of Reed-Sternberg cells. What disease is
meant?
A. Lymphogranulomatosis
B. Lymphoblastic leukosis
C. Myeloblastic leukosis
D. Myelocytic leukosis
E. Lymphocytic leukosis

98. A patient had been suffering from profuse diarrhea and vomiting for 2 days.
He died from acute dehydration. Autopsy revealed that the intestinal wall was
edematic and hyperemic, with multiple haemorrhages in the mucous
membrane. Intestine lumen contains whitish fluid resembling of rice water.
What disease caused death?
A. Cholera
B. Dysentery
C. Salmonellosis
D. Typhoid fever
E. Enterocolitis

99. Examination of a 66 year old patient revealed a lytic tumour in the locus of
pathological rib fracture. Histologically this tumour consists of atypical
plasmoblasts. Further examination revealed osteoporosis in the bones of
vertebral column and pelvis. These changes are typical for:
A. Myelomatosis
B. Tuberculous osteomyelitis
C. Ewing's osteosarcoma
D. Neuroblastoma
E. Metastatic lung cancer

100. A patient died from acute cardiac insufficiency, among clinical


presentations there was gastrointestinal haemorrhage. Examination of
mucous membrane of sromach revealed some defects reaching
myenteron; their edges and bottom were mostly even and loose, some
of them contained dark-red blood. What pathological process was
revealed?
A. Acute ulcers
B. Chronic ulcers
C. Erosions
D. Thrombosis
E. Inflammation
101. A 33 year old man died from uraemia. Autopsy revealed enlarged
kidneys weighing 500,0 each and consisting of multiple cavities 0,5 -2 cm
in diameter. The cavities were full of light-yellow transparent liquid.
Renal pelvis and ureters had no pecularities. What renal disease caused
uraemia?
A. Bilateral polycystic renal disease
B. Chronic pyelonephritis
C. Renal tumour
D. Renl tuberculosis
E. Rapidly progressing glomerulonephritis

102. A patient ill with tuberculosis died from progressing cardiopulmonary


decompensation. Autopsy in the area of the right lung apex revealed a cavity
5 cm in diameter communicating with lumen of a segmental bronchus. On
the inside cavity walls are covered with caseous masses with epithelioid and
Langhans cells beneath them. What morphological form of tuberculosis is
it?
A. Acute cavernous tuberculosis
B. Tuberculoma
C. Caseous pneumonia
D. Infiltrative tuberculosis
E. Acute focal tuberculosis

103. A 30 year old woman has applied a lipstick with a fluorescent


substance for a long time. Then she got a limited erythema and slight peeling
on her lip border, later there appeared transversal striae and cracks. Special
methods of microscopic examination of the affected area helped to reveal
sensibilized lymphocytes and macrophages in the connective tissue;
cytolysis. What type of immunological hypersensitivity was developed?
A. IV type (cellular cytotoxicity)
B. I type (reaginic)
C. II type (antibody cytotoxicity)
D. III type (immune complex cytotoxicity)
E. Granulomatosis

104. Examination of a young woman revealed a node-like, soft and elastic


homogenous tumour of pinkish-white colour along the acoustic nerve. The
tumour contains cell bundles with oval nuclei. Cellular fibrous bundles form
rhythmic structures made up by parallel rows of regularly oriented cells
arranged in form of a palisade with cell-free homogenous zone (Verocay
bodies) between them. What tumour is it?
A. Neurinoma
B. Malignant neurinoma
C. Ganglioneurinoma
D. Neuroblastoma
E. Ganglioneuroblastoma
105. A 23 year old man has perforation of hard palate. In the area of this
perforation there was a compact well-defined formation. Microscopic
examination of the resected formation revealed a large focus of caseous
necrosis surrounded by granulation tissue with endovasculitis, cellular
infiltration composed of lymphocytes, epithelioid cells (mainly
plasmocytes). What is the most probable diagnosis?
A. Syphilis
B. Tuberculosis
C. Scleroma
D. Sarcoma
E. Leprosy

106. A 50 year old man who was referred to the hospital for treatment of
cervical lymphadenitis underwent test for induvidual sensitivity to penicillin.
30 seconds after he went hot all over, AP dropped down to 0 mm Hg that led
to cardiac arrest. Resuscitation was unsuccessful. Autopsy results: acute
venous plethora of internal organs; histological examination of skin (from the
site of injection) revealed degranulation of mast cells (tissue basophils).
Degranulation was also revealed in myocardium and lungs. What type of
hypersensitivity reaction is it?
A. Anaphylactic
B. Delayed-type hypersensitivity
C. Complement-mediated cytotoxic
D. Immunocomplex-mediated

107. A 2 year old child had acute respiratory viral infection and died from
cardiopulmonary decompensation. Autopsy revealed that his right lung was
hyperemic; in the 2nd, 6th and 10 th segments and on the incision there were
airless yellowish foci of irregular form, from several mm up to 1 cm large.
Microscopical examination revealed exudate consisting mainly of neutrophils
in the given areas of pulmonary tissue in the alveoles, bronchioles and
bronchial tubes. What is the most probable diagnosis?
A. Focal pneumonia
B. Interstitial pneumonia
C. Croupous pneumonia
D. Acute bronchitis
E. Pulmonary abscess

108. The upper lobe of the right lung is enlarged, grey and airless, the
inscision surface is dripping with turbid liquid, the pleura has many
fibrinogenous films; microscopical examination of alveoles revealed exudate
containing neutrophils, desquamated alveolocytes and fibrin fibers. The
bronchus wall is intact. What is the most probable diagnosis?
A. Croupous pneumonia
B. Interstitial pneumonia
C. Pulmonary abscess
D. Focal pneumonia
E. Influenzal pneumonia
109. A 28 year old patient had high arterial pressure, hematuria and facial
edemata. In spite of treatment renal insufficiency was progressing. 6 months
later the patient died from uremia. Microscopic examination of his kidneys
and their glomerules revealed proliferation of capsule nephrothelium and of
podocytes with "demilune" formation, sclerosis and hyalinosis of
glomerules. What disease corresponds with the described picture?
A. Subacute glomerulonephritis
B. Acute pyelonephritis
C. Nephrotic syndrome
D. Chronic glomerulonephritis
E. Acute glomerulonephritis

110. Autopsy of a man ill with severe hypothyroidism revealed that


connective tissue, organ stroma, adipose and cartilaginous tissues were
swollen, semitransparent, mucus-like. Microscopic examination of
tissues revealed stellate cells having processes with mucus between
them. What type of dystrophy is it?
A. Stromal-vascular carbohydrate
B. Stromal-vascular adipose
C. Stromal-vascular proteinaceous
D. Parenchymatous proteinaceous
E. Parenchymatous adipose

111. Examination of the anterior abdominal wall of a pregnant woman


revealed a tumour-like formation that arose on the spot of a tumour that
was removed two years ago. The neoplasm was well-defined, dense, 2х1 cm
large. Histological examination revealed that the tumour was composed of
differentiated connective tissue with prevailing collagen fibres. What
tumour might be suspected?
A. Desmoid
B. Lipoma
C. Fibrosarcoma
D. Hibernoma
E. Leiomyoma
Krok 1 – 2015 Path Physiology Base
1. Patient with hypochromic anemia has splitting hair and loss of hair,
increased nail brittling and taste alteration. What is the mechanism of the
development of these symptoms?
A. Deficiency of iron-containing enzymes
B. Deficiency of vitamin В12
C. Decreased production of parathyrin
D. Deficiency of vitamin А
E. Decreased production of thyroid hormones

2. A 27- year-old woman has dropped penicillin containing eye drops. In few
minutes there appeared feeling of itching, burning of the skin, lips and
eyelids edema, whistling cough, decreasing of BP. What antibodies take
part in the development of this allergic reaction?
A. IgE and IgG
B. B IgM and IgG
C. IgA and IgM
D. IgM and IgD
E. IgG and IgD

3. A patient underwent a surgery for excision of a cyst on pancreas. After this


he developed haemorrhagic syndrome with apparent disorder of blood
coagulation. Development of this complication can be explained by:
A. Activation of fibrinolytic system
B. Insufficient fibrin production
C. Reduced number of thrombocytes
D. Activation of anticoagulation system
E. Activation of Christmas factor

4. A 43-year-old patient has thrombopenia, reduction of fibrinogen, products of


degradation of fibrin presented in the blood, petechial haemorrhage along
with septic shock. What is the most likely cause of the changes?
A. DIC-syndrome
B. Autoimmune thrombocytopenia
C. Haemorrhagic diathesis
D. Disorder of thrombocytes production
E. Exogenous intoxication

5. Pyruvate concentration in the patient’s urine has increased 10 times from


normal amount. What vitamin deficiency can be the reason of this
change:
A. Vitamin B1
B. Vitamin C
C. Vitamin A
D. Vitamin E
E. Vitamin B6
6. Person has stable HR, not more than 40 bpm. What is the pacemaker of the
heart rhythm in this person?
A. Atrioventricular node
B. Sinoatrial node
C. His' bundle
D. Branches of His' bundle
E. Purkinye' fibers

7. A 32-year-old patient was admitted to the hospital with gross bloodloss due
to auto accident trauma. Ps - 110Bpm, RR- 22 pm, BP- 100/60mm Hg. What
changes in the blood will occur in an hour after the bloodloss?
A. Hypovolemia
B. Erythropenia
C. Hypochromia of erythrocytes
D. Leukopenia
E. Hypoproteinemia

8. A 70-year-old patient suffers from atherosclerosis complicated by the lower


limb thrombosis that has caused gangrene on his left toes. What is the most
likely cause of the thrombosis origin?
A. Thrombocyte adhesion
B. Prothrombinase activation
C. Transformation of prothrombin into thrombin
D. Transformation of fibrinogen into fibrin
E. Impaired heparin synthesis

9. ECG of a 44-year-old patient shows signs of hypertrophy of both ventricles


and the right atrium. The patient was diagnosed with the tricuspid valve
insufficiency. What pathogenetic variant of cardiac dysfunction is usually
observed in case of such insufficiency?
A. Heart overload by volume
B. Heart overload by resistance
C. Primary myocardial insufficiency
D. Coronary insufficiency
E. Cardiac tamponade

10. Shock and signs of acute renal failure (ARF) developed in the patient due
to permanent injury. What is the leading cause of development of ARF in
the case?
A. Decreased arterial pressure
B. Urine excretion violation
C. Increased pressure in the nephron capsule
D. Increased pressure in the renal arteries
E. Decreased oncotic BP
11. Substitution of the glutamic acid on valine was revealed while examining
initial molecular structure. For what inherited pathology is this typical?
A. Sickle-cell anemia
B. Thalassemia
C. Minkowsky-Shauffard disease
D. Favism
E. Hemoglobinosis

12. Inflamation is characterised by increasing penetration of vessels of


microcirculation stream, increasing of their fluid dynamic blood pressure.
Increasing of the osmotic concentration and dispersity of protein structures
present in the intercellular fluid. What kind of edema will appear in this case?
A. Mixed
B. Hydrodynamic
C. Colloid-osmotic
D. Lymphogenic
E. Membranogenic

13. Disorder of the airways passage in small and middle bronchi was revealed
in the patient. What disorder of the acid-base equilibrium can be detected
in the blood?
A. Respiratory acidosis
B. Metabolic acidosis
C. Respiratory alkalosis
D. Metabolic alkalosis

14. A 62-year-old patient was admitted to the neurological department due to


cerebral haemorrage. Condition is grave. There is observed progression of
deepness and frequency of breath that turnes into reduction to apnoea,and
the cycle repeates. What respiration type has developed in the patient?
A. Cheyne-Stockes respiration
B. Kussmaul respiration
C. Biot's respiration
D. Gasping respiration
E. Apneustic respiration

15. In a 45-year-old patient on ECG it was revealed: sinus rhythm, the number of
auricular complexesexceeds number of ventricular complexes; progressing
extension of the P-Q interval from complex to complex; fallout of some
ventricular complexes; Р waves and QRST complexes are without changes.
Name the type of heart rhythm disfunction.
A. Atrioventricular block of the II degree
B. Synoauricular block
C. Atrioventricular blockade of the I degree
D. Intraatrial block
E. Complete atrioventricular block
16. A 57-year-old patient was admitted to the gastroenterological department
with suspicion on Zollinger-Ellison syndrom because of rapid increase of
gastrin level in the blood serum. What disorder of the secretory function of
the stomach is the most likely?
A. Hyperacid hypersecretion
B. Hyperacid hyposecretion
C. Achylia
D. Hypoacid hyposecretion
E. Hypoacid hypersecretion

17. A healthy woman has three sons affected by color blindness who were
born after her two marriages. Children both of her husbands are healthy.
What is the most possible pattern of inheritance of this disease?
A. X-linked recessive
B. Y-linked
C. Autosomal recessive
D. Autosomal dominant
E. X-linked dominant

18. Upper neck node of sympathetic trunk was removed from the
rabbit on experiment. Reddening and increased temperature of the
skin of head is observed. What form of peripheral circulation of the
blood developed in the rabbit?
A. Neuroparalytic arterial hyperemia
B. Neurotonic arterial hyperemia
C. Metabolic arterial hyperemia
D. Venous hyperemia
E. Stasis

19. A couple came for medical genetic counseling. The man has hemophilia,
the woman is healthy and there were no cases of hemophilia in her family.
What is the risk of having a sick child in this family?
A. 0
B. 100%
C. 75%
D. 50%
E. 25%

20. Oval and round organelles with double wall are seen at the electron
micrograph. The outer membrane is smooth, the inner membrane folded into
cristae contain enzyme ATPase synthetase. These are:
A. Mitochondria
B. Golgi complex
C. Lysosomes
D. Centrioles
E. Ribosomes
21. A tissue sample of benign tumor was studied under the electron
microscope. A lot of small (15-20 nm) spherical bodies, consisting of 2
unequal subunits were detected. These are:
A. Ribosomes
B. Golgi complex
C. Smooth endoplasmic reticulum
D. Microtubules
E. Mitochondria

22. A woman who was sick with rubella during the pregnancy gave birth to a
deaf child with hare lip and cleft palate. This congenital defect is an example
of:
A. Phenocopy
B. Edward’s syndrome
C. Genocopy
D. Patau’s syndrome
E. Down’s syndrome

23. A woman who was infected with toxoplasmosis during the pregnancy
has a child with multiple congenital defects.This is a result of:
A. Teratogenesis
B. Cancerogenesis
C. Biological mutogenesis
D. Chemical mutogenesis
E. Recombination

24. A patient with tissue trauma was taken a blood sample for the
determination of blood clotting parameters. Specify the right sequence of
extrinsic pathway activation.
A. III – VIIa – Xa
B. III – IV – Xa
C. IV – VIII: TF – Xa
D. IV – VIIa – Xa
E. III – VIII: TF – Xa

25. A patient with the symptoms of acute alcoholic poisoning was brought to
the hospital. What carbohydrates metabolism changes are typical for this
condition?
A. The gluconeogenesis velocity in liver is decreased
B. The gluconeogenesis is increased in liver
C. The breakage of glycogen is increased in liver
D. The anaerobic glucose metabolism predominates in muscles
E. The anaerobic breakage of glucose is increased in muscles

26. In result of the damage of one of the Atomic Power Plant reacttor the run-out
of radioelements happened. People in the increased radiation zone were
radiated with approximately 250-300 r. They were immediately hospitalized.
What changes in the blood count would be typical?
A. Lymphopenia
B. Leukopenia
C. Anemia
D. Thrombopenia
E. Neutropenia

27. Dystrophic changes of the heart muscle are accompanied with cardiac cavity
enlargement, decrease of the strength of heart contraction, increased
amount of blood, which remains in the heart during systolic phase,
overfilled veins. For what state of heart is it characteristic?
A. Myogenic dilatation
B. Tonogenic dilatation
C. Emergency stage of hyperfunction and hypertrophy
D. Cardiosclerosis
E. Tamponage of the heart

28. Transmural myocardial infarction in the patient was complicated with


progressive acute left ventricle insufficiency. What is the most typical for
this state?
A. Edema of the lungs
B. Edema of the extremities
C. Cyanosis
D. Ascites
E. Arterial hypertension

29. Arterial hypertention is caused by the stenosis of the renal arteries in the
patient. Activation of what system is the main link in the pathogenesys of this
form of hypertension?
A. Renin-angiotensin
B. Sympathoadrenal
C. Parasympathetic
D. Kallikrein-kinin
E. Hypothalamic-pituitary

30. A 12-year-old boy often suffers from virus and bacterial infections and
eczematous skin lesions. Enlargement of T-lymphocytes and IgM with
normal IgA and IgG was revealed on examination. What type of immune
system pathology is presented in the patient?
A. Composite immunedefficiency
B. Hypoplasia of thymus
C. Bruton's hypogammaglobulinemia
D. Turner's syndrome
E. Hereditary immundeficiency of the complement system

31. Patient with diabetes didn't get insulin injection in time that caused
hyperglycemic coma (glucose in the blood 50mmol/L). What mechanism
is prevalent in the development of the coma?
A. Hyperosmia
B. Hypokaliemia
C. Hypoxia
D. Hyponatremia
E. Acidosis

32. Necrosis focus appeared in the area of hyperemia and skin edema in few
hours after burn. What mechanism strengthens destructive events in the
inflammation area?
A. Secondary alteration
B. Primary alteration
C. Emigration of lymphocytes
D. Diapedesis of erythrocytes
E. Proliferation of fibroblasts

33. Processes of repolarisation are disturbed in ventricular myocardium in


examined person. It will cause amplitude abnormalities of configuration
and duration of the wave:
A. Т
B. Q
C. R
D. S
E. P

34. On simulation of inflammation of the lower extremity the animal exrerienced


raise of the temperature, increase of amount of antibodies and leucocytes in
the blood. What substances caused this general reaction of the organism on
inflammation?
A. Interleukin
B. Glucocorticoid
C. Mineralcorticoid
D. Leucotriens
E. Somatomedins

35. A 68-year-old woman can't move by the upper and lower right extremities
due to insult. Muscle tone of these extremities and reflexes are increased.
There are pathological reflexes. What form of the paralysis is it?
A. Hemiplegia
B. Paraplegia
C. Tetraplegia
D. Monoplegia
E. Dissociation

36. After a serious psycho-emotional stress a 45-year-old patient suddenly felt


constricting heart pain irradiating to the left arm, neck and left scapula. His
face turned pale, the cold sweat stood out on it. The pain attack was stopped
with nitroglycerine. What process has developed in this patient?
A. Stenocardia
B. Myocardial infarction
C. Stroke
D. Psychogenic shock
E. Stomach ulcer perforation

37. Periodic renal colics attackes are observed in the woman with primery
hyperparathyroidizm. Ultrasonic examination revealed small stones in the
kidneys. What is the cause of the formation of the stones?
A. Hypercalcemia
B. Hyperphosphatemia
C. Hypercholesterinemia
D. Hyperuricemia
E. Hyperkalemia

38. While having the dinner the child choked and aspirated the food. Meavy
cough has started, skin and mucose are cyanotic, rapid pulse, rear
breathing, expiration is prolonged. What disorder of the external
breathing developed in the child?
A. Stage of expiratory dyspnea on asphyxia
B. Stage of inspiratory dyspnea on asphyxia
C. Stenotic breathing
D. Alternating breathing
E. Biot's breathing

39. Chronic glomerulonephritis was diagnosed in a 34 -year-old patient 3 years


ago. Edema has developed in the last 6 monthes. What caused it?
A. Proteinuria
B. Hyperproduction of vasopressin
C. Disorder of albuminous kidneys function
D. Hyperosmolarity of plasma
E. Hyperaldosteronism

40. While playing volleyball a sportsman made a jump and landed on the
outside edge of his foot. He felt acute pain in the talocrural joint, active
movements are limited, passive movements are unlimited but painful. A
bit later there appeared a swelling in the area of external ankle, the skin
became red and warm. What type of peripheral circulation disturbance
is the case?
A. Arterial hyperemia
B. Stasis
C. Embolism
D. Venous hyperemia
E. Thrombosis

41. Having helped to eliminate consequences of a failure at a nuclear power


plant, a worker got an irradiation doze of 500 roentgen. He complains of
headache, nausea, dizziness. What changes in leukocytes quantity can be
expected 10 hours after irradiation?
A. Neutrophilic leukocytosis
B. Lymphocytosis
C. Leukopenia
D. Agranulocytosis
E. Leukemia

42. A 12 y.o. boy who suffers from bronchial asthma has an acute attack of
asthma: evident expiratory dyspnea, skin pallor. What type of alveolar
ventilation disturbance is it?
A. Obstructive
B. Restrictive
C. Throracodiaphragmatic
D. Central
E. Neuromuscular

43. A 46-year-old patient suffering from the diffuse toxic goiter underwent
resection of the thyroid gland. After the surgery the patient presents with
appetite loss, dyspepsia, increased neuromuscular excitement. The body
weight remained unchanged. Body temperature is normal. Which of the
following has caused such a condition in this patient?
A. Reduced production of parathormone
B. Increased production of thyroxin
C. Increased production of calcitonin
D. Increased production of thyroliberin
E. Reduced production of thyroxin

44. To prevent the transplant rejection after organ transplantation it is


required to administer hormonotherapy for the purpose of
immunosuppression. What hormones are used for this purpose?
A. Glucocorticoids
B. Mineralocorticoids
C. Sexual hormones
D. Catecholamines
E. Thyroid

45. A patient caught a cold after which there appeared facial expression
disorder. He cannot close his eyes, raise his eyebrows, bare his teeth.
What nerve is damaged?
A. Facial
B. Vagus
C. Trigeminus
D. Glossopharyngeal
E. Infraorbital
46. A group of mountain climbers went through the blood analysis at the
height of 3000 m. It revealed decrease of HCO3 to 15 micromole/l
(standard is 22-26 micromole/l). What is the mechanism of HCO3
decrease?
A. Hyperventilation
B. Intensification of acidogenesis
C. Hypoventilation
D. Decrease of ammoniogenesis
E. Decrease of bicarbonate reabsorption in kidneys

47. ECG of a patient shows such alterations: P-wave is normal, P-Q-


interval is short, ventricular QRST complex is wide, R-wave is double-
peak or two-phase. What form of arrhythmia is it?
A. WPW syndrome (Wolff-Parkinson-White)
B. Frederick's syndrome (atrial flutter)
C. Atrioventricular block
D. Ventricular fibrillation
E. Ciliary arrhythmia

48. A patient who suffers from pneumonia has high body temperature. What
biologically active substance plays the leading part in origin of this
phenomenon?
A. Interleukin-I
B. Histamine
C. Bradykinin
D. Serotonin
E. Leukotrienes

49. A 34 year old woman was diagnosed with hereditary microspherocytic


hemolytic anemia (Minkowsky-Shauffard disease). What mechanism
caused haemolysis of erythrocytes?
A. Membranopathy
B. Enzymopathy
C. Hemoglobinopathy
D. Autoimmune disorder
E. Bone marrow hypoploasia

50. From the group of children who were eating sweet sappy watermelon two
kids developed the signs of poisoning: rapid weakness, dizziness, headache,
vomiting, edema, tachycardia, cyanosis of mouth, ears, tips of the fingers
cyanosis. High concentration of nitrates was detected. What is the leading
mechanism of the pathogenesis of the poisoning in the two children?
A. Insufficiency of met-Hb-reductase
B. Insufficiency of superoxiddismutase
C. Block cytochrome oxidase
D. Insufficiency glutathione pyroxidase
E. Insufficiency of catalase
51. 2 years ago a patient underwent resection of pyloric part of stomach.
He complains of weakness, periodical dark shadows beneath his eyes,
dyspnea. In blood: Hb - 70 g/l, erythrocytes - 3,0*1012/l, colour index -
0,7. What changes of erythrocytes in blood smears are the most typical
for this condition?
A. Microcytes
B. Megalocytes
C. Schizocytes
D. Ovalocytes
E. Macrocytes

52. During a prophylactic medical examination a 7-year-old boy was diagnosed


with daltonism. His parents are healthy and have normal colour vision, but
his grandfather on his mother’s side has the same abnormality. What is the
type of the abnormality inheritance?
A. Recessive, sex-linked
B. Dominant, sex-linked
C. Semidominance
D. Autosomal recessive
E. Autosomal dominant

53. A 27 y.o. patient put eye drops that contain penicillin. After a few minutes she
felt itching and burning of her body, there appeared lip and eye-lid edemata;
arterial pressure began to drop. What immunoglobulins took part in the
development of this allergic reaction?
A. lgE and lgG
B. IgM and IgG
C. IgA and IgM
D. IgM and IgD
E. IgG and IgD

54. Inflammation of a patient's eye was accompanied by accumulation of turbid


liquid with high protein at the bottom of anterior chamber that was called
hypopyon. What process underlies the changes under observation?
A. Disturbance of microcirculation
B. Primary alteration
C. Secondary alteration
D. Proliferation

55. A 48 y.o. patient was admitted to the hospital with complaints about
weakness, irritability, sleep disturbance. Objectively: skin and scleras are
yellow. In blood: conjugated bilirubin, cholalemia. Feces are acholic. Urine
is of dark colour (bilirubin). What jaundice is it?
A. Mechanic
B. Hemolytic
C. Parenchymatous
D. Gilbert's syndrome
E. Crigler-Najjar syndrome

56. A patient who suffers from severe disorder of water-salt metabolism


experienced cardiac arrest in diastole. What is the most probable
mechanism of cardiac arrest in diastole?
A. Hyperkaliemia
B. Hypernatremia
C. Organism dehydratation
D. Hypokaliemia
E. Hyponatremia

57. A patient who suffers from heart failure has enlarged liver, edemata of
lower extremities, ascites. What is the leading mechanism in the
development of this edema?
A. Hydrodynamic
B. Colloid osmotic
C. Lymphogenous
D. Membranogenic

58. Two weeks after lacunar tonsillitis a 20-year-old man started complaining
about general weakness, lower eyelid edemata. After examination the
patient was diagnosed with acute glomerulonephritis. What are the most
likely pathological changes in the urine formula?
A. Proteinuria
B. Cylindruria
C. Presence of fresh erythrocytes
D. Pyuria
E. Natriuria

59. A 32 y.o. man is tall, he has gynecomastia, adult woman pattern of hair
distribution, high voice, mental deficiency, sterility. Provisional diagnosis is
Klinefelter's syndrome. In order to specify diagnosis it is necessary to
analize:
A. Caryotype
B. Leukogram
C. Spermatogenesis
D. Blood group
E. Genealogy

60. Examination of a miner revealed pulmonary fibrosis accompanied by


disturbance of alveolar ventilation. What is the main mechanism of this
disturbance?
A. Limitation of respiratory surface of lungs
B. Constriction of superior respiratory tracts
C. Disturbance of neural respiration control
D. Limitation of breast mobility
E. Bronchi spasm
61. After transfusion of 200 ml of blood a patient presented with body
temperature rise up to 37,9oC. Which of the following substances is the
most likely cause of temperature rise?
A. Interleukin-1
B. Interleukin-2
C. Tumour necrosis factor
D. Interleukin-3
E. Interleukin-4

62. A 49 y.o. woman consulted a doctor about heightened fatigue and dyspnea
during physical activity. ECG: heart rate is 50/min, PQ is extended, QRS is
unchanged, P wave quanity exceeds quantity of QRS complexes. What type
of arrhythmia does the patient have?
A. Atrioventricular block
B. Extrasystole
C. Sinus bradycardia
D. Ciliary arhythmia
E. Sinoatrial block

63. A woman has been applying a new cosmetic preparation for a week that
resulted in eye-lid inflammation accompanied by hyperemia, infiltration
and painfulness. What type of allergic reaction was developed?
A. ІV
B. I
C. II
D. III
E. V

64. A patient is followed up in an endocrinological dispensary on account of


hyperthyreosis. Weight loss, tachycardia, finger tremor are accompanied by
hypoxia symptoms - headache, fatigue, eye flicker. What mechanism of
thyroid hormones action underlies the development of hypoxia?
A. Disjunction, oxydation and phosphorilation
B. Inhibition of respiratory ferment synthesis
C. Competitive inhibition of respiratory ferments
D. Intensification of respiratory ferment synthesis
E. Specific binding of active centres of respiratory ferments

65. A 56 y.o. patient has been suffering from thyreotoxicosis for a long time.
What type of hypoxia can be developed?
A. Tissue
B. Hemic
C. Circulatory
D. Respiratory
E. Mixed
66. An animal with aortic valve insufficiency got hypertrophy of its left heart
ventricle. Some of its parts have local contractures. What substance
accumulated in the myocardiocytes caused these contractures?
A. Calcium
B. Potassium
C. Lactic acid
D. Carbon dioxide
E. Sodium

67. A girl is diagnosed with adrenogenital syndrome


(pseudohermaphroditism). This pathology was caused by hypersecretion
of the following adrenal hormone:
A. Androgen
B. Estrogen
C. Aldosterone
D. Cortisol
E. Adrenalin

68. A patient has extrasystole. ECG shows no P wave, QRS complex is


deformed, there is a full compensatory pause. What extrasystoles are
these?
A. Ventricular
B. Atrial
C. Atrioventricular
D. Sinus

69. A 12-year-old teenager has significantly put off weight within 3 months;
glucose concentration rose up to 50 millimole\l. He fell into a coma. What is
the main mechanism of its development?
A. Hyperosmolar
B. Hypoglycemic
C. Ketonemic
D. Lactacidemic
E. Hypoxic

70. As a result of increased permeability of the erythrocyte membrane in a


patient with microspherocytic anaemia (Minkowsky-Shauffard disease)
cells receive sodium ions and water. Erythrocytes take form of
spherocytes and can be easily broken down. What is the leading
mechanism of erythrocyte damage in this case?
A. Electrolytic osmotic
B. Calcium
C. Acidotic
D. Protein
E. Nucleic
71. A 56 year old patient suffering from cardiac insufficiency has edema of feet
and shins, edematous skin is pale and cold. What is the leding mechanism of
edema pathogenesis?
A. Rise of hydrostatic pressure in venules
B. Drop of oncotic pessure in capillaries
C. Increase of capillary permeability
D. Disorder of lymph outflow
E. Positive water balance

72. A disaster fighter at a nuclear power plant developed hemorrhagic


syndrome on the background of acute radiation disease. What is the
most important factor of syndrome pathogenesis?
A. Thrombocytopenia
B. Vascular wall damage
C. Increased activity of fibrinolysis factors
D. Increased activity of anticoagulative system factors
E. Decreased activity of coagulative factors

73. Prophylactic medical examination of a 36 year old driver revealed that his AP
was 150/90 mm Hg. At the end of working day he usually hears ear noise,
feels slight indisposition that passes after some rest. He was diagnosed with
essential hypertension. What is the leading pathogenetic mechanism in this
case?
A. Neurogenetic
B. Nephric
C. Humoral
D. Endocrinal
E. Reflexogenic

74. Violation of safety rules resulted in calomel intoxication. Two days later the
daily diuresis was 620 ml. A patient experienced headache, vomiting,
convulsions, dyspnea, moist rales in lungs. What pathology is it?
A. Acute renal insufficiency
B. Chronic renal insufficiency
C. Uraemic coma
D. Glomerulonephritis
E. Pyelonephritis

75. A newborn child with pylorostenosis has often repeating vomiting


accompanied by apathy, weakness, hypertonicity, sometimes convulsions.
What disorder form of acid-base balance is it?
A. Nongaseous alkalosis
B. Gaseous alkalosis
C. Gaseous acidosis
D. Metabolic acidosis
E. Excretory acidosis
76. 24 hours after appendectomy blood of a patient presents neutrophilic
leukocytosis with regenerative shift. What is the most probable
mechanism of leukocytosis development?
A. Amplification of leukopoiesis
B. Redistribution of leukocytes in the organism
C. Decelerated leukocyte destruction
D. Deceleratied emigration of leukocytes to the tissues
E. Amplification of leukopoiesis and decelerated emigration of leukocytes to the
tissues

77. A 59 year old patient is a plant manager. After the tax inspection of his plant
he felt intense pain behind his breastbone irradiating to his left arm. 15
minutes later his condition came to normal. Which of the possible
mechanisms of stenocardia development is the leading in this case?
A. High catecholamine concentration in blood
B. Coronary atherosclerosis
C. Intravascular aggregation of blood corpuscles
D. Coronary thrombosis
E. Functional heart overload

78. Arterial pressure of a surgeon who performed a long operation rised up to


140/110 mm Hg. What changes of humoral regulation could have caused
the rise of arterial pressure in this case?
A. Activation of sympathoadrenal system
B. Activation of formation and excretion of aldosterone
C. Activation of renin angiotensive system
D. Activation of kallikrein kinin system
E. Inhibition of sympathoadrenal system

79. A 50-year-old patient complains of thirst, drinking of a lot of water,


marked polyuria. Blood glucose is 4,8 mmol/L, urine glucose and acetone
bodies are absent, urine is colorless, specific gravity is 1,002-1,004. What
is the cause of polyuria?
A. Vasopressin insufficiency
B. Hypothyroidism
C. Insulin insufficiency
D. Aldosteronism
E. Thyrotoxicosis

80.A patient was ill with burn disease that was complicated by DIC syndrome.
What stage of DIC syndrome can be suspected if it is known that the patient's
blood coagulates in less than 3 minutes?
A. Hypercoagulation
B. Transition phase
C. Hypocoagulation
D. Fibrinolysis
E. Terminal
81. A 55 y.o. woman consulted a doctor about having continuous cyclic uterine
hemorrhages for a year, weakness, dizziness. Examination revealed skin
pallor. Hemogram: Hb - 70 g/l, erythrocytes - 3,2*1012/l, color index - 0,6,
leukocytes - 6,0*109 /l, reticulocytes - 1%; erythrocyte hypochromia. What
anemia is it?
A. Chronic posthemorrhagic anemia
B. Hemolytic anemia
C. Aplastic anemia
D. B12-folate-deficiency anemia
E. Iron-deficiency anemia

82. A 56 year old patient came to a hospital with complaints about general
weakness, tongue pain and burning, sensation of limb numbness. In the past
he underwent resection of forestomach. In blood: Hb- 80 g/l; erythrocytes -
2,0*1012/l; colour index - 1,2, leukocytes - 3,5*109 /l. What anemia type is it?
A. B12-folate deficient
B. Hemolytic
C. Posthemorrhagic
D. Aplastic
E. Iron-deficient

83. The patient with acute miocardial infarction was given intravenously
different solutions during 8 hours with medical dropper 1500ml and
oxygen intranasally. He died because of pulmonary edema. What caused
the pulmonary edema?
A. Volume overload of the left ventricular
B. Decreased oncotic pressure due to hemodilution
C. Allergic reaction
D. Neurogenic reaction
E. Inhalation of the oxygen

84. A 25 year old Palestinian woman complains of weakness, dizziness, dyspnea.


In anamnesis: periodically exacerbating anemia. In blood: Hb - 60 g/l,
erythrocytes - 2,5*101 2/l, reticulocytes - 35 o/oo, anisocytosis and
poikilocytosis of erythrocytes, a lot of target cells and polychromatophils.
What type of anemia is it?
A. Thalassemia
B. Sickle-cell anemia
C. Minkowsky-Shauffard disease
D. Addison-Biermer disease
E. Glucose 6-phosphate dehydrogenase-deficient anemia

85. A 23 y.o. patient complains of weakness, temperature rise up to 38-40oC.


Objectively: liver and spleen are enlarged. Hemogram: Hb- 100 g/l,
erythrocytes - 2,9*1012/l, leukocytes - 4,4*109 /l, thrombocytes – 48*109 /l,
segmentonuclear neutrophils - 17%, lymphocytes - 15%, blast cells - 68%.
All cytochemical reactions are negative. Make a hematological conclusion:
A. Undifferentiated leukosis
B. Chronic myeloleukosis
C. Acute myeloblastic leukosis
D. Acute lymphoblastic leukosis
E. Acute erythromyelosis

86. Inflammatory processes cause synthesis of protein of acute phase in an


organism. What substances stumulate their synthesis?
A. Interleukin-1
B. Immunoglobulins
C. Interferons
D. Biogenic amins
E. Angiotensin

87. A chemical burn caused esophagus stenosis. Difficulty of ingestion led to


the abrupt loss of weight. In blood: 3,0*10 1 2/l, Hb - 106 g/l, crude protein -
57 g/l. What type of starvation is it?
A. Incomplete
B. Proteinic
C. Complete
D. Water
E. Absolute

88. A 42 year old woman with neuralgia of trifacial nerve complains about
periodical reddening of the right part of her face and neck, sense of warmth
gush, increased skin sensitivity. These effects can be explained by the
following type of arterial hyperemia:
A. Neurotonic
B. Neuroparalytic
C. Metabolic
D. Functional
E. Reactive

89. A patient who suffers from acute myocarditis has clinical signs of cardiogenic
shock. What of the under-mentioned pathogenetic mechanisms plays the
main part in shock development?
A. Disturbance of pumping ability of heart
B. Depositing of blood in organs
C. Reduction of diastolic flow to the heart
D. Decrease of vascular tone
E. Increase of peripheral vascular resistance

90. On the 6th day of treatment a patient with acute renal insufficiency
developed polyuria. Diuresis intensification at the beginning of polyuria
stage of acute renal insufficiency is caused by:
A. Renewal of filtration in nephrons
B. Volume expansion of circulating blood
C. Growth of natriuretic factor
D. Reduction of aldosteron content in plasma
E. Reduction of vasopressin content in plasma

91. A 30 year old woman has face edemata. Examination revealed


proteinuria (5,87 g/l), hypoproteinemia, dysproteinemia,
hyperlipidemia. What condition is the set of these symptoms typical
for?
A. Nephrotic syndrome
B. Nephritic syndrome
C. Chronic pyelonephritis
D. Acute renal failure
E. Chronic renal failure

92. A patient with nephrotic syndrome has massive edemata of his face and
limbs. What is the leading pathogenetic mechanism of edemata
development?
A. Drop of oncotic blood pressure
B. Increase of vascular permeability
C. Rise of hydrodynamic blood pressure
D. Lymphostasis
E. Increase of lymph outflow

93. A patient staying in the pulmonological department was diagnosed with


pulmonary emphysema accompanied by reduced elasticity of pulmonary
tissue. What type of respiration is observed?
A. Expiratory dyspnea
B. Inspiratory dyspnea
C. Superficial respiration
D. Infrequent respiration
E. Periodic respiration

94. An unconscious young man with signs of morphine poisoning entered


admission office. His respiration is shallow and infrequent which is caused
by inhibition of respiratory centre. What type of respiratory failure is it?
A. Ventilative dysregulatory
B. Ventilative obstructive
C. Ventilative restrictive
D. Perfusive
E. Diffusive

95. A 62 year old patient who previously worked as stoker was admitted to a
hospital with complaints about general weakness, abrupt weight loss, hoarse
voice, dyspnea, dry cough. Laryngoscopy revealed a tumour in the pharynx
that invaded vocal cords and epiglottis. What is the most probable cause of
tumour development?
A. Polycyclic aromatic carbohydrates
B. Nitrosamines
C. Aromatic amines and amides
D. Retroviruses
E. Ionizing radiation

96. Tuberculine was injected intraperitoneally to the animal sensibilized with


tuberculine. Venous hyperemia and peritonial edema were detected on the
laparotomy in 24 hours. Increased amount of lymphocytes and monocytes
were in the smear-print from the peritonium. What pathological process is in
the animal?
A. Allergic inflammation
B. Serous inflammation
C. Suppurative inflammation
D. Fibrinous inflammation
E. Aseptic inflammation

97. Daltonism was diagnosed in a 7-year-old boy while prophylactic medical


examination. Parents are healthy, color vision is normal. Grandfather from
the mother's side has the same disorder. What is the type of inheriting of
this anomaly?
A. Recessive, connected with sex
B. Dominant, connected with sex
C. Incomplete domination
D. Autosomal-recessive
E. Autosomal-dominant

98. A married couple came to the genetic counseling. The husband suffers
from the insulin-dependant diabetes, the wife is healthy. What is the
probability that this couple will have an insulin-dependant child?
A. Higher than throughout the population
B. The same as throughout the population
C. Lower than throughout the population
D. 100%
E. 50%

99. A patient who had been working hard under conditions of elevated
temperature of the environment, has now a changed quantity of blood plasma
proteins. What penomenon is the case?
A. Relative hyperproteinemia
B. Absolute hyperproteinemia
C. Absolute hypoproteinemia
D. Disproteinemia
E. Paraproteinemia

100. An infant has pylorospasm, weakness, hypodynamia, convulsions


as a result of frequent vomiting. What kind of acid-base disbalance is it?
A. Excretory alkalosis
B. Excretory acidosis
C. Metabolic acidosis
D. Exogenous nongaseous acidosis
E. Gaseous alkalosis

101. An experimental rat with extremity paralysis has no tendon and


cutaneous reflexes, muscle tone is decreased, but muscles of the affected
extremity maintain their ability to react with excitation to the direct action
of continious current. What type of paralysis is it?
A. Flaccid peripheral
B. Flaccid central
C. Spastic peripheral
D. Spastic central
E. Extrapyramidal

102. A 28 year old man had a gunshot wound of shin that resulted in an
ulcer from the side of the injury. What is the main factor of neurodystrophy
pathogenesis in this case?
A. Traumatization of peripheral nerve
B. Psychical stress
C. Microcirculation disturbance
D. Infection
E. Tissue damage

103. A 45 year old patient was admitted to the cardiological department.


ECG data: negative P wave overlaps QRS complex, diastolic interval is
prolonged after extrasystole. What type of extrasystole is it?
A. Atrioventricular
B. Sinus
C. Atrial
D. Ventricular
E. Bundle-branch

104. A rabbit's nerve that innervates the right ear was cut and its right
superior cervical ganglion was removed. Immediately after operation the
temperature of ear skin was measured. It was revealed that the temperature
of the rabbit's ear skin on the side of denervation was by 1,5 oC higher than on
the opposite intact side. What of the following is the most probable
explanation of the above-mentioned effects?
A. Arterial neuroparalytic hyperemia
B. Arterial neurotopical hyperemia
C. Atrerial hyperemia induced by metabolic factors
D. Reactive arterial hyperemia
E. Physiological arterial hyperemia

105. Two hours after an exam a student had a blood count done and it was
revealed that he had leukocytosis without significant leukogram
modifications. What is the most probable mechanism of leukocytosis
development?
A. Redistribution of leukocytes in the organism
B. Leukopoiesis intensification
C. Deceleration of leukocyte lysis
D. Deceleration of leukocyte migration to the tissues
E. Leukopoiesis intensification and deceleration of leukocyte lysis

106. A patient presents with icteritiousness of skin, scleras and mucous


membranes. Blood plasma the total bilirubin is increased, stercobilin is
increased in feces, urobilin is increased in urine. What type of jaundice is it?
A. Haemolytic
B. Gilbert's disease
C. Parenchymatous
D. Obturational
E. Cholestatic

107. Hepatitis has led to the development of hepatic failure. Mechanism of


edemata formation is activated by the impairment of the following liver
function:
A. Protein-synthetic
B. Barrier
C. Chologenetic
D. Antitoxic
E. Glycogen-synthetic

108. As a result of a trauma a patient has developed traumatic shock that


led to the following disorders: AP is 140/90 mm Hg, Ps is 120 bpm. The
patient is fussy, talkative, pale. Such state relates to the following shock
phase:
A. Erectile
B. Latent period
C. Terminal
D. Torpid

109. Examination of a patient admitted to the surgical department with


symptoms of acute appendicitis revealed the following changes in the
white blood cells: the total count of leukocytes is 16*10 9 /l. Leukocyte
formula: basophils - 0, eosinophils - 2%, juvenile forms - 2%, stabnuclear
- 8%, segmentonuclear - 59%, lymphocytes - 25%, monocytes-4%. The
described changes can be classified as:
A. Neutrophilia with regenrative left shift
B. Neutrophilia with right shift
C. Neutrophilia with degenerative left shift
D. Neutrophilic leukemoid reaction
E. Neutrophilia with hyperregenerative left shift
110. A patient being treated for viral hepatitis type B got symptoms of
hepatic insufficiency. What blood changes indicative of protein metabolism
disorder will be observed in this case?
A. Absolute hypoalbuminemia
B. Absolute hyperalbuminemia
C. Absolute hyperfibrinogenemia
D. Proteinic blood composition is unchanged
E. Absolute hyperglobulinemia

111. A patient was stung by a bee. Examination revealed that his left hand
was hot, pink, edematic, there was a big red blister on the site of sting. What
is the leading mechanism of edema development?
A. Increased vessel permeability
B. Reduced vessel filling
C. Injury of vessels caused by the sting
D. Drop of oncotic pressure in tissue
E. Drop of osmotic pressure in tissue

112. A patient suffering from pheochromocytoma complains of thirst, dry


mouth, hunger. Blood test for sugar revealed hyperglycemia. What type of
hyperglycemia is it?
A. Adrenal
B. Hypercorticoid
C. Alimentary
D. Somatotropic
E. Hypoinsulinemic

113. A patient suffering from stenocardia was taking nitroglycerine


which caused restoration of blood supply of myocardium and relieved
pain in the cardiac area. What intracellular mechanism provides
restoration of energy supply of insulted cells?
A. Intensification of ATP resynthesis
B. Reduction of ATP resynthesis
C. Increased permeability of membranes
D. Intensification of oxygen transporting into the cell
E. Intensification of RNA generation

114. A couple had a child with Down's disease. Mother is 42 years


old. This disease is most probably caused by the following impairment
of prenatal development:
A. Gametopathy
B. Blastopathy
C. Embryopathy
D. Non-specific fetopathy
E. Specific fetopathy
115. There are several groups of molecular mechanisms playing
important part in pathogenesis of insult to cells which contributes to the
pathology development. What processes are stimulated by proteinic
damage mechanisms?
A. Enzyme inhibition
B. Lipid peroxidation
C. Phospholipase activation
D. Osmotic membrane distension
E. Acidosis

116. A child was born with cleft palate. Examination revealed aorta defects
and reduced number of T-lymphocytes in blood. What immunodeficient
syndrome is it?
A. DiGeorge
B. Wiskott-Aldrich
C. Chediak-Higashi
D. Louis-Bar
E. Swiss-type

117. Examination of a child who frequently suffers from infectious diseases


revealed that IgG concentration in blood serum was 10 times less than
normal, IgA and IgM concentration was also significantly reduced. Analysis
showed also lack of B-lymphocytes and plasmocytes. What disease are these
symptoms typical for?
A. Bruton's disease
B. Swiss-type agammaglobulinemia
C. Dysimmunoglobulinemia
D. Louis-Bar syndrome
E. Di George syndrome

118. A driver who got a trauma in a road accident and is shocked has
reduction of daily urinary output down to 300 ml. What is the main
pathogenetic factor of such diuresis change?
A. Drop of arterial pressure
B. Drop of oncotic blood pressure
C. Increased vascular permeability
D. Decreased number of functioning glomerules
E. Secondary hyperaldosteronism

119. Examination of a 42 year old patient revealed a tumour of


adenohypophysis. Objectively: the patient's weight is 117 kg, he has moon-
like hyperemic face, red-blue striae of skin distension on his belly.
Osteoporosis and muscle dystrophy are present. AP is 210/140 mm Hg.
What is the most probable diagnosis?
A. Cushing's disease
B. Cushing's syndrome
C. Conn's disease
D. Diabetes mellitus
E. Essential hypertension

120. Examination of a 12 year old boy with developmental lag revealed


achondroplasia: disproportional constitution with evident shortening of
upper and lower limbs as a result of growth disorder of epiphyseal
cartilages of long tubal bones. This disease is:
A. Inherited, dominant
B. Inherited, recessive
C. Inherited, sex-linked
D. Congenital
E. Acquired

121. A patient was diagnosed with autoimmune hemolitic cytotoxic


anemia. What substances are antigens in II type allergic reactions?
A. Modified receptors of cell membranes
B. Antibiotics
C. Hormones
D. Serum proteins
E. Inflammation modulators

122. A patient suffers from the haemorrhagic syndrome that shows itself
in frequent nasal bleedings, posttraumatic and spontaneous intracutaneous
and intra-articular haemorrhages. After a laboratory study a patient was
diagnosed with the type B haemophilia. This disease is provoked by the
deficit of the following factor of blood coagulation:
A. IX
B. VIII
C. XI
D. V
E. VII

123. A 58-year-old patient suffers from the cerebral atherosclerosis.


Examination revealed hyperlipoidemia. What class of lipoproteins
will most probably show increase in concentration in this patient’s
blood serum?
A. Low-density lipoproteins
B. High-density lipoproteins
C. Fatty acid complexes with albumins
D. Chylomicrons
E. Cholesterol

124. In course of a preventive examination of a miner a doctor revealed


changes of cardiovascular fitness which was indicative of cardiac
insufficiency at the compensation stage. What is the main proof of cardiac
compensation?
A. Myocardium hypertrophy
B. Tachycardia
C. Rise of arterial pressure
D. Dyspnea
E. Cyanosis

125. A 47 year old man with myocardium infarction was admitted to the
cardiological department. What changes of cellular composition of peripheral
blood are induced by necrotic changes in the myocardium?
A. Neutrophilic leukocytosis
B. Monocytosis
C. Eosinophilic leukocytosis
D. Thrombocytopenia
E. Lymphopenia

126. A patient with skin mycosis has disorder of cellular immunity. The
most typical characteristic of it is reduction of the following index:
A. T-lymphocytes
B. Immunoglobulin G
C. Immunoglobulin E
D. B-lymphocytes
E. Plasmocytes

127. A patient with massive burns developed acute renal insufficiency


characterized by a significant and rapid deceleration of glomerular
filtration. What is the mechanism of its development?
A. Reduction of renal blood flow
B. Damage of glomerular filter
C. Reduction of functioning nephron number
D. Rise of pressure of tubular fluid
E. Renal artery embolism

128. A child is pale, pastose, muscular tissue is bad developed, lymph nodes
are enlarged. He often suffers from angina and pharyngitis, blood has signs of
lymphocytosis. The child is also predisposed to autoallergic diseases. What
type of diathesis can be presumed in this case?
A. Lymphohypoplastic
B. Exudative
C. Gouty
D. Asthenic
E. Hemorrhagic

129. Parents of a 3 year old child have been giving him antibiotics with
purpose of preventing enteric infections for a long time. A month later the
child's condition changed for the worse. Blood examination revealed
apparent leukopenia and granulocytopenia. What is the most probable
mechanism of blood changes?
A. Myelotoxic
B. Autoimmune
C. Redistributive
D. Age-specific
E. Hemolytic

130. A patient ill with enteritis accompanied by massive diarrhea has low
water rate in the extracellular space, high water rate inside the cells and
low blood osmolarity. What is such disturbance of water-electrolytic
metabolism called?
A. Hypo-osmolar hypohydration
B. Hyperosmolar hypohydration
C. Osmolar hypohydration
D. Hypo-osmolar hyperhydration
E. Hyperosmolar hyperhydration

131. A patient with obliterating atherosclerosis underwent sympathectomy


of femoral artery in the region of femoral trigone. What type of arterial
hyperemia was induced by the operation?
A. Neuroparalytic
B. Reactive
C. Metabolic
D. Neurotonic
E. Functional

132. A 15 year old girl has pale skin, glossitis, gingivitis. Blood
count: erythrocytes - 3,3*101 2/l, hemoglobin - 70 g/l, colour
index - 0,5. Examination of blood smear revealed hypochromia,
microcytosis, poikilocytosis. What type of anemia is it?
A. Iron-deficient
B. B12-folic acid- deficient
C. Sickle-cell
D. Hemolytic
E. Thalassemia

133. A 70 year old man is ill with vascular atherosclerosis of lower


extremities and coronary heart disease. Examination revealed disturbance of
lipidic blood composition. The main factor of atherosclerosis pathogenesis is
the excess of the following lipoproteins:
A. Low-density lipoproteins
B. Cholesterol
C. High-density lipoproteins
D. Intermediate density lipoproteins
E. Chylomicrons

134. A patient ill with essential arterial hypertension had a hypertensic


crisis that resulted in an attack of cardiac asthma. What is the leading
mechanism of cardiac insufficiency in this case?
A. Heart overload caused by high pressure
B. Heart overload caused by increased blood volume
C. Absolute coronary insufficiency
D. Myocardium damage
E. Blood supply disturbance

135. A 5 year old child is ill with measles. Blood analysis revealed increase
of total number of leukocytes up to 13*109 /l. Leukogram: basophils - 0,
eosinophils - 1, myelocytes - 0, juvenile neutrophils - 0, band neutrophils -
2, segmented neutrophils - 41, lymphocytes - 28, monocytes - 28. Name this
phenomenon:
A. Monocytosis
B. Agranulocytosis
C. Lymphocytosis
D. Eosinopenia
E. Neutropenia

136. Rats being under stress have muscular hypertonia and high arterial
pressure, high glucose concentration in blood and intensified secretion of
corticotropin and corticosteroids. In what stress phase are these animals?
A. Antishock phase
B. Exhaustion
C. Shock phase
D. Erectile
E. Terminal

137. After taking poor-quality food a patient developed repeated episodes of


diarrhea. On the next day he presented with decreased arterial pressure,
tachycardia, extrasystole. Blood pH is 7,18. These abnormalities were caused
by the development of:
A. Nongaseos acidosis
B. Gaseous acidosis
C. Nongaseous alkalosis
D. Gaseous alkalosis
E. Metabolic alkalosis

138. A patient has been diagnosed with influenza. His condition became
drastically worse after taking antipyretic drugs. His consciousness is
confused, AP is 80/50 mm Hg, Ps is 140/m, body temperature droped
down to 35,8oC. What complication developed in this patient?
A. Collapse
B. Hyperthermia
C. Hypovolemia
D. Acidosis
E. Alkalosis

139. A patient was admitted to the infectious department. His symptoms:


dry skin, decreased skin turgor, rice-water stool. The patient was diagnosed
with cholera. What disorder of water-electrolytic balance is most often
observed in this disease?
A. Isoosmotic hypohydration
B. Hyperosmotic hyperhydration
C. Hypoosmotic hypohydration
D. Hyperosmotic hypohydration
E. Hypoosmotic hyperhydration

140. A 45 year old woman is ill with breast cancer. Her left arm has
symptoms of lymphatic system insufficiency - limb edema, lymph node
enlargement. What form of lymphatic circulation insufficiency is it?
A. Mechanic insufficiency
B. Dynamic insufficiency
C. Resorption insufficiency
D. Combined insufficiency

141. A 25 year old man has spent a long time in the sun under high air
humidity. As a result of it his body temperature rose up to 39 oC. What
pathological process is it?
A. Hyperthermia
B. Infectious fever
C. Hypothermia
D. Noninfectious fever
E. Burn disease

142. A 26 year old man is in the torpid shock phase as a result of a


car accident. In blood: 3,2*10 9 /l. What is the leading mechanism of
leukopenia development?
A. Redistribution of leukocytes in bloodstream
B. Leikopoiesis inhibition
C. Disturbed going out of mature leukocytes from the marrow into the blood
D. Lysis of leukocytes in the blood-forming organs
E. Intensified elimination of leukocytes from the organism
Krok 1 – 2015 Pharmacology Base
1. A 30-year-old patient complains about having abdominal pain and diarrhea
for five days; body temperature rise up to 37,5 oC along with chills. The day
before a patient had been in a forest and drunk from an open water
reservoir. Laboratory analyses enabled to make the following diagnosis:
amebic dysentery. What is the drug of choice for its treatment?
A. Metronidazole
B. Furazolidonum
C. Levomycetin
D. Phthalazol
E. Emetine hydrochloride

2. A woman works as railway traffic controller. She suffers from seasonal


vasomotor rhinitis and gets treatment in the outpatient setting. She was
prescribed an antihistamine that has no effect upon central nervous system.
What drug is it?
A. Loratadine
B. Dimedrol
C. Promethazine
D. Suprastin
E. Tavegil

3. During an operation a patient got injection of muscle relaxant dithylinum.


Relaxation of skeletal muscles and inhibition of respiration lasted two
hours. This condition was caused by absence of the following enzyme in
blood serum:
A. Butyrylcholin esterase
B. Catalase
C. Acetylcholinesterase
D. Glucose 6-phosphatase
E. Glutathione peroxidase

4. A 60-year-old patient was admitted to the surgical department because of


infection caused by blue pus bacillus (Pseudomonas aeruginosa) which is
sensative to penicillin antibiotics. Indicate which of the given penicillins has
marked activity to the Pseudomonas aeruginosa?
A. Carbenicillin disodium
B. Benzylpenicillin
C. Phenoxymethylpenicillin
D. Oxacillin
E. Methicillin

5. A 45-year-old woman suffers from allergic seasonal coryza caused by


the ambrosia blossoming. What medicine from the stabilizer of the
adipose cells group can be used for prevention of this disease?
A. Ketotifen
B. Diazoline
C. Phencarol
D. Tavegyl
E. Dimedrol

6. Systemic amebiasis with involvment of intestines, liver, lungs was


diagnosed in a 52-year-old patient. What drug should be
prescribed?
A. Metronidasol
B. Quiniofone
C. Tetracycline
D. Quingamine
E. Enteroseptol

7. A 38-year-old man who poisoned himself with mercury dichloride was


taken to the admission room in grave condition. What antidote should be
immediately introduced?
A. Unithiol
B. Dipiroxim
C. Atropine
D. Nalorphine
E. Isonitrosine

8. A patient who suffers from insomnia caused by emotional disorder was


prescribed a hypnotic drug with tranquillizing effect. What hypnotic was
prescribed?
A. Nitrazepam
B. Phenobarbital
C. Chloral hydrate
D. Sodium ethaminal
E. Bromisoval

9. A patient had to go through an operation. Doctors introduced him


dithylinum (listenone) and performed intubation. After the end of operation
and cessation of anesthesia the independent respiration wasn't restored.
Which enzyme deficit prolongs the action of muscle relaxant?
A. Pseudocholinesterase
B. Succinate dehydrogenase
C. Carbanhydrase
D. N-acetyltransferase
E. K-Na-adenosine triphosphatase

10. The alternate usage of dichlotiazide, etacrin acid and lasex didn't cause
marked diuretic effect in patient with marked peripheral edema. Increased
amount of aldosterone is in the blood. Indicate the medicine to be
prescribed.
A. Spironolacton
B. Mannit
C. Clopamid
D. Urea
E. Amilorid
11. A 56-year-old patient with complains of thirst and frequent urination was
diagnosed to have diabete mellitus and butamin was prescribed. What is
the mechanism of action of this medicine?
A. It stimulates beta-cells of Langergans' islets
B. It helps to absorb the glucose by the cells of the organism tissues
C. It relieves transport of glucose through the cells' membranes
D. It inhibits alpha cells of Langergans' islets
E. It inhibits absorption of glucose in the intestines

12. A 37 year old patient suffering from obliterating vascular endarteritis of


lower limbs takes daily 60 microgram/kilogram of phenylin. Because of
presentations of convulsive disorder (craniocerebral trauma in anamnesis)
he was prescribed phenobarbital. Withholding this drug caused nasal
hemorhage. What is this complication connected with?
A. Induction of enzymes of microsomal oxidation in liver caused by
phenobarbital
B. Aliphatic hydroxylation of phenobarbital
C. Conjugation of phenylin with glucuronic acid
D. Oxidative deamination of phenylin
E. Inhibition of microsomal oxidation in liver caused by phenobarbital

13. Patient with complaints of dryness in the mouth, photophobia and vision
violation was admitted to the reception-room. Skin is hyperemic, dry,
pupils are dilated, tachycardia. Poisoning with belladonna alkaloids was
diagnosed on further examination. What medicine should be prescribed?
A. Prozerin
B. Diazepam
C. Pilocarpine
D. Armine
E. Dipyroxim

14. Patient was on glucocorticoids for a long time, discontinuation of usage


caused exacerbation of the illness, decreased BP, weakness. How can you
explain it?
A. Insufficiency of adrenal glands
B. Adaptation to the medicine
C. Sensitization
D. Hyperproduction of ACTH
E. Cumulation

15. Patient complaines of weakness, dyspnea, low extremities oedema.


Diagnosis: chronic cardiac insufficiency. What medicine should be
prescribed first of all?
A. Digitoxin
B. Caffeine
C. Papaverine
D. Propranolol
E. Raunatin
16. Signs of gastropathy develop in the patient with rheumatoid arthritis who
was treated with indometacin. With what activity of the drug can this
complication be connected?
A. Anticyclooxygenase
B. Antiserotonin
C. Antihistamine
D. Antikinine
E. Local irritating

17. Testosterone and it's analogs increase the mass of skeletal muscles that allows
to use them for treatment of dystrophy. Due to interaction of the hormon with
what cell substance is this action caused?
A. Nuclear receptors
B. Membrane receptors
C. Ribosomes
D. Chromatin
E. Proteins- activators of transcription

18. A patient ill with neurodermatitis has been taking prednisolone for a long
time. Examination revealed high rate of sugar in his blood. This
complication is caused by the drug influence upon the following link of
carbohydrate metabolism:
A. Gluconeogenesis activation
B. Glycogenogenesis activation
C. Intensification of glucose absorption in the bowels
D. Inhibition of glycogen synthesis
E. Activation of insulin decomposition

19. Proserin increases skeletal muscle tone when given systematically.


Halothane induces relaxation of skeletal muscles and reduces proserin
effects. What is the nature of proserin and halothane interaction?
A. Indirect functional antagonism
B. Direct functional antagonism
C. Competitive antagonism
D. Independent antagonism
E. Noncompetitive antagonism

20. A 70-year-old man, who suffered from chronic bronchitis, was prescribed
medicine for the cough - codeine. What is the mechanism of anticoughing
effect?
A. Central
B. Reflex
C. Competitive
D. Local effect
E. Peripheral effect

21. A patient with frequent attacks of stenocardia was prescribed sustak-forte to


be taken one tablet twice a day. At first the effect was positive but on the
second day stenocardia attacks resumed. What can explain inefficiency of
the prescribed drug?
A. Tachyphylaxis
B. Cumulation
C. Sensibilization
D. Idiosyncrasy
E. Dependence

22. A 37-year-old man was admitted to the surgical department with


symptoms of acute pancreatitis: vomiting, diarrhea, bradycardia,
hypotention, weakness, dehydration of the organism. What medicine
should be used first of all?
A. Contrycal
B. No-spa
C. Platyphylline
D. Etaperazine
E. Ephedrine

23. The patient has taken the mixture prescribed by neuropathologist for
neurasthenia for 2 weeks. Patient felt better but developed coryza,
conjunctivitis, rash, inertia, decrease of memory. Bromizm was
diagnosed. What should be prescribed to decrease symptoms?
A. Natrium chloride
B. Glucose solution 5%
C. Asparcam
D. Polyglucin

24. Analeptical remedy of reflective type from the H-cholinomimetics group was
given to the patient for restoration of breathing after poisoning with carbon
monoxide. What medicine was prescribed to the patient?
A. Lobeline hydrochloride
B. Atropine sulphate
C. Adrenalin hydrochloride
D. Mesaton
E. Pentamin

25. An aged patient complains of headache, dizziness, quick tiredness,


worsening of memory. Anamnesis: craniocerebral injury. Medicine of
what group should be prescribed?
A. Nootropics
B. Somnific
C. Neuroleptics
D. Analgetics
E. Sedatives

26. The patient was treated medically for psychosis for 2 weeks. Patient's
condition improved but rigidity, tremor, hypokinesia developed. Which of
the drugs can cause such complications?
A. Aminazine
B. Diphenine
C. Sydnocarb
D. Imipramine
E. Chlordiazepoxide

27. Gonorrhoea was revealed in the patient on bacterioscopy of the smear


from urethra. Taking into account that medecines for gonorrhoea are
fluorquinolones, patient should be prescribed:
A. Ciprofloxacin
B. Furazolidone
C. Fluorouracil
D. Urosulfan
E. Cefazoline

28. Patient with bronchial asthma was taking tablets which caused
insomnia, headache, increased blood pressure. What medecine can
cause such complications?
A. Ephedrine
B. Adrenaline
C. Chromolin sodium
D. Euphyline
E. Izadrine

29. Diuretic drug was prescribed to the patient with hypertension in the course
of complex treatment. In a few days BP decreased but signs of hypokaliemia
developed. What drug could cause such complications?
A. Lasix
B. Spironolactone
C. Clophelin
D. Triamterene
E. Enalapril

30. Patient was admitted to the infection unit with diagnosis of bacterial
dysentery. On laboratory studies it was revealed that causative element is
sensative to the many antimicrobial medicines, but patient has anemia.
What medicine is contra-indicated to the patient?
A. Levomycetin
B. Phthalazol
C. Furazolidone
D. Enteroseptol
E. Ampicillin

31. Patient with pneumonia has intolerance to antibiotics. Which of the


conbined sulfanilamide medicines should be prescribed to the patient?
A. Biseptol
B. Aethazol
C. Natrium sulfacyl
D. Streptocid
E. Sulfadimethoxine
32. A patient suffering from myasthenia has been administered proserin. After
its administration the patient has got nausea, diarrhea, twitch of tongue
and skeletal muscles. What drug would help to eliminate the intoxication?
A. Atropine sulfate
B. Physostigmine
C. Pyridostigmine bromide
D. Isadrine
E. Mesatonum

33. A patient who had myocardial infarction was administered 75 mg of


acetylsalicinic acid a day. What is the purpose of this administration?
A. Reduction of thrombocyte aggregation
B. Inflammation reduction
C. Pain relief
D. Temperature reduction
E. Coronary vessel dilatation

34. Patient with mercury poisoning was admitted to the toxicological


department from the chemical industry. What medicine should be
used?
A. Unithiol
B. Isonitrozin
C. Naloxone
D. Activated carbon
E. Enterosorbent

35. A 5-year-old child has been diagnosed with acute right distal pneumonia.
Sputum inoculation revealed that the causative agent is resistant to
penicillin, but it is senstive to macrolides. What drug should be prescribed?
A. Azithromycin
B. Tetracycline
C. Gentamycin
D. Streptomycin
E. Ampicillin

36. A patient suffering from chronic hyperacidic gastritis takes an antacid drug
for heartburn elimination. After its ingestion the patient feels better but at
the same time he has a sensation of stomach swelling. Which of the
following drugs might be the cause of such side effect?
A. Sodium hydrocarbonate
B. Magnesium oxide
C. Magnesium trisilicate
D. Aluminium hydrooxide
E. Pepsin

37. A patient consulted a doctor about bowels disfunction. The doctor


established symptoms of duodenitis and enteritis. Laboratory examination
helped to make the following diagnosis: lambliosis. What medication
should be administered?
A. Metronidazole
B. Erythromycin
C. Monomycin
D. Chingamin
E. Tetracycline

38. Introduction of a pharmaceutical substance to an experimental animal


resulted in reduction of salivation, pupil mydriasis. Next intravenous
introduction of acetylcholine didn't lead to any significant changes of heart
rate. Name this substance:
A. Atropine
B. Adrenaline
C. Propranolol
D. Proserin
E. Salbutamol

39. Continious taking of a drug can result in osteoporosis, erosion of stomach


mucous membrane, hypokaliemia, retention of sodium and water, reduced
content of corticotropin in blood. Name this drug:
A. Prednisolone
B. Hydrochlorothiazide
C. Digoxin
D. Indometacin
E. Reserpine

40. A 63 y.o. man with collapse symptoms was delivered to the emergency
hospital. A doctor chose noradrenaline in order to prevent hypotension.
What is the action mechanism of this medication?
A. Activation of α1-adrenoreceptors
B. Activation of serotonin receptors
C. Activation of β-adrenoreceptors
D. Activation of dopamine receptors
E. Block of M-cholinoreceptors

41. Anapriline therapy caused positive effect in the dynamic of the disease
of a 44-year-old woman suffering from stenocardia. What is the main
mechanism of the effect of this medicine?
A. Blockade of β-adrenoreceptors and decrease myocardial
requirements to the oxygen
B. Decrease of oxidative exchange in myocardium due to enzyme blockade of
Krebs' cycle
C. Decreased power inputs of myocardium due to reduced loading
D. Increased oxygen supply to the myocardium
E. Decreased need in increasing of oxygen supply to the myocardium

42. A patient suffering from syphilis has been treated with bismuth
preparations. As a result of it some grey spots turned up on the mucous
membrane of the oral cavity; nephropathy symptoms were also present.
What drug should be used for treatment of bismuth intoxication?
A. Unithiol
B. Methylene blue
C. Naloxone
D. Bemegride
E. Nalorphine

43. A 4 year old child was admitted to the orthopaedic department with shin
fracture together with displacement. Bone fragments reposition requires
preliminary analgesia. What preparation should be chosen?
A. Promedol
B. Analgin
C. Morphine hydrochloride
D. Panadol

44. Patient with abscess of the cut wound applied to the traumatological
department. Doctor for the cleaning of the wound from the pus washed it
with 3% hydrogen peroxide. Foam was absence. What caused the absents
on the drug activity?
A. Inherited insufficiency of catalase
B. Low concentration H2O2
C. Inherited insufficiency phosphatdehydrogenase of erythrocyte
D. Shallow wound
E. Pus in the would

45. A patient was diagnosed with active focal pulmonary tuberculosis.


What drug should be prescribed in the first place?
A. Isoniazid
B. Sulfalen
C. Cyclocerine
D. Ethionamide
E. Ethoxide

46. A patient taking clonidine for essential hypertension treatment was using
alcohol that caused intense inhibition of central nervous system. What may
it be connected with?
A. Effect potentiating
B. Effect summation
C. Cumulation
D. Intoxication
E. Idiosyncrasy

47. Examination of a patient revealed extremely myotic pupils, sleepiness,


infrequent Chain-Stoke's respiration, urinary retention, slowing-down
of heart rate, enhancement of spinal reflexes. What substance caused
the poisoning?
A. Morphine
B. Atropine
C. Phosphacole
D. Caffeine
E. Barbital

48. A patient suffers from diabetes melitus. After the regular insulin injection
his condition grew worse: there appeared anxiety, cold sweat, tremor of
limbs, general weakness, dizziness. What preparation can eliminate these
symptoms?
A. Adrenaline hydrochloride
B. Butamide
C. Caffeine
D. Noradrenaline
E. Glibutide

49. Continuous taking of some drugs foregoing the pregnancy increase the risk
of giving birth to a child with genetic defects. What is this effect called?
A. Mutagenic effect
B. Embryotoxic effect
C. Teratogenic effect
D. Fetotoxical effect
E. Blastomogenic effect

50. A doctor administered a patient with allergic dermatitis a H1 -


histamine blocker as a part of complex treatment. Name this
medication:
A. Loratadine
B. Cromolyn sodium
C. Prednisolone
D. Adrenaline
E. Hydrocortisone

51. A 19 year old woman suffers from primary syphilis. Doctor administered
her complex therapy that includes benzylpenicillin sodium salt. What is the
mechanism of action of this drug?
A. It blocks synthesis of peptidoglycan of microbal membrane
B. It blocks synthesis of cytoplasm proteins
C. It blocks thiol enzymes
D. It blocks RNA synthesis
E. It blocks DNA synthesis

52. A 65 year old female patient suffers from chronic renal insufficiency
accompanied by evident edemata caused by chronic glomerulonephritis.
What diuretic should be administered for forced diuresis?
A. Furosemide
B. Hydrochlorothiazide
C. Chlorthalidone
D. Cyclometazide
E. Acetazolamide
53. A patient suffering from coronary artery disease had taken a certain drug
many times a day in order to arrest stenocardia attacks. Overdose of this
drug finally caused intoxication. Objectively: cyanotic skin and mucous
membranes, dramatic fall in the arterial pressure, tachycardia, respiration
inhibition. Blood has increased concentration of methemoglobin. The drug
the patient had taken relates to the following group:
A. Organic nitrates
B. alpha-adrenoceptor blockers
C. Calcium channel blockers
D. Adenosine drugs
E. Myotropic spamolytics

54. A patient with II stage hypertension has been taking one of hypotensive
medications for the purpose of treatment. After a time arterial pressure
decreased, but the patient started complaining of flaccidity, sleepiness,
indifference. A bit later he felt stomach pain. He was diagnosed with ulcer.
What hypotensive medication has the patient been taking?
A. Reserpine
B. Dibazole
C. Furosemide
D. Verapamil
E. Captopril

55. A child suffers from drug idiosyncrasy. What is the cause of


such reaction?
A. Hereditary enzymopathy
B. Exhaustion of substrate interacting with pharmaceutical substance
C. Accumulation of pharmaceutical substance
D. Inhibition of microsomal liver enzymes
E. Associated disease of target organ

56. Examination of a 70 year old patient rrevealed insulin-dependent diabetes.


What drug should be administered?
A. Glibenclamid
B. Insulin
C. Mercazolilum
D. Parathyroidin
E. Cortisone

57. For the preparation of a patient's burn skin surface a certain


medication was used. Its antiseptic action is provided by free oxygen
that segregates in presence of organic substances. Choose the right
answer:
A. Potassium permanganate
B. Furacilin
C. Chlorhexidine
D. Boric acid
E. Sodium bicarbonate
58. A patient suffering from chronic cardiac insufficiency was
recommended to undergo a prophylactic course of treatment with a
cardiological drug from the group of cardiac glycosides that is to be
taken enterally. What drug was recommended?
A. Digoxin
B. Strophanthine
C. Corglycon
D. Cordiamin
E. Cordarone

59. A 50 y.o. patient with chronic cardiac insufficiency and tachyarrythmia


was prescribed a cardiotonic drug. What drug was prescribed?
A. Digoxin
B. Dopamine
C. Dobutamine
D. Amyodarone
E. Mildronate

60. An ophthalmologist used a 1% mesaton solution for the diagnostic purpose


(pupil dilation for eye-ground examination). What is the cause of mydriasis
induced by the drug?
A. Activation of α1 adrenoreceptors
B. Activation of α2 adrenoreceptors
C. Block of α1 adrenoreceptors
D. Activation of β1 adrenoreceptors
E. Activation of M-cholinoreceptors

61. Mother of a 2 year old child consulted a stomatologist. In the period of


pregnancy she was irregularly taking antibiotics for an infectious disease.
Examination of the child revealed incisor destruction, yellow enamel,
brown rim around the dental cervix. What drug has apparent teratogenic
effect?
A. Doxacycline
B. Furosemide
C. Ampiox
D. Xantinol nicotinate
E. Octadine

62. A 36 y.o. man has a craniocerebral trauma. Objectively: diminished breath


sounds, thready pulse, no reflexes. What way of pyracetam introduction
will be the most apropriate in this case?
A. Intravenous
B. Rectal
C. Subcutaneous
D. Peroral
E. Inhalation

63. A patient suffering from initial hypertension has been taking an


antihypertensive preparation for a long time. Suddenly he stopped taking
this preparation. After this his condition grew worse, this led to
development of hypertensive crisis. This by -effect can be classified as:
A. Abstinence syndrome
B. Cumulation
C. Tolerance
D. Sensibilization
E. Dependence

64. A patient ill with bronchial asthma didn't inform his doctor that he had
attacks of stenocardia. Doctor administered him a medication, which taking
resulted in less frequent attacks of bronchial asthma, but stenocardia attacks
became more frequent. What medication was administered?
A. Isadrin
B. Salbutamol
C. Aminophylline
D. Cromolyn sodium
E. Phenotherol

65. A patient who has been suffering from cardiac insufficiency for several
months has been taking digoxin on an outpatient basis. At a certain stage
of treatment there appeared symptoms of drug overdose. What
phenomenon underlies the development of this complication?
A. Material cumulation
B. Habituation
C. Sensibilization
D. Functional cumulation
E. Tachyphylaxis

66. A patient suffers from vision impairment - hemeralopy (night


blindness). What vitamin preparation should be administered the
patient in order to restore his vision?
A. Retinol acetate
B. Vicasol
C. Pyridoxine
D. Thiamine chloride
E. Tocopherol acetate

67. A patient suffers from severe postoperative pseudomonadous infection.


What of the following antibiotics should be administered in this case?
A. Amicacin sulfate
B. Benzylpenicillin
C. Cephazolin
D. Erythromycin
E. Doxycycline

68. After a tooth extraction a patient felt persistent pain behind his breast
bone. After sublingual intake of an antianginal drug the pain behind the
breast bone disappeared, but the patient complained of headache and
dizziness. What drug are these properties typical for?
A. Nitroglycerin
B. Propranolol
C. Metoprolol
D. Validol
E. Verapamil

69. A patient with fracture of his lower jaw was admitted to the maxillofacial
department. It was decided to fix his bones surgically under anaesthetic.
After intravenous introduction of muscle relaxant there arose short fibrillar
contractions of the patient's facial muscles. What muscle relaxant was
applied?
A. Dithylinum
B. Tubocurarin chloride
C. Pipecuronium bromide
D. Diazepam
E. Melictine

70. Patient in the unconscious state was admitted to the emergency room. Skin
is cold, pupils are delayed, breathing is heavy, with cycles of the Cheyne-
Stokes type, blood pressure is decreased, urinary bladder is overloaded.
Poisoning with what substance is the most likely?
A. Narcotic analgesics
B. Sedatives
C. Non-narcotic analgesics
D. М- cholinergic antagonists

71. Examination of a 60 y.o. patient revealed hyperglycemia and


glucosuria. A doctor administered him a medication for internal
use. What medication is it?
A. Glibenclamid
B. Furosemide
C. Oxytocin
D. Pancreatine
E. Corglycon

72. An elderly female patient suffers from the type 2 diabetes mellitus
accompanied by obesity, atherosclerosis, coronary artery disease. Basal
hyperinsulinemia is also present. What treatment would be the most
appropriate?
A. Glibenclamid
B. Insulin
C. Retabolil
D. Lovastatin
E. Amlodipine

73. A patient suffers from stenocardia and takes isosorbide mononitrate. He


was prescribed a complementary drug with disaggregating effect. What
drug is it?
A. Acetylsalicinic acid
B. Nitroglycerine
C. Propranolol
D. Nifedipine
E. Validol

74. A patient in postoperative period was prescribed an anticholinesterase drug


for stimulation of intestinal peristalsis and tonus of urinary bladder. What
drug is it?
A. Proserin
B. Dichlothiazide
C. Reserpine
D. Mannitol
E. Propanolol

75. A liquidator of a breakdown at a nuclear power plant who was irradiated


complained about vomiting that occurs all of a sudden. What medication
should be prescribed?
A. Metoclopramide
B. Reserpine
C. Atropine
D. Aeron
E. De-Nol

76. A patient with chronic cardiac insufficiency has been treated with
cardiotonic drugs and a thiazide diuretic, but in spite of it there are still
edemata and risk of ascites. What medication should be prescribed to
amplify diuretic effect of the applied drugs?
A. Spironolactone
B. Furosemide
C. Amyloride
D. Clopamide
E. Manitole

77. A patient ill with collagenesis has been taking prednisolone for a long time.
Hypokaliemia development caused spastic pain of skeletal muscles. What
medication should be used in order to correct potassium exchange?
A. Panangin
B. Dithylinum
C. Diazepam
D. Noshpa
E. Thyrocalcitonin

78. A patient ill with essential hypertension was recommended a drug that
prevents thrombosis. It is to be taken parenterally. What drug is it?
A. Heparin
B. Amben
C. Protamine sulfate
D. Neodicumarin
E. Syncumar

79. A patient presents with twilight vision impairment. Which of the following
vitamins should be administered?
A. Retinol acetate
B. Cyanocobalamin
C. Pyridoxine hydrochloride
D. Ascorbic acid
E. Nicotinic acid

80.A female patient consulted a doctor about pain and limited movements in
the knee joints. Which of the following nonsteroid anti-inflammatory drugs
should be administered taking into consideration that the patient has a
history of chronic gastroduodenitis?
A. Celecoxib
B. Diclofenac sodium
C. Promedol
D. Acetylsalicilic acid
E. Butadiounum

81. A 66 year old female patient got intravenous injection of magnesium


sulfate solution for the purpose of elimination of hypertensive crisis. But
arterial pressure didn't go down and after repeated introduction of the
same preparation there appered sluggishness, slow response, inhibition of
consciousness and respiration. What preparation is antagonist of
magnesium sulfate and can eliminate symptoms of its overdose?
A. Calcium chloride
B. Potassium chloride
C. Sodium chloride
D. Activated carbon
E. Potassium permanganate

82. A 64 year old woman has impairment of twilight vision (hemeralopy). What
vitamin should be recommended in the first place?
A. Vitamin A
B. Vitamin B2
C. Vitamin E
D. Vitamin C
E. Vitamin B6

83. A patient underwent appendectomy. In the postoperative period he has


been taking an antibiotic. The patient complains about hearing
impairment and vestibular disorders. What group of antibiotics has such
by-effects?
A. Aminoglycosides
B. Penicillins
C. Tetracyclines
D. Macrolides
E. Cephalosporins
84. A student came to see a doctor and asked to administer him a drug for
treatment of allergic rhinitis that occurs in the period of linden flowering.
What drug may be used?
A. Loratadine
B. Noradrenaline hydrotartrate
C. Propanolol
D. Ambroxol
E. Losartan

85. A 7 year old child is ill with bronchitis. It is necessary to administer him an
antibacterial drug. What drug of fluoroquinolone group is CONTRA-
INDICATED at this age?
A. Cyprofloxacin
B. Ampicillin
C. Amoxicillin
D. Sulfadimethoxine
E. Ampiox

86. A patient consulted a physician about muscle rigidity, constrained


movements, permanent arm tremor. The patient was diagnosed with
Parkinson's disease. What preparation should be administered?
A. Levodopa
B. Phenytoin
C. Phenobarbital
D. Diazepam
E. Ethosuximide

87. A patient takes digoxin for treatment of cardiac insufficiency. What


diuretic may increase digoxin toxicity due to the intensified excretion of
K+ ions?
A. Hydrochlorothiazide
B. Spironolactone
C. Panangine
D. Siliborum
E. Lisinopril

88. A patient with coronary artery disease was admitted to the cardiological
department. For stenocardia prevention a drug from the group of \beta-
adrenoceptor blockers was administered. What drug is it?
A. Metoprolol
B. Atropine sulfate
C. Morphine hydrochloride
D. Oxytocin
E. Furosemide

89. A woman was delivered to a hospital for trachea intubation. What of the
following drugs should be applied in this case?
A. Dithylinum
B. Nitroglycerine
C. Metronidazole
D. Atropine sulfate
E. Gentamycin sulfate

90. A patient suffers from pulmonary tuberculosis. During treatment neuritis of


visual nerve arose. What drug has caused this by -effect?
A. Isoniazid
B. Ethambutol
C. Kanamycin
D. Rifampicin
E. Streptomycin

91. A patient ill with amebiasis was prescribed a certain drug. The use of
alcohol together with this drug is contra-indicated because the drug inhibits
metabolism of ethyl alcohol. What drug is it?
A. Metronidazole
B. Reserpine
C. Clonidine
D. Diazepam
E. Aminazine

92. A patient suffers from chronic left-ventricular insufficiency. What


medication should be administered?
A. Digoxin
B. Bemegride
C. Etimizole
D. Vinpocetine
E. Pyracetam

93. A patient ill with chronic cardiac insufficiency was prescribed an average
therapeutic dose of digoxin. Two weeks after begin of its taking there
appeared symptoms of drug intoxication (bradycardia, extrasystole,
nausea). Name the phenomenon that caused accumulation of the drug in
the organism?
A. Material cumulation
B. Functional cumulation
C. Tolerance
D. Tachyphylaxis
E. Idiosyncrasy

94. A patient with drug intoxication presented with the dryness of oral
mucous membrane and mydriatic pupils. Such action of this drug is
associated with the following effect:
A. Muscarinic cholinoreceptor block
B. Muscarinic cholinoreceptor stumulation
C. Nicotinic cholinoreceptor stumulation
D. Adrenoreceptor stimulation
E. Adrenoreceptor block

95. A patient with a limb fracture must be administered a depolarizing drug


from the myorelaxant group for the purpose of a short-time surgery. What
drug is it?
A. Dithylinum
B. Tubocurarine chloride
C. Cytitonum
D. Atropine sulfate
E. Pentaminum

96. A patient suffering from chronic bronchitis takes a synthetic mucolytic drug
that facilitates the sputum thinning. What drug is it?
A. Acetylcysteine
B. Diazepam
C. Heparin
D. Furosemide
E. Enalapril

97. Burned skin surface was treated with a certain preparation. Its
antiseptic properties are provided by atomic oxygen that is formed in
presence of organic substances. What preparation was applied?
A. Potassium permanganate
B. Furacillin
C. Chlorhexidine bigluconate
D. Alcoholic iodine solution
E. Sodium hydrocarbonate

98. On the 2-3rd day after stomach resection intestinal peristalsis wasn't
restored. What is to be administered for stimulation of gastrointestinal
tract?
A. Proserin
B. Prasosin
C. Cyclodole
D. Atropine sulfate
E. Acetylcholine

99. A stomatologist injected a patient with a certain drug in order to reduce


salivation during tooth filling. What drug is it?
A. Atropine sulfate
B. Adrenaline hydrochloride
C. Proserin
D. Pilocarpine hydrochloride
E. Mesaton

100. A patient with chronic cardiac insufficiency has been taking foxglove
(Digitalis) preparations for a long time. Due to the violation of intake
schedule the woman got symptoms of intoxication. These symptoms result
from:
A. Material cumulation
B. Tachyphylaxis
C. Idiosyncrasy
D. Antagonism
E. Sensibilization

101. Before tooth extraction a patient was advised to take a certain


drug for haemorrhage prevention. What drug was advised?
A. Vicasolum
B. Heparin
C. Asparcam
D. Magnesium sulfate
E. Dimedrol

102. A man who has been taking a drug for a long time cannot withhold
it because this causes impairment of psychic, somatic and vegetative
functions. Name the syndrome of different disturbances caused by drug
discontinuation:
A. Abstinence
B. Sensibilization
C. Idiosyncrasy
D. Tachyphylaxis
E. Cumulation

103. A patient consulted a stomatologist about purulent inflammation of


his gums. What drug will be the most effective if it is suspected that a
causative agent is an anaerobe?
A. Metronidazole
B. Gentamicin
C. Oxacillin sodium
D. Co-trimoxazole
E. Nitroxoline

104. A 45-year-old patient suffers from neurosis characterized by


irritability, sleeplessness, motiveless anxiety. What drug would
eliminate all the symptoms?
A. Diazepam
B. Valerian extract
C. Pyracetam
D. Caffeine sodium benzoate
E. Levodopa

105. A 12 year old child has intolerance to some foodstuffs. Their


consumption causes an allergic reaction in form of itching skin eruptions.
What antihistaminic drug should be admistered so that the child could
attend school?
A. Loratadine
B. Dimedrol
C. Diclofenac
D. Aminophylline
E. Ephedrine

106. A patient that entered the admission office had the following signs of
acute cardiac insufficiency: paleness, acrocyanosis, frequent shallow
respiration. What drug is indicated in this case?
A. Corglycon
B. Digitoxin
C. Cordiamin
D. Nitroglycerine
E. Adrenaline hydrochloride

107. A patient with myocardial infarction was admitted to the


cardiological department. For pain relief it was decided to potentiate
fentanyl action with a neuroleptic. Which of the following neuroleptics is
the most suitable for neuroleptanalgesia?
A. Droperidol
B. Aminazine
C. Triftazine
D. Haloperidol
E. Sulpiride

108. A patient with bacterial pneumonia was prescribed benzylpenicillin.


What is the mechanism of its antibacterial effect?
A. Inhibition of synthesis of microorganism wall
B. Inhibition of intracellular protein synthesis
C. Abnormal permeability of cytoplasmic membrane
D. Inhibition of SH-groups of microorganism enzymes
E. Antagonism with p-amino-benzoic acid

109. A patient has herpetic rash. What medication should be administered?


A.Acyclovir
B.Gentamycin
C.Clotrimazole
D.Benzylpenicillin sodium salt
E.Biseptol

110. A patient who has been treated in a neural clinic and has been
taking a sedative for a long time got the following complication: cough,
rhinitis, epiphora. What drug caused these disturbances?
A. Sodium bromide
B. Diazepam
C. Valerian
D. Phenazepam
E. Reserpine
111. In order to accelerate healing of a radiation ulcer a vitamin drug was
administered. What drug is it?
A. Retinol acetate
B. Retabolil
C. Prednisolone
D. Levamisole
E. Methyluracil

112. A patient with hip fracture was prescribed a narcotic analgetic.


Its anesthetic action is determined by interaction with the following
receptors:
A. Opiate receptors
B. Adrenoreceptors
C. Cholinoreceptors
D. Benzodiazepine receptors
E. GABA-ergic receptors

113. A patient with hypertensic crisis was admitted to the cardiological


department, he was injected intravenously with an antihypertensive drug -
salt of an alkaline-earth metal. What drug was injected?
A. Magnesium sulfate
B. Potassium chloride
C. Sodium hydrocarbonate
D. Calcium lactate
E. Benzohexamethonium

114. A patient with acute morphine poisoning was delivered to a hospital.


What specific narcotic antagonist should be chosen in this case?
A. Naloxone
B. Paracetamol
C. Methacin
D. Digoxin
E. Unithiol

You might also like